Vous êtes sur la page 1sur 170

MyPasTest MRCS A Online - Jan Exam 2015

10. Physiology; System Specific Physiology (399Qs)


----------------------------------------------------------------------------------------------------------------------------------
01. A 45-year-old woman is seen for follow-up six weeks after hemithyroidectomy. Her blood results are:
TSH 2.1 mU/l, T3 5.6 pmol/l and T4 110nmol/l. What is combined to generate T3 and T4 in the thyroid?
Single best answer select one answer only.
Iodide YOUR ANSWER
Mono-iodotyrosine and di-iodotyrosine CORRECT ANSWER
Page |
Calcitonin
842
Thyroglobulin and iodide
TSH.

The thyroid cells actively absorb and retain iodide for incorporation into the mono-iodotyrosine and di-
iodotyrosine they synthesize. These molecules combine to form T3 and T4. These are stored in the
thyroid cells bound to thyroglobulin. On stimulation of the cell by thyroid-stimulating hormone (TSH)
(from the anterior pituitary) the T3/T4 is released and expelled from the cell into the bloodstream where it
travels to its target organ bound to a plasma carrier protein.

Calcitonin is produced by the parafollicular C cells of the thyroid. Calcitonin lowers the circulating
calcium and phosphate.

02. You are on an ITU ward round when the consultant asks you how best to measure renal function.
Which of the following statements concerns an ideal substance for measuring glomerular filtration rate
(GFR)? Single best answer - select one answer only.
Creatinine is an ideal substance for measuring the GFR YOUR ANSWER
Insulin is an ideal substance for measuring the GFR
The substance should be secreted by the renal tubule
The substance may be reabsorbed by the renal tubule
The substance should be freely filtered by the glomerulus CORRECT ANSWER.

An ideal substance for measuring the glomerular filtration rate (GFR) should be freely filtered at the
glomerulus and neither secreted nor reabsorbed by the tubules. Inulin (not insulin), a polysaccharide
molecule, meets these criteria for an ideal substance. It can be administered intravenously to patients to
measure their GFR. Creatinine is not an ideal substance for measuring GFR as a small amount is secreted
by the tubules. This means that slightly more creatinine is excreted in the urine than is filtered. However,
the laboratory usually underestimates the plasma creatinine level and so these two errors cancel one
another out. Therefore creatinine provides a useful clinical estimation of GFR.

03. A 54-year-old with ESRF presents 2 days post dialysis with chills, fever and nausea. Which of the
following statements regarding renal replacement therapy is correct? Single best answer - select one
answer only.
May involve replacing filtered fluid in the peritoneal cavity so-called haemofiltration YOUR ANSWER
May be indicated first line for fluid overload
---------------------------------------------------------------------------------------------------------------------------------------------------------------------------------------------------------------
Dr Mohammed Shamsul Islam Khan, Medical Officer, Clinical Neuro-Surgery, National Institute of Neuro-Sciences and Hospital
Sher-E-Bangla Nagar, Dhaka-1207, Bangladesh. Mobile: +880 1713 455 662, +880 1685 811979. E-mail: drsikhan@gmail.com
MyPasTest MRCS A Online - Jan Exam 2015
10. Physiology; System Specific Physiology (399Qs)
----------------------------------------------------------------------------------------------------------------------------------
May be indicated in hypokalaemia
May require the formation of an arteriovenous fistula CORRECT ANSWER
Should not be considered in cases where rapid ultrafiltration is required.

Renal replacement therapy is indicated in intractable fluid overload (ITU), hyperkalemia, acidosis and Page |
uraemia when medical therapy has failed. It can be administered as a holding measure to cover an acute
crisis or in the chronic setting. In the acute setting, such as on ITU, haemofiltration is usually used, where 843
the patients own blood pressure drives the formation of an ultrafiltrate. The advantage over
haemodialysis is that the machinery involved is less complex and it can continue for many hours.

In the chronic setting, where the maintenance of a reasonable lifestyle of the patient is requisite and the
process must be rapid/intermittent, haemodialysis or peritoneal dialysis is commonly used.
Haemodialysis like haemofiltration involves the patients blood being pumped through an artificial kidney
where waste products are exchanged with dialysis fluid, which surrounds the semi-permeable tunes
through which the blood is passed.

Often an arteriovenous fistula is formed surgically to allow easy vascular access for haemodialysis.
Peritoneal dialysis involves fluid being pumped into the abdominal cavity into which waste products
diffuse. The fluid must be changed episodically (often four times per day).

04. Gastrointestinal (GI) resistance vessels are affected by their unique vascular properties, by the
presence of vasodilator metabolites, by autonomic nerves and by a variety of circulating agents. Which of
the following circulating factors dilates GI resistance vessels? Single best answer question choose ONE
true option only.
Adrenaline YOUR ANSWER
Angiotensin II
Noradrenaline
Vasoactive intestinal peptide CORRECT ANSWER
Vasopressin.

A number of agents released into the circulation may affect GI blood flow. Important vasoconstrictors
include noradrenaline, adrenaline and dopamine released from the adrenal medulla, angiotensin II
generated as a consequence of renal rennin release and vasopressin released from the posterior
pituitary. Important vasodilators include vasoactive intestinal peptide and the hormones gastrin,
cholecystokinin and glucagon.

05. There are three ways by which carbon dioxide is transported in the blood. About 70% of the carbon
dioxide is transported to the lungs: Single best answer question choose ONE true option only.
In the form of bicarbonate ions YOUR ANSWER (Correct Answer)
In the form of carbonic acid
In the form of carbaminohaemoglobin

---------------------------------------------------------------------------------------------------------------------------------------------------------------------------------------------------------------
Dr Mohammed Shamsul Islam Khan, Medical Officer, Clinical Neuro-Surgery, National Institute of Neuro-Sciences and Hospital
Sher-E-Bangla Nagar, Dhaka-1207, Bangladesh. Mobile: +880 1713 455 662, +880 1685 811979. E-mail: drsikhan@gmail.com
MyPasTest MRCS A Online - Jan Exam 2015
10. Physiology; System Specific Physiology (399Qs)
----------------------------------------------------------------------------------------------------------------------------------
In chemical combination with albumin
In the dissolved state in the water of the plasma and cells.

Dissolved carbon dioxide reacts with water inside red blood cells to form carbonic acid. This reaction is
catalysed by a protein enzyme in the red cells called carbonic anhydrase. Most of the carbonic acid
immediately dissociates into bicarbonate ions and hydrogen ions, the hydrogen ions in turn combining
Page |
with haemoglobin. Approximately 23% of the carbon dioxide produced in the tissues combines directly 844
with haemoglobin to form carbaminohaemoglobin and an additional 7% is transported in the dissolved
state in the water of the plasma and cells.

06. A 26-year-old female presents to the pre assessment clinic with elevated serum calcium levels. She
tells you she has familial hypocalciuric hypercalcaemia. Which of the following statements is correct
regarding familial hypocalciuric hypercalcaemia? Single best answer - select one answer only.
Hypermagnesaemia may be seen YOUR ANSWER (Correct Answer)
It is autosomal recessive
It is associated with increased urinary calcium excretion more than 200 mg per 24 hours
It is associated with low parathyroid hormone (PTH) levels
The condition responds to parathyroidectomy.

This rare autosomal dominant condition is associated with decreased urinary calcium excretion less
than 200 mg per 24 hours. There is no response to parathyroidectomy but the prognosis is excellent.
Hypermagnesaemia may be seen, parathyroid hormone (PTH) levels are normal or slightly elevated.

07. A 69-year-old women with a previous myocardial infarct is admitted to HDU with shortness of breath,
peripheral oedema and signs of chest sepsis. A line is inserted to measure her central venous pressure.
Which of the following is correct regarding CVP? Single best answer select one answer only.
CVP can be estimated by examining the external jugular vein YOUR ANSWER (Correct Answer)
Indicates left ventricular filling pressure in left ventricular failure
Correlates well with cardiac afterload
Is most useful when taken as a one-off measurement
Is low in hypervolemia.

The central venous pressure (CVP) is an important concept in clinical cardiology because it is a major
determinant of the filling pressure and therefore the preload of the right ventricle (and not the cardiac
afterload). The CVP can be measured invasively by using a manometer after central venous cannulation.
The CVP can be estimated by examining the height of the pulsation in the external jugular vein.

The height of the pulsation should be compared with the angle of Louis as the patient sits at 45 to the
horizontal. Zero is taken at the level of the right atrium. This is somewhat arbitrary and so one-off
measurements are not very useful unless there is a profoundly low CVP indicating profound hypovolemia.
More useful is to measure the changes in CVP occurring during and after the administration of a bolus of
---------------------------------------------------------------------------------------------------------------------------------------------------------------------------------------------------------------
Dr Mohammed Shamsul Islam Khan, Medical Officer, Clinical Neuro-Surgery, National Institute of Neuro-Sciences and Hospital
Sher-E-Bangla Nagar, Dhaka-1207, Bangladesh. Mobile: +880 1713 455 662, +880 1685 811979. E-mail: drsikhan@gmail.com
MyPasTest MRCS A Online - Jan Exam 2015
10. Physiology; System Specific Physiology (399Qs)
----------------------------------------------------------------------------------------------------------------------------------
200 ml of colloid intravenously. If the patient is hypovolemic, there will be only a very small transient
change in CVP.

In normovolemia there will be a rise in CVP, which will be sustained for 10 min or so. In hypervolemia the
rise in CVP will be sustained for longer. A high CVP indicates myocardial failure. CVP indicates the right
and left ventricular filling pressure only when these are equal. If there is ventricular failure and the filling
pressure of the two ventricles is not equivalent then CVP is a measure only of the right atrial pressure.
Page |
845
08. You attend a trauma call to a 40-year-old patient in resus who has sustained a significant isolated
closed head injury and has a GCS of 7. Which is the most appropriate way to definitively manage the
patients airway? Select one answer only.
Cricothyroidotomy YOUR ANSWER
Nasopharyngeal tube
Oropharyngeal tube
Orotracheal intubation CORRECT ANSWER
Tracheostomy.

A definitive airway requires a tube present in the trachea with the cuff inflated (although uncuffed for
prepubertal children), the tube connected to some form of oxygen-enriched assisted ventilation and the
airway secured in place with tape. Oropharyngeal or nasopharyngeal tubes (airways) are not forms of
definitive airways. Definitive airways are of three varieties: orotracheal tube; nasotracheal tube and;
surgical airway (cricothyroidotomy or tracheostomy). Rapid sequence induction of anaesthesia with
orotracheal intubation is preferred by most anaesthetists as the primary method of securing a definitive
airway. There are not features in this case which suggest the need for a surgical airway over an
orotracheal tube.

The decision to provide a definitive airway is based on clinical findings including:


01. presence of apnoea
02. inability to maintain a patent airway by other means
03. need to protect the lower airway from aspiration of blood, vomitus or other foreign bodies
04. impending or potential compromise of the airway, eg, following inhalation injury or facial
fractures
05. major facial burns
06. compression from neck haematoma, eg, retropharyngeal haematoma
07. upper airway, pharyngeal or facial haemorrhage
08. presence of a closed head injury requiring assisted ventilation in patients with a GCS score <8
09. sustained seizure activity
10. inability to maintain adequate oxygenation by face mask oxygen supplementation

---------------------------------------------------------------------------------------------------------------------------------------------------------------------------------------------------------------
Dr Mohammed Shamsul Islam Khan, Medical Officer, Clinical Neuro-Surgery, National Institute of Neuro-Sciences and Hospital
Sher-E-Bangla Nagar, Dhaka-1207, Bangladesh. Mobile: +880 1713 455 662, +880 1685 811979. E-mail: drsikhan@gmail.com
MyPasTest MRCS A Online - Jan Exam 2015
10. Physiology; System Specific Physiology (399Qs)
----------------------------------------------------------------------------------------------------------------------------------
09. A 62-year-old man was admitted with an exacerbation of chronic obstructive pulmonary disease. His
arterial blood gases on air showed pH = 7.29, p(CO2) = 65.3 mmHg, p(O2) = 62 mmHg and standard
bicarbonate = 30.5 mmol/l. This patient had: Single best answer question choose ONE true option only.
Metabolic acidosis YOUR ANSWER
Metabolic alkalosis
Page |
Mixed acidosis
846
Respiratory acidosis CORRECT ANSWER
Respiratory alkalosis.

This patient had an acidosis with a high p(CO2) and normal standard bicarbonate-respiratory acidosis.
This is a common finding in acute exacerbations of chronic obstructive pulmonary disease. It is usually
managed with nebulisers, steroids and antibiotics and non-invasive ventilation.

10. A three-month-old infant is admitted to the paediatric ward with a persistent fever. As part of the
workup, a lumbar puncture is performed. When the CSF is analysed in the lab, it is found to be normal.
Which of the following would you expect to be true with regards the CSF? Single best answer select one
answer only.
Contains less glucose than blood YOUR ANSWER (Correct Answer)
Contains less glucose than nasal mucus
Contains more lactic acid than blood
Contains more protein than blood
Has a more alkalotic pH than blood.

The total volume of CSF is approximately 150 ml. The rate of CSF production is about 550 ml/day. The
composition of CSF is essentially the same as that of brain extracellular fluid. The pH of CSF is 7.33,
whereas that of plasma is 7.4 due to the higher PCO2 in CSF than in plasma. The protein, glucose, lactic
acid, cholesterol, potassium and calcium concentrations of CSF are all lower than in plasma. However,
CSF does contain more glucose than nasal mucus; so a positive glucose dipstix test of clear nasal
discharge indicates a CSF leak. The lumbar CSF pressure is normally 718 cmH2O.

11. A 46-year-old woman is diagnosed with primary hyperparathyroidism. Which of the following
symptoms may she complain of? Single best answer select one answer only.
Diarrhoea YOUR ANSWER
Dysuria
Neck pain
Euphoria
Depression CORRECT ANSWER.

Hypercalcaemia typically produces the symptoms of bones, stones, moans and groans:
01. bones (arthralgia and osteoporosis)
---------------------------------------------------------------------------------------------------------------------------------------------------------------------------------------------------------------
Dr Mohammed Shamsul Islam Khan, Medical Officer, Clinical Neuro-Surgery, National Institute of Neuro-Sciences and Hospital
Sher-E-Bangla Nagar, Dhaka-1207, Bangladesh. Mobile: +880 1713 455 662, +880 1685 811979. E-mail: drsikhan@gmail.com
MyPasTest MRCS A Online - Jan Exam 2015
10. Physiology; System Specific Physiology (399Qs)
----------------------------------------------------------------------------------------------------------------------------------
02. stones (renal)
03. abdominal groans (pancreatitis, peptic ulcers and constipation)
04. moans (depression, fatigue and weakness).
At an extreme, elevated serum calcium can also cause confusion, seizures and coma.

12. A 79-year-lady presents to the pre-admission clinic with untreated hypertension. After discussion with
Cardiology you start her on lisinopril, which acts to block angiotensin secretin enzyme and lowers
Page |
aldosterone. Of the following, which one reflects a true action of aldosterone? Single best answer - select 847
one answer only.
Decreases excretion of ammonium YOUR ANSWER
Decreases secretion of hydrogen
Decreases secretion of potassium
Increases excretion of magnesium
Increases secretion of potassium CORRECT ANSWER.

The primary effects of aldosterone on the renal system are to increase reabsorption of sodium and
increase secretion of potassium and hydrogen ions. Aldosterone acts by increasing the number and
activity of the active transport systems in the distal renal tubule to sodium and potassium. Owing to the
enhanced reabsorption of sodium, water reabsorption is also increased. Aldosterone increases excretion
of both ammonium and magnesium.

13. A 28-year-old man was admitted with status epilepticus. He was given intravenous diazepam. Arterial
blood gases on 15 l/min via a reservoir bag mask showed pH 7.05, p(CO2) = 61.5 mmHg (8.20Kpa), p(O2) =
115 mmHg (15.33Kpa) (and standard bicarbonate = 16 mmol/l. His other results were sodium = 140 mmol/l,
potassium = 4 mmol/l and chloride = 98 mmol/l. This patient most probably had: Single best answer
question choose ONE true option only.
Metabolic acidosis YOUR ANSWER
Metabolic alkalosis
Mixed acidosis CORRECT ANSWER
Respiratory acidosis
Respiratory alkalosis.

This patient had acidosis with both a high p (CO2) and a low standard bicarbonate-mixed acidosis. The
anion gap is 30 mmol/l (increased). The p(O2) is lower than expected because the patient was breathing
around 70% oxygen. Does this fit with the clinical picture? Yes, he had a lactic acidosis from prolonged
fitting and a respiratory acidosis from intravenous diazepam. This disturbance will return to normal with
attention to either A - airway manoeuvres and oxygen, B - assisted ventilation if needed or C - treatment
with fluids.

14. Increased venous return to the heart is most likely to be caused by Single best answer question
choose ONE true option only.

---------------------------------------------------------------------------------------------------------------------------------------------------------------------------------------------------------------
Dr Mohammed Shamsul Islam Khan, Medical Officer, Clinical Neuro-Surgery, National Institute of Neuro-Sciences and Hospital
Sher-E-Bangla Nagar, Dhaka-1207, Bangladesh. Mobile: +880 1713 455 662, +880 1685 811979. E-mail: drsikhan@gmail.com
MyPasTest MRCS A Online - Jan Exam 2015
10. Physiology; System Specific Physiology (399Qs)
----------------------------------------------------------------------------------------------------------------------------------
Deep inspiration YOUR ANSWER (Correct Answer)
Forced expiration
Hypovolaemia
Positive pressure ventilation
Page |
Tension pneumothorax.
848
Blood returns to the heart from the lower limbs via the action of the calf muscle pumps, valves in the
veins of the leg, and the effect of negative intra-thoracic pressure generated during inspiration. Anything
causing the intra thoracic pressure to become less negative will decrease the venous return to the right
atrium. Tension pneumothorax, positive pressure ventilation and forced expiration all cause this effect,
and therefore reduce the venous return. Although hypovolaemia may cause vasoconstriction in an
attempt to increase venous return, it is unlikely to increase above normal levels.

15. The normal glomerular filtration rate (GFR) for both kidneys is 125 ml/min per 1.73 m2 of body surface
area. GFR increases by: Single best answer question choose ONE true option only.
Afferent arteriolar constriction YOUR ANSWER
Angiotensin II release
Efferent arteriolar constriction CORRECT ANSWER
Reducing renal blood flow
Sympathetic stimulation.

Increased renal blood flow increases the GFR, while reduced renal blood flow reduces GFR. Afferent
arteriolar constriction results in decreased renal blood flow and decreased glomerular pressures, leading
to decreased GFR. Efferent arteriolar constriction results in increased resistance to outflow from
glomeruli. This results in increased glomerular pressure with resultant increased GFR. Sympathetic
stimulation causes preferential afferent arteriolar constriction, which decreases renal blood flow and
decreases glomerular pressure, resulting in decreased GFR. Angiotensin II induces contraction of
mesangial cells, causing decreased glomerular capillary area with resultant reduction in GFR.

16. You attend a trauma call in the emergency department. The patient is a 32-year-old male who has been
stabbed in the left side of the chest, just below the fifth rib. You suspect that the patient has cardiac
tamponade. Which of the following signs would be consistent with the diagnosis of cardiac tamponade?
Select one answer only.
A globular cardiac outline on chest X-ray (CXR) YOUR ANSWER (Correct Answer)
Charcots triad
Collapsed neck veins
Hypertension
Prominent heart sounds.
The classical symptom triad of cardiac tamponade (Becks triad) includes muffled heart sounds,
distended neck veins and hypotension, and is seen in the majority of patients. Pulsus paradoxus is
---------------------------------------------------------------------------------------------------------------------------------------------------------------------------------------------------------------
Dr Mohammed Shamsul Islam Khan, Medical Officer, Clinical Neuro-Surgery, National Institute of Neuro-Sciences and Hospital
Sher-E-Bangla Nagar, Dhaka-1207, Bangladesh. Mobile: +880 1713 455 662, +880 1685 811979. E-mail: drsikhan@gmail.com
MyPasTest MRCS A Online - Jan Exam 2015
10. Physiology; System Specific Physiology (399Qs)
----------------------------------------------------------------------------------------------------------------------------------
defined as a fall in systolic blood pressure of over 10 mmHg on inspiration and this only occurs in 1 in 10
cases of tamponade. In cardiac tamponade, the cardiac outline on CXR is classically globular. Charcots
triad refers to fever, jaundice and abdominal pain, and is suggestive of cholangitis.

17. A 24-year-old woman undergoes resection of the terminal ileum with fashioning of an ileostomy for
Crohns disease. Some 2 weeks after surgery, she is making a good recovery, and is eating a high-energy,
low-residue diet, but has a high ileostomy volume, necessitating intravenous fluid replacement. Her
Page |
serum calcium concentration is 1.82 mmol/l, phosphate 1.28 mmol/l, alkaline phosphatase 82 U/l (normal 849
< 150), albumin 30 g/l, creatinine 80 m mol/l. Prior to surgery, her serum calcium concentration was 2.18
mmol/l, albumin 36 g/l. What is the most likely cause of her hypocalcaemia? Single best answer question
choose ONE true option only.
Formation of insoluble calcium salts in the intestine YOUR ANSWER
Hypoalbuminaemia
Hypomagnesaemia CORRECT ANSWER
Malabsorption of calcium
Malabsorption of vitamin D.

Impaired fat absorption can lead to the formation of insoluble calcium salts in the gut. Fat and calcium are
absorbed in the proximal small intestine, so, too, is vitamin D. Although bile salts are absorbed distally,
and impaired absorption can lead to a secondary decrease in proximal fat absorption, this is unlikely to
be responsible for hypocalcaemia developing so quickly. The normal alkaline phosphatase level also
mitigates against vitamin D deficiency.

Hypocalcaemia would normally be expected to stimulate parathyroid hormone secretion and cause the
plasma phosphate concentration to fall (PTH is phosphaturic). Patients with ileostomies can lose large
amounts of magnesium through their stomas; hypomagnesaemia impairs PTH secretion and can cause
hypocalcaemia that is resistant to an increased provision of calcium.

18. A 12-year-old boy is taken to his GP with a history of increasing lethargy and fatigue. On examination
he is noted to have increased skin pigmentation in his skin creases and buccal mucosa. His blood tests
show hyperkalaemia and a mild acidosis, he is thought to have Addisons disease. This is caused by a
deficiency in cortisol and aldosterone. Which of the following tissue types is the main source of
aldosterone? Single best answer - select one answer only.
Anterior pituitary gland YOUR ANSWER
Chromaffin cells of the adrenal medulla
Zonaglomerulosa of the adrenal cortex CORRECT ANSWER
Zonafasciculata of the adrenal cortex
Zonareticularis of the adrenal cortex.

Chromaffin cells are specialised sympathetic post-ganglionic neurones that release a variety of hormones
when stimulated to do so including adrenaline, noradrenaline, dopamine. The adrenal cortex produces
corticosteroids (mineralocorticoids such as aldosterone and glucocorticoids such as cortisol) and sex
---------------------------------------------------------------------------------------------------------------------------------------------------------------------------------------------------------------
Dr Mohammed Shamsul Islam Khan, Medical Officer, Clinical Neuro-Surgery, National Institute of Neuro-Sciences and Hospital
Sher-E-Bangla Nagar, Dhaka-1207, Bangladesh. Mobile: +880 1713 455 662, +880 1685 811979. E-mail: drsikhan@gmail.com
MyPasTest MRCS A Online - Jan Exam 2015
10. Physiology; System Specific Physiology (399Qs)
----------------------------------------------------------------------------------------------------------------------------------
steroids. The adrenal cortex is composed of the zonaglomerulosa (which produces mineralocorticoids),
zonafasciculata (which produces glucocorticoids) and zonareticularis (which produces sex steroids).

19. A 35-year-old maintenance engineer falls from a flagpole and is admitted with a head injury and is
profoundly hypotensive. Which of the following regarding neurogenic shock is correct? Single best
answer select one answer only.
Page |
Causes decreased cardiac pre-load YOUR ANSWER (Correct Answer) 850
Is characterised by tachycardia
Is associated with cool peripheries
Leads to a narrowed pulse pressure
Can be caused by isolated intracranial injuries.

Neurogenic shock follows spinal transaction, usually a high cervical spine injury or brainstem injury with
loss of sympathetic outflow below the level of injury and consequent vasodilatation. The rapid increase in
size of the vascular bed, including venous capacitance vessels (venous pooling of blood), leads to a
reduction in the venous return and cardiac output (due to less blood being returned to the heart
decreased pre-load). Loss of sympathetic tone also leads to hypotension. The classic picture of
neurogenic shock is hypotension without tachycardia or cutaneous vasoconstriction (causing warm pink
peripheries).

The other signs and symptoms may include tachypnoea, diaphragmatic breathing and quadriplegia. A
narrowed pulse pressure is not seen in neurogenic shock. Isolated intracranial injuries do not cause
neurogenic shock or any other forms of shock. The presence of shock in a patient with a head injury
necessitates a search for another cause of shock. Patients sustaining a spinal injury would have also
suffered a concurrent thoracic, abdominal or pelvic trauma.

The loss of sympathetic tone compounds the physiological effects of hypovolemia and hypovolemia
compounds the physiological effects of sympathetic denervation. Therefore, patients with diagnosed or
suspected neurogenic shock should be treated initially for hypovolemia. The inability to restore organ
perfusion with adequate fluid resuscitation suggests either continuing haemorrhage or a neurogenic
shock.

20. A 26-year-old man was seen in the Accident and Emergency Department following a stab injury to his
left chest in the third intercostal space in the mid-axillary line. He was haemodynamically stable and his
chest X-ray showed a pneumothorax. The most likely response in this patient upon entry of air into the
chest would be for the: Single best answer question choose ONE true option only.
Lung to collapse inward and the chest wall to collapse inward YOUR ANSWER
Lung to collapse inward and the chest wall to spring outward CORRECT ANSWER
Lung to expand outward and the chest wall to spring inward
Lung to expand outward and the chest wall to spring outward
Lung volume to be unaffected and chest wall to spring outward.
---------------------------------------------------------------------------------------------------------------------------------------------------------------------------------------------------------------
Dr Mohammed Shamsul Islam Khan, Medical Officer, Clinical Neuro-Surgery, National Institute of Neuro-Sciences and Hospital
Sher-E-Bangla Nagar, Dhaka-1207, Bangladesh. Mobile: +880 1713 455 662, +880 1685 811979. E-mail: drsikhan@gmail.com
MyPasTest MRCS A Online - Jan Exam 2015
10. Physiology; System Specific Physiology (399Qs)
----------------------------------------------------------------------------------------------------------------------------------
The response to a stab wound that punctures the lung demonstrates the elasticity of the lung and chest
wall. The tendency of the lung to collapse is normally balanced by the tendency of the chest wall to spring
out. Therefore, intrapleural pressures are subatmospheric.Introduction of air in this space allows the lung
to collapse and not to expand outward. At the same time the chest wall will spring outward, not inward.
Lung collapse obviously results in very low lung volume.

21. A 45-year old man is being screened for secondary hypertension. He complains of muscle weakness
Page |
and his blood results show hypernatraemia, hypokalaemia and metabolic alkalosis. What is the most 851
likely diagnosis? Single best answer select one answer only.
Addisons YOUR ANSWER
Coarctation of the aorta
Conns syndrome CORRECT ANSWER
Incidental findings in patient with essential hypertension
Phaeochromocytoma.

Conns syndrome is due to the excessive production of aldosterone. The syndrome is characterised by
hypertension, renal damage, hypernatraemia, hypokalaemic alkalosis and muscle weakness. The body
potassium is depleted, but urinary potassium content is high.

Addisons would cause hypotension rather than hypertension and the blood results would show
hyponatraemia, hyperkalaemia and metabolic acidosis.

Phaeochromocytoma and coarctation of the aorta would not give these biochemical derangements.

22. A patient in the Intensive Care Unit demonstrates hypoxia secondary to severe pneumonia, and
suffers a subsequent derangement in acid-base balance. Which of the following options is a consequence
of the normal physiological response to hypoxia? Select one answer only.
Decrease in arterial blood pressure YOUR ANSWER
Decrease in blood flow to skeletal muscles
Respiratory acidosis
Respiratory alkalosis CORRECT ANSWER
Respiratory depression.

Chemoreceptors found in the carotid and aortic bodies are sensitive to changes in the chemistry of blood,
resulting in stimulation of the respiratory centre. So, increasing respiratory rate causing respiratory
alkalosis. Arterial blood pressure would also rise indirectly, increasing blood flow to the brain and
skeletal muscles.

23. You insert a subclavian central line for one of you patients, but forget to close it following insertion.
Which recognised complication of central venous line insertion is most likely to occur as a result? Select
one answer only.
Arterial air embolism YOUR ANSWER
---------------------------------------------------------------------------------------------------------------------------------------------------------------------------------------------------------------
Dr Mohammed Shamsul Islam Khan, Medical Officer, Clinical Neuro-Surgery, National Institute of Neuro-Sciences and Hospital
Sher-E-Bangla Nagar, Dhaka-1207, Bangladesh. Mobile: +880 1713 455 662, +880 1685 811979. E-mail: drsikhan@gmail.com
MyPasTest MRCS A Online - Jan Exam 2015
10. Physiology; System Specific Physiology (399Qs)
----------------------------------------------------------------------------------------------------------------------------------
Chylothorax
Haemorrhage
Pneumothorax
Venous air embolism CORRECT ANSWER.
Page |
Central venous line insertion may cause trauma to adjacent tissues with consequent haemorrhage and 852
pneumothorax. If it is left open then air may enter the blood, causing venous air embolism. Systemic
arterial air embolism may occur if the cannula is mistakenly placed in the carotid artery.

24. Flow through a vessel or lumen is: S ingle best answer quest ion choose ONE true opt ion only.
Is inversely proportional to the pressure head of flow YOUR ANSWER
Is inversely proportional to the radius
Is directly proportional to the length of the tube
Is directly proportional to the viscosity of blood passing through it
Is directly proportional to the fourth power of radius CORRECT ANSWER.

The Hagen-Poiseuille law states that the flow through a vessel is:
01. Directly proportional to the pressure head of flow
02. Directly proportional to the fourth power of radius
03. Inversely proportional to the viscosity
04. Inversely proportional to the length of the tube.

The radius of the tube is therefore the most important determinant of flow through a blood vessel. Thus,
doubling the radius of the tube will lead to a 16-fold increase in flow at a constant pressure gradient. The
implications of this are several fold.

First, owing to the fourth power effect on resistance and flow, active changes in radius constitute an
extremely powerful mechanism for regulating both the local blood flow to a tissue and central arterial
pressure. The arterioles are the main resistance vessels of the circulation and their radius can be actively
controlled by the tension of smooth muscle within its wall.

Second, in terms of intravenous fluid replacement in hospital, flow is greater through a peripheral cannula
than through central lines. The reason is that peripheral lines are short and wide (and therefore of lower
resistance and higher flow) compared to central lines, which are long and possess a narrow lumen. A
peripheral line is therefore preferential to a central line when urgent fluid resuscitation, or blood, is
required.

25. Cerebral blood flow is the blood supply to the brain in any given moment. Which of the following
statement is correct? Single best answer - select one answer only.
Averages approximately 1 litre per minute per 100 g of brain YOUR ANSWER
Decreases at mean arterial pressures of less than 60 mmHg CORRECT ANSWER

---------------------------------------------------------------------------------------------------------------------------------------------------------------------------------------------------------------
Dr Mohammed Shamsul Islam Khan, Medical Officer, Clinical Neuro-Surgery, National Institute of Neuro-Sciences and Hospital
Sher-E-Bangla Nagar, Dhaka-1207, Bangladesh. Mobile: +880 1713 455 662, +880 1685 811979. E-mail: drsikhan@gmail.com
MyPasTest MRCS A Online - Jan Exam 2015
10. Physiology; System Specific Physiology (399Qs)
----------------------------------------------------------------------------------------------------------------------------------
It is supplied entirely by the carotid arterial system
It is diminished with a loss of 10% of the circulating volume of blood in the normal young person
It is increased as intracranial pressure increases.

Blood supply to the brain is by the internal carotid and vertebral arteries, which form the circle of Willis. Page |
The brain lies within the rigid skull. So an increase in the volume of the skull contents will increase the
intracranial pressure. Cerebral blood flow is proportional to the difference between mean arterial pressure 853
and intracranial pressure. CBF averages 50100 ml per min per 100 g of brain. It is sustained in
preference to other organs to which blood flow is diminished when there is loss of blood. Mean arterial
pressure must decrease to less than 60 mmHg before there is a decrease in CBF. This does not occur
until there is a loss of over 40% of the circulating volume in the normal individual (2 l itres in a 75 kg man).

26. Which of the following are true regarding calcium balance? Select one answer only.
Bone resorption occurs due to the actions of osteoblasts YOUR ANSWER
Calcitonin has a profound effect on the control of calcium levels in the human
24,25 Dihydroxyvitamin D3 is the active form of vitamin D
Depends on vitamin D, which must be ingested in the diet only
Active vitamin D3 is produced by hydroxylation in response to the actions of PTH CORRECT ANSWER.

The inorganic element of bone is resorbed through the actions of osteoclasts. It is laid down by
osteoblasts. Control of bone resorption/deposition is by parathyroid hormone (PTH) and vitamin D3.
Vitamin D3 can be made in the skin through the activity of sunlight, but in times of low sun exposure it
must be taken in the diet. PTH from the parathyroid glands is released in response to low serum calcium
levels.

PTH has direct effects on bone resorption and promotes calcium resorption in the kidney and bowel. It
also promotes activation of vitamin D3 by hydroxylation in the kidney to form 1,25-dihydroxyvitamin D3.
The effects of 1,25-dihydroxyvitamin D3 complement those of parathyroid hormone (PTH) and serve to
increase the serum calcium levels. Calcitonin has a minimal effect on calcium levels in the human but can
be extracted (usually from salmon), for use therapeutically to increase bone deposition in osteoporosis.

27. The electrocardiogram (ECG) of a patient shows a prolonged QT interval. A normal QT interval is
usually about: Single best answer question choose ONE true option only.
0.10 s YOUR ANSWER
0.20 s
0.30 s
0.40 s CORRECT ANSWER
0.50 s.

The QT interval is measured from the beginning of the QRS complex to the end of the T-wave. A normal
QT interval is usually about 0.40 s. The QT interval, as well as, the corrected QT interval is important in
---------------------------------------------------------------------------------------------------------------------------------------------------------------------------------------------------------------
Dr Mohammed Shamsul Islam Khan, Medical Officer, Clinical Neuro-Surgery, National Institute of Neuro-Sciences and Hospital
Sher-E-Bangla Nagar, Dhaka-1207, Bangladesh. Mobile: +880 1713 455 662, +880 1685 811979. E-mail: drsikhan@gmail.com
MyPasTest MRCS A Online - Jan Exam 2015
10. Physiology; System Specific Physiology (399Qs)
----------------------------------------------------------------------------------------------------------------------------------
the diagnosis of long-QT syndrome and short-QT syndrome. The QT interval varies based on the heart
rate and various correction factors have been developed to correct the QT interval for the heart rate.

28. A 67-year-old man with severe community acquired pneumonia is admitted to ITU for respiratory
support. Which of the following statements regarding the pathophysiological effects of intermittent
positive pressure ventilation is correct? Single best answer - select one answer only.
Page |
It results in hepatic failure YOUR ANSWER
854
It lowers intracranial pressure
It increases splanchnic blood flow
It causes hypertension
It can cause a pneumothorax CORRECT ANSWER.

IPPV creates a positive intrathoracic pressure (this is normally negative)and a compression tamponade,
thereby reducing venous return, cardiac output and therefore blood pressure. With a reduction in cardiac
output there is reduction in liver, kidney and intestinal blood flow. The alveolimay be subjected to high
inflation pressures and result in subsequent PCO2barotrauma and pneumothorax. IPPV causes an
increase in intracranialpressure, as do positive end-expiratory pressure (PEEP) ventilation
andobstruction of central venous drainage.

29. You review a 39-year-old sportsman who complains of knee pain. Arthroscopy reveals damage to the
cartilage. Which of the following stems best describes a property of hyaline cartilage? Single best answer
question choose ONE true option only.
It has a blood supply from small arterioles YOUR ANSWER
It is rich in type 1 collagen
Chondrocytes secrete collagen only
It is avascular CORRECT ANSWER
Pressure from normal joint loading accelerates damage to cartilage.

Hyaline cartilage forms the articular surface and is avascular, relying on diffusion from synovial fluid for
nutrients. It is rich in type II collagen and forms a meshwork containing proteoglycan molecules that
retain water. Intermittent pressure from joint loading is essential to maintain normal cartilage function.
Chondrocytes secrete proteoglycans and collagen and are embedded in the cartilage. They migrate to the
joint surface along with the matrix that they produce.

30. Which of the following hormones stimulates respiration and causes the arterial p (CO2) to fall during
pregnancy? Single best answer question choose ONE true option only.
Cortisol YOUR ANSWER
Glucose
Insulin

---------------------------------------------------------------------------------------------------------------------------------------------------------------------------------------------------------------
Dr Mohammed Shamsul Islam Khan, Medical Officer, Clinical Neuro-Surgery, National Institute of Neuro-Sciences and Hospital
Sher-E-Bangla Nagar, Dhaka-1207, Bangladesh. Mobile: +880 1713 455 662, +880 1685 811979. E-mail: drsikhan@gmail.com
MyPasTest MRCS A Online - Jan Exam 2015
10. Physiology; System Specific Physiology (399Qs)
----------------------------------------------------------------------------------------------------------------------------------
Oestrogen
Progesterone CORRECT ANSWER.

The high circulating levels of progesterone that occur during pregnancy increase the sensitivity of the
hypothalamic respiratory centres to small increases in p(CO2) and lower the 'set point'. A rise of 1 mmHg
p(CO2) results in an increased ventilation of 6 l/min, which is several times the change noted in non-
Page |
pregnant individuals. During pregnancy normal p(CO2) values fall from about 40 to 32 mmHg. 855
31. You are on an ITU ward round where you review a patient admitted with severe sepsis and multiorgan
failure. As a result he has low urine output and peripheral oedema. He is acidotic and hyperkalaemic.
Which of the following is correct regarding renal replacement therapy? Single best answer - select one
answer only.
It can only be delivered via a central line YOUR ANSWER
It is a cure for kidney disease
It may involve replacing filtered fluid in the peritoneal cavity CORRECT ANSWER
It should be considered first line for fluid overload
Rapid ultrafiltration of the blood cannot be undertaken.

Renal replacement therapy is indicated in intractable fluid overload (ITU), hyperkalaemia, acidosis and
uraemia when medical therapy has failed. It can be administered as a holding measure to cover an acute
crisis or in the chronic setting. In the acute setting, such as on ITU, haemofiltration is usually used, where
the patients own blood pressure drives the formation of an ultrafiltrate.

The advantage over haemodialysis is that the machinery involved is less complex and it can continue for
many hours. In the chronic setting, where the maintenance of a reasonable lifestyle of the patient is
requisite and the process must be rapid/intermittent, haemodialysis or peritoneal dialysis is commonly
used. Haemodialysis like haemofiltration involves the patients blood being pumped through an artificial
kidney where waste products are exchanged with dialysis fluid, which surrounds the semi-permeable
tunes through which the blood is passed.

Often an arteriovenous fistula is formed surgically to allow easy vascular access for haemodialysis.
Peritoneal dialysis involves fluid being pumped into the abdominal cavity into which waste products
diffuse. The fluid must be changed episodically (often four times per day).

32. Growth hormone and glucagon are both polypeptide hormones. Both growth hormone and glucagon:
Single best answer question choose ONE true option only.
Decrease blood glucose concentration YOUR ANSWER
Increase blood glucose concentration CORRECT ANSWER
Increase gluconeogenesis
Increase glycogenolysis
---------------------------------------------------------------------------------------------------------------------------------------------------------------------------------------------------------------
Dr Mohammed Shamsul Islam Khan, Medical Officer, Clinical Neuro-Surgery, National Institute of Neuro-Sciences and Hospital
Sher-E-Bangla Nagar, Dhaka-1207, Bangladesh. Mobile: +880 1713 455 662, +880 1685 811979. E-mail: drsikhan@gmail.com
MyPasTest MRCS A Online - Jan Exam 2015
10. Physiology; System Specific Physiology (399Qs)
----------------------------------------------------------------------------------------------------------------------------------
Increase lipolysis.

Glucagon increases blood glucose concentration, mainly by causing glycogenolysis and


gluconeogenesis in the liver. Growth hormone increases blood glucose concentration by decreasing
glucose utilisation and uptake by the cells. In addition, growth hormone increases the mobilisation of
fatty acids from adipose tissue and increases the use of fatty acids for energy. Page |
33. A 72-year-old lady is admitted with severe community acquired pneumonia and goes into acute renal 856
failure. Which of the following can be used to estimate GFR? Single best answer - select one answer only.
ADH YOUR ANSWER
Glucose
Inulin CORRECT ANSWER
Para-amino hippuric acid
Sodium.

Inulin (not insulin) can be used to measure the glomerular filtration rate. Inulin is freely filtered across the
glomerulus into Bowman's capsule. It is neither reabsorbed, secreted nor metabolised by the cells of the
nephron. These serve as criteria for the measurement of the glomerular filtration rate (GFR). Inulin is
usually used in experimental studies.

Creatinine can be used to estimate GFR in clinical practice, creatinine is a by-product of skeletal muscle
and is produced at a relatively constant rate, any amount produced is proportional to the muscle mass. In
most clinical situations, creatinine clearance provides a reasonably accurate measure of GFR. Para-amino
hippuric acid is an organic ion filtered across the glomerulus and is used to measure renal plasma flow.

34. You are describing the Bohr effect to a group of medical students. What displaces the oxygen-
haemoglobin dissociation curve to the left? Select one answer only.
Anaemia YOUR ANSWER
A fall in 2,3-DPG CORRECT ANSWER
A fall in pH
An increase in PCO2
Pyrexia.

The Bohr effect describes the factors that alter the position of the oxyhaemoglobin dissociation curve. A
left shift signifies increased affinity for O2 whereas a shift to the right signifies a reduced affinity for O 2.
The oxygenhaemoglobin dissociation curve is sigmoid-shaped and reaches a plateau at 70100 mmHg
(PO2).

A left shift is produced by a high pH, alkalosis, a fall in PCO 2, a fall in temperature or a fall in 2,3-DPG
concentration. As a result of a left shift, less oxygen is released (a higher percentage saturation for a

---------------------------------------------------------------------------------------------------------------------------------------------------------------------------------------------------------------
Dr Mohammed Shamsul Islam Khan, Medical Officer, Clinical Neuro-Surgery, National Institute of Neuro-Sciences and Hospital
Sher-E-Bangla Nagar, Dhaka-1207, Bangladesh. Mobile: +880 1713 455 662, +880 1685 811979. E-mail: drsikhan@gmail.com
MyPasTest MRCS A Online - Jan Exam 2015
10. Physiology; System Specific Physiology (399Qs)
----------------------------------------------------------------------------------------------------------------------------------
given PO2, leading to a fall in oxygen delivery). Anaemia produces no shift of the curve but alters the
percentage oxygen saturation of the blood.

35. A 65-year-old woman is admitted with right iliac fossa pain, nitrate positive urine dip and impaired
renal function. Her urea is 24, creatinine is 215, potassium is 6.4. You initiate treatment to counteract this.
Which treatment would be the best initial management? Single best answer - select one answer only.
Page |
Calcichew YOUR ANSWER
857
Inhaled steroids
Insulin and dextrose CORRECT ANSWER
Intravenous fluids
Gentamicin.

Insulin increases cellular potassium uptake and is therefore useful in the treatment of hyperkalaemia. IV
fluids will help with improving pre-renal failure but the raised potassium can be life threatening so needs
urgent correction. Inhaled salbutamol rather than steroids reduce high potassium. This patient potentially
has a renal tract infection if nitrate positive though needs correlation with patients history but gentamicin
is nephrotoxic and should be avoided. Intravenous calcium should be given for myocardial stability oral
calcichew is not indicated.

36. Resection of the terminal ileum is most commonly associated with the malabsorption of which of the
following components? Single best answer - select one answer only.
Bile salts YOUR ANSWER (Correct Answer)
Calcium
Cholesterol
Folic Acid
Sodium.

The terminal ileum is the primary site for absorption of the fat-soluble vitamins B12 and K. In addition, bile
salts are predominantly absorbed here. Calcium and folic acid are mainly absorbed in the jejunum.

37. A 21-year-old man is involved in a road-traffic accident and undergoes an emergency splenectomy.
Which of the following may be seen in the blood film of this patient? Single best answer select one
answer only.
HowellJolly bodies YOUR ANSWER (Correct Answer)
Leukopenia
Neutrophilia
Punctate basophilia
Rouleaux formation.

---------------------------------------------------------------------------------------------------------------------------------------------------------------------------------------------------------------
Dr Mohammed Shamsul Islam Khan, Medical Officer, Clinical Neuro-Surgery, National Institute of Neuro-Sciences and Hospital
Sher-E-Bangla Nagar, Dhaka-1207, Bangladesh. Mobile: +880 1713 455 662, +880 1685 811979. E-mail: drsikhan@gmail.com
MyPasTest MRCS A Online - Jan Exam 2015
10. Physiology; System Specific Physiology (399Qs)
----------------------------------------------------------------------------------------------------------------------------------
One of the functions of the spleen is the sequestration and phagocytosis of old or abnormal red cells.
Therefore, in a postsplenectomy patient, red-cell inclusion bodies may be present, producing target cells,
HowellJolly bodies and sideroblasts. Punctate basophilia is seen in patients with lead poisoning,
thalassaemia, and haemolytic anaemia due to pyrimidine-5-nucleotidase deficiency. Rouleaux formation
is the tendency for red cells to stack up like a pile of coins. The amount of rouleaux formation is
determined by the concentration of protein in the plasma and is therefore seen in Myeloma, where blood
viscosity is high.
Page |
858
38. A 72-year-old man is being consented by the Urology SpR for a transurethral resection of the prostate
(TURP). Which of the following risks should he be warned about? Single best answer select one answer
only.
Hypernatraemia YOUR ANSWER
Hypokalaemia
Hypomagnesaemia
Impotence
Retrograde ejaculation CORRECT ANSWER.

Transurethral resection of the prostate (TURP) syndrome due to absorption of large amounts of irrigation
fluid (glycine) produces hyponatraemia. Patients should be warned of the risk of retrograde ejaculation.

39. An 82-year old man falls at home and fractures hip left hip whilst trying to use the bathroom at night.
He is found lying on the floor the following morning by his carers and brought to Accident and
Emergency. He undergoes fixation with dynamic hip screw. The following day he develops low urine
output, peripheral oedema with bibasal crepitations noted on chest auscultation. Which of the following is
correct in acute renal tubular dysfunction? Single best answer - select one answer only.
Following surgery it frequently requires haemofiltration YOUR ANSWER
Following surgery it is usually due to cortical ischaemia
It can be treated with osmotic diuretics CORRECT ANSWER
It is commonly due to renal vein thrombosis
It produces a high urine osmolality (> 350 mOsml/kg).

Acute renal tubular dysfunction is due to ischaemia of the medulla. It can be initially treated by osmotic
diuretics, which may also have a free radical scavenging effect. The majority of cases of acute tubular
necrosis resolve with supportive treatment and careful fluid management, only rarely do they require
haemofiltration or haemodialysis. Acute renal failure and liver failure together constitute the hepatorenal
syndrome.

40. You are reviewing a patient in the neurosurgical clinic who has had a severe traumatic brain injury
some months previously. The GP has noted upper motor neuron signs throughout. What would you
expect to find on examination? Single best answer - select one answer only.
Reduced tone in the affected limbs YOUR ANSWER

---------------------------------------------------------------------------------------------------------------------------------------------------------------------------------------------------------------
Dr Mohammed Shamsul Islam Khan, Medical Officer, Clinical Neuro-Surgery, National Institute of Neuro-Sciences and Hospital
Sher-E-Bangla Nagar, Dhaka-1207, Bangladesh. Mobile: +880 1713 455 662, +880 1685 811979. E-mail: drsikhan@gmail.com
MyPasTest MRCS A Online - Jan Exam 2015
10. Physiology; System Specific Physiology (399Qs)
----------------------------------------------------------------------------------------------------------------------------------
Hyperreflexia CORRECT ANSWER
Marked wasting
Fasciculations
Extensor contractures.
Page |
Upper motor neuron lesions such as cortical injury due to trauma are associated with spasticity (initial 859
resistance to passive movement that decreases as continued pressure is applied), weakness,
hyperreflexia, clonus and up going plantars. There is usually no muscle wasting and flexure contractures
are seen. Lower motor neurone lesions are associated with muscle wasting, fasciculations, weakness and
decreased reflexes.

41. A 45-year-old man with previous peptic ulcer disease was admitted with persistent vomiting. He
looked dehydrated. His blood results were sodium = 140 mmol/l, potassium = 2.5 mmol/l, chloride = 86
mmol/l, pH = 7.5, p(CO2) = 50 mmHg, p(O2) = 107 mmHg, standard bicarbonate = 40 mmol/l. This patient
had: Single best answer question choose ONE true option only.
Metabolic acidosis YOUR ANSWER
Metabolic alkalosis CORRECT ANSWER
Mixed acidosis
Respiratory acidosis
Respiratory alkalosis.

This patient had alkalosis due to a high standard bicarbonate-metabolic alkalosis. This was hypokalemic
hypochloremic metabolic alkalosis because of potassium and chloride loss from vomiting (and
compensatory increase in bicarbonate). Treatment was of the underlying cause (pyloric stenosis) and
intravenous sodium chloride with potassium.

42. An elderly man with a history of prostatism presents with acute retention of urine. His serum
creatinine concentration is 520 mol/l.
Which of the following additional abnormal serum biochemistry test
results is most suggestive of a chronic component to his renal failure? Single best answer - choose ONE
true option only.
Hyperkalaemia YOUR ANSWER
Hyperuricaemia
Hypocalcaemia CORRECT ANSWER
Hyponatraemia
Low serum bicarbonate concentration.

Hyperkalaemia and hyperuricaemia (due to decreased excretion), hyponatraemia (due mainly to continued
water intake with decreased ability to excrete it) and metabolic acidosis occur in both acute and chronic

---------------------------------------------------------------------------------------------------------------------------------------------------------------------------------------------------------------
Dr Mohammed Shamsul Islam Khan, Medical Officer, Clinical Neuro-Surgery, National Institute of Neuro-Sciences and Hospital
Sher-E-Bangla Nagar, Dhaka-1207, Bangladesh. Mobile: +880 1713 455 662, +880 1685 811979. E-mail: drsikhan@gmail.com
MyPasTest MRCS A Online - Jan Exam 2015
10. Physiology; System Specific Physiology (399Qs)
----------------------------------------------------------------------------------------------------------------------------------
renal failure. The presence of hypocalcaemia in renal failure suggests that this is, at least in part, of
longstanding, and is due to decreased renal synthesis of calcitriol (1,25-dihydroxycholecalciferol).

43. A 67-year-old suffers a myocardial infarction. She has a central line placed for monitoring and is
transferred to CCU. Which of the following statements regarding central venous catheterisation is
correct? Single best answer - select one answer only.
Page |
Allows central arterial pressure monitoring YOUR ANSWER
860
Allows pulmonary artery catheterisation via a SchwannGanz catheter CORRECT ANSWER
Allows continuous blood pressure monitoring
Can be complicated by fat embolism
Is often of the external jugular vein.

Central venous cannulation is usually through the internal jugular or the subclavian veins. It allows the
measurement of the central venous pressure. Complications include air embolism (if the patient is not
head down and the cap is left off the cannula), pneumothorax and haemothorax are dangerous
complications. Haematoma or infections are relatively common minor complications. Allows assessment
of the state of hydration of the patient, by monitoring the response to filling.

44. You are consenting a patient for carpal tunnel decompression and are reading the pre-operative
neurophysiology report. Which of the following statements is correct regarding the role of neurons?
Single best answer - Select one answer only.
Have dendrites that transmit stimuli away from the cell body YOUR ANSWER
Have axons that transmit stimuli towards the cell body
Transmit action potentials more rapidly if they are non-myelinated
Are supported by glial cells in the central nervous system CORRECT ANSWER
Transmit impulses less rapidly if they are of a larger diameter.

The anatomy of the neurone consists of a cell body, which receives stimuli via the dendrites. Impulses are
transmitted away from the cell body via axons. The axonal diameter is proportional to the rate of
transmission of the action potential along the axon. The fastest large diameter axons transmit at 120 m/s
in the human. Myelination of axons speeds transmission as the impulse jumps from one gap in the myelin
sheath to the next these gaps are called the nodes of Ranvier. The supportive cells of the neurons are
known collectively as the glial cells in the central nervous system (oligodendrocytes, astrocytes,
ependymal cells and microglia) and the Schwann cells and satellite cells in the peripheral nervous
system.

45. You are reviewing an insulin dependent diabetic patient the night before their surgery. Which of the
following statesments is correct regarding insulin? Single best answer - select one answer only.
Inhibits gluconeogenesis YOUR ANSWER (Correct Answer)
Inhibits protein synthesis
---------------------------------------------------------------------------------------------------------------------------------------------------------------------------------------------------------------
Dr Mohammed Shamsul Islam Khan, Medical Officer, Clinical Neuro-Surgery, National Institute of Neuro-Sciences and Hospital
Sher-E-Bangla Nagar, Dhaka-1207, Bangladesh. Mobile: +880 1713 455 662, +880 1685 811979. E-mail: drsikhan@gmail.com
MyPasTest MRCS A Online - Jan Exam 2015
10. Physiology; System Specific Physiology (399Qs)
----------------------------------------------------------------------------------------------------------------------------------
Inhibits potassium entry into cells
Is synthesised by C cells of the pancreas
Increases glucose uptake by the brain.

Insulin affects many organs and intracellular pathways. Insulin is synthesised by the islet cells of the Page |
pancreas and is an anabolic hormone. It stimulates glucose storage but inhibits glucose production. In
addition, it enhances protein synthesis and inhibits proteolysis. The uptake of glucose by the brain is
861
independent of insulin.

46. The cardiovascular effects of raised intracranial pressure include: Single best answer quest ion
choose ONE true opt ion only.
Decreased blood pressure, decreased heart rate, decreased cerebral perfusion pressure YOUR ANSWER
Decreased blood pressure, increased heart rate, decreased cerebral perfusion pressure
Increased blood pressure, increased heart rate, decreased cerebral perfusion pressure
Increased blood pressure, decreased heart rate, decreased cerebral perfusion pressure CORRECT
ANSWER
Decreased blood pressure, increased heart rate, increased cerebral perfusion pressure.

The important relationship between the cerebral perfusion, mean arterial blood pressure and intracranial
pressure is as follows:
CPP = MABP ICP, where-
1. CPP = cerebral perfusion pressure
2. MABP = mean arterial blood pressure
3. ICP = intracranial pressure.

It stems from the fact that the adult brain is enclosed in a rigid, incompressible box, with the result that
the volume inside it must remain constant (Monroe-Kelly doctrine). A rise in intracranial pressure
therefore decreases cerebral perfusion pressure (and hence cerebral blood flow).

In raised intracranial pressure, as the brainstem becomes compressed, local neuronal activity causes a
rise in sympathetic vasomotor drive and thus a rise in blood pressure. This is known as the Cushings
reflex. This elevated blood pressure evokes a bradycardia via the baroreceptor reflex. The Cushings
reflex helps to maintain cerebral blood flow and protect the vital centres of the brain from loss of nutrition
if the intracranial pressure rises high enough to compress the cerebral arteries.

47. A 35-year-old woman is diagnosed as having diabetes insipidus. In this patient although vasopressin
(ADH) significantly contributes to fluid and electrolyte balance, it does not appear to closely regulate
blood volume in the long run. Blood volume is maintained at near normal levels in this patient because:
Single best answer question choose ONE true option only.
Plasma oncotic pressure increases YOUR ANSWER
Renal blood flow decreases

---------------------------------------------------------------------------------------------------------------------------------------------------------------------------------------------------------------
Dr Mohammed Shamsul Islam Khan, Medical Officer, Clinical Neuro-Surgery, National Institute of Neuro-Sciences and Hospital
Sher-E-Bangla Nagar, Dhaka-1207, Bangladesh. Mobile: +880 1713 455 662, +880 1685 811979. E-mail: drsikhan@gmail.com
MyPasTest MRCS A Online - Jan Exam 2015
10. Physiology; System Specific Physiology (399Qs)
----------------------------------------------------------------------------------------------------------------------------------
Sympathetic reflexes decrease glomerular filtration
The peripheral renin-angiotensin system is stimulated
Water intake is appropriately adjusted CORRECT ANSWER.

In diabetes insipidus, appropriate water intake from thirst will adequately compensate for the potential Page |
excess volume loss. Only if access to appropriate intake is prevented will a large volume loss occur. All
of the other changes listed would tend to maintain blood volume, but all are either short-term effects or 862
the result of extreme stimuli, such as haemorrhage or intense sympathetic activity.

48. A 50-year-old obese man with known unstable angina is admitted to the coronary care unit following a
myocardial infarct. Your consultant places a swan-ganz catheter. He asks what parameters the catheter
can measure. Which of the following can be applied? Single best answer select one answer only.
Jugular venous pressure YOUR ANSWER
Pulmonary capillary wedge pressure CORRECT ANSWER
Venous return
Left ventricular pressure
Right atrial pressure.

A SwanGanz catheter is a balloon-tipped catheter that can be floated through the right side of the heart
into the pulmonary artery. It can be wedged in the pulmonary artery when the balloon is inflated to
measure the pulmonary artery wedge pressure. This gives a good estimation of left atrial pressure. The
catheter tip also comprises a thermistor. Cardiac output can be calculated using the thermodilution
principle. Systemic vascular resistance can also be calculated if blood pressure and cardiac output are
known.

49. Which of the following is correct with regard to the standard base excess? Single best answer - Select
one answer only.
It has a normal range of +6 to 6 YOUR ANSWER
It is likely to be negative in a patient with ischaemic bowel CORRECT ANSWER
It requires albumin concentration for its calculation
It is likely to be negative in a patient with persistent vomiting
It is of little direct clinical use.

The normal range is +2 to 2 and is calculated from the amount of acid or base required to return the
blood sample to a pH of 7.4. It is a useful indicator of the severity of metabolic disturbance and is likely to
be negative with most intra-abdominal conditions arising due to ischaemia or infection. With persistent
vomiting, there is a flow of hydrogen ions from the body, leading to a metabolic alkalosis.
50. A 46-year-old woman has had dry eyes and dry mouth for many years and is diagnosed with Sjogrens
syndrome. Which of the following would be true regarding her saliva? Single best answer select one
answer only.
---------------------------------------------------------------------------------------------------------------------------------------------------------------------------------------------------------------
Dr Mohammed Shamsul Islam Khan, Medical Officer, Clinical Neuro-Surgery, National Institute of Neuro-Sciences and Hospital
Sher-E-Bangla Nagar, Dhaka-1207, Bangladesh. Mobile: +880 1713 455 662, +880 1685 811979. E-mail: drsikhan@gmail.com
MyPasTest MRCS A Online - Jan Exam 2015
10. Physiology; System Specific Physiology (399Qs)
----------------------------------------------------------------------------------------------------------------------------------
Flow rate would increase during nausea YOUR ANSWER (Correct Answer)
Secretion from the parotid gland would be predominantly mucous in nature
Secretion would cease altogether at night
Volume production would be reduced to 1.5 litres a day
Page |
It would contain only a low concentration of potassium.
863
The salivary glands normally produce between 0.75 and 1.5 litres of saliva per day. The parotid gland
produces predominantly serous saliva, the sublingual mainly mucous, and the submandibular a mixture
of the two. There are also many hundreds of tiny glands distributed across the buccal mucosa, secreting
predominantly mucous saliva. Owing to active transport against a concentration gradient, saliva contains
a higher concentration of potassium and a lower concentration of sodium compared to plasma.
Consequently, if a large volume is lost hypokalaemia may result. Although secretion rates are reduced
during sleep, secretion does not stop completely.

Sjgren syndrome is a systemic chronic inflammatory disorder characterized by lymphocytic infiltrates in


exocrine organs. Most common presentation is with sicca symptoms, ie xerophthalmia (dry eyes),
xerostomia (dry mouth) and parotid gland enlargement. The saliva in these patients may have a higher
sodium concentration and volume would be reduced, but would be unlikely to cease altogether, even at
night. Salivary flow increases during nausea and this would not change in someone with Sjogrens,
although the absolute volume, would of course be less.

51. A 14-year-old girl is admitted with anorexia nervosa. In her case, which of the following is the most
likely direct energy source for the brain? Single best answer select one answer only.
Amino acids YOUR ANSWER
Fat
Glucose
Ketone bodies CORRECT ANSWER
Lactic acid.

Glucose is the major direct source of energy for the brain under normal conditions. Glucose enters the
brain via a glucose transporter system. In prolonged starvation, pregnancy and in the newborn, the brain
develops the capacity to use ketone bodies for energy. Ketone bodies are formed from the condensation
of acetyl CoA following the degradation of fatty acids in the liver.

52. Concerning the salivary glands. Single best answer question choose ONE true option only.
They secrete around 150 ml of saliva per day YOUR ANSWER
They secrete saliva with a pH of 4-5
They secrete saliva which is hypertonic
They are supplied by the parasympathetic nervous system CORRECT ANSWER
They secrete saliva containing trypsinogen.
---------------------------------------------------------------------------------------------------------------------------------------------------------------------------------------------------------------
Dr Mohammed Shamsul Islam Khan, Medical Officer, Clinical Neuro-Surgery, National Institute of Neuro-Sciences and Hospital
Sher-E-Bangla Nagar, Dhaka-1207, Bangladesh. Mobile: +880 1713 455 662, +880 1685 811979. E-mail: drsikhan@gmail.com
MyPasTest MRCS A Online - Jan Exam 2015
10. Physiology; System Specific Physiology (399Qs)
----------------------------------------------------------------------------------------------------------------------------------
Saliva is secreted from the acini, and transported via the salivary ducts to the oral cavity. The secretion
from the sublingual gland is predominately mucous, the parotid serous and the submandibular mixed.
The pH of saliva varies from 7-8, and around 1.5L is produced per day.

As well as a-amylase, saliva contains lipase and glycoproteins to lubricate food and protect the oral
mucosa. Lysozyme, IgA and lactoferrin act as bacteriostatic agents, and proteins protect the tooth Page |
enamel. 864
The saliva is isotonic when it is excreted from the acini; Na + and Cl- are exchanged for K+ and HC03- in the
ducts, and the saliva becomes hypotonic by the time it reaches the mouth.

53. Which of the following is correct regarding adrenals and surgical hypertension? Select one answer
only.
Renovascular disease is a rare cause of secondary hypertension YOUR ANSWER
A phaeochromocytoma can be shown by MIBG scintigraphy CORRECT ANSWER
Most adrenal incidentalomas are hormone secreting
A raised midnight cortisol is diagnostic of Cushings syndrome
20% of phaeochromocytomas are extra adrenal.

Renovascular disease is the most common cause of surgical hypertension and can be seen in young
women and in older patients with diffuse atherosclerosis. A renal bruit may be heard. Other causes
include Cushings syndrome, Conn s syndrome and phaeochromocytomas. Phaeochromocytomas can
be diagnosed by urinary catecholamine determination, which has superseded vanillylmandelic acid (VMA)
determination. Incidental adrenal tumours found often on computed tomography (CT) scan are usually
non-functioning. Risk of secretion and malignancy arise if they are greater than 5 cm in diameter.

Normally, cortisol levels are highest at about 07000900 h and decline to <50 nanomoles per litre (nmol/l),
by midnight cortisol is > 50 nmol/l, ie there is loss of diurnal variation. Although measuring a midnight
cortisol > 50 nmol/l is not diagnostic of Cushings syndrome. This is because other factors can cause a
'midnight' cortisol > 50 nmol/l, for example stress due to physical illness, being awake at the time and
depression. Cushings can be diagnosed using the dexamethasone suppression test.

54. A 57-year-old man is diagnosed with a pituitary macroadenoma. Which of the following is LEAST likely
to be raised as a result? Single best answer select one answer only.
Calcitonin YOUR ANSWER (Correct Answer)
Thyroid stimulating hormone (TSH)
Luteinising hormone (LH)
Growth hormone (GH)
Prolactin.
The anterior pituitary gland secretes six hormones: thyroid-stimulating hormone (TSH, thyrotropin),
growth hormone (GH), adrenocorticotropic hormone (ACTH), luteinising hormone (LH), follicle-stimulating
---------------------------------------------------------------------------------------------------------------------------------------------------------------------------------------------------------------
Dr Mohammed Shamsul Islam Khan, Medical Officer, Clinical Neuro-Surgery, National Institute of Neuro-Sciences and Hospital
Sher-E-Bangla Nagar, Dhaka-1207, Bangladesh. Mobile: +880 1713 455 662, +880 1685 811979. E-mail: drsikhan@gmail.com
MyPasTest MRCS A Online - Jan Exam 2015
10. Physiology; System Specific Physiology (399Qs)
----------------------------------------------------------------------------------------------------------------------------------
hormone (FSH) and prolactin. The posterior pituitary secretes oxytocin and vasopressin. Macroadenomas
cause hormonal imbalance by mass effect. The most sensitive cells are the somatotrophs and the
gonadotrophs, whereas corticotrophs and thyrotrophs tend to be more resistant. Calcitonin is produced
by the parafollicular C cells of the thyroid. Calcitonin lowers the circulating calcium and phosphate,
although the exact physiological role of calcitonin is unknown.

55. In the respiratory system, physiological shunt? Single best answer question choose ONE true option
Page |
only. 865
Is greater than the anatomical shunt YOUR ANSWER (Correct Answer)
Is not present in healthy adult
Affects arterial carbon dioxide more than arterial oxygen tension
Has the same effect on respiratory gas exchange as does physiological dead space
Is abolished when the subject breathes pure oxygen.

The physiological shunt is the sum of the anatomical shunt (blood passing from the right ventricle to the
systemic circulation via normal anatomical pathways, e.g. the bronchial vessels, without passing through
the pulmonary alveolar capillaries), and the element of pulmonary alveolar capillary blood that has passed
through non or poorly aerated alveoli. Therefore physiological shunt is always at least as great as or
greater than the anatomical shunt.

There is always a normal anatomical shunt even in the young healthy adult. The difference in carbon
dioxide tension between arterial and mixed venous blood is a little less than 1 kPa, and therefore even a
50% shunt only increases arterial carbon dioxide tension by about 0.5 kPa. A 50% shunt would reduce
arterial oxygen tension from 13.5kPa to below 9 kPa.

The physiological dead space results primarily in a failure to remove carbon dioxide from alveolar gas, i.e.
a rise in arterial carbon dioxide tension if ventilation not increased. The breathing of pure oxygen cannot
eliminate the anatomical right to left portion of the physiological shunt.

56. A parathyroid adenoma will be most likely to cause. Single best answer question choose ONE true
option only.
Decreased osteoclastic activity YOUR ANSWER
Decreased urinary phosphate excretion
Hypocalcaemia
Increased osteoblastic activity
Increased osteoclastic activity CORRECT ANSWER.

The parathyroid glands produce parathyroid hormone (PTH) in response to serum calcium levels via a
negative feedback mechanism. High levels of serum Ca2+ inhibit PTH secretion, and low levels stimulate
PTH secretion. The response to Ca2+ levels is very rapid, so effects are seen very quickly after removal of
---------------------------------------------------------------------------------------------------------------------------------------------------------------------------------------------------------------
Dr Mohammed Shamsul Islam Khan, Medical Officer, Clinical Neuro-Surgery, National Institute of Neuro-Sciences and Hospital
Sher-E-Bangla Nagar, Dhaka-1207, Bangladesh. Mobile: +880 1713 455 662, +880 1685 811979. E-mail: drsikhan@gmail.com
MyPasTest MRCS A Online - Jan Exam 2015
10. Physiology; System Specific Physiology (399Qs)
----------------------------------------------------------------------------------------------------------------------------------
the glands. PTH affects calcium levels by its action on the bone, kidney and gut. In bone, increased
osteoclastic activity causes calcium levels to rise. This is due firstly to acid secretion onto the bone
surface, and secondly to proteases dissolving the matrix.

In the kidney, PTH controls the hydroxylation of 25,hydroxy cholecalciferol D to 1,25 hydroxy
cholecalciferol. This has the indirect effect of increasing calcium uptake in the gut. In the proximal tubule, Page |
PTH increases the urinary excretion of phosphate, which in turn increases the ionisation of calcium. 866
There is also an increase in Ca2+ reabsorption in the distal tubule. Bicarbonate resorption is inhibited in
the kidney, causing a hyperchloraemic acidosis which increase calcium ionisation and resorption from
bone.

PTH excess therefore causes hypercalcaemia, hypophosphataemia and hyperchloraemia, as well as


raised urinary phosphate.

57. In metabolic alkalosis associated with prolonged nasogastric aspiration in postoperative ileus, what is
the most important cause of the acidbase disturbance? Single best answer question choose ONE true
option only.
Hypoventilation YOUR ANSWER
Increased renal bicarbonate reabsorption
Loss of gastric acid CORRECT ANSWER
Potassium depletion
Secondary aldosteronism.

Loss of unbuffered gastric acid is the cause of the metabolic alkalosis seen under these circumstances if
there is inadequate replacement of the fluid lost with intravenous physiological saline. Increased renal
bicarbonate reabsorption (needed to allow adequate renal sodium reabsorption in the presence of
hypochloraemia), potassium depletion (gastric secretions contain about 10 mmol/l of potassium) and
secondary aldosteronism (a result of extracellular fluid loss) all help to maintain the alkalosis, but they do
not cause it. Hypoventilation is a compensatory change: on its own, hypoventilation causes carbon
dioxide retention and a respiratory acidosis.

58. Which is the correct calculation for normal lung volumes? Single best answer quest ion choose ONE
true opt ion only.
Vital capacity = expiratory reserve volume + inspiratory reserve volume YOUR ANSWER
V ital capac ity = exp iratory reserve volume + tidal volume + insp iratory reserve volume CORRECT
ANSWER
Total lung capacity = vital capacity + functional reserve capacity
Total lung capac ity = insp iratory reserve volume + res idual volume
Funct ional res idual capac ity = res idual volume + t idal volume.
---------------------------------------------------------------------------------------------------------------------------------------------------------------------------------------------------------------
Dr Mohammed Shamsul Islam Khan, Medical Officer, Clinical Neuro-Surgery, National Institute of Neuro-Sciences and Hospital
Sher-E-Bangla Nagar, Dhaka-1207, Bangladesh. Mobile: +880 1713 455 662, +880 1685 811979. E-mail: drsikhan@gmail.com
MyPasTest MRCS A Online - Jan Exam 2015
10. Physiology; System Specific Physiology (399Qs)
----------------------------------------------------------------------------------------------------------------------------------
Normal adult lung volumes:
1. Tidal volume (TV) = 7ml/kg (400-500ml).
2. Vital capacity (VC) = 4.5L = expiratory reserve volume + tidal volume + inspiratory reserve volume,
i.e. the combined maximum expiration + the maximum inspiration.
3. Total lung capacity (TLC) = 7.5L = VC + residual volume (RV).
4. Functional residual capacity (FRC) = 2.8L = expiratory reserve volume + RV (ie the amount of air in
the lungs after a normal breath).
Page |
867
59. Your consultant asks you to note the jugular venous pressure in a 59-year-old diabetic with known
heart failure. This is noted to be high. Which of the following would not be a possible cause? Select one
answer only.
Fluid overload YOUR ANSWER
Pericardial effusion
Hypovolaemia CORRECT ANSWER
Pulmonary hypertension
Superior vena cava (SVC) obstruction.

The JVP is a measure of pressure in the right atrium and usually reflects intravascular volume reasonably
well. However, there are a few instances where a falsely elevated JVP may occur, as in conditions that
cause or lead to, right-sided cardiac failure or pulmonary hypertension.

60. Which of the following statements is correct regarding pulse oximetry? Select one answer only.
Gives a direct reflection of the p(O2) YOUR ANSWER
Shows a linear relationship with oxygen carriage
Is inaccurate in the presence of high levels of carbon monoxide CORRECT ANSWER
Indicates adequate tissue oxygen delivery if readings are over 95%
Is the best means of indicating inadequate ventilation.

Oxygen saturation reflects the oxygen carried in the blood. The relationship between pulse oximetry and
oxygen carriage is a sigmoid curve. It does not reflect whether delivery to the tissues is adequate as this
is dependent also on cardiac output and haemoglobin levels. The adequacy of ventilation is better
reflected by the p(CO2). Carbon monoxide leads to the presence of carboxyhaemoglobin which decreases
the oxygen available to the tissues.

61. A 35-year-old woman was admitted with a large postpneumonic effusion. Nearly 2 litres of purulent
fluid was drained from the chest following insertion of an intercostals drain. What is the normal amount of
pleural fluid? Single best answer question choose ONE true option only.
10 ml YOUR ANSWER (Correct Answer)
100 ml
250 ml

---------------------------------------------------------------------------------------------------------------------------------------------------------------------------------------------------------------
Dr Mohammed Shamsul Islam Khan, Medical Officer, Clinical Neuro-Surgery, National Institute of Neuro-Sciences and Hospital
Sher-E-Bangla Nagar, Dhaka-1207, Bangladesh. Mobile: +880 1713 455 662, +880 1685 811979. E-mail: drsikhan@gmail.com
MyPasTest MRCS A Online - Jan Exam 2015
10. Physiology; System Specific Physiology (399Qs)
----------------------------------------------------------------------------------------------------------------------------------
500 ml
1000 ml.

The total volume of fluid present in the intrapleural space is estimated to be only 2-10 ml. A small amount
of protein is present in intrapleural fluid. At end expiration, mean intrapleural pressure (ppl) is about 5
cmH2O below atmospheric pressure (-5 cmH2O) and ppl becomes more subatmospheric with inspiration
Page |
such that, at end inspiration of a typical tidal volume, ppl may be -10 cmH2O. 868
The magnitude of the subatmospheric or 'negative' intrapleural pressure reflects the tendency of the lung
to recoil or pull away from the inner chest cage. Because the intrapleural fluid is a liquid that cannot be
expanded or contracted, it results in a 'negative' intrapleural pressure that becomes more 'negative' with
lung inflation.

62. A 21-year old male is admitted following an RTA in which he sustains an open tibial fracture with
heavy blood loss. He develops low blood pressure, and is catheterised and resuscitated for low urine
output. Which of the following is correct in acute renal failure? Single best answer - select one answer
only.
Acute cortical necrosis heralds a good prognosis YOUR ANSWER
Dialysis should be commenced early
Fluid resuscitation is rarely required
Pain control with non-steroidal anti-inflammatory drugs (NSAIDs) should continue
Systemic infection characteristically complicates the illness CORRECT ANSWER.

Fluid resuscitation is essential to ensure that the patient is adequately filled, with a blood pressure
sufficient for renal perfusion (prerenal failure). All nephrotoxic drugs should be stopped. The initial
management is supportive; dialysis is used for chronic renal failure. In the emergency setting,
haemofiltration is used when there is fluid overload, hyperkalaemia (> 6.0 mmol/l) or metabolic acidosis
(pH < 7.2 and deteriorating base excess) resistant to treatment.

63. A 56-year-old woman sustains a myocardial infarction. ST elevation and Q waves are present in leads
V4V6, I and AVL. Which of the following aspects of the heart is most likely to have been involved in the
infarct? Single best answer question choose ONE true option only.
Anterior YOUR ANSWER
Anterolateral CORRECT ANSWER
Anteroseptal
Inferior
Lateral.

This combination suggests an anterolateral infarct. Purely anterior infarcts tend to involve the chest leads
only (typically V2V5),anteroseptal V1V3, lateral infarcts chest leads only (I, II, AVL) and inferior infarcts II,
III and AVF.
---------------------------------------------------------------------------------------------------------------------------------------------------------------------------------------------------------------
Dr Mohammed Shamsul Islam Khan, Medical Officer, Clinical Neuro-Surgery, National Institute of Neuro-Sciences and Hospital
Sher-E-Bangla Nagar, Dhaka-1207, Bangladesh. Mobile: +880 1713 455 662, +880 1685 811979. E-mail: drsikhan@gmail.com
MyPasTest MRCS A Online - Jan Exam 2015
10. Physiology; System Specific Physiology (399Qs)
----------------------------------------------------------------------------------------------------------------------------------
64. You are reviewing a patient who is peripherally cyanosed. What would increase the release of oxygen
from blood to the peripheral tissues? Select one answer only.
Decreased blood volume YOUR ANSWER
Decreased temperature
Increased PCO2 CORRECT ANSWER Page |
Increased pH 869
Increased osmotic pressure.

The mechanisms of oxygen release from blood to the peripheral tissues are determined by factors
shifting the oxygenhaemoglobin dissociation curve. Hence, a low pH state, increase in temperature,
increase in PCO2 and an increase in 2,3-DPG all facilitate the delivery of oxygen to the tissues, by raising
the PCO2 at which is released in the peripheral capillaries (shift of the curve to the right).

65. The cardiac index is defined as which of the following? Select one answer only.
Cardiac output/body surface area YOUR ANSWER (Correct Answer)
Stroke volume x heart rate
Mean arterial pressure x systemic vascular resistance
Cardiac output/body weight
Cardiac output/heart rate.

Cardiac index = cardiac output/body surface area. (It allows comparison of cardiac function corrected for
size of the individual.) Cardiac output = stroke volume x heart rate. Systemic vascular resistance = [(mean
arterial pressure CVP)/cardiac output] x 80.

66. You are participating in your hospital's annual medics Vs surgeons 5 a side football game. Which of
the following is associated with the increased heart rate? Select one answer only.
Hypothyroidism YOUR ANSWER
Intravenous adenosine
Verapamil
Salbutamol CORRECT ANSWER
Metronidazole.

At rest, there is tonic vagal tone on the heart. This is lost after cardiac-denervation and, as a result, the
heart rate increases. Adenosine induces heart block, and if given in sufficient doses can cause transient
asystole. Verapamil is a calcium channel blocker that slows cardiac electrical conduction and heart rate.
Salbutamol has -adrenoceptor activity resulting in tachycardia. Metronidazole has no effect on the heart
rate.

67. Vasopressin (ADH). Single best answer question choose ONE true option only.
---------------------------------------------------------------------------------------------------------------------------------------------------------------------------------------------------------------
Dr Mohammed Shamsul Islam Khan, Medical Officer, Clinical Neuro-Surgery, National Institute of Neuro-Sciences and Hospital
Sher-E-Bangla Nagar, Dhaka-1207, Bangladesh. Mobile: +880 1713 455 662, +880 1685 811979. E-mail: drsikhan@gmail.com
MyPasTest MRCS A Online - Jan Exam 2015
10. Physiology; System Specific Physiology (399Qs)
----------------------------------------------------------------------------------------------------------------------------------
Is synthesised in the posterior pituitary gland YOUR ANSWER
Deficiency leads to a risk of water intoxication
Excessive secretion usually results in diabetes insipidus
Increased plasma osmolarity is the primary physiological stimulus CORRECT ANSWER
Page |
Acts on the proximal convoluted tubules of the kidney.
870
Vasopressin is synthesised in the supraoptic nucleus of the hypothalamus and transported to the
posterior pituitary via the axons. Excessive secretion is associated with the risk of impaired water
excretion. Diabetes insipidus results from deficient secretion or action of this hormone leading to thirst
and polyuria. It acts mainly on the distal convoluted tubules and the collecting ducts of the kidney.

68. You are assessing a patient who has suffered a blunt head injury approximately 2 hours ago. You
suspect raised intracranial pressure. Which of the following signs would be consistent with this
diagnosis?
Bloody tap following lumbar puncture YOUR ANSWER
Decreased blood pressure
Increased pulse rate
Irregular breathing CORRECT ANSWER
Neck stiffness.

Rising intracranial pressure presents with headache, drowsiness, vomiting and seizures and there is
often a history of trauma. Signs include listlessness, irritability, drowsiness, falling pulse, rising blood
pressure, coma, irregular breathing (respiratory depression from compression of the medulla) and, later,
papilloedema. Lumbar puncture is contraindicated in raised intracranial pressure.

69. A 65-year old man is diagnosed with a parathyroid adenoma. He attends the outpatient clinic for
advice on management options. What is the best option for treatment? Single best answer - select one
answer only.
Only surgery is curative YOUR ANSWER (Correct Answer)
Bisphosphonates and calcitonin result in good long-term control
Octreotide results in good control of parathyroid hormone (PTH) levels
Surgery is reserved for those not improving on bisphosphonate treatment
Reducing dietary calcium intake can be curative.

Parathyroidectomy is the only curative treatment for primary hyperparathyroidism caused by a


parathyroid adenoma. Bisphosphonates, calcitonin and octreotide do not have a role in treatment. Low
calcium intake stimulates the parathyroids and high intake accentuates hypercalcaemia, so the
recommendation is moderate intake.

---------------------------------------------------------------------------------------------------------------------------------------------------------------------------------------------------------------
Dr Mohammed Shamsul Islam Khan, Medical Officer, Clinical Neuro-Surgery, National Institute of Neuro-Sciences and Hospital
Sher-E-Bangla Nagar, Dhaka-1207, Bangladesh. Mobile: +880 1713 455 662, +880 1685 811979. E-mail: drsikhan@gmail.com
MyPasTest MRCS A Online - Jan Exam 2015
10. Physiology; System Specific Physiology (399Qs)
----------------------------------------------------------------------------------------------------------------------------------
70. A 71-year-old man with known chronic obstructive pulmonary disease is admitted to A&E with severe
shortness of breath. Blood gas analysis shows: arterial [H+] 55 nmol/l (pH 7.26), p(CO2) 9.4 kPa, p(O2) 9.1
kPa, derived [HCO3] 31 mmol/l. Which of the following types of acidbase disturbance is most likely?
Single best answer question choose ONE true option only.
Acute respiratory acidosis YOUR ANSWER Page |
Chronic, compensated respiratory acidosis
871
Exacerbation of chronic respiratory acidosis CORRECT ANSWER
Mixed respiratory acidosis and metabolic alkalosis
Severe metabolic acidosis.

The patient is acidotic but the elevated bicarbonate concentration is incompatible with a metabolic
acidosis. The high p(CO2) indicates that there is a respiratory acidosis. Were this to be acute, the
bicarbonate concentration would be normal or only marginally elevated and a higher hydrogen-ion
concentration would be expected (hydrogen-ion concentration increases by approximately 5.5 nmol/l for
each increase in p(CO2) by 1 kPa in an acute disturbance.) Were compensation to be complete, it would
be expected that the patient would be less acidotic, and the bicarbonate concentration higher.
The data are compatible with both an acute exacerbation of a chronic respiratory acidosis and a mixed
respiratory acidosis and metabolic alkalosis, but the clinical setting makes the former the more likely.

71. A 43-year-old lady has a history of medullary thyroid carcinoma and parathyroid hyperplasia. Which
other condition would be associated with Multiple Endocrine Neoplasia type 2A (MEN2A)? Single best
answer select one answer only.
Chromophobe pituitary adenoma YOUR ANSWER
Gastrinoma
Marfanoid habitus
Neurofibromatosis
Phaeochromocytoma CORRECT ANSWER.

Two syndromes of multiple endocrine neoplasia (MEN) are described. Both are fairly common.

MEN1 is inherited in an autosomal dominant manner and is due to mutation of the menin gene. It includes
the following features:
01. Parathyroid hyperplasia and adenomas
02. Pituitary adenomas

03. Pancreatic islet cell tumours, eg gastrin secreting (ZollingerEllison syndrome) and insulinoma

04. Adrenal cortical adenoma (usually nonfunctioning)/nodular hyperplasia


05. Thymic carcinoid tumours
---------------------------------------------------------------------------------------------------------------------------------------------------------------------------------------------------------------
Dr Mohammed Shamsul Islam Khan, Medical Officer, Clinical Neuro-Surgery, National Institute of Neuro-Sciences and Hospital
Sher-E-Bangla Nagar, Dhaka-1207, Bangladesh. Mobile: +880 1713 455 662, +880 1685 811979. E-mail: drsikhan@gmail.com
MyPasTest MRCS A Online - Jan Exam 2015
10. Physiology; System Specific Physiology (399Qs)
----------------------------------------------------------------------------------------------------------------------------------
06. Thyroid adenomas
07. Lipomas

MEN2 is also autosomal-dominant. It is associated with mutations in the RET proto-oncogene. It is


subdivided into three clinical syndromes: Page |
MEN2A medullary thyroid cancer (MTC), phaeochromocytoma, primary parathyroid hyperplasia 872
MTC, phaeochromocytoma, mucosal neuromas (lips and tongue), intestinal
MEN2B
ganglioneuromas, Marfanoid habitus. Not associated with parathyroid hyperplasia.
Familial
variant of 2A with MTC only and no other manifestations.
MTC

MTC usually presents as a thyroid mass or local lymphadenopathy. It may present with diarrhoea (due to
calcitonin), but occasionally with other paraneoplastic manifestations such as ACTH-dependent
Cushing's syndrome.

72. You are reviewing a female patient who is in the intensive care unit, 1 day post-op, following an
emergency thoracotomy for a stab wound to the left side of the chest. The patient has a raised cardiac
output. Which of the following can cause an increase in cardiac output? Single best answer - select one
answer only.
Late pregnancy YOUR ANSWER (Correct Answer)
Long-term habitation at altitude
Rapid dysrhythmias
Standing from a lying position
Starvation.

Cardiac output = stroke volume x heart rate.


Cardiac output is decreased on standing from a lying position and above a certain heart rate stroke
volume falls reducing cardiac output. Eating increases cardiac output and starvation decreases it. The
cardiac output increases during pregnancy and reaches its maximal for the pregnancy by approximately
20 weeks gestation. Cardiac output is increased only short term when arriving at altitude.

73. You are reviewing Pulmonary function tests for one of your pre-op patients who is known to have
airways disease. Which pulmonary function test results would assist you in confirming whether this is
restrictive or obstructive airways disease? Select one answer only.
Forced expiratory volume : forced vital capacity (FEV1 : FVC) ratio of <0.75 typically occurs in obstructive
disease YOUR ANSWER (Correct Answer)
Forced expiratory volume : forced vital capacity (FEV1 : FVC) ratio of <0.75 typically occurs in restrictive
disease

---------------------------------------------------------------------------------------------------------------------------------------------------------------------------------------------------------------
Dr Mohammed Shamsul Islam Khan, Medical Officer, Clinical Neuro-Surgery, National Institute of Neuro-Sciences and Hospital
Sher-E-Bangla Nagar, Dhaka-1207, Bangladesh. Mobile: +880 1713 455 662, +880 1685 811979. E-mail: drsikhan@gmail.com
MyPasTest MRCS A Online - Jan Exam 2015
10. Physiology; System Specific Physiology (399Qs)
----------------------------------------------------------------------------------------------------------------------------------
Obstructive disease typically results in an increased FEV1
Obstructive disease typically results in an increased FVC
Restrictive disease typically results in an increased FEV1.

Pulmonary function tests are normally performed in a special lung function laboratory. They are not Page |
usually performed at the bedside. PEFR can be used as a simple bedside test for obstruction to
expiration. This approximates to the FEV1, the maximum amount of air a patient can exhale in the first 873
second of expiration. FVC refers to the volume of air a patient can exhale after maximal inspiration. The
ratio FEV1 : FVC describes the proportion of the maximum amount of air a patient can hold in their lungs,
which they can expel in the first second of respiration.

In restrictive airways disease the FEV1 and FVC are both usually decreased, giving a normal FEV1:FVC
ratio. In obstructive airways disease the FEV1 is usually decreased with a normal FVC, giving a decreased
FEV1:FVC ratio.

74. A 25-year-old female presents with painful urination. Which of the following symptoms would you also
expect to find in an irritative lower urinary tract infection? Single best answer - select one answer only.
Hesitancy YOUR ANSWER
Incomplete voiding
Nocturia CORRECT ANSWER
Overflow incontinence
Poor flow.

Lower urinary tract symptoms can be divided into filling (irritative) and voiding (obstructive) symptoms.
Filling symptoms consist of frequency, nocturia, urgency (with or without urge incontinence) and
suprapubic pain. Voiding symptoms consist of hesitancy, poor flow, a feeling of incomplete emptying and
postmicturition dribble.

75. Biochemical mechanisms of signal transduction follow the pharmacological subdivisions of the
adrenergic receptors. Stimulation of either
1- or
2-receptors: Single best answer question choose ONE
true option only.
Activates adenylate cyclase YOUR ANSWER (Correct Answer)
Activates guanylate cyclase
Activates gene transcription
Inhibits adenylate cyclase
Inhibits guanylate cyclase.

Biochemical mechanisms of signal transduction follow the pharmacological subdivisions of the


adrenergic receptors. Stimulation of either 1- or
2-receptors activates adenylate cyclase. -Adrenergic
responses typically result from increased production of cAMP. The only salient difference between
1- and

---------------------------------------------------------------------------------------------------------------------------------------------------------------------------------------------------------------
Dr Mohammed Shamsul Islam Khan, Medical Officer, Clinical Neuro-Surgery, National Institute of Neuro-Sciences and Hospital
Sher-E-Bangla Nagar, Dhaka-1207, Bangladesh. Mobile: +880 1713 455 662, +880 1685 811979. E-mail: drsikhan@gmail.com
MyPasTest MRCS A Online - Jan Exam 2015
10. Physiology; System Specific Physiology (399Qs)
----------------------------------------------------------------------------------------------------------------------------------
2-receptors is the low sensitivity of the 2-receptors to noradrenaline. Stimulation of the
2-receptors
inhibits adenylate cyclase and may block the increase in cAMP produced by other agents. For beta
effects, the recognition subunit of the adrenergic receptor communicates with adenylate cyclase through
the stimulatory guanosine binding subunit of the receptor complex (G protein).

76. A 17-year-old male with cystic fibrosis receives supplements of pancreatin to help with the digestion
of starch, fat and protein. Secretion of pancreatic juice is primarily under the control of which two
Page |
hormones? Single best answer - select one answer only. 874
Cholecystokinin and glucagon YOUR ANSWER
Cholecystokinin and insulin
Cholecystokinin and secretin CORRECT ANSWER
Glucagon and insulin
Glucagon and secretin.

Supplements of pancreatin are given by mouth to compensate for reduced or absent exocrine secretion in
cystic fibrosis, chronic pancreatitis, and following a pancreatectomy and gastrectomy. The pancreas is an
essential endocrine organ that produces insulin, glucagon and other hormones. Pancreatic secretion is
alkaline, which maintains the correct pH levels for enzyme action and function. The exocrine function of
the pancreas is to produce enzymes for normal digestion.

Inactive precursors of these enzymes are secreted in pancreatic juice and cleaved to form trypsin and
chymotrypsin, as well as carboxypeptidases, lipase, co-lipase and amylase. Secretion of pancreatic juice
is primarily under the control of two hormones secretin and cholecystokinin (CCK) secreted by the
duodenum. The rate of pancreatic secretion is typically 150 ml/h and therefore approximately 3500 ml/day.

77. Pulmonary gas exchange occurs under which of the following physiological principles? Single best
answer question choose ONE true option only.
Gas exchange can occur in the final seven branches of the bronchoalveolar tree YOUR ANSWER (Correct
Answer)
The first 12 branches of the bronchial tree are collectively known as the conducting zone
The equilibration of gases takes about 2.5 s in the resting lung
Only about 0.15% of oxygen is carried in solution in the plasma
Carbon dioxide is less water-soluble than oxygen.

Gas exchange can occur in the final seven branches of the bronchoalveolar tree (the respiratory zone).
The first 16 branches of the bronchial tree are collectively known as the conducting zone. The
equilibration of gases takes about 0.25 s in the resting lung. Only about 1.5% of oxygen is carried in
solution in the plasma.

Carbon dioxide is more water-soluble than oxygen, between 5 and 10% of and this is the predominant
method of carriage of CO2 is carried in dissolved form.
---------------------------------------------------------------------------------------------------------------------------------------------------------------------------------------------------------------
Dr Mohammed Shamsul Islam Khan, Medical Officer, Clinical Neuro-Surgery, National Institute of Neuro-Sciences and Hospital
Sher-E-Bangla Nagar, Dhaka-1207, Bangladesh. Mobile: +880 1713 455 662, +880 1685 811979. E-mail: drsikhan@gmail.com
MyPasTest MRCS A Online - Jan Exam 2015
10. Physiology; System Specific Physiology (399Qs)
----------------------------------------------------------------------------------------------------------------------------------
78. Which of the following would be expected if an antibody that neutralises the activity of human
chorionic gonadotrophin (hCG) is administered for 7 days starting 4 weeks after conception? Single best
answer question choose ONE true option only.
Death of the embryo and its subsequent expulsion YOUR ANSWER (Correct Answer)
Decreased secretion of pituitary LH
Page |
Increased secretion of estradiol
875
Increased secretion of progesterone
Increased secretion of prolactin.

hCG is a glycoprotein hormone, produced in pregnancy, which is made by the embryo soon after
conception and later by the syncytiotrophoblast (part of the placenta). Its role is to prevent the
disintegration of the corpus luteum of the ovary and thereby maintain progesterone production that is
critical for a pregnancy in humans.

hCG may have additional functions, for instance it is thought that it affects the immune tolerance of the
pregnancy. Early pregnancy testing generally is based on the detection or measurement of hCG.
Neutralisation of hCG activity will result in disintegration of the corpus luteum, with subsequent death
and expulsion of embryo.

79. You take arterial blood gases from a patient on the ward who is short of breath, you suspect chronic
rather than acute respiratory failure. What findings on the blood gases would assist in confirming that the
respiratory failure is chronic rather than acute? Select one answer only.
Arterial pH of 7.0 YOUR ANSWER
Arterial pH of 7.2
PaO2 of 7 kPa
PaO2 of 9 kPa
Plasma bicarbonate of 39 mmol/l CORRECT ANSWER.

In acute respiratory failure the PaO2 falls and the PaCO2 rises with a consequent drop in pH. In chronic
respiratory failure, the serum bicarbonate concentration increases to correct the acidaemia. The rise of
bicarbonate concentration in the CSF renders the respiratory centre in the brainstem insensitive to
hypercarbia but not to hypoxia. So, a hypoxic respiratory drive is the predominant mechanism and is the
only stimulus to increase the ventilation rate.

80. Which of the following hormones prevents menstrual cycle during the early post-partum period?
Single best answer question choose ONE true option only.
FSH YOUR ANSWER
Oestrogen
Oxytocin
Progesterone
---------------------------------------------------------------------------------------------------------------------------------------------------------------------------------------------------------------
Dr Mohammed Shamsul Islam Khan, Medical Officer, Clinical Neuro-Surgery, National Institute of Neuro-Sciences and Hospital
Sher-E-Bangla Nagar, Dhaka-1207, Bangladesh. Mobile: +880 1713 455 662, +880 1685 811979. E-mail: drsikhan@gmail.com
MyPasTest MRCS A Online - Jan Exam 2015
10. Physiology; System Specific Physiology (399Qs)
----------------------------------------------------------------------------------------------------------------------------------
Prolactin CORRECT ANSWER.

The high levels of prolactin that occur during lactation suppress release of gonadotrophins, which in turn
prevents a normal menstrual cycle. Because of this, lactation has been exploited as a means of birth
control. However, after about 40 days post partum, prolactin sensitivity to suckling decreases markedly
and so loses its value as a contraceptive method. Page |
81. With respect to vomiting which of the following statements is the best answer? Single best answer 876
question choose ONE true option only.
The main receptor in the chemoreceptor trigger zone (CTZ) is adrenergic YOUR ANSWER
The CTZ is outside the blood brain barrier CORRECT ANSWER
5HT3 agonists may be effective in controlling cisplatin induced vomiting
H2 receptors are abundant in the vomiting centre
The vomiting centre is present in the reticular formation of the mid brain.

The vomiting centre is present in the reticular formation of the medulla, the CTZ is outside the blood brain
barrier and the main receptors are dopaminergic D2 receptors. 5HT3 antagonist is effective in controlling
vomiting. H1 receptors have been identified in the vomiting centre.

82. Bile is an important compound in the absorption of vitamins and secretion of waste products. Which
of the following controls the secretion of bile? Single best answer - select one answer only.
ADH YOUR ANSWER
Cholecystokinin CORRECT ANSWER
Parasympathetic nervous system
Renin
Somatostatin.

Biliary secretion is controlled by:


bile salts delivered to the liver by the portal circulation hormonal control exerted largely by secretin as
well as CCK and gastrin sympatheticnervous control, which plays a minor role the postganglionic
parasympathetic fibres produces a small increase in secretion, whereas sympathetic activity leads to a
decrease in secretion.

83. Which of the following takes place during inspiration? Single best answer question choose ONE true
option only.
The diaphragm drops by 10 cm during normal breathing YOUR ANSWER
A negative pressure of 13 mmHg is created CORRECT ANSWER
The rib cage recoils
Abdominal muscles contract

---------------------------------------------------------------------------------------------------------------------------------------------------------------------------------------------------------------
Dr Mohammed Shamsul Islam Khan, Medical Officer, Clinical Neuro-Surgery, National Institute of Neuro-Sciences and Hospital
Sher-E-Bangla Nagar, Dhaka-1207, Bangladesh. Mobile: +880 1713 455 662, +880 1685 811979. E-mail: drsikhan@gmail.com
MyPasTest MRCS A Online - Jan Exam 2015
10. Physiology; System Specific Physiology (399Qs)
----------------------------------------------------------------------------------------------------------------------------------
Accessory muscles relax.

During normal breathing the diaphragm may drop 1 cm, creating a pressure drop of 13 mmHg and an air
intake of 500 ml. During strenuous exercise the diaphragm may drop 10 cm, producing a pressure drop of
100 mmHg and an air intake of 3000 ml. During passive expiration the rib cage recoils, the abdominal
muscles contract and the accessory muscles relax. Page |
877
84. You have performed a tracheostomy and are writing the post-op notes. When would you recommend
the earliest time, the first tube change takes place? Select one answer only.
Day 1 post-op YOUR ANSWER
Day 3 post-op CORRECT ANSWER
Day 5 post-op
Six hours post-op
Twelve hours post-op.

Tube changing should be avoided until 3 days post-operation until a track is well established and the tube
changed easily.

85. You have taken a patient's arterial blood gases, and they have a raised PCO2. Regarding carbon
dioxide in arterial blood, which of the following statements is correct? Single best answer - select one
answer only.
It diffuses with difficulty into red blood cells YOUR ANSWER
It enters red cells by active transport
It is carried predominantly as bicarbonate (HCO3-) CORRECT ANSWER
It is less soluble than oxygen
It is mainly dissolved in plasma.

In arterial blood, 89.4% of the CO2 is in the form of HCO3 . Only 5.3% is in carbamino compounds and
5.3% is dissolved in plasma. CO2 is about 20 times more soluble than O2 in simple solutions at equal
partial pressure. CO2 readily diffuses into red blood cells and is rapidly hydrated to H2CO3 in the presence
of carbonic anhydrase. The H2CO3 dissociates into H+ and HCO3 , and the H+ is buffered mainly by
haemoglobin, while the HCO3 enters the plasma.

86. You are seeing a patient in neurosurgical clinic with a posterior cerebral artery infarction. What
symptoms would you expect to find? Single best answer - select one answer only.
Nystagmus YOUR ANSWER
Dysdiadochokinesis
Dysphasia
Loss of light reflex
---------------------------------------------------------------------------------------------------------------------------------------------------------------------------------------------------------------
Dr Mohammed Shamsul Islam Khan, Medical Officer, Clinical Neuro-Surgery, National Institute of Neuro-Sciences and Hospital
Sher-E-Bangla Nagar, Dhaka-1207, Bangladesh. Mobile: +880 1713 455 662, +880 1685 811979. E-mail: drsikhan@gmail.com
MyPasTest MRCS A Online - Jan Exam 2015
10. Physiology; System Specific Physiology (399Qs)
----------------------------------------------------------------------------------------------------------------------------------
Homonymous hemianopia CORRECT ANSWER.

The posterior cerebral artery supplies the cerebral peduncle and the optic tract. It supplies the
inferomedial surface of the temporal and occipital lobes. Homonymous hemianopia is caused by a lesion
beyond the optic chiasm opposite to the field defect, e.g. due to stroke or a tumour. Nystagmus and
dysdiadochokinesis are cerebellar signs. Page |
878
87. A 26-year-old man is seen in the Accident and Emergency Department with a lower motor neurone
lesion of long standing. This patient will have: Single best answer question choose ONE true option
only.
Hyperaesthesia YOUR ANSWER
Hyper-reflexia
Muscle wasting CORRECT ANSWER
Positive Babinski's sign
Spasticity.

Lower motor neurones (LMNs) are the motoneurons connecting the brainstem and spinal cord to muscle
fibres, bringing the nerve impulses from the upper motor neurones out to the muscles. LMN are classified
based on the type of muscle fibres they innervate. Alpha-motor neurones (-MNs) innervate extrafusal
muscle fibres, the most numerous type of muscle fibre and the one most involved in contraction of a
muscle. Gamma-motor neurones (-MNs) innervate intrafusal muscle fibres, which are involved with
muscle spindles and the sense of body position. Damage to lower motor neurones is indicated by
abnormal electromyographic potentials, fasciculations, paralysis, weakening muscles and neurogenic
atrophy (wasting) of skeletal muscle(s).

88. You have a patient who is undergoing a hemihepatectomy and needs central venous pressure
monitoring during anaesthesia and post-operatively whilst in the intensive care unit. The anaesthetist is
teaching you how to insert a central venous pressure line. Regarding insertion of the line, where should
the tip lie? Single best answer - select one answer only.
The tip should lie in the aorta YOUR ANSWER
The tip should lie in the left atrium
The tip should lie in the right atrium
The tip should lie in the right ventricle
The tip should lie in the superior vena cava CORRECT ANSWER.

The tip of a central venous pressure line should lie low in the superior vena cava (SVC). The reading
reflects right ventricular end-diastolic pressure and the left can only be assumed. The reading is usually
measured in cm H2O, and is raised during positive pressure ventilation.

A subclavian approach is usually carried out on the right to avoid the thoracic duct although it may also
be carried out on the left.
---------------------------------------------------------------------------------------------------------------------------------------------------------------------------------------------------------------
Dr Mohammed Shamsul Islam Khan, Medical Officer, Clinical Neuro-Surgery, National Institute of Neuro-Sciences and Hospital
Sher-E-Bangla Nagar, Dhaka-1207, Bangladesh. Mobile: +880 1713 455 662, +880 1685 811979. E-mail: drsikhan@gmail.com
MyPasTest MRCS A Online - Jan Exam 2015
10. Physiology; System Specific Physiology (399Qs)
----------------------------------------------------------------------------------------------------------------------------------
89. A 56-year-old man suffered a myocardial infarction following which the velocity of impulse conduction
through the atrioventricular node decreased. This patient will have: Single best answer question choose
ONE true option only.
Atrial fibrillation YOUR ANSWER
Decreased PR interval
Page |
Disappearance of the T-wave
879
Increased heart rate
Increased PR interval CORRECT ANSWER.

Damage to the atrioventricular node resulting from ischaemia or inflammation may increase the PR
interval from a normal value of about 0.12-0.20 s to as much as 0.25-0.40 s. If conduction through the
arteriovenous node becomes severely impaired, complete block of impulses may occur, in which case the
atria and ventricles will beat independently.

90. Consider the following data;


paO2 7.6 KPa
paCO2 3.2 KPa
pH 7.45
HCO3- 24 mmol/l.
This blood gas analysis is most consistent with: Single best answer question choose ONE true option
only.
Compensated metabolic acidosis YOUR ANSWER
Normal blood gas analysis
Metabolic acidosis
Respiratory acidosis
Type 1 respiratory failure CORRECT ANSWER.

The blood gases show a picture of type 1 respiratory failure, with a low pa O 2 and low pa CO2, possibly
due to consolidation of lung with compensatory hyperventilation. This pattern may be seen in conditions
producing a ventilation/perfusion mismatch, such as pneumonia or pulmonary embolism. If ventilation is
able to increase enough to keep CO2 normal, then a respiratory acidosis will not develop.

A metabolic acidosis would have a pH of less than 7.35 and low bicarbonate. A compensated metabolic
acidosis would show decreased bicarbonate as well as a low CO2 due to respiratory compensation.

91. Deficiency of maltase in the brush border of small intestine: Single best answer question choose
ONE true option only.
Does not prevent hydrolysis of maltose because it can be hydrolysed by pancreatic amylase YOUR
ANSWER

---------------------------------------------------------------------------------------------------------------------------------------------------------------------------------------------------------------
Dr Mohammed Shamsul Islam Khan, Medical Officer, Clinical Neuro-Surgery, National Institute of Neuro-Sciences and Hospital
Sher-E-Bangla Nagar, Dhaka-1207, Bangladesh. Mobile: +880 1713 455 662, +880 1685 811979. E-mail: drsikhan@gmail.com
MyPasTest MRCS A Online - Jan Exam 2015
10. Physiology; System Specific Physiology (399Qs)
----------------------------------------------------------------------------------------------------------------------------------
Decreases rate of absorption of ingested glucose
Results in lack of absorption of lactose
Results in decreased frequency of stool
Results in increased passage of maltose in stool CORRECT ANSWER.
Page |
Maltase is one enzyme produced by the cells lining the small intestine to break down disaccharides. It 880
comes under the enzyme category carbohydrase (which is a subcategory of hydrolase) and the
disaccharide it hydrolyses is maltose. Maltase is secreted by the surface cells of the villi, which are thin
projections on the mucosa. These are found throughout the small intestine, but differ in shape in the
duodenal and ileal sections.

Maltase works like any other enzyme, with the substrate (maltose) binding with the active site. When the
maltose binds with maltase, the former is hydrolysed, that is to say it is split into its component parts, ie
two molecules of -D-glucose. This is done by breaking the glycosidic bond between the 'first' carbon of
one glucose and the 'fourth' carbon of the other (a 1-4 bond). Maltase deficiency will result in increased
passage of maltose in the stool.

92. Lung compliance: Single best answer question choose ONE true option only.
Is defined as the change in pressure per unit volume YOUR ANSWER
Is synonymous with elastance
Is increased in emphysema CORRECT ANSWER
Is equal in inflation and deflation
Is reduced by the presence of surfactant.

Compliance is expressed as volume change per unit change in pressure. Elastance is the reciprocal of
compliance. The pressure-volume curve of the lung is non-linear with the lungs becoming stiffer at high
volumes. The curves which the lung follows in inflation and deflation are different. This behaviour is
known as hysteresis. The lung volume at any given pressure during deflation is larger than during
inflation. This behaviour depends on structural proteins (collagen, elastin), surface tension and the
properties of surfactant..

Surfactant is formed in and secreted by type II pneumocytes. The active ingredient is dipalmitoyl
phosphatidylcholine. It helps prevent alveolar collapse by lowering the surface tension between water
molecules in the surface layer. In this way it helps to reduce the work of breathing (makes the lungs more
compliant) and permits the lung to be more easily inflated.

Various disease states are associated with either a decrease or increase in the lung compliance. Fibrosis,
atelectasis and pulmonary oedema all result in a decrease in lung compliance (stiffer lungs). An increased
lung compliance occurs in emphysema where an alteration is elastic tissue is probably responsible
(secondary to the long term effects of smoking). The lung effectively behaves like a soggy bag so that a

---------------------------------------------------------------------------------------------------------------------------------------------------------------------------------------------------------------
Dr Mohammed Shamsul Islam Khan, Medical Officer, Clinical Neuro-Surgery, National Institute of Neuro-Sciences and Hospital
Sher-E-Bangla Nagar, Dhaka-1207, Bangladesh. Mobile: +880 1713 455 662, +880 1685 811979. E-mail: drsikhan@gmail.com
MyPasTest MRCS A Online - Jan Exam 2015
10. Physiology; System Specific Physiology (399Qs)
----------------------------------------------------------------------------------------------------------------------------------
given pressure change results in a large change in volume (i.e. the lungs are more compliant). However,
during expiration the airways are less readily supported and collapse at higher lung volumes resulting in
gas trapping and hyperinflation.

93. Pulmonary vascular resistance increases: Single best answer question choose ONE true option
only. Page |
At high altitude YOUR ANSWER (Correct Answer) 881
During space flight
On exercise
On inspiring 100% oxygen
With anaemia.

Pulmonary vascular resistance increases at high altitude as a result of the global alveolar hypoxia. The
exact mechanism is still unknown but is apparently a local effect on the smooth muscle of the pulmonary
arterial wall. The increase causes right ventricular hypertrophy with characteristic electrocardiogram
(ECG) changes.

Pulmonary vascular resistance during space flight would, if anything, decrease as blood flow becomes
more uniform. Anaemia would decrease viscosity - a term in the resistance formula. With exercise,
pulmonary arterial pressure tends to rise causing recruitment and distension of pulmonary vessels
leading to a fall in pulmonary vascular resistance. Inspiring 100% oxygen would also cause the
pulmonary vascular resistance to drop.

94. Features of atrial fibrillation (AF) include which one of the following? Select one answer only.
Absent P waves YOUR ANSWER (Correct Answer)
Regular pulse
T wave changes
A regular heart rhythm
Will convert to sinus rhythm with carotid massage.

Features of atrial fibrillation include absent P waves (on electrocardiogram (ECG)) and an irregular pulse.
Atrial fibrillation does not produce any T wave changes. T wave changes are associated with ischaemia.
Carotid sinus massage will convert atrial flutter but not fibrillation.

95. With regards to Tracheostomy which of the following is correct? Select one answer only.
Is uncomplicated by thyroid disease YOUR ANSWER
May be needed for bronchial toilet CORRECT ANSWER
Is straightforward in people with a short neck
will increase the anatomical dead space
Increases the ventilationperfusion mismatch.
---------------------------------------------------------------------------------------------------------------------------------------------------------------------------------------------------------------
Dr Mohammed Shamsul Islam Khan, Medical Officer, Clinical Neuro-Surgery, National Institute of Neuro-Sciences and Hospital
Sher-E-Bangla Nagar, Dhaka-1207, Bangladesh. Mobile: +880 1713 455 662, +880 1685 811979. E-mail: drsikhan@gmail.com
MyPasTest MRCS A Online - Jan Exam 2015
10. Physiology; System Specific Physiology (399Qs)
----------------------------------------------------------------------------------------------------------------------------------
Emergency tracheostomy is a formal operation and should be carried out under controlled circumstances
under general anaesthetic. However, percutaneous tracheostomy is performed under local anaesthetic in
some centres. It is useful in enabling adequate toilet of the lungs and the reduction in dead space may aid
weaning from ventilation. Anatomical dead space of the respiratory tract refers to all the areas not
involved in gas exchange, such as the oropharynx.

96. You review a patient on the ward who underwent a right femoral nailing 2 days ago. The patient is
Page |
short of breath and has chest pain on inspiration. You suspect a pulmonary embolus. Which of the 882
following signs points to this diagnosis? Single best answer - select one answer only.
Cyanosis YOUR ANSWER (Correct Answer)
A fourth heart sound
Hypertension
Low jugulovenous pressure
A Q-wave and inverted T-wave in lead V1 on the electrocardiogram (ECG).

In massive pulmonary embolism, the patient may have acute circulatory failure with cyanosis,
hypotention and a raised JVP. A third heart sound may be heard. The most common ECG abnormality is a
sinus tachycardia, but the classical S-wave occurs in lead 1 and the Q-wave and inverted T-wave in lead 3.

97. Oxygen is carried in the blood in two forms. Most is carried combined with haemoglobin but there is a
very small amount dissolved in the plasma. How much oxygen is normally carried in the blood? Single
best answer question choose ONE true option only.
2 ml oxygen/100 ml blood YOUR ANSWER
5 ml oxygen/100 ml blood
10 ml oxygen/100 ml blood
20 ml oxygen/100 ml blood CORRECT ANSWER
30 ml oxygen/100 ml blood.

In a normal person each 100 ml of blood contains about 15 g of haemoglobin and each gram of
haemoglobin can bind with about 1.34 ml of oxygen when it is 100% saturated (15 1.34 = 20 ml O 2/100 ml
blood). The haemoglobin in venous blood leaving the peripheral tissues is about 75% saturated with
oxygen, so the amount of oxygen transported by haemoglobin in venous blood is about 15 ml O 2/100 ml
blood. Therefore, about 5 ml of oxygen is normally transported to the tissues in each 100 ml of blood.

At a p(O2) > 100 mmHg only 3 ml of oxygen will dissolve in every litre of plasma. If the p(O2) of oxygen in
arterial blood (pA(O2)) is increased significantly (by breathing 100% oxygen) then a small amount of extra
oxygen will dissolve in the plasma (at a rate of 0.003 ml O2/100 ml of blood per mmHg p(O2)) but there will
normally be no significant increase in the amount carried by haemoglobin, which is already > 95%
saturated with oxygen.
98. A 69-year-old women with a previous myocardial infarct is admitted to HDU with shortness of breath,
peripheral oedema and signs of chest sepsis. A line is inserted to measure her central venous pressure.

---------------------------------------------------------------------------------------------------------------------------------------------------------------------------------------------------------------
Dr Mohammed Shamsul Islam Khan, Medical Officer, Clinical Neuro-Surgery, National Institute of Neuro-Sciences and Hospital
Sher-E-Bangla Nagar, Dhaka-1207, Bangladesh. Mobile: +880 1713 455 662, +880 1685 811979. E-mail: drsikhan@gmail.com
MyPasTest MRCS A Online - Jan Exam 2015
10. Physiology; System Specific Physiology (399Qs)
----------------------------------------------------------------------------------------------------------------------------------
Which of the following is correct regarding central venous lines? Single best answer select one answer
only.
They may involve cannulation of the internal jugular vein YOUR ANSWER (Correct Answer)
They are commonly used for pulmonary artery catheterisation via a SwannGanz catheter
They are usually tunnelled under the skin Page |
Line infection is a problem when left in situ for greater than 21 days 883
Allows continuous blood pressure monitoring.

Central venous cannulation is usually through the internal jugular or the subclavian veins. It allows the
measurement of the central venous pressure. Complications include air embolism (if the patient is not
head down and the cap is left off the cannula), pneumothorax and haemothorax are dangerous
complications. Haematoma or infections are relatively common minor complications. Allows assessment
of the state of hydration of the patient, by monitoring the response to filling. Lines should be withdrawn
when clinical needs warrants it, careful consideration to remove or replace should be taken after 10 days.

99. You admit a patient who has fallen 6 meters sustaining a significant head injury. When treating this
patient's intracranial pressure and cerebral blood flow following head injury, which of the following
statesments is correct?
Cerebral blood flow remains constant despite fluctuation in blood pressure YOUR ANSWER
Hyperventilation increases cerebral blood flow
Increased PCO2 increases intracranial pressure CORRECT ANSWER
Intracranial pressure can be reduced by controlled hypoventilation
Raised blood pressure and tachycardia are signs of elevated intracranial pressure.

The first step in any resuscitation procedure including following head injury is establishment of A
(airway), B (breathing), C (circulation). Hyperventilation reduces the carbon dioxide tension in the blood;
this leads to cerebral vasoconstriction thereby reducing cerebral blood flow and hence reduces
intracranial pressure. Cushings reflex (i.e. raised blood pressure and bradycardia) is also a consequence
of raised intracranial pressure.

100. A 21-year-old intravenous drug abuser is found unconscious at home lying on his left arm for an
unknown period of time. He develops rhabdomyolysis, requires a surgical decompression and daily
dialysis. A central line is inserted to monitor his intravascular fluid needs. Which of the following is true
of central venous pressure? Select one answer only.
Is affected by the Valsalva manoeuvre YOUR ANSWER (Correct Answer)
Is reduced in right ventricular failure
Is normal in septic shock
Accurately reflects cardiac output
Is raised in hypertension.

---------------------------------------------------------------------------------------------------------------------------------------------------------------------------------------------------------------
Dr Mohammed Shamsul Islam Khan, Medical Officer, Clinical Neuro-Surgery, National Institute of Neuro-Sciences and Hospital
Sher-E-Bangla Nagar, Dhaka-1207, Bangladesh. Mobile: +880 1713 455 662, +880 1685 811979. E-mail: drsikhan@gmail.com
MyPasTest MRCS A Online - Jan Exam 2015
10. Physiology; System Specific Physiology (399Qs)
----------------------------------------------------------------------------------------------------------------------------------
The CVP reflects the pressure in the right atrium and so is elevated in right-sided heart failure. The
Valsalva manoeuvre raises CVP by impeding venous return to the heart. CVP is low in all types of shock,
except for cardiogenic shock. CVP does not directly reflect cardiac output and, at best, is an indicator of
intravascular fluid volume.

101. The largest contribution to systemic vascular resistance (SVR) is made by the---- Single best answer Page |
question choose ONE true option only.
884
Aortic valve YOUR ANSWER
Great arteries
Arterioles CORRECT ANSWER
Venules
Great veins.

The capillaries and arterioles each account for around 25% of the SVR. The large surface area of the
capillaries, as well as the low flow and pressure drop through the capillary beds is vital to their function in
exchange of gases and nutrients. The arterioles have abundant smooth muscle in their walls, and flow is
regulated to a large degree by the sympathetic nervous system. They therefore exert a great deal of
control over the flow through the capillary beds, as well as which capillary beds are open at a given time.

102. You are assessing a patient who needs an emergency appendicectomy under general anaesthesia.
The patient is 60 years of age and has a past medical history of a Myocardial Infarction (MI) within the last
year and hypertension. The patient asks you about the risk of further MI. What is the correct information
to convey? Single best answer - select one answer only.
Operation within 1 week of an MI carries a 50% risk of mortality YOUR ANSWER
Operation within 3 months of an MI carries a risk of re-infarction of more than 50%
Operation within 3-6 months of an MI carries a risk of re-infarction of approximately 15% CORRECT
ANSWER
The resting electrocardiogram is the best indicator of ischaemic heart disease
Well-controlled hypertension carries a significant postoperative myocardial risk.

The most important indicator of ischaemic heart disease is in the patients history. Risk factors in the
history include smoking, diabetes or hypercholesterolaemia. They should lead one to suspect myocardial
disease. Well-controlled hypertension does not appear to be an adverse risk factor.

The risk of re-infarction during general anaesthetic is 37% within 3 months, 16% at 3-6 months and 4-5%
after this. These figures are obviously dependent upon the nature of the infarct and the nature of the
operative intervention proposed. www.anesthesia-analgesia.org/content/56/3/455.full.pdf

103. You are playing football in your annual medics Vs surgeons match, which has gone into overtime. As
you struggle to keep up the pace of the match, you stop to consider your body's response to exertion.
Which of the following is NOT true of Starlings law of the heart? Single best answer select one answer
only.
---------------------------------------------------------------------------------------------------------------------------------------------------------------------------------------------------------------
Dr Mohammed Shamsul Islam Khan, Medical Officer, Clinical Neuro-Surgery, National Institute of Neuro-Sciences and Hospital
Sher-E-Bangla Nagar, Dhaka-1207, Bangladesh. Mobile: +880 1713 455 662, +880 1685 811979. E-mail: drsikhan@gmail.com
MyPasTest MRCS A Online - Jan Exam 2015
10. Physiology; System Specific Physiology (399Qs)
----------------------------------------------------------------------------------------------------------------------------------
States that the energy of cardiac contraction is dependent upon the resting length of the cardiac muscle
fibre YOUR ANSWER
Means that the more the heart is filled during diastole the greater the force of contraction during systole
Means that stroke volume is reduced during exercise CORRECT ANSWER
Describes how the cardiac output is matched to venous return Page |
Explains how congestion in the left or right side of the heart is prevented. 885
Starling described the intrinsic response of the heart to changes in right atrial and aortic pressure. With
an increase in ventricular filling during diastole there is an increase in fibre length, which results in an
increase in the force of contraction and stroke volume. This is an adaptation of stroke volume to venous
return.

104. In a breathless pat ient, a pleural effus ion w ith less than 2.5g of prote in per 100ml of flu id is most l
ikely to be caused bySingle best answer quest ion choose ONE true opt ion only.
Bronchial carcinoma YOUR ANSWER
Mitral regurgitation CORRECT ANSWER
Pneumonia
Tuberculosis
Tricuspid regurgitation.

An effusion with less than 2.5g of protein per 100ml is a transudate. Other biochemical characteristics of
a transudate include LDH < 200 iU/l, WCC < 1000/ml, glucose <3 mmol/l. Transudative effusions are most
commonly due to factors such as decompensated liver failure and left ventricular failure. Malignancy and
infection are causes of an exudative pleural effusion. Pulmonary embolism can cause either an exudative
or transudative effusion, although the former is more common.

In this question, B is more likely than E to be associated with left ventricular failure, and therefore a
pleural effusion. Tricuspid regurgitation is usually functional and secondary to an enlarged right ventricle
in right ventricular failure, and causes a pulsatile liver, peripheral oedema and ascites.

105. Which of the following statements correctly defines cardiac index? Single best answer - select one
answer only.
Cardiac output/body weight YOUR ANSWER
Cardiac output/body surface area CORRECT ANSWER
Cardiac output x stroke volume
Mean arterial pressure x systemic vascular resistance
Stroke volume x heart rate.

Cardiac index is cardiac output/body surface area (CI = CO/BSA). The normal value for the cardiac index
is between 2.5-4.2 l/min per m2. Cardiac output is stroke volume x heart rate (CO = SV x HR). Systemic
---------------------------------------------------------------------------------------------------------------------------------------------------------------------------------------------------------------
Dr Mohammed Shamsul Islam Khan, Medical Officer, Clinical Neuro-Surgery, National Institute of Neuro-Sciences and Hospital
Sher-E-Bangla Nagar, Dhaka-1207, Bangladesh. Mobile: +880 1713 455 662, +880 1685 811979. E-mail: drsikhan@gmail.com
MyPasTest MRCS A Online - Jan Exam 2015
10. Physiology; System Specific Physiology (399Qs)
----------------------------------------------------------------------------------------------------------------------------------
vascular resistance is (mean arterial pressure - central venous pressure)/cardiac output (SVR = [MAP -
CVP]/CO ).

106. A 36-year-old man was brought to the Accident and Emergency Department following an automobile
accident in which he suffered a pelvic fracture with significant internal blood loss resulting in
haemorrhagic shock. Which of the following organs is especially vulnerable during the shock phase in Page |
this patient? Single best answer question choose ONE true option only.
Brain YOUR ANSWER
886
Heart muscle
Kidneys CORRECT ANSWER
Skin
Skeletal muscle.

During resting conditions, approximately 15% of the cardiac output goes to the brain, 15% to the muscles,
30% to the gastrointestinal (GI) tract and 20% to the kidneys. However, when normalised by organ weight,
the kidneys receive the largest specific blood flow (400 ml/min 100 g) at rest and are particularly
vulnerable during haemorrhagic shock. The brain also receives relatively high specific blood flow (50
ml/min 100 g).

Blood flow through the skin varies between 1 and 100 ml/min 100 g and serves temperature regulation.
Heart muscle not surprisingly also has a relatively high resting specific blood flow (60 ml/min 100 g),
which may increase five-fold during exercise. Skeletal muscles have low specific blood flow (2 to 3 ml/min
100 g) at rest, which may increase up to 20-fold during strenuous exercise.

107. The lung has two circulations, the pulmonary circulation that perfuses alveoli and the bronchial
circulation that provides nutrients and gas exchange for the conducting airways. What percentage of
cardiac output is received by bronchial circulation? Single best answer question choose ONE true
option only.
2% YOUR ANSWER (Correct Answer)
12%
20%
50%
70%.

The bronchial circulation is part of the systemic circulation and receives about 2% of the cardiac output
from the left heart. Bronchial arteries arise from branches of the aorta, intercostal, subclavian or internal
mammary arteries. The bronchial arteries supply the tracheobronchial tree with both nutrients and O 2.
Vascular pressures in the bronchial circulation are similar to those in other systemic vascular beds.

About a third of the venous drainage from the bronchial circulation is via the azygos, hemiazygos and
intercostals veins, which return bronchial venous blood to the right atrium. However, about two-thirds of
bronchial capillary blood is thought to drain into anastomoses or communicating vessels that empty into
---------------------------------------------------------------------------------------------------------------------------------------------------------------------------------------------------------------
Dr Mohammed Shamsul Islam Khan, Medical Officer, Clinical Neuro-Surgery, National Institute of Neuro-Sciences and Hospital
Sher-E-Bangla Nagar, Dhaka-1207, Bangladesh. Mobile: +880 1713 455 662, +880 1685 811979. E-mail: drsikhan@gmail.com
MyPasTest MRCS A Online - Jan Exam 2015
10. Physiology; System Specific Physiology (399Qs)
----------------------------------------------------------------------------------------------------------------------------------
the pulmonary veins. This vascular connection between the bronchial and pulmonary circulation is called
the bronchopulmonary circulation. This communicating circulation adds a small volume of poorly
oxygenated bronchial venous blood to the freshly oxygenated blood in the pulmonary vein.

108. A 69-year-old man is admitted via A and E with a 24 hour history of slurred speech and right sided
weakness. A diagnosis of stroke is made. Which of the following increases cerebral blood flow? Select
one answer only.
Page |
A decrease in body temperature YOUR ANSWER 887
An increase in PO2
Decrease in arterial PCO2
Raised intracranial pressure
Noradrenaline CORRECT ANSWER.

Cerebral blood flow is affected by arterial blood pressure, venous bloodpressure, intracranial pressure,
viscosity of the blood and the vascular resistance of the cerebral arterioles. The calibre of the arterioles
are controlled by local vasodilator metabolites such as PCO2, angiotensin and autoregulatory
mechanisms. Raised intracranial pressure would reduce cerebral blood flow. Noradrenaline would
increase arterial blood pressure and therefore cerebral blood flow .

109. A 34-year-old lady has her blood pressure checked by her GP and it is found to be 180/100. She is
diagnosed with a phaeochromocytoma causing secondary hypertension. Which of the following is most
likely regarding this patients phaeochromocytoma? Single best answer select one answer only.

It is usually bilateral YOUR ANSWER


It is usually extra-adrenal
It is usually malignant
It may be screened for by urinary catecholamines CORRECT ANSWER
It is best treated medically.

Phaeochromocytoma is derived from neural crest and is solitary in 90% of cases. 510% are bilateral and
only 10% are malignant. If suspected, it is best screened by assessing urinary total catecholamines,
which has a 90% sensitivity and specificity compared with vanillylmandelic acid (VMA) in a 24-hour
sample of urine, which has only a 60% sensitivity. Once diagnosed, an adrenal computed tomography
(CT) scan should be undertaken which will show 90% of tumours. When identified, a phaeochromocytoma
should be excised surgically but with adequate preparation of the patient. There is no effective medical
treatment.

110. Carbon dioxide is principally transported in the blood in the form of: Single best answer question
choose ONE true option only.
CO2 physically dissolved in solution YOUR ANSWER

---------------------------------------------------------------------------------------------------------------------------------------------------------------------------------------------------------------
Dr Mohammed Shamsul Islam Khan, Medical Officer, Clinical Neuro-Surgery, National Institute of Neuro-Sciences and Hospital
Sher-E-Bangla Nagar, Dhaka-1207, Bangladesh. Mobile: +880 1713 455 662, +880 1685 811979. E-mail: drsikhan@gmail.com
MyPasTest MRCS A Online - Jan Exam 2015
10. Physiology; System Specific Physiology (399Qs)
----------------------------------------------------------------------------------------------------------------------------------
Carboxyhaemoglobin
Bicarbonate CORRECT ANSWER
Carbaminohaemoglobin
Carbonic anhydrase.
Page |
Carbon dioxide is transported in the blood in the form of: 888
Bicarbonate accounts for about 80-90% of the total CO2 in the blood
As carbamino compounds (5-10%)
Physically dissolved in solution (only 5%)

Carbon dioxide is carried on the haemoglobin molecule as carbamino-haemoglobin; carboxyhaemoglobin


is the combination of haemoglobin with carbon monoxide.
Erythrocytes contain the enzyme carbonic anhydrase that catalyses the reaction------
CO2 + H2O = H+ + HCO3- ..and requires zinc as a co-factor. This plays an important role in carbon
dioxide transport and in the buffering of pH.

111. A deep sea diver loses consciousness soon after resurfacing. A diagnosis of air embolus is made.
Which of the following is correct regarding air embolus? Single best answer select one answer only.
Open-heart surgery may lead to air embolism YOUR ANSWER (Correct Answer)
Death is caused by air trapped in the left ventricular outflow tract
May lead to hypertension
Accidental intravenous injection of 2530 ml of air is frequently fatal
The cause of death is usually rupture of the pulmonary vessels.

Air embolism is a well-recognised cause of death in deep-sea divers. The other important causes include
laparoscopic surgery; open-heart surgery, insertion of central venous catheter, injury to the chest wall,
arterial or venous surgery and massive blood transfusions. The signs and symptoms depend on the site
of the obstruction and the amount of the embolus. A large air embolism affecting the cerebral circulation
arteries will cause immediate loss of consciousness and convulsions.

The common clinical features of air embolism include joint pain, hypotension, visual disturbances,
hypoxia, apnoea and irregular heartbeats. Accidental injection of up to 50 ml of air is usually absorbed by
the body without any serious effects. However, injections of between 100 and 300 ml can be frequently
fatal. The usual cause of death in air embolism is from circulatory obstruction and cardiovascular
collapse resulting from the air being trapped in the right ventricular outflow tract.

112. Gastric acid secretion is stimulated by: Single best answer question choose ONE true option only.
Somatostatin YOUR ANSWER
Gastrin CORRECT ANSWER
Secretin
---------------------------------------------------------------------------------------------------------------------------------------------------------------------------------------------------------------
Dr Mohammed Shamsul Islam Khan, Medical Officer, Clinical Neuro-Surgery, National Institute of Neuro-Sciences and Hospital
Sher-E-Bangla Nagar, Dhaka-1207, Bangladesh. Mobile: +880 1713 455 662, +880 1685 811979. E-mail: drsikhan@gmail.com
MyPasTest MRCS A Online - Jan Exam 2015
10. Physiology; System Specific Physiology (399Qs)
----------------------------------------------------------------------------------------------------------------------------------
The glossopharyngeal nerve
Cholecystokinin.

Gastric acid is stimulated by 3 factors:


Acetylcholine: From parasympathetic neurones of the vagus nerve that innervate parietal cells Page |
directly.
Gastrin: produced by pyloric G-cells. 889
Histamine: Produced by mast cells. This stimulates the parietal cells directly and also
potentiates parietal cell stimulation by gastrin and neuronal stimulation. H2 blockers such as
ranitidine are therefore an effective way of reducing acid secretion.
Gastric acid is inhibited by 3 factors:
Somatostatin
Secretin
Cholecystokinin
There are 3 classic phases of gastric acid secretion:
Cephalic (preparatory) phase [significant]: Results in the production of gastric acid before food
actually enters the stomach. Triggered by the sight, smell, thought and taste of food acting via
the vagus nerve.
Gastric phase [most significant]: Initiated by the presence of food in the stomach, particularly
protein rich food.
Intestinal phase [least significant]: The presence of amino acids and food in the duodenum
stimulate acid production.

113. Growth hormone is a 191-amino acid, single-chain polypeptide hormone that is synthesised, stored
and secreted by the somatotroph cells within the lateral wings of the anterior pituitary gland, which
stimulates growth and cell reproduction in humans. Secretion of growth hormone is increased by: Single
best answer question choose ONE true option only.
Cortisol YOUR ANSWER
Exercise CORRECT ANSWER
Free fatty acids
Hyperglycaemia
Somatostatin.

Peptides released by neurosecretory nuclei of the hypothalamus into the portal venous blood
surrounding the pituitary are the major controllers of growth hormone (GH) secretion by the
somatotrophs. Growth hormone releasing hormone (GHRH) from the arcuate nucleus and ghrelin
promote GH secretion; somatostatin from the periventricular nuclei inhibits it.

GH secretion is also affected by negative feedback from circulating concentrations of GH and insulinlike
growth factor-1. Although the balance of these stimulating and inhibiting peptides determines GH release,
this balance is affected by many physiological stimulators and inhibitors of GH secretion. Stimulators of

---------------------------------------------------------------------------------------------------------------------------------------------------------------------------------------------------------------
Dr Mohammed Shamsul Islam Khan, Medical Officer, Clinical Neuro-Surgery, National Institute of Neuro-Sciences and Hospital
Sher-E-Bangla Nagar, Dhaka-1207, Bangladesh. Mobile: +880 1713 455 662, +880 1685 811979. E-mail: drsikhan@gmail.com
MyPasTest MRCS A Online - Jan Exam 2015
10. Physiology; System Specific Physiology (399Qs)
----------------------------------------------------------------------------------------------------------------------------------
GH secretion include (among others) sleep, exercise, hypoglycaemia, dietary protein and estradiol.
Inhibitors of GH secretion include dietary carbohydrate and glucocorticoids.
114. You are attending a trauma call in the emergency department. The patient is a 38-year-old man who
has been stabbed in the thorax. The location of the stab wound is at the level of the sixth costal cartilage
just to the left of the sternum. What is the most likely structure to have been injured? Select one answer
only.
Page |
Aorta YOUR ANSWER
890
Left atrium
Left ventricle
Right atrium
Right ventricle CORRECT ANSWER.

The right ventricle is the most anterior chamber of the heart and most likely to be injured in penetrating
trauma to the chest.

The inferior border of the heart is mostly the right ventricle, with the left ventricle at the apex. The inferior
border surface marking is the right 6th costal cartilage to the left 5th intercostal space (apex). The left
border of the heart is mainly left ventricle with a small amount of left atrium at the upper part. The right
border is comprised of the right atrium.

115. A 48-year-old female is seen by her GP and describes a history of lethargy, reduced appetite but
increased weight over the previous 6 months. Biochemistry reveals she has hypothyroidism and
treatment is commenced with thyroxine. In normal physiology which of the following thyroid hormones
has the most potent effect on metabolism of peripheral tissues? Single best answer - select one answer
only.
Calcitonin YOUR ANSWER
Tetraiodothyronine (T4)
Thyroid stimulating hormone (TSH)
Thyrotrophin releasing hormone (TRH)
Triiodothyronine (T3) CORRECT ANSWER.

Calcitonin is involved in the control of plasma Calcium. TRH from the hypothalamus releases TSH from
the anterior pituitary gland which acts on the thyroid gland to produce T3 and T4 which act on peripheral
tissues and have a negative feedback effect on TSH production. T3 is around 3 5 times more active on
metabolism than T4.

Thyroxine has wide-ranging effects on metabolism. This hormone increases oxygen consumption in
every tissue in the body and stimulates protein synthesis. Thyroxine affects both the synthesis and
degradation of lipids, the net effect being a decrease in lipid stores.
By influencing the mechanisms by which cholesterol is eliminated from the body, thyroxine decreases
plasma cholesterol levels. Owing to its stimulatory effect on metabolic processes, thyroxine will also have
the effect of increasing the demand for coenzymes and vitamins.
---------------------------------------------------------------------------------------------------------------------------------------------------------------------------------------------------------------
Dr Mohammed Shamsul Islam Khan, Medical Officer, Clinical Neuro-Surgery, National Institute of Neuro-Sciences and Hospital
Sher-E-Bangla Nagar, Dhaka-1207, Bangladesh. Mobile: +880 1713 455 662, +880 1685 811979. E-mail: drsikhan@gmail.com
MyPasTest MRCS A Online - Jan Exam 2015
10. Physiology; System Specific Physiology (399Qs)
----------------------------------------------------------------------------------------------------------------------------------
116. A 22-year-old woman has puffy eyes and hoarse voice. Her plasma thyroid-stimulating hormone
(TSH) concentration is low but increases markedly when she is given thyrotrophin-releasing hormone
(TRH). She probably has: Single best answer question choose ONE true option only.
Hyperthyroidism due to a thyroid tumour YOUR ANSWER
Hyperthyroidism due to a primary abnormality in the hypothalamus
Page |
Hypothyroidism due to a primary abnormality in the hypothalamus CORRECT ANSWER
891
Hypothyroidism due to a primary abnormality in pituitary gland
Hypothyroidism due to a primary abnormality in thyroid gland.

Tertiary hypothyroidism, also called hypothalamic-pituitary axis hypothyroidism, results when the
hypothalamus fails to instruct the pituitary to produce sufficient TSH. This young woman's
hypothyroidism improves with TRH administration, confirming the diagnosis of tertiary hypothyroidism.

117. Pasta is an important dietary source of complex starches. Complex starches are mainly digested by
enzymes secreted from the: Single best answer question choose ONE true option only.
Pancreas YOUR ANSWER (Correct Answer)
Large intestine
Salivary glands
Small intestine
Stomach.

Pancreatic secretion contains a large quantity of


-amylase, a digestive enzyme that hydrolyses starches,
glycogen and most other carbohydrates to form disaccharides and a few trisaccharides.

118. A 23-year-old student has severe diarrhoea resulting from an intestinal parasite that he acquired on a
trip to the Far East. He loses 4.0 l of intestinal fluid over a 6-hour period, but feels too sick to drink any
fluids. His urine output during the 6-hour ordeal is only 0.1 l. Which of the following physiological
changes is most likely to be found in this patient at the end of the 6-hour period as compared with the
hour before the onset of diarrhoea? Single best answer question choose ONE true option only.
Decreased plasma angiotensin II levels YOUR ANSWER
Decreased urine osmolarity
Increased extracellular fluid volume
Increased glomerular filtration rate
Increased plasma aldosterone levels CORRECT ANSWER.

Severe dehydration as experienced by the student in this clinical vignette will result in increased plasma
angiotensin II levels, increased urine osmolarity, decreased glomerular filtration rate and increased
plasma aldosterone levels - all attempts on the part of the body to conserve fluid. Severe diarrhoea will
lead to decreased extracellular fluid volume.

---------------------------------------------------------------------------------------------------------------------------------------------------------------------------------------------------------------
Dr Mohammed Shamsul Islam Khan, Medical Officer, Clinical Neuro-Surgery, National Institute of Neuro-Sciences and Hospital
Sher-E-Bangla Nagar, Dhaka-1207, Bangladesh. Mobile: +880 1713 455 662, +880 1685 811979. E-mail: drsikhan@gmail.com
MyPasTest MRCS A Online - Jan Exam 2015
10. Physiology; System Specific Physiology (399Qs)
----------------------------------------------------------------------------------------------------------------------------------
119. You are reviewing pre-operative lung function tests for a 70Kg 21 year old male patient. What would
you expect the vital capacity to be? Select one answer only.
2000ml YOUR ANSWER
3000ml
4000ml Page |
5000ml CORRECT ANSWER 892
6000ml.

The vital capacity is the inspiratory reserve volume + expiratory reserve volume + tidal volume. It
decreases with advancing age from age 20 onwards. In females it is 50-60ml/Kg, in males 70ml/Kg.
Therefore in this case the vital capacity would be expected to be 70x70=4900ml.

120. A child has ingested an overdose of a drug. Which of the following is most likely to reduce the renal
excretion of this drug? Single best answer question choose ONE true option only.
Alkalinisation of urine YOUR ANSWER
Extensive binding of drug to plasma proteins CORRECT ANSWER
Haemodialysis
Induced diuresis
Induction of hepatic microsomal enzymes.

Renal excretion of drugs will be reduced if they are extensively bound to plasma proteins, as plasma
proteins are not filtered under normal conditions. All the rest of the options will either enhance renal
excretion of drugs or increase their removal from the body.

121. A 37-year-old man is admitted with headache and papilloedema. An urgent CT scan is requested.
Which of the following is true of a space occupying lesion in the left cerebral hemisphere? Single best
answer - select one answer only.
May reduce intracranial pressure YOUR ANSWER
May shift the third ventricle to the left
May cause nausea and vomiting CORRECT ANSWER
Must be large before it causes symptoms
Rarely presents with symptoms.

The skull is a rigid compartment and its three main contents (brain, blood and cerebrospinal fluid) are in
dynamic equilibrium (Monroe-Kellie doctrine). An uncompensated increase in one component will result
in raised intracranial pressure (ICP). However, relatively large lesions may develop in the frontal lobes
before any signs of raised ICP are seen. Space-occupying lesions cause a shift of cerebral matter to the
opposite side and so may compress the lateral ventricle. Nausea and vomiting are signs of raised ICP.

---------------------------------------------------------------------------------------------------------------------------------------------------------------------------------------------------------------
Dr Mohammed Shamsul Islam Khan, Medical Officer, Clinical Neuro-Surgery, National Institute of Neuro-Sciences and Hospital
Sher-E-Bangla Nagar, Dhaka-1207, Bangladesh. Mobile: +880 1713 455 662, +880 1685 811979. E-mail: drsikhan@gmail.com
MyPasTest MRCS A Online - Jan Exam 2015
10. Physiology; System Specific Physiology (399Qs)
----------------------------------------------------------------------------------------------------------------------------------
122. You are reviewing a patient in the neurosurgical clinic with neuropathic or chronic pain following a
severe limb injury. Which of the following statements is correct with regards to the pain pathways in
normal individuals? Single best answer - Select one answer only.
Pain is perceived in the thalamus YOUR ANSWER (Correct Answer)
The left precentra lgyrus receives pain stimuli from the right side of the body
Page |
Proprioception is conveyed along with pain in the anterior spinothalamic tracts
893
Spinal reflex activity cannot be modulated by descending spinal pathways
Neurones conveying thermal sensation rarely cross to the contralateral spinal cord.

Proprioceptive information is conveyed in the dorsal columns of the spinal cord. The sensory afferents
for cold (A and C fibres) and warm (C fibres) relay information to the postcentral gyrus via the antero-
lateral spinothalamic tract and the thalamic radiation. Nociceptive afferents end in the spinal cord and are
processed via brainstem, thalamus, postcentral gyrus (sensory cortex) and peri-aqueductal grey matter.

123. You have a post-operative patient, in the intensive care unit, who has decreased coronary blood flow.
The intensive care consultant asks you to tell her what would increase the coronary blood flow. How do
you respond? Single best answser - select one answer ony.
Antidiuretic hormone (ADH)/vasopressin YOUR ANSWER
Distension of a hollow viscus (eg bladder)
Increasing p CO2 CORRECT ANSWER
Pain
Serotonin (5HT).

Coronary blood flow is decreased by ADH, pain and distension of the gut and bladder. Coronary blood
flow is increased by an increase in p(CO2) and decreased by alkalosis.
124. A patient undergoes respiratory function tests. Which of the following are normal readings for a 70-
kg man? Single best answer question choose ONE true option only.
Peak expiratory flow of 376 l/min YOUR ANSWER
Total lung capacity of 3.5 litres
Functional residual capacity of 3.5 litres
Tidal volume of 250 ml
Inspiratory reserve volume of 3 litres CORRECT ANSWER.
Normal readings for such a patient would be:
peak expiratory flow 520700 l/min
total lung capacity 56.5 litres
functional residual capacity 23 litres
tidal volume 500700 ml

---------------------------------------------------------------------------------------------------------------------------------------------------------------------------------------------------------------
Dr Mohammed Shamsul Islam Khan, Medical Officer, Clinical Neuro-Surgery, National Institute of Neuro-Sciences and Hospital
Sher-E-Bangla Nagar, Dhaka-1207, Bangladesh. Mobile: +880 1713 455 662, +880 1685 811979. E-mail: drsikhan@gmail.com
MyPasTest MRCS A Online - Jan Exam 2015
10. Physiology; System Specific Physiology (399Qs)
----------------------------------------------------------------------------------------------------------------------------------
125. A 17-year-old male is brought into A&E with a one day history of feeling generally unwell and
feverish. On further questioning it emerges he has photophobia and is increasingly drowsy. He is
commenced on antibiotics and a lumbar puncture is arranged for possible meningitis. In a healthy
individual what is the relationship between glucose and protein levels in the cerebrospinal fluid (CSF)
relative to plasma?
CSF glucose equal, CSF protein equal YOUR ANSWER Page |
CSF glucose equal, CSF protein lower 894
CSF glucose lower, CSF protein equal
CSF glucose lower, CSF protein higher
CSF glucose lower, CSF protein lower CORRECT ANSWER.

CSF is formed mainly by the choroid plexus by an active secretory process. It circulates in the
subarachnoid space and is absorbed into the circulation by the arachnoid villi. CSF glucose and protein
levels are both much lower than those of plasma. Changes in these concentrations are helpful in
detecting pathological processes such as tumour or infection, where the bloodbrain barrier is breached.

126. A 59-year-old, poorly-controlled diabetic, is admitted with sepsis secondary to a necrotic first and
second toe and undergoes radical debridement. Which of the following statements on the diabetic foot is
correct? Single best answer - select one answer only.
Ankle brachial pressure index (ABPI) is low YOUR ANSWER
It is usually painful
It is associated with a peripheral neuropathy CORRECT ANSWER
Lumbar sympathectomy increases blood supply to the muscles
Once foot ulcers are established strict diabetic control will not help in the healing process.

The diabetic foot is often painless due to an associated peripheral neuropathy. The ABPI is obtained
using a Doppler probe by comparing the pressure in the dorsalispedis, peroneal and posterior tibial
arteries at the ankle with the brachial systolic pressure. It may be spuriously elevated in diabetic patients.
In these patients, the increased calcification of the peripheral vessels withstands compression using a
tourniquet resulting in an elevated reading for Doppler pressure. A lumbar sympathectomy increases
blood flow to the skin, and not muscle, as the sympathetic nerve supply is predominantly to cutaneous
blood vessels.

127. The autonomic system is part of the peripheral nervous system that control visceral functions. Which
of the following statements regarding autonomic neurons is correct? Single best answer - select one
answer only.
Activity of autonomic ganglionic neurons is modulated by postganglionic neurons YOUR ANSWER
Generally involve one neurone in the efferent pathway
Of the sympathetic variety are formed in the mid-brain and the sacrum

---------------------------------------------------------------------------------------------------------------------------------------------------------------------------------------------------------------
Dr Mohammed Shamsul Islam Khan, Medical Officer, Clinical Neuro-Surgery, National Institute of Neuro-Sciences and Hospital
Sher-E-Bangla Nagar, Dhaka-1207, Bangladesh. Mobile: +880 1713 455 662, +880 1685 811979. E-mail: drsikhan@gmail.com
MyPasTest MRCS A Online - Jan Exam 2015
10. Physiology; System Specific Physiology (399Qs)
----------------------------------------------------------------------------------------------------------------------------------
Of the parasympathetic variety are formed in the mid-brain and the sacrum CORRECT ANSWER
Of the sympathetic variety commonly have synapses in which the neurotransmitter is noradrenaline.

The parasympathetic neurons are formed at the upper and lower extremities of the spinal cord, in the mid-
brain and hindbrain (fibres are carried by cranial nerves III, VII, IX and X) but also in the sacral spinal cord
(S2S4). The neurons of the sympathetic nervous system are formed throughout the thoracic and upper
Page |
lumbar spine. All autonomic nerves are composed of two neurons, which synapse in the autonomic 895
ganglia.

The ganglia of the sympathetic nervous system are found in a chain either side of the spine. The ganglia
of the parasympathetic nervous system tend to lie near the target organ (eg coeliac ganglion which
supplies the foregut). The neurotransmitter at the synapses in the ganglia of both the sympathetic and
parasympathetic nervous systems is acetylcholine. The synapses of the post-ganglionic fibres with the
end organs in the sympathetic nervous system involve catecholaminergic neurotransmission.

128. Various lung intravascular and extravascular pressures influence pulmonary blood flow and its
distribution in the lung. What is the normal mean pulmonary arterial pressure? Single best answer
question choose ONE true option only.
5 mmHg YOUR ANSWER
10 mmHg
15 mmHg CORRECT ANSWER
50 mmHg
90 mmHg.

The hydrostatic pressure of the pulmonary circulation refers to the actual pressure inside pulmonary
vessels relative to atmospheric pressure. Hydrostatic (blood pressure) in the pulmonary vascular bed is
low compared with that of similar systemic vessels. The mean pulmonary arterial pressure is about 15
mmHg (ranging from about 13 to 19 mmHg) and is much lower than the average systemic arterial
pressure of 90 mmHg.

The pressure drop between pulmonary artery and capillary is small compared with that of systemic
vessels because the lung lacks high resistance arterioles. The lack of arterioles minimises active
regulation of lung blood flow distribution. In addition, there may be little damping of the arterial pressure
pulse associated with right heart ejection (systole). During late diastole, blood flow in some lung
capillaries, particularly at the lung apex, may momentarily cease.

129. Glucagon is secreted by the -cells of the pancreatic islets. Which of the following is most likely to
induce glucagon secretion? Single best answer question choose ONE true option only.
High serum concentration of glucose YOUR ANSWER
Low serum concentration of amino acids
Low serum concentration of glucose CORRECT ANSWER
Secretion of somatostatin by the pancreatic d-cells
---------------------------------------------------------------------------------------------------------------------------------------------------------------------------------------------------------------
Dr Mohammed Shamsul Islam Khan, Medical Officer, Clinical Neuro-Surgery, National Institute of Neuro-Sciences and Hospital
Sher-E-Bangla Nagar, Dhaka-1207, Bangladesh. Mobile: +880 1713 455 662, +880 1685 811979. E-mail: drsikhan@gmail.com
MyPasTest MRCS A Online - Jan Exam 2015
10. Physiology; System Specific Physiology (399Qs)
----------------------------------------------------------------------------------------------------------------------------------
Parasympathetic stimulation.

Hypoglycaemia is the most potent stimulus for glucagon secretion. High serum levels of amino acids
(especially alanine and arginine) will also induce glucagon release. Hyperglycaemia is a potent stimulus
for insulin release from pancreatic -cells. Somatostatin inhibits glucagon secretion. While
parasympathetic stimulation to the pancreas stimulates acinar secretion, it does not stimulate -cells to Page |
secrete glucagon.
896
130. Vitamin K is involved in carboxylation of glutamic acid residues of the following clotting factors
EXCEPT for? Single best answer question choose ONE correct option only.
XI YOUR ANSWER (Correct Answer)
Prothrombin
VII
IX
X.

Vitamin K is essential for activation of clotting factors II (prothrombin), VII, IX, X.

131. A 69-year-old man in heart failure is commenced on inotrope therapy to increase the force of heart
muscular contractions. Which of the following have a direct positive inotropic effect on the heart? Select
one answer only.
Isoprenaline infusion YOUR ANSWER (Correct Answer)
GTN infusion
Verapamil
Intravenous furosemide (frusemide)
Disopyramide.

Dopamine at low doses has an effect on pure dopaminergic receptors;only at higher doses does it have a
truly inotropic effect via ? receptors. Calcium and digoxin are both inotropes. Furosemide (frusemide) and
GTN may have an indirect inotropic effect if used to offload a failing ventricle, but they themselves are not
inotropic. Isoprenaline is a chronotropic agentand is occasionally used in bradycardic states.

132. At a given pO2, haemoglobin gives up oxygen more readily in the following situation. Single best
answer question choose ONE true option only.
Decreased pCO2 YOUR ANSWER
Decreased levels of 2,3 DPG
Presence of fetal haemoglobin
Increased pH

---------------------------------------------------------------------------------------------------------------------------------------------------------------------------------------------------------------
Dr Mohammed Shamsul Islam Khan, Medical Officer, Clinical Neuro-Surgery, National Institute of Neuro-Sciences and Hospital
Sher-E-Bangla Nagar, Dhaka-1207, Bangladesh. Mobile: +880 1713 455 662, +880 1685 811979. E-mail: drsikhan@gmail.com
MyPasTest MRCS A Online - Jan Exam 2015
10. Physiology; System Specific Physiology (399Qs)
----------------------------------------------------------------------------------------------------------------------------------
Increased temperature CORRECT ANSWER.

The oxygen dissociation curve is a sigmoid shape; a shift to the right enables haemoglobin to give up
oxygen more readily at a given pO2. This occurs where temperature rises, PCO2 rises and pH falls. Levels
of 2,3, DPG rise in conditions causing chronic hypoxia, such as airway obstruction or living at high
altitude, and also cause a right shift of the curve hence B is false. The increased affinity of fetal Page |
haemoglobin for oxygen (i.e. a left shift of the curve) facilitates oxygen transport across the placenta 897
hence C is false.

133. A 15-year-old boy is referred to an endocrine clinic as his GP has noted a lack of secondary sexual
characteristics. In the clinic following further tests he is commenced on testosterone for delayed puberty.
Which of the following cell types is the main source of testosterone in males? Single best answer - select
one answer only.
Adrenal cortex YOUR ANSWER
Adrenal medulla
Anterior pituitary
Leydig cells CORRECT ANSWER
Sertoli cells.

The germ cells of the man are known as spermatogonia. They reside in the outer regions of the
seminiferous tubules. The diploid cells undergo two meiotic divisions to form haploid gametes
(spermatids). These spermatids are formed in response to testosterone. Testosterone is produced mainly
by Leydig cells in the testes. The developing spermatozoa are supported by Sertoli cells, which line the
seminiferous tubules. Sertoli cells maintain the immunologically privileged environment in the testes.

They secrete a substance that makes T cells apoptose, in case they generate an immune response
against the spermatozoa, which would be recognized as non-self. A bloodtestis barrier prevents
formation of antispermatozoan antibodies. Follicle-stimulating hormone (FSH), in addition to
testosterone, is required for the latter parts of spermatogenesis. The spermatozoa are released into the
seminiferous tubules as non-motile structures. They become fully mature and motile in the epididymis.

134. Endometrial biopsy from a 24-year-old healthy woman with regular menstrual cycle shows the
presence of highly coiled arteries with oedema. Which of the following stages of the menstrual cycle
correlates with this description? Single best answer question choose ONE true option only.
Early proliferative YOUR ANSWER
Late proliferative
Menstruation
Ovulation
Secretory CORRECT ANSWER.
The secretory phase of the menstrual cycle begins soon after ovulation occurs. Under the influence of
progesterone produced by the corpus luteum in the ovary, the endometrium continues to grow to reach a
---------------------------------------------------------------------------------------------------------------------------------------------------------------------------------------------------------------
Dr Mohammed Shamsul Islam Khan, Medical Officer, Clinical Neuro-Surgery, National Institute of Neuro-Sciences and Hospital
Sher-E-Bangla Nagar, Dhaka-1207, Bangladesh. Mobile: +880 1713 455 662, +880 1685 811979. E-mail: drsikhan@gmail.com
MyPasTest MRCS A Online - Jan Exam 2015
10. Physiology; System Specific Physiology (399Qs)
----------------------------------------------------------------------------------------------------------------------------------
maximum thickness of 5-7 mm as measured from the muscle wall to the endometrial cavity. The stromal
cells continue to increase in size and number. Blood supply to the endometrium increases. The important
change in this phase occurs in the endometrial glands.

The glands increase in size and become actively secretory, hence the name - 'secretory phase'. In the
early stage, the secretions collect in the cells of the glands. However, by the 19th to the 22nd day of the
cycle, the secretions are pushed out of the cells and collect in the endometrial cavity. This secretion is
Page |
rich in glycogen, fructose and glucose. Its main function is to supply nutrition to any fertilized ovum 898
reaching the uterus.

135. When a person is dehydrated, hypotonic fluid will be found in the: Single best answer question
choose ONE true option only.
Glomerular filtrate YOUR ANSWER
Proximal tubule
Distal end of the ascending loop of Henle CORRECT ANSWER
Late distal convoluted tubule
Collecting duct.

Fluid in the distal end of the ascending limb of loop of Henle is hypotonic regardless of the state of
hydration because of the active reabsorption of sodium chloride but not water, which is impermeant in
this tubular segment. In dehydration, the late distal convoluted tubule and the collecting duct become
more and more hypertonic in the presence of ADH (antidiuretic hormone), which increases permeability of
these tubular segments. The glomerular filtrate and proximal tubule have the same tonicity as plasma.

136. In a lung function test, the functional residual capacity: Single best answer question choose ONE
true option only.
Is the sum of the tidal volume and residual volume YOUR ANSWER
Is the sum of the inspiratory reserve volume, the expiratory reserve volume and the tidal volume
Can be measured directly by spirometry
Is equal to the sum of the residual volume and the expiratory reserve volume CORRECT ANSWER
Is that volume of air that remains in the lung after forced expiration.

Spirometry traces are easy to understand if you remember the following two rules:
1. There are 4 lung volumes and 5 capacities that you need to remember.
2. A capacity is made up of 2 or more lung volumes
The 4 lung volumes are:
Tidal volume = volume of air inspired or expired with each normal breath in quiet breathing;
approximately 500mls.
Residual volume = that volume of air that remains in the lung after forced expiration.
Inspiratory reserve volume = extra volume of air that can be inspired over and above the normal
tidal volume.
---------------------------------------------------------------------------------------------------------------------------------------------------------------------------------------------------------------
Dr Mohammed Shamsul Islam Khan, Medical Officer, Clinical Neuro-Surgery, National Institute of Neuro-Sciences and Hospital
Sher-E-Bangla Nagar, Dhaka-1207, Bangladesh. Mobile: +880 1713 455 662, +880 1685 811979. E-mail: drsikhan@gmail.com
MyPasTest MRCS A Online - Jan Exam 2015
10. Physiology; System Specific Physiology (399Qs)
----------------------------------------------------------------------------------------------------------------------------------
Expiratory reserve volume = extra volume of air that can be expired by forceful expiration after the
end of a normal tidal expiration.
The 5 lung capacities are:
Functional residual capacity = that volume of air that remains in the lung at the end of quiet
expiration. Equal to the sum of the residual volume and the expiratory reserve volume.
Inspiratory capacity = inspiratory reserve volume + tidal volume
Expiratory capacity = expiratory reserve volume + tidal volume
Page |
Vital capacity = inspiratory reserve volume + tidal volume + expiratory reserve volume (or total 899
lung capacity residual volume)
Total lung capacity = vital capacity + residual volume.

The residual volume (and therefore functional residual capacity and total lung capacity) cannot be
measured directly by spirometry. They are measured by either whole body plethysmography, or by using
the helium dilution or nitrogen washout techniques.

137. In a trial to test the effectiveness of a new drug against the standard treatment for improving forced
expiratory volume in 1 s (FEV1) amongst asthmatics, the members of age- and sex-matched pairs are
randomly allocated to the new or standard treatment. The difference (new-standard) in forced vital
capacity (FVC) between the pairs one hour after treatment was 0.5 0.2 (mean standard error), P <0.005.
What are the chances of this difference having occurred by chance? Single best answer - select one
answer only.
Less than one time in 100 YOUR ANSWER
Less than one time in 500
Less than one time in 1000
Less than two in 1000
Less than five in 1000 CORRECT ANSWER.

P <0.005 shows that the observed differences would have occurred less than five times in 1000 if the new
drug was no more, or less, effective than the standard (5 times in 1000 = 1 time in 200).

138. Hypoxic pulmonary vasoconstriction: Single best answer question choose ONE true option only.
Acts reflexively via the central nervous system YOUR ANSWER
Improves matching of ventilation and blood flow in some lung diseases CORRECT ANSWER
Is not important in the perinatal period
Is irreversible
Requires a p(O2) of less than 40 mmHg in mixed venous blood.

Suppose a lobe or lobule of lung is poorly ventilated because of partial bronchial obstruction. The
resulting alveolar hypoxia will reduce the blood flow through the mechanism of hypoxic pulmonary
vasoconstriction. The result is improvement in the matching of ventilation and blood flow. Although the
mechanism of hypoxic pulmonary vasoconstriction is not fully understood, we know that central nervous
connections are not required because the phenomenon can be demonstrated in isolated lungs. Reducing
---------------------------------------------------------------------------------------------------------------------------------------------------------------------------------------------------------------
Dr Mohammed Shamsul Islam Khan, Medical Officer, Clinical Neuro-Surgery, National Institute of Neuro-Sciences and Hospital
Sher-E-Bangla Nagar, Dhaka-1207, Bangladesh. Mobile: +880 1713 455 662, +880 1685 811979. E-mail: drsikhan@gmail.com
MyPasTest MRCS A Online - Jan Exam 2015
10. Physiology; System Specific Physiology (399Qs)
----------------------------------------------------------------------------------------------------------------------------------
the p(O2) of the blood entering the lung results in much less vasoconstriction than reducing the p(O2) of
alveolar gas. Hypoxic pulmonary vasoconstriction is important in the perinatal period.

When the newborn baby makes the transition from placental to air breathing, it is important for pulmonary
vascular resistance to fall precipitously within a few seconds. As a consequence, pulmonary blood flow
dramatically increases from its value of only about 15% of the cardiac output in utero. The increase in
pulmonary blood flow is assisted by closure of both the ductus arteriosus and the foramen ovale.
Page |
Hypoxic pulmonary vasoconstriction is reversible unless it is long standing. 900
139. Abolition of the cephalic phase of pancreatic secretion: Single best answer question choose ONE
true option only.
Will mainly affect HCO3- secretion YOUR ANSWER
Will mainly affect enzymatic secretion
Will result in 10% reduction in maximal pancreatic secretion
Will result after vagotomy CORRECT ANSWER
Will result after administration of a sympathetic antagonist.

The cephalic phase of pancreatic secretion is triggered by the central nervous system through
recognition and integration of the sight, smell and taste of food. When human subjects see, smell and
chew food without swallowing, HCO3- and pancreatic enzyme secretion increases. The degree of enzyme
secretion in the cephalic phase is about 50% of the maximal response elicited with exogenous CCK and
secretin. The vagus nerve modulates secretion by means of cholinergic fibres, which innervate pancreatic
acinar cells. The vagus also modulates peptidergic nerve fibres, which innervate duct cells.

140. A 68-year-old female is admitted to coronary care following a myocardial infarct. Her renal blood flow
and renal perfusion pressure fall, causing a drop in urine output. Which of the following factors are
responsible for increasing these? Single best answer - select one answer only.
Angiotensin II YOUR ANSWER
Antidiuretic hormone
Endothelin
Insulin
Prostaglandins CORRECT ANSWER.

The renal blood flow and renal perfusion are controlled by several factors including hormones and the
sympathetic nervous system. Atrial natriuretic peptide causes dilatation of the afferent arterioles and
increases the renal blood flow and the glomerular filtration rate. Prostaglandins (PGE2 and PGI2) increase
the renal blood flow and the glomerular filtration rate. Angiotensin II produced systemically, and within
the kidney, constricts the afferent and the efferent arterioles and decreases the renal blood flow and the
glomerular filtration rate. Adenosine triphosphate selectively vasoconstricts the afferent arteriole and so
reduces the renal blood flow. Endothelin is a potent vasoconstrictor secreted by endothelial cells of
arterioles, which decreases the renal blood flow. Glucocorticoids dilate the afferent arterioles and
---------------------------------------------------------------------------------------------------------------------------------------------------------------------------------------------------------------
Dr Mohammed Shamsul Islam Khan, Medical Officer, Clinical Neuro-Surgery, National Institute of Neuro-Sciences and Hospital
Sher-E-Bangla Nagar, Dhaka-1207, Bangladesh. Mobile: +880 1713 455 662, +880 1685 811979. E-mail: drsikhan@gmail.com
MyPasTest MRCS A Online - Jan Exam 2015
10. Physiology; System Specific Physiology (399Qs)
----------------------------------------------------------------------------------------------------------------------------------
increase the renal blood flow and the glomerular filtration rate. Likewise, nitric oxide also causes
vasodilatation and increases the renal blood flow.

141. The plateau phase of the cardiac action potential is due to: Single best answer question choose
ONE true option only.
Page |
Magnesium influx YOUR ANSWER
Potassium influx
901
Calcium influx CORRECT ANSWER
Chloride efflux
Sodium influx.

The most important source of activator calcium in cardiac muscle remains its release from the
sarcoplasmic reticulum. Calcium however also enters from the extracellular space during the plateau
phase of the action potential. This calcium entry provides the stimulus that induces calcium release from
the sarcoplasmic reticulum (calcium induced calcium release).

The result is that tension generated in cardiac, but not in skeletal, muscle is profoundly influenced both
by extracellular calcium levels and factors that affect the magnitude of the inward calcium current. This is
of practical value in two key clinical situations; in heart failure where digoxin is utilised to increase
cardiac contractility (by increasing the intracellular calcium concentration) and in hyperkalaemia where
calcium gluconate is used to stabilise the myocardium.

The plateau phase of the action potential in cardiac muscle (principally due to calcium influx) maintains
the membrane at a depolarised potential for as long as 500ms. The result is that the cell membrane is
refractory throughout most of the mechanical response, largely due to the inactivation of fast sodium
channels. This prevents tetany upon repetitive stimulation which would be detrimental to cardiac output.
Furthermore, the prolonged refractory period in cardiac muscle allows the impulse that originates in the
sino-atrial node to propagate throughout the entire myocardium just once, thereby preventing re-entry
arrhythmias.

142. The vessels of the pulmonary circulation are very compliant (easily distensible) and so can serve as
a reservoir for the left ventricle. How much blood volume can the pulmonary vessels accommodate in an
adult man under normal resting conditions? Single best answer question choose ONE true option only.
150 ml YOUR ANSWER
250 ml
500 ml CORRECT ANSWER
1000 ml
1500 ml.

---------------------------------------------------------------------------------------------------------------------------------------------------------------------------------------------------------------
Dr Mohammed Shamsul Islam Khan, Medical Officer, Clinical Neuro-Surgery, National Institute of Neuro-Sciences and Hospital
Sher-E-Bangla Nagar, Dhaka-1207, Bangladesh. Mobile: +880 1713 455 662, +880 1685 811979. E-mail: drsikhan@gmail.com
MyPasTest MRCS A Online - Jan Exam 2015
10. Physiology; System Specific Physiology (399Qs)
----------------------------------------------------------------------------------------------------------------------------------
The vessels of the pulmonary circulation are very compliant (easily distensible) and so typically
accommodate about 500 ml of blood in an adult man. This large lung blood volume can serve as a
reservoir for the left ventricle, particularly during periods when left ventricular output momentarily
exceeds venous return. Therefore, cardiac output can be increased rapidly by drawing upon pulmonary
blood volume without depending on an instantaneous increase in venous return. Because of this
function, the lung is sometimes referred to as an 'accessory' heart.
Page |
143. A 52-year-old obese male has recently been diagnosed with type 2 diabetes mellitus by his GP. Part 902
of his treatment has involved the GP commencing orlistat, a pancreatic lipase inhibitor to help reduce
weight and improve glycaemic control. Lipase is also produced at the base of the tongue. What
proportion of triglyceride digestion occurs due to lipase from this source in a healthy individual? Single
best answer - select one answer only.
1% YOUR ANSWER
10%
30% CORRECT ANSWER
50%
75%.

Orlistat is a pancreatic lipase inhibitor and prevents the breakdown of dietary fat to fatty acid and
glycerols. Side effects of its use can include abdominal cramps, flatus, oily spotting and intestinal
borborygmi.

Neural and hormonal signals control pancreatic exocrine secretion. The main trigger is the presence of
acid and products of digestion in the duodenum, not the presence of triglycerides. Lipase is secreted by
glands near the base of the tongue (responsible for up to 30% of triglyceride digestion), and the
remainder is digested by pancreatic lipase which requires a cofactor (co-lipase) to become active. The
products of triglyceride breakdown are free fatty acids and 2-monoglycerides.

144. Your anaesthetic colleague is drawing up drugs for neuromuscular blockade in the prep room whilst
the team goes through the WHO checklist before an operation. Regarding these drugs, which of the
following work primarily by blocking transmission in postsynaptic autonomic fibres? Single best answer -
select one answer only.
Suxamethonium YOUR ANSWER
Hexamethonium CORRECT ANSWER
Bupivacaine
Pentazocine
Fentanyl.

Suxamethonium is a very rapid-acting muscle relaxant and is ideal if crash intubation is required.
Suxamethonium acts by mimicking acetylcholine at the neuromuscular junction but undergoes hydrolysis
much more slowly than acetylcholine. Depolarisation is therefore prolonged, which results in
neuromuscular blockade. Guanethidine is an adrenergic antagonist, preventing the release of
---------------------------------------------------------------------------------------------------------------------------------------------------------------------------------------------------------------
Dr Mohammed Shamsul Islam Khan, Medical Officer, Clinical Neuro-Surgery, National Institute of Neuro-Sciences and Hospital
Sher-E-Bangla Nagar, Dhaka-1207, Bangladesh. Mobile: +880 1713 455 662, +880 1685 811979. E-mail: drsikhan@gmail.com
MyPasTest MRCS A Online - Jan Exam 2015
10. Physiology; System Specific Physiology (399Qs)
----------------------------------------------------------------------------------------------------------------------------------
noradrenaline (norepinephrine) from the postganglionic adrenergic neurones. It is rarely used clinically,
only for resistant hypertension. Pentazocine, like fentanyl, is an opioid analgesic.

Bupivacaine is a long-acting amide local anaesthetic, commonly used for epidural and spinal anaesthesia.
Local anaesthetics depress conduction in small unmyelinated fibres first and larger myelinated fibres
last. It can also affect autonomic nerve fibres but this is not its primary mechanism of action. Page |
Hexamethonium is an agent that causes a depolarising block on the postganglionic cell body. Although it 903
was used for the control of blood pressure several years ago, it is now no longer in use.

145. You are assessing a patient in the emergency department who has been stabbed in the left side of
the chest and you suspect a cardiac tamponade. Which of the following features would be expected with
such a presentation? Single best answer - Select one answer only.
Bradycardia YOUR ANSWER
Elevated central venous pressure but a lowered pulmonary artery pressure
Hypotension CORRECT ANSWER
Increased jugular venous pressure on expiration
Pulsusalternans.

In cardiac tamponade, patients have an elevated central venous pressure (CVP) and pulmonary artery
pressure. The jugular venous pressure (JVP) increases on inspiration (instead of falling) this is called
Kussmauls sign. Pulsusparadoxus not pulsusalternans is seen.

146. A 25-year-old female with suspected meningitis presents in shock and is admitted to ITU. She has a
central line placed within the subclavian vein and she has a urinary catheter. Her urine output tailors off in
the proceeding 24 hours to 14mls / hr. Which of the following statements on glomerular filtration is
correct? Single best answer - select one answer only.
ADH acts on the proximal tubule YOUR ANSWER
Allows the passage of glucose into the ultrafiltrate in the normal individual CORRECT ANSWER
Depends on venous return
Is normally governed by the blood pressure
Usually takes place at 24 ml/min.

In the normal individual the glomerular filtration rate is kept fairly constant at 120 ml/min. The ultrafiltrate
is essentially plasma without the proteins (these are not able to cross the basement membrane of the
glomerulus). The GFR is governed by the rate of glomerular blood flow, which is governed by the tone of
BOTH the afferent and efferent arterioles. This is kept constant over a range of blood pressures. Glucose
is able to cross the glomerular basement membrane into the ultrafiltrate. In normal individuals it is then
actively re-absorbed. The resorption mechanism has a maximal rate. If the glucose concentration of the
serum, and consequently the ultrafiltrate is too high, then the mechanism is overwhelmed and glycosuria
occurs (as occurs in diabetes mellitus).
---------------------------------------------------------------------------------------------------------------------------------------------------------------------------------------------------------------
Dr Mohammed Shamsul Islam Khan, Medical Officer, Clinical Neuro-Surgery, National Institute of Neuro-Sciences and Hospital
Sher-E-Bangla Nagar, Dhaka-1207, Bangladesh. Mobile: +880 1713 455 662, +880 1685 811979. E-mail: drsikhan@gmail.com
MyPasTest MRCS A Online - Jan Exam 2015
10. Physiology; System Specific Physiology (399Qs)
----------------------------------------------------------------------------------------------------------------------------------

147. A 72-year-old female has been noted at the diabetes clinic to have increasingly poor glycaemic
control despite metformin, gliclazide and dietary advice. It is decided to commence insulin to improve her
glycaemic control. Which of the following cell membranes is insulin effective in facilitating the diffusion of
glucose across? Single best answer - select one answer only.
Brain cells YOUR ANSWER Page |
Cardiac muscle cells CORRECT ANSWER 904
Large bowel mucosa
Renal tubular epithelium
Small bowel mucosa.

Glucose will not diffuse through a cell membrane against a concentration gradient. Insulin facilitates the
diffusion of glucose across many cell membranes including cardiac muscle cells, skeletal muscle cells,
some smooth muscle cells and adipose tissue cells. Insulin does not affect transport of glucose into brain
cells, renal tubular epithelium or across intestinal mucosa.

148. Lactation does not occur during pregnancy because the action of prolactin is blocked by increased
concentrations of which of the following hormones? Single best answer question choose ONE true
option only.
Growth hormone YOUR ANSWER
Human chorionic somatomammotrophin
Insulin
Progesterone CORRECT ANSWER
Thyroxine.

Progesterone inhibits lactation during pregnancy. The fall in serum progesterone with passage of the
placenta seems to be the important event in establishment of lactogenesis. Furthermore, progesterone
administration inhibits casein and -lactalbumin synthesis in vitro. Once lactogenesis is initiated,
prolactin appears to be the key hormone in maintenance of milk synthesis (galactopoiesis) and secretion.
Oestrogen and progesterone or even ovariectomy have no effect on lactation once it has been initiated.

149. A 14-year-old boy has generally well controlled asthma but needs to use his inhaler prior to playing
football. At present he only uses a salbutamol inhaler. Which of the following adrenoceptor subtypes
does salbutamol act mainly on? Single best answer - select one answer only.
1 YOUR ANSWER
2
1
2 CORRECT ANSWER
3.

---------------------------------------------------------------------------------------------------------------------------------------------------------------------------------------------------------------
Dr Mohammed Shamsul Islam Khan, Medical Officer, Clinical Neuro-Surgery, National Institute of Neuro-Sciences and Hospital
Sher-E-Bangla Nagar, Dhaka-1207, Bangladesh. Mobile: +880 1713 455 662, +880 1685 811979. E-mail: drsikhan@gmail.com
MyPasTest MRCS A Online - Jan Exam 2015
10. Physiology; System Specific Physiology (399Qs)
----------------------------------------------------------------------------------------------------------------------------------
Adrenaline exerts its effects by stimulation of 1 and 2 receptors: it increases the force (inotropic) and
rate (chronotropic) of contraction of the heart; 1 effects adrenaline dilates blood vessels in skeletal
muscle and liver; 2 effects splanchnic vasoconstriction occurs with administration of adrenaline
through its effect (small). Decreased bowel motility is seen with adrenaline, mediated through and 2
effects. Bronchodilatation is produced through relaxation of the bronchial muscle (2 effect).

150. Which one of the following is higher at the apex of the lung than at the base when a person is
Page |
standing? Single best answer question choose ONE true option only. 905
V/Q ratio YOUR ANSWER (Correct Answer)
Ventilation
PaCO2
Compliance
Blood flow.

The alveoli at the apex of the lung are larger than those at the base so their compliance is less. Because
of the reduced compliance, less inspired gas goes to the apex than to the base. Also, because the apex is
above the heart level, less blood flows through the apex than through the base. However, the reduction in
air flow is less than the reduction in blood flow, so that the V/Q ratio at the top of the lung is greater than
it is at the bottom. The increased V/Q ratio at the apex makes PaCO2 lower and PaO2 higher at the apex
than they are at the base.

151. You are asked to give a short description of the normal function of the colon to a medical student.
Which of the following best describes the colon? Single best answer - select one answer only.
Absorption of sodium and water YOUR ANSWER (Correct Answer)
Absorption of bicarbonate
Absorption of bile salts
Secretion of sodium
Secretion of chloride.

The ascending and transverse colon are involved in the regulation ofintraluminal fluid volume as well as
sodium and water absorption. The left colon is the site for final modulation of intraluminal contents
beforeevacuation. More than 90% of short-chain fatty acids are taken up bycolonic mucosal cells.
Chloride absorption occurs by a passive energy-independent mechanism, being absorbed together with
sodium. Bicarbonate is secreted by the colon, by an energy-requiring process. Bilesalts are absorbed in
the terminal ileum.

152. On reviewing a patient's pre-operative lung function tests you note an increased functional residual
capacity (FRC). What can increase functional residual capacity (FRC)? Select one answer only.
Emphysema YOUR ANSWER (Correct Answer)
Fibrosing alveolitis
---------------------------------------------------------------------------------------------------------------------------------------------------------------------------------------------------------------
Dr Mohammed Shamsul Islam Khan, Medical Officer, Clinical Neuro-Surgery, National Institute of Neuro-Sciences and Hospital
Sher-E-Bangla Nagar, Dhaka-1207, Bangladesh. Mobile: +880 1713 455 662, +880 1685 811979. E-mail: drsikhan@gmail.com
MyPasTest MRCS A Online - Jan Exam 2015
10. Physiology; System Specific Physiology (399Qs)
----------------------------------------------------------------------------------------------------------------------------------
General anaesthesia
Obesity
Recent abdominal surgery.

FRC represents the volume of the lungs at the end of a normal tidal breath. It is reduced intraoperatively Page |
because of the increased activity of muscles of respiration and elevation of the diaphragm. Pain and
splinting of the diaphragm occur postoperatively in abdominal surgery, reducing it. Fibrosing alveolitis is 906
a restrictive lung disease and decreases FRC, emphysema decreased lung recoil and increases it.

153. You are attending a trauma call for a patient who has sustained a neck injury. What level of cervical
spine injury would you expect to result in significant problems with ventilation? Select one answer only.
C3 YOUR ANSWER (Correct Answer)
C4
C5
C6
C7.

Expiration is passive and results from the elastic recoil of the lungs, unless forced, when it is brought
about by contraction of the intercostals and the generation of an increased intra-abdominal pressure.
Inspiration is usually active. In normal inspiration contraction of the diaphragm is the most important
generator of negative intrathoracic pressure, which results in air inflow. The diaphragm is innervated by
the phrenic nerve (C3, 4, 5). The diaphragm can be damaged, resulting in respiratory difficulty in high
spinal cord injuries. The intercostal muscles also assist in normal inspiration. The accessory muscles of
ventilation (scalenus anterior and sternomastoid) are only utilised when there is respiratory
embarrassment and inspiration is laboured.

154. You attend a trauma call to a 25-year-old patient who has a liver laceration and is in class III
haemorrhagic shock. What physiological effects would haemorrhagic shock cause? Select one answer
only.
A decreased ventilationperfusion mismatch YOUR ANSWER
Decreased atrial pressure and subsequent reduction in antidiuretic hormone (ADH) secretion
Increased catecholamine secretion by the adrenal cortex
Increased tidal ventilation CORRECT ANSWER
The oxygenhaemoglobin dissociation curve to shift to the left.

Catecholamines are secreted by the adrenal medulla, and a decrease in atrial pressure stimulates ADH
secretion by the HenryGauer reflex. Decreased PaO2 stimulates chemoreceptors, and with increased
ventilation (tidal ventilation) and reduced cardiac output there is an increase in V / Q mismatch. The
oxygenhaemoglobin dissociation curve shifts to the right when there is a decrease in pH (ie acidosis), or
an increase in pCO2 , temperature or 2,3-DPG.

---------------------------------------------------------------------------------------------------------------------------------------------------------------------------------------------------------------
Dr Mohammed Shamsul Islam Khan, Medical Officer, Clinical Neuro-Surgery, National Institute of Neuro-Sciences and Hospital
Sher-E-Bangla Nagar, Dhaka-1207, Bangladesh. Mobile: +880 1713 455 662, +880 1685 811979. E-mail: drsikhan@gmail.com
MyPasTest MRCS A Online - Jan Exam 2015
10. Physiology; System Specific Physiology (399Qs)
----------------------------------------------------------------------------------------------------------------------------------
155. A 27-year-old male is rushed in the emergency department after being stabbed in the neck. He is
bleeding very heavily and there is concern he is haemodynamically compromised due to the bleeding and
the massive haemorrhage protocol is therefore activated. Which of the following combinations most
accurately describe the physiological response to a loss of circulating blood volume? Select one answer
only.
Increased peripheral venular vasoconstriction, increased peripheral arteriolar vasoconstriction, increased Page |
resistance to blood flow, haemodilution, bradycardia YOUR ANSWER
907
Reduced peripheral venularvasoconstriction,reduced peripheral arteriolar vasoconstriction, reduced
resistance to blood flow, haemodilution, bradycardia
Increased peripheral venular vasoconstriction, increased peripheral arteriolar vasoconstriction,increased
resistance to blood flow, haemodilution, tachycardia CORRECT ANSWER
Reduced peripheral venular vasoconstriction, reduced peripheral arteriolar vasoconstriction,increased
resistance to blood flow, haemoconcentration, bradycardia
Increased peripheral venularvasoconstriction, reduced peripheral arteriolar vasoconstriction, increased
resistance to blood flow, haemodilution ,bradycardia..

Vasoconstriction is widespread and occurs on both the venous and arterial sides of the circulation. The
result is increased peripheral resistance toblood flow. Changes in vascular pressures lead to an influx of
extra cellular water into the circulation, a phenomenon known as transcapillary refilling. This leads to
haemodilution. Tachycardia occurs as the bloodpressure falls, which is sensed by baroreceptors in the
carotid arteries andaortic arch.

156. An independence of the P-waves and the QRS complexes of the electrocardiogram (ECG) indicates:
Single best answer question choose ONE true option only.
A conduction block in the left bundle branch YOUR ANSWER
A failure of the atrioventricular (AV) node to conduct CORRECT ANSWER
An early repolarisation of ventricular fibres
Depression of the sinoatrial node
Slowing of conduction at the atrioventricular node.

A heart block can be a blockage at any level of the electrical conduction system of the heart. Blocks that
occur within the sinoatrial (SA) node are described as SA nodal blocks. Blocks that occur within the AV
node are described as AV nodal blocks. Blocks that occur below the AV node are known as infra-Hisian
blocks (named after the bundle of His). Clinically speaking, most of the important heart blocks are AV
nodal blocks and infra-Hisian blocks. Complete failure of the AV node to conduct results in third-degree
heart block, also known as complete heart block or third-degree AV block.

It is a defect of the electrical system of the heart, in which the impulse generated in the SA node does not
propagate to the ventricles. Because the impulse is blocked, an accessory pacemaker below the level of
the block will typically activate the ventricles. This is known as an escape rhythm. Since this accessory
pacemaker activates independently of the impulse generated at the SA node, two independent rhythms
can be noted on the ECG. The first will activate the atria and create the P-waves, typically with a regular P
---------------------------------------------------------------------------------------------------------------------------------------------------------------------------------------------------------------
Dr Mohammed Shamsul Islam Khan, Medical Officer, Clinical Neuro-Surgery, National Institute of Neuro-Sciences and Hospital
Sher-E-Bangla Nagar, Dhaka-1207, Bangladesh. Mobile: +880 1713 455 662, +880 1685 811979. E-mail: drsikhan@gmail.com
MyPasTest MRCS A Online - Jan Exam 2015
10. Physiology; System Specific Physiology (399Qs)
----------------------------------------------------------------------------------------------------------------------------------
to P interval. The second will activate the ventricles and produce the QRS complex, typically with a
regular R to R interval. The PR interval will be variable, as the hallmark of complete heart block is no
apparent relationship between P-waves and QRS complexes.

157. A 67-year old man presents with diarrhoea and abdominal pain. Which of the following symptoms or
signs, in addition to his presentation, is most likely to go with a diagnosis of carcinoid syndrome? Single Page |
best answer select one answer only.
Alternating constipation YOUR ANSWER 908
Dysmenorrhoea
Flushing CORRECT ANSWER
Mitral regurgitation
Nausea.

The symptoms of carcinoid syndrome are:


01. Flushing
02. Telangiectasia
03. Diarrhea
04. Bronchospasm
05. right heart valve lesions (tricuspid regurgitation and stenosis, pulmonary stenosis)
pellagra (rarely)

158. The cardiac index is equal to..............Single best answer question choose ONE true option only.
Blood pressure x heart rate YOUR ANSWER
Cardiac output / body surface area CORRECT ANSWER
Cardiac output / height
Cardiac output x peripheral resistance
Stroke volume x heart rate.

The cardiac index is a measure that allows cardiac output to be compared between patients of differing
size for direct comparison. It is measured in L/min/m2, and usually ranges from 2.5-3.5. Stroke volume x
heart rate is equal to the cardiac output, and cardiac output x peripheral resistance equals the blood
pressure. Cardiac output/ height and blood pressure x heart rate are not useful measurements.

159. Spontaneous respiration ceases after? Single best answer question choose ONE true option only.
Transection of the brain stem above the pons YOUR ANSWER
Transection of the brain stem at the caudal end of the medulla CORRECT ANSWER
Bilateral vagotomy
Bilateral vagotomy combined with transection of the brain stem at the superior border of pons
Transection of the spinal cord at the level of first thoracic segment.

---------------------------------------------------------------------------------------------------------------------------------------------------------------------------------------------------------------
Dr Mohammed Shamsul Islam Khan, Medical Officer, Clinical Neuro-Surgery, National Institute of Neuro-Sciences and Hospital
Sher-E-Bangla Nagar, Dhaka-1207, Bangladesh. Mobile: +880 1713 455 662, +880 1685 811979. E-mail: drsikhan@gmail.com
MyPasTest MRCS A Online - Jan Exam 2015
10. Physiology; System Specific Physiology (399Qs)
----------------------------------------------------------------------------------------------------------------------------------
Rhythmic discharge of neurons in the medulla and pons produces automatic respiration, and so
transection of the brain stem below the medulla stops the respiration. The respiratory muscles are
supplied by C3 C5 segments (diaphragm) and thoracic segments for intercostal & flat muscles of the
trunk.

160. A 56-year-old diabetic and hypertensive woman suffered a haemorrhagic stroke affecting the primary Page |
somatic sensory cortex. Where is the primary somatic sensory cortex located? Single best answer
question choose ONE true option only.
909
Central sulcus YOUR ANSWER
Occipital lobe
Precentral gyrus
Postcentral gyrus CORRECT ANSWER
Sylvian fissure.

The primary somatic sensory (S1) cortex in the postcentral gyrus is the first and largest cortical receiving
area for the analysis of information from somatosensory receptors. It receives and analyses information
from the ventrobasal (VPL + VPM) nucleus of the thalamus.

S1 cortex is necessary for the conscious awareness:


(1) that a stimulus has occurred;
(2) of stimulus quality;
(3) of stimulus location;
(4) of stimulus amplitude; and
(5) of stimulus duration.

The S1 cortex is not, however, the 'end station' in the somatic sensory pathway. S1 cortex sends
information, via corticocortical fibres, to other areas of the neocortex. Further important analysis of
somatosensory information occurs in the posterior parietal association cortex.

161. Which of the following physiological characteristics relates to the lining of the respiratory tract?
Single best answer question choose ONE true option only.
About 1 litre of mucus is produced every day YOUR ANSWER
The cilia are under the control of a physiological motor, dynein CORRECT ANSWER
The mucociliary escalator moves at 0.2 cm/minute
The bronchioles have cartilage in their wall
The bronchioles have diameters up to 5 mm.
About 100 ml of mucus is produced every day. The cilia are under the control of a physiological motor,
dynein (which is absent in Kartageners syndrome). The mucociliary escalator moves at 2 cm/minute. The
bronchioles do not have cartilage in their wall (which distinguishes them from bronchi). The bronchioles
can be up to 1 mm in diameter.

---------------------------------------------------------------------------------------------------------------------------------------------------------------------------------------------------------------
Dr Mohammed Shamsul Islam Khan, Medical Officer, Clinical Neuro-Surgery, National Institute of Neuro-Sciences and Hospital
Sher-E-Bangla Nagar, Dhaka-1207, Bangladesh. Mobile: +880 1713 455 662, +880 1685 811979. E-mail: drsikhan@gmail.com
MyPasTest MRCS A Online - Jan Exam 2015
10. Physiology; System Specific Physiology (399Qs)
----------------------------------------------------------------------------------------------------------------------------------
162. Which of the following hormones exerts the least effect on growth? Single best answer question
choose ONE true option only.
Growth hormone YOUR ANSWER
Testosterone
T4 Page |
Insulin 910
Vasopressin CORRECT ANSWER.

Growth is stimulated by growth hormone which stimulates IGF-I secretion, androgens and estrogens
initially stimulate the growth, they then ultimately terminate the growth by causing the epiphysis to fuse
to the long bones. Thyroid hormones have a permissive effect to the action of growth hormone possibly
via somatomedines. Insulin promotes growth as it has an anabolic effect on protein metabolism.
Vasopressin has no effect on growth.

163. A 22-year-old man is admitted to accident and emergency having sustained a gun shot injury to his
abdomen. He has lost a significant amount of blood and is being stabilised prior to surgery. Which of the
following regarding renal blood flow is correct? Single best answer - select one answer only.
It can be estimated by para-aminohippuric acid (PAH) clearance YOUR ANSWER (Correct Answer)
It decreases in hypercarbia
It increases in hypovolaemic shock
Renal plasma flow is given by the Starlings equation
The kidneys receive roughly 75% of cardiac outflow.

At rest, the kidney receives 25% of the cardiac output, approx 1.2 litres of blood/minute. Renal plasma
flow can be measured by infusing PAH and determining its urine and plasma concentrations. PAH is
filtered by the glomeruli and secreted by the renal tubular cells, so its excretion ratio is high. The effective
renal plasma flow is equivalent to the urine clearance of PAH.

Blood flow in the renal cortex is much higher than in the medulla.The factors affecting renal blood flow
include: blood volume (hypovolaemia reduces renal blood flow) and catecholamines (constrictrenal
vessels so reducing flow). ADH causes water retention, the effect of which is to increase renal blood flow.
164. You are on an ITU ward round when the consultant asks you how best to measure renal function. Of
the following, which can be best used to measure glomerular filtration rate (GFR)? Single best answer -
select one answer only.
ADH YOUR ANSWER
Glucose
Inulin CORRECT ANSWER
Insulin
---------------------------------------------------------------------------------------------------------------------------------------------------------------------------------------------------------------
Dr Mohammed Shamsul Islam Khan, Medical Officer, Clinical Neuro-Surgery, National Institute of Neuro-Sciences and Hospital
Sher-E-Bangla Nagar, Dhaka-1207, Bangladesh. Mobile: +880 1713 455 662, +880 1685 811979. E-mail: drsikhan@gmail.com
MyPasTest MRCS A Online - Jan Exam 2015
10. Physiology; System Specific Physiology (399Qs)
----------------------------------------------------------------------------------------------------------------------------------
Para-amino hippuric acid (PAH).

GFR can be measured using the relationship GFR = UV/P for any substrate that is freely filtered and not
reabsorbed or secreted (UV is the amount of urine per minute and P is the concentration in plasma). Inulin
fulfils those criteria. Clinically GFR is estimated using creatinine. Glucose is reabsorbed and PAH is
secreted and therefore they are so not suitable for the measurement of GFR. Page |
911
165. You are called to see a post-operative patient in the high dependency unit, who underwent an
elective abdominal aortic aneurysm repair 36 hours ago. The patient has a reduced cardiac output. Which
of the following could cause a reduction in cardiac output? Single best answer - select one answer only.
Acute hypoxia YOUR ANSWER (Correct Answer)
Alkalosis
Hypervolaemia
Systemic vasodilatation
Tachycardia.

The Starling curve demonstrates the relationship between cardiac performance (stroke volume) and
cardiac filling (preload). Cardiac performance improves with increased filling until the heart becomes
overfull and stretched. Thus hypovolaemia (reduced filling) will adversely influence cardiac output.
Similarly, cardiac performance will improve with better myocardial contractility. Both acidosis and
hypoxia will cause impaired contractility and thus adversely influence cardiac output.

Cardiac Output = Heart Rate x Stroke Volume.


Thus tachycardia will improve cardiac output.

Cardiac Output = [Mean Arterial Pressure - Central Venous Pressure] / Systemic Vascular Resistance.
Systemic vasodilatation is reflected by a reduced systemic vascular resistance and will therefore cause
an increased cardiac output.

166. Aldosterone is a steroid hormone that is involved in the regulation of blood pressure. Which of the
follow statements regarding aldosterone is correct? Single best answer - select one answer only.
It is secreted in excess in Addisons disease YOUR ANSWER
In excess results in hyperkalemia and metabolic acidosis
It is produced by the zonareticularis of the adrenal gland
Its secretion is inhibited in Conns Disease
It is produced in response to increased levels of angiotensin II CORRECT ANSWER.

Aldosterone is a steroid hormone produced in the zona glomeruli adrenal cortex. Its main effect is to
promote water and sodium resorption in response to diminished circulating volume. A decrease in
circulating volume is detected by diminished blood flow and hypoxia at the juxtaglomerular apparatus of

---------------------------------------------------------------------------------------------------------------------------------------------------------------------------------------------------------------
Dr Mohammed Shamsul Islam Khan, Medical Officer, Clinical Neuro-Surgery, National Institute of Neuro-Sciences and Hospital
Sher-E-Bangla Nagar, Dhaka-1207, Bangladesh. Mobile: +880 1713 455 662, +880 1685 811979. E-mail: drsikhan@gmail.com
MyPasTest MRCS A Online - Jan Exam 2015
10. Physiology; System Specific Physiology (399Qs)
----------------------------------------------------------------------------------------------------------------------------------
the nephron. This results in the release of renin into the bloodstream. Renin catalyses the transformation
of angiotensinogen into angiotensin I in the kidney.

This angiotensin I is then converted into angiotensin II as it is transported through the lungs, by
angiotensin converting enzyme (ACE). This angiotensin II stimulates the adrenal cortex to secrete the
mineralocorticoid aldosterone. Excess secretion of aldosterone occurs in primary hyperaldosteronism
and Conns syndrome. These are characterised by hypokalemia and metabolic alkalosis. A deficiency of
Page |
aldosterone, as occurs in Addisons disease, can produce hyperkalemia and mild acidosis. 912
167. You are called to see one of your ward patients who is hyperventilating. What physiological effects
does hyperventilation cause? Select one answer only.
Decreased blood flow to the brain YOUR ANSWER (Correct Answer)
Decrease in the basal metabolic rate
Increased serum calcium levels
Increased CO2 levels
Respiratory acidosis.

Hyperventilation leads to a fall in the blood CO2 levels and so respiratory alkalosis. The fall in the blood
CO2 levels leads to a reduction in the blood flow to the brain, causing dizzy spells, agitation, confusion,
hallucinations and sometimes collapse. The patient becomes very anxious and breathes through the
mouth, which leads to dryness of the mouth. Hyperventilation results in a fall in serum calcium levels;
this causes tingling and numbness in the arms, perioral tingling, muscle twitching and spasms or cramps
in the arms and legs. Hyperventilation stems from over activity of the muscles of respiration, particularly
the intercostal muscles, which soon leads to muscle fatigue (not when the patient is ventilated). This
causes the patient to feel as being unable to breathe. The basal metabolic rate of the patient increases
during hyperventilation.

168. A 28-year-old man presents to A and E in haemorrhaging shock having sustained an open tibial
fracture. Which of the following statements concerning blood volume is correct? Single best answer -
select one answer only.
It is around 2.5 litres in a 70kg man YOUR ANSWER
It is increased by diuretics
It is reduced by renin
It equals the volume of plasma plus the volume of red blood cells CORRECT ANSWER
Males have less blood volume than females.
Cardiac output (not blood volume) = heart rate stroke volume. Blood volume is around 5 litres in a
typical 70 kg man. The blood volume is decreased by diuretics. Renin is involved in the formation of
angiotensin 2 and ultimately in the secretion of aldosterone. Aldosterone will increase the intravascular
and so the blood volume, through the retention of sodium.
169. For which of the following substances would you expect the renal clearance to be the lowest, under
normal conditions? Single best answer question choose ONE true option only.

---------------------------------------------------------------------------------------------------------------------------------------------------------------------------------------------------------------
Dr Mohammed Shamsul Islam Khan, Medical Officer, Clinical Neuro-Surgery, National Institute of Neuro-Sciences and Hospital
Sher-E-Bangla Nagar, Dhaka-1207, Bangladesh. Mobile: +880 1713 455 662, +880 1685 811979. E-mail: drsikhan@gmail.com
MyPasTest MRCS A Online - Jan Exam 2015
10. Physiology; System Specific Physiology (399Qs)
----------------------------------------------------------------------------------------------------------------------------------
Urea YOUR ANSWER
Creatinine
Sodium
Glucose CORRECT ANSWER
Page |
Water.
913
Under normal conditions the renal clearance of glucose is zero, since glucose is completely reabsorbed
in the renal tubules and not excreted.

170. You are reviewing a patient on the ward who underwent a hemiarthroplasty for a fractured neck of
femur 2 hours ago. The patient is hypotensive. The hypotension is most likely to be caused by which of
the following? Single best answer - select one answer only.
Narcotics producing vasoconstriction YOUR ANSWER
Narcotics producing myocardial depression
Pain
Regional anaesthesia producing vasoconstriction by increasing sympathetic outflow
Regional anaesthesia producing vasodilation by blocking sympathetic outflow CORRECT ANSWER.

Narcotics produce their hypotensive effect via a vagally induced bradycardia, and spinal and epidural
anaesthesia produces hypotension via a vasodilator effect from blocking sympathetic outflow. Pain
results in hypertension.

171. A 32-year-old man is admitted to accident and emergency having sustained a stab injury to his
abdomen. He has lost a significant amount of blood and is being stabilised prior to surgery. Which of the
following regarding renal blood flow is correct? Single best answer - select one answer only.
Autoregulation is unimpeded by shock, sepsis and low cardiac states YOUR ANSWER
Renal blood flow is higher in the medulla than in the cortex
Renal blood flow is unaffected by hypertension
Renal blood flow is normally kept constant by autoregulation CORRECT ANSWER
Renal blood flow may be increased by 50% in acute renal failure.

At normal systemic blood pressure the renal vascular resistance adjusts the pressure in the renal
arterioles so that renal blood flow remains fairly constant (autoregulation). In hypertensive patients there
is a shift of autoregulation, so that they have a slightly reduced renal blood flow for a given blood
pressure. Blood flow in the cortex is much higher than in the medulla. Autoregulation is impaired in
shock, sepsis and low cardiac output states.

172. Normal Cerebrospinal fluid (CSF): Single best answer question choose ONE true option only.
---------------------------------------------------------------------------------------------------------------------------------------------------------------------------------------------------------------
Dr Mohammed Shamsul Islam Khan, Medical Officer, Clinical Neuro-Surgery, National Institute of Neuro-Sciences and Hospital
Sher-E-Bangla Nagar, Dhaka-1207, Bangladesh. Mobile: +880 1713 455 662, +880 1685 811979. E-mail: drsikhan@gmail.com
MyPasTest MRCS A Online - Jan Exam 2015
10. Physiology; System Specific Physiology (399Qs)
----------------------------------------------------------------------------------------------------------------------------------
Is produced within arachnoid granulations YOUR ANSWER
Has an identical composition to that of plasma
Has a protein content that is 0.5% that of plasma CORRECT ANSWER
Has a higher potassium content than that of plasma
Page |
pH is heavily buffered.
914
Most of the CSF is produced by the choroid plexus, which is situated in the lateral, 3 rd and 4th ventricles.
CSF is absorbed directly into the cerebral venous sinuses through the arachnoid villi, or granulations, by
a process known as mass or bulk flow.

The composition of CSF is different to plasma. Of importance to mention are the concentrations of K +,
Ca2+, bicarbonate and protein which are lower in CSF than in plasma. This is to prevent high
concentrations of these electrolytes inadvertently exciting neurones present within the brain substance.
The potassium content of the CSF in this respect is particularly important. Further buffering of the K +
content of CSF takes place through astrocytes.

Likewise, the low protein content of the CSF (the CSF protein content is 0.5% that of plasma) is deliberate
to prevent some proteins and amino acids acting as false neurotransmitters. The CSF is more acidic
than plasma because pH of the CSF plays a critical role in the regulation of pulmonary ventilation and
cerebral blood flow. Another reason why the CSF protein is kept deliberately low is to prevent proteins
buffering pH. The result is that the pH of the CSF accurately reflects carbon dioxide levels of the blood. In
this way changes in pH act as a powerful regulator of the respiratory system (through the action of pH on
central chemoreceptors) and on cerebral blood flow.

173. Intermittent positive pressure ventilation (IPPV) has which one of the following physiological
effects?Single best answer - select one answer only.
an increase in mean airway pressure YOUR ANSWER (Correct Answer)
an increase in the work of breathing
an increase in urinary output
Preservation of the patients inspiratory/expiratory ratio
Reduction of dead space.
Intermittent positive pressure ventilation (IPPV) causes an increase in pulmonary vascular resistance and
decreases preload. IPPV increases antidiuretic hormone (ADH) secretion and decreases cardiac output,
reducing renal blood flow and urine output.

174. An overweight 32-year-old woman presents with a short history of painless jaundice. There is no
previous history of illness and, apart from the jaundice, she has no signs of chronic liver disease. Initial
investigations reveal a haemoglobin of 12.7 g/dl, MCV 105 fl, serum bilirubin 162 mol/l, AST 145 U/l,

---------------------------------------------------------------------------------------------------------------------------------------------------------------------------------------------------------------
Dr Mohammed Shamsul Islam Khan, Medical Officer, Clinical Neuro-Surgery, National Institute of Neuro-Sciences and Hospital
Sher-E-Bangla Nagar, Dhaka-1207, Bangladesh. Mobile: +880 1713 455 662, +880 1685 811979. E-mail: drsikhan@gmail.com
MyPasTest MRCS A Online - Jan Exam 2015
10. Physiology; System Specific Physiology (399Qs)
----------------------------------------------------------------------------------------------------------------------------------
alkaline phosphatase 224 U/l, gamma-glutamyltransferase 200 U/l. Which of the following is the most
likely diagnosis? Single best answer question choose ONE true option only.
Alcoholic liver disease YOUR ANSWER (Correct Answer)
Autoimmune chronic hepatitis
Carcinoma of the head of the pancreas Page |
Cholecystitis 915
Hepatitis A infection.

Jaundice with an elevation of both AST and alkaline phosphatase suggests mixed hepatocellular
damage and cholestatic liver disease, typical of acute alcoholic hepatitis on a background of chronic
liver disease (and is not excluded by the lack of physical signs). The high gamma-glutamyltransferase
lends support to this (although it may be increased in liver disease of any cause). Macrocytosis is typical
of chronic excessive alcohol intake and is not a feature of the other conditions; although were it not
present, autoimmune liver disease would need to be considered.
In hepatitis A, AST is typically higher than alkaline phosphatase, while the reverse is true of pancreatic
carcinoma. Chronic cholecystitis can cause jaundice but it would be unusual for there to be no history of
acute episodes.

175. The symptoms of dumping syndrome in patients with intestinal short circuits such as anastomosis
of the jejunum to the stomach are caused by? Single best answer question choose ONE true option
only.
Increased blood pressure YOUR ANSWER
Increase secretion of glucagons
Increased secretion of CCK
Hypoglycemia CORRECT ANSWER
Hyperglycemia.

There are two types of dumping syndromes resulting from rapid gastric emptying;
1. Early dumping which manifests by dizziness, sweating and palpitations within 5-45 minutes after
eating.
2. Late dumping occurs 2-4 hours after eating and is due to rebound hypoglycemia, small meals and
glucose help to improve symptoms

176. A 58-year-old man presents with jaundice and itch. Which of the following clinical and biochemical
characteristics of hepatic jaundice is correct? Single best answer - select one answer only.
Dark urine YOUR ANSWER (Correct Answer)
Dark stool
Serum bilirubin is conjugated

---------------------------------------------------------------------------------------------------------------------------------------------------------------------------------------------------------------
Dr Mohammed Shamsul Islam Khan, Medical Officer, Clinical Neuro-Surgery, National Institute of Neuro-Sciences and Hospital
Sher-E-Bangla Nagar, Dhaka-1207, Bangladesh. Mobile: +880 1713 455 662, +880 1685 811979. E-mail: drsikhan@gmail.com
MyPasTest MRCS A Online - Jan Exam 2015
10. Physiology; System Specific Physiology (399Qs)
----------------------------------------------------------------------------------------------------------------------------------
Serum transaminase is normal
Serum alkaline phosphatase is mildly decreased.

The three types of jaundice and their biochemical and clinical differences:
Prehepatic Hepatic Posthepatic Page |
Jaundice Usually mild Variable Variable often deep 916
Colour of urine Normal Dark Dark
Colour of faeces Normal Normal Pale
Serum bilirubin Unconjugated Unconjugated + conjugated Conjugated
Serum transaminases Normal Grossly increased Normal or mild increase
Serum alkaline phosphatase Normal Mild elevation Grossly elevated

177. While reviewing lung function tests on a pre-operative patient in clinic you note a decreased
pulmonary functional residual capacity (FRC). What underlying condition could cause a decrease in FRC?
Select one answer only.
Asthma YOUR ANSWER
Chronic bronchitis
Emphysema
Pneumothorax
Pulmonary fibrosis CORRECT ANSWER.

FRC is the volume of gas in the lung after a normal expiration. It can be calculated from the expiratory
reserve volume plus the residual volume. FRC can be measured by the helium dilution technique or body
plethysmography. FRC is often increased in diseases where there is increased airway resistance, such as
emphysema, chronic bronchitis and asthma. A reduced FRC is seen in patients with reduced lung
compliance, as in diffuse interstitial fibrosis.

178. A defect in which part of the renal tubule will affect absorption of amino acids and glucose the most?
Single best answer question choose ONE true option only.
Collecting duct YOUR ANSWER
Collecting tubule
Distal convoluted tubule
Loop of Henle
Proximal convoluted tubule CORRECT ANSWER.
Glucose, amino acids, inorganic phosphate and some other solutes are reabsorbed via secondary active
transport in the proximal renal tubule through co-transport channels driven by the sodium gradient out of
the nephron.
---------------------------------------------------------------------------------------------------------------------------------------------------------------------------------------------------------------
Dr Mohammed Shamsul Islam Khan, Medical Officer, Clinical Neuro-Surgery, National Institute of Neuro-Sciences and Hospital
Sher-E-Bangla Nagar, Dhaka-1207, Bangladesh. Mobile: +880 1713 455 662, +880 1685 811979. E-mail: drsikhan@gmail.com
MyPasTest MRCS A Online - Jan Exam 2015
10. Physiology; System Specific Physiology (399Qs)
----------------------------------------------------------------------------------------------------------------------------------

179. A middle-aged patient following a stroke developed dysarthria, nystagmus and a tremor that worsens
with directed movement. This patient most probably has: Single best answer question choose ONE true
option only.
Cerebellar disease YOUR ANSWER (Correct Answer)
Damage to pontine and caudate nuclei
Page |
Hyperthyroidism 917
Parkinsonism
Spinal cord transaction.

The patient in this vignette has cerebellar tremor as suggested by the signs and symptoms. Cerebellar
tremor is a slow, broad tremor of the extremities that occurs at the end of a purposeful movement, such
as trying to press a button or touching a finger to the tip of one's nose. Cerebellar tremor is caused by
lesions in or damage to the cerebellum resulting from stroke, tumour or disease such as multiple
sclerosis or some inherited degenerative disorder. It can also result from chronic alcoholism or overuse
of some medicines.

In classic cerebellar tremor, a lesion on one side of the brain produces a tremor in that same side of the
body that worsens with directed movement. Cerebellar damage can also produce a 'wing-beating' type of
tremor called rubral or Holmes' tremor - a combination of rest, action and postural tremors. The tremor is
often most prominent when the affected person is active or is maintaining a particular posture. Cerebellar
tremor may be accompanied by dysarthria (speech problems), nystagmus (rapid, involuntary rolling of the
eyes), gait problems and postural tremor of the trunk and neck.

180. If the end-diastolic ventricular volumes are increased (within physiological limits): Single best
answer question choose ONE true option only.
Cardiac output would be decreased YOUR ANSWER
The force of cardiac contraction would be decreased
The output of the right ventricle would exceed the output of the left ventricle
The stroke volume would be increased CORRECT ANSWER
Venous pressure would be decreased.

If a strip of muscle is passively stretched, it will exhibit an increasing amount of tension disproportionate
to the amount of stretch. This passive tension is related to non-contractile elements that comprise the
series and parallel elastic components in the muscle fibre. At any given length, if the muscle is stimulated
to contract isometrically (ie the muscle is not allowed to shorten), it will develop an additional amount of
tension, which is a function of the initial fibre length.
At lengths shorter or longer than this, developed tension (active) decreases. At lengths in which there is
little or no overlap of actin and myosin, active tension is zero, even though there may still be passive
tension. These principles were first promulgated for skeletal muscle and later applied to cardiac muscle
by O. Frank in 1895 for the frog heart, followed (1914) by Starling for the dog heart-lung preparation. The
concept, known as Starling's Law of the Heart, relates that, within physiological limits, the force
---------------------------------------------------------------------------------------------------------------------------------------------------------------------------------------------------------------
Dr Mohammed Shamsul Islam Khan, Medical Officer, Clinical Neuro-Surgery, National Institute of Neuro-Sciences and Hospital
Sher-E-Bangla Nagar, Dhaka-1207, Bangladesh. Mobile: +880 1713 455 662, +880 1685 811979. E-mail: drsikhan@gmail.com
MyPasTest MRCS A Online - Jan Exam 2015
10. Physiology; System Specific Physiology (399Qs)
----------------------------------------------------------------------------------------------------------------------------------
generated by the contracting heart, as reflected by cardiac output or stroke volume, is increased if the
muscle fibres have been stretched previously.

If cardiac parameters of length and tension are used, there is an optimal end-diastolic volume (EDV) at
which the work output (pressure developed or volume pumped) of the heart is maximal. At lower or higher
volumes or filling pressures (preload), the amount of developed tension or work produced, such as
measured by stroke volume, is reduced. Therefore, an increase in right atrial pressure, caused by an
Page |
increase in venous pressure, causes increased right ventricular end-diastolic volume, increased right 918
ventricular end-diastolic pressure and increased stroke volume.

181. You have taken a patient's arterial blood gases and the PCO2 is 8KPa. Carbon dioxide retention can
cause which of the following? Select one answer only.
A decrease in cerebrospinal fluid (CSF) pressure YOUR ANSWER
A faint pulse
An increase in serum potassium CORRECT ANSWER
Decreased cardiac output
Reduced plasma catecholamines.

CO2 retention causes a respiratory acidosis, which increases serum potassium, in addition to
catecholamine release causing an increased risk of cardiac arrhythmias and sweating. The pulse is
classically bounding and the patient may vasodilate. The O2Hb dissociation curve moves to the right.
Increased CO2 causes a rise in cerebrospinal (CSF) pressure and hyperventilation to reduce CO2 will
decrease CSF pressure in head in juries. CO2 retention also increases cardiac output. With high levels a
reduced level of consciousness is seen and eventually coma.

182. You are called to see a patient on the neuro-ICU with a closed head injury and raised intracranial
pressure. Which of the following statements is correct regarding raised intracranial pressure? Single best
answer - select one answer only.
Can be caused by shearing injury to the scalp YOUR ANSWER
Occurs immediately after an extradural haematoma
Leads to a rise in cerebral perfusion pressure
Leads to a fall in systemic blood pressure
Can lead to a rise in blood pressure and a bradycardia CORRECT ANSWER.

Although shearing injuries to the scalp can result in severe blood loss and external haematoma, they do
not cause raised intracranial pressure (ICP). Both extradural and subdural haematomas, and any
condition leading to cerebral oedema or cerebral engorgement could lead to a rise in ICP. Likewise,
obstruction to the cerebrospinal fluid (CSF) pathway could lead to a raised ICP, although this is
uncommon after head injuries. In the early stages of head injury, there is a non-linear relationship
between an expanding haematoma and elevation of ICP a haematoma may expand without any
significant rise in pressure. Once this early compliance is lost, the pressure will rise rapidly. This severely
jeopardises cerebral perfusion: cerebral perfusion pressure (CPP) is equal to mean arterial blood
---------------------------------------------------------------------------------------------------------------------------------------------------------------------------------------------------------------
Dr Mohammed Shamsul Islam Khan, Medical Officer, Clinical Neuro-Surgery, National Institute of Neuro-Sciences and Hospital
Sher-E-Bangla Nagar, Dhaka-1207, Bangladesh. Mobile: +880 1713 455 662, +880 1685 811979. E-mail: drsikhan@gmail.com
MyPasTest MRCS A Online - Jan Exam 2015
10. Physiology; System Specific Physiology (399Qs)
----------------------------------------------------------------------------------------------------------------------------------
pressure minus ICP (normal CPP is approximately 70 mmHg and the normal ICP in adults is 1015
mmHg). Therefore, any rise in ICP will cause a corresponding fall in CPP.

As ICP rises, CSF is driven out of the intracranial compartment the first stage of compensation. This
follows the MonroKellie hypothesis whichstates that: 'The sum of intracranial volumes of blood, brain,
CSF and other components is constant, and that an increase in any one of these must be offset by a
corresponding decrease in another, or else ICP will rise.' With continuing rise in ICP, brain shifts occur
Page |
within the cranial cavity. The most important of these brain shifts is uncal transtentorial herniation or 919
coning.

This causes impairment of conscious level, development of an ipsilateral fixed dilated pupil (due to 3rd
cranial nerve compression), hemiparesis of the contralateral side (due to compression of the cerebral
peduncle), and later cardiovascular and respiratory abnormalities (CheyneStokes breathing) due to
brainstem compression. The agonal event is often accompanied by hypertension and bradycardia
Cushings reflex. Frank papilloedema may be seen at this stage.

183. A 34-year-old lady undergoes caesarean section because of foetal distress. Which of the following is
true with regard to her prolactin levels: Single best answer select one answer only.
Secretion is increased by dopamine YOUR ANSWER
Secretion is under hypothalamic control CORRECT ANSWER
Secretion would be lower as she has undergone surgery
Secretion would be lower than normal because of her pregnancy
Secretion would be reduced due to the stress response.

Prolactin secretion is controlled by the hypothalamus through the release of dopamine, which has an
inhibitory effect. There are no known hypothalamic prolactin-releasing hormones, although both TRH and
VIP stimulate prolactin secretion this is not, however, of physiological importance. Prolactin secretion
increases after surgery, and trauma, as part of the stress response. A physiological rise in prolactin levels
is seen during pregnancy and breast-feeding. Prolactin stimulates the secretion of breast milk and
reduces gonadal activity. Other common causes of hyperprolactinaemia are a prolactin-secreting tumour
and drugs such as dopaminergic-receptor blockers.

184. Which of the following substances is transported by facilitated diffusion across the placental barrier?
Single best answer question choose ONE true option only.
Amino acids YOUR ANSWER
Conjugated steroids
Glucose CORRECT ANSWER
Nucleotides
Water-soluble vitamins.

The syncytiotrophoblastic cell layer is the limiting barrier to diffusion and active transport. Other
components such as connective tissue and endothelium of fetal capillaries may limit diffusion for large
---------------------------------------------------------------------------------------------------------------------------------------------------------------------------------------------------------------
Dr Mohammed Shamsul Islam Khan, Medical Officer, Clinical Neuro-Surgery, National Institute of Neuro-Sciences and Hospital
Sher-E-Bangla Nagar, Dhaka-1207, Bangladesh. Mobile: +880 1713 455 662, +880 1685 811979. E-mail: drsikhan@gmail.com
MyPasTest MRCS A Online - Jan Exam 2015
10. Physiology; System Specific Physiology (399Qs)
----------------------------------------------------------------------------------------------------------------------------------
molecules and cells. Simple diffusion-exchange is determined by concentration gradient for low-
molecular-weight molecules, such as blood gases, sodium, water, urea, non-polar molecules such as
cholesterol and steroid hormones. Hexose sugars (eg, glucose) are transported by facilitated diffusion,
whereas active transport systems exist for amino acids, conjugated steroids, nucleotides and water-
soluble vitamins. Pinocytosis occurs for transfer of plasma proteins, immunoglobulins and lipoproteins.

185. A 68-year old man presents to his GP with weight gain, bruising and generalised weakness and is
Page |
diagnosed with Cushings syndrome. He is found to be hyperglycaemic and serum ACTH levels are very 920
high. Overnight high dose dexamethasone suppression test shows a raised morning cortisol level. Which
of the following is most likely to be found? Single best answer select one answer only.
Adrenocortical adenoma YOUR ANSWER
Adrenocortical carcinoma
Basophil pituitary adenoma
Small cell bronchial carcinoma CORRECT ANSWER
Bronchial carcinoid.

Excessive plasma concentration of cortisol is most commonly due to treatment with


cortisol/adrenocorticotropic hormone (ACTH) or their synthetic analogues, ie iatrogenic.

Cushings syndrome can be ACTH-dependent or ACTH-independent. The raised serum ACTH shows that
in this case, it is ACTH-dependent, so adrenal causes (eg primary adrenocortical adenoma/carcinoma
without the normal pituitary/hypothalamic regulatory mechanism) are excluded.

In turn, ACTH-dependent Cushings can then be classified into pituitary or ectopic, depending on the
source of ACTH.

Excess stimulation of the adrenal glands by ACTH from the pituitary may be secondary to chromophobe
or basophil pituitary adenoma. A pituitary source for ACTH-dependent Cushings should result in a
suppressed cortisol level with the dexamethasone-suppression test.

Ectopic sources of ACTH (such as oat cell carcinoma of the bronchus, bronchial carcinoid, thymic
tumours, islet cell tumours and phaeochromocytoma) will not respond to dexamethasone suppression, as
they are independent of the hypothalamic-pituitary-adrenal axis. Small cell bronchial carcinoma is more
common than bronchial carcinoid and is therefore the appropriate answer.

186. A 43-year-old man sustains pericardial injury from a stab wound to the chest, resulting in cardiac
tamponade and subsequent cardiogenic shock. Which of the following regarding cardiogenic shock is
correct? Single best answer select one answer only.
May be caused by sepsis YOUR ANSWER
Leads to a fall in the pulmonary capillary wedge pressure
May be treated with sodium nitroprusside CORRECT ANSWER
Leads to an increase in coronary blood flow

---------------------------------------------------------------------------------------------------------------------------------------------------------------------------------------------------------------
Dr Mohammed Shamsul Islam Khan, Medical Officer, Clinical Neuro-Surgery, National Institute of Neuro-Sciences and Hospital
Sher-E-Bangla Nagar, Dhaka-1207, Bangladesh. Mobile: +880 1713 455 662, +880 1685 811979. E-mail: drsikhan@gmail.com
MyPasTest MRCS A Online - Jan Exam 2015
10. Physiology; System Specific Physiology (399Qs)
----------------------------------------------------------------------------------------------------------------------------------
Has a 2030% mortality rate.

Cardiogenic shock is caused primarily by the failure of the heart to maintain normal circulation. As in
hypovolemic shock, there is a failure of organ perfusion and tissue oxygenation, characterized by
hypotension, low cardiac output and signs of poor tissue perfusion such as oliguria, cold extremities and
poor cerebral function. The causes of cardiogenic shock include myocardial infarction, acute massive Page |
pulmonary embolism, pericardial tamponade, valve destruction, aortic dissection and pneumothorax. The
pulmonary capillary wedge pressure is reduced in hypovolemic shock. 921
Cardiogenic shock is diagnosed when critical impairment of tissue perfusion occurs despite an adequate
or elevated pulmonary capillary wedge pressure and in the absence of mechanical circulatory
obstruction, ie in cardiogenic shock, the wedge pressure will be normal or elevated. Short acting venous
dilators such as glyceryl trinitrate or sodium nitroprusside may be given if the pulmonary capillary wedge
pressure is 25 mmHg or higher. Cardiogenic shock also leads to a reduced coronary blood flow, which
results in further impairment of cardiac function. The mortality rate is nearly 90% in cardiogenic shock.

187. You are about to administer oxygen therapy to a 79-year-old patient with chronic bronchitis. Which
one of the following hazards should be borne in mind with regards to oxygen therapy? Single best answer
- select one answer only.
Formation of pulmonary bullae YOUR ANSWER
Bronchopulmonary dysplasia
Supraventricular tachycardia
Bronchospasm
Hypercarbia CORRECT ANSWER.

The administration of oxygen to patients such as those with chronic bronchitis, who rely on hypoxic
respiratory drive, can lead to respiratory depression and carbon dioxide retention (hyper rather than
hypocarbia). This is often referred to as CO2 narcosis and should not be treated by the sudden cessation
of oxygen therapy, as this is likely to cause a dangerous drop in the partial pressure of oxygen. Such
patients should be encouraged to breathe or, if unconscious, immediately ventilated.

In paediatric practice, bronchopulmonary dysplasia may result when immature lungs are ventilated at
high concentrations of inspired oxygen. Premature babies receiving high oxygen concentrations may
suffer retrolental fibroplasia when neovascularisation and haemorrhage lead to retinal detachment.
Neither bronchospasm nor the formation of bullae is caused by oxygen therapy. Supraventricular
tachycardia is not a recognised hazard of oxygen therapy.

188. In an experiment the juxtaglomerular cells in the kidney were selectively destroyed. This will result in
loss of production of: Single best answer question choose ONE true option only.
Aldosterone YOUR ANSWER
Angiotensin
Erythropoietin

---------------------------------------------------------------------------------------------------------------------------------------------------------------------------------------------------------------
Dr Mohammed Shamsul Islam Khan, Medical Officer, Clinical Neuro-Surgery, National Institute of Neuro-Sciences and Hospital
Sher-E-Bangla Nagar, Dhaka-1207, Bangladesh. Mobile: +880 1713 455 662, +880 1685 811979. E-mail: drsikhan@gmail.com
MyPasTest MRCS A Online - Jan Exam 2015
10. Physiology; System Specific Physiology (399Qs)
----------------------------------------------------------------------------------------------------------------------------------
Renin CORRECT ANSWER
Urodilatin.

In the kidney, the juxtaglomerular cells (JG cells) are cells that synthesise, store and secrete the enzyme
renin. They are specialised smooth muscle cells in the wall of the afferent arteriole that delivers blood to
the glomerulus. In synthesising renin, they play a critical role in the renin-angiotensin system and so in
Page |
renal autoregulation, the self-governance of the kidney. In appropriately stained slides, juxtaglomerular 922
cells are distinguished by their granulated cytoplasm.

189. A 13-year-old girl develops exophthalmos, nervousness, diarrhoea and weight loss following the
death of her mother. Her blood pressure is 170/90. What is the most likely diagnosis?
multiple endocrine adenoma (MEN Type 2) YOUR ANSWER
a phaeochromocytoma
Graves disease CORRECT ANSWER
retro-orbital pseudo-tumour
acute appendicitis.

Whilst hypertension, weight loss, diarrhoea and anxiety are features of phaeochromocytoma,
exophthalmos is not. Moreover, Graves disease is more likely to occur in adolescence than
phaeochromocytoma.

190. A 32-year-old IVDU is admitted with reduced consciousness, severe sepsis and a reduced blood
pressure requiring ionotrophic support. Which of the following is correct in regard to the renin-
angiotensin system? Single best answer - select one answer only.
Renin is decreased immediately after surgery YOUR ANSWER
Renin is produced by the juxtaglomerular cells CORRECT ANSWER
Renin is responsible for the conversion of angiotensin I to angiotensin II
Renin is secreted in response to a increased renal blood flow
Renin is secreted in response to low oxygen tension in the renal artery.

Renin secretion is increased by stimuli that decrease ECF volume and blood pressure or increase
sympathetic output. So, the stimuli that increase renin secretion include: sodium depletion, cardiac
failure, dehydration, constriction of the renal artery, hypotension, haemorrhage and an upright posture.
Renin is responsible for the conversion of angiotensinogen to angiotensinI.

Angiotensin-converting enzyme is responsible for the conversion of angiotensin I to angiotensin II


(predominantly in the lungs). Renin is produced by the juxtaglomerular cells, which, together with the
macula densa and lacis cells, constitute the juxtaglomerular apparatus.

191. Which of the following metabolic effects is most likely to be caused by thyroid hormone?
---------------------------------------------------------------------------------------------------------------------------------------------------------------------------------------------------------------
Dr Mohammed Shamsul Islam Khan, Medical Officer, Clinical Neuro-Surgery, National Institute of Neuro-Sciences and Hospital
Sher-E-Bangla Nagar, Dhaka-1207, Bangladesh. Mobile: +880 1713 455 662, +880 1685 811979. E-mail: drsikhan@gmail.com
MyPasTest MRCS A Online - Jan Exam 2015
10. Physiology; System Specific Physiology (399Qs)
----------------------------------------------------------------------------------------------------------------------------------
Single best answer question choose ONE true option only.
Decreased glycogenolysis in the liver YOUR ANSWER
Increased glucose absorption in the gut CORRECT ANSWER
Decreased lipolysis
Decreased expression of adrenergic receptors Page |
Decreased oxygen uptake in the mitochondria. 923
Thyroid hormone has widespread metabolic effects. Increased glycogenolysis in the liver, increased
glucose absorption in the gut and increased insulin breakdown all tend to increase blood glucose. The
glycogenolytic effects of catecholamines are also potentiated. These effects can make the diagnosis and
management of diabetes in thyrotoxicosis difficult. There is an overall lipolytic effect, with decreased
serum cholesterol seen in thyrotoxicosis, and an increase in hypothyroidism.

There is an increased expression of -adrenergic receptors in many tissues including skeletal and
cardiac muscle. There is a positive inotropic effect with increased cardiac output and heart rate.
A raised metabolic rate and increased heat production are due to increased oxygen uptake and ATP
production in the mitochondria.

There are also effects on bone, with an overall breakdown of bone, sometimes leading to hypercalcaemia.
Increased serum 2,3 DPG leads to a right shift of the haemoglobin dissociation curve. Thyroid hormones
are also essential for fetal development, with deficiency leading to cretinism. The fetus produces its own
hormone from 18 weeks of gestation.

192. Considering the ABO and rhesus (Rh) systems. Single best answer question choose ONE true
option only.
If the patient blood group is AB, his serum will have anti A and anti B antibodies YOUR ANSWER
Naturally occurring anti A and anti B antibodies are usually IgG
Red blood cells are the only carrier of the antigen A,B and H
The presence of the D antigen makes the subject rhesus positive CORRECT ANSWER
Rhesus antibodies are naturally occurring antibodies.

In AB blood group, the patient will have A & B antigen but not anti-A and anti-B antibodies. These
antibodies are usually Ig M and are present on most of body cells. Rhesus antibodies do not occur
naturally and are formed as immune antibodies upon exposure to antigen.

193. You are advising a patient about diet and following the conversation a prospective medical student
who has been shadowing you ask from which area fat is most predominantly absorbed. How do you
respond? Single best answer - select one answer only.
Antrum YOUR ANSWER
---------------------------------------------------------------------------------------------------------------------------------------------------------------------------------------------------------------
Dr Mohammed Shamsul Islam Khan, Medical Officer, Clinical Neuro-Surgery, National Institute of Neuro-Sciences and Hospital
Sher-E-Bangla Nagar, Dhaka-1207, Bangladesh. Mobile: +880 1713 455 662, +880 1685 811979. E-mail: drsikhan@gmail.com
MyPasTest MRCS A Online - Jan Exam 2015
10. Physiology; System Specific Physiology (399Qs)
----------------------------------------------------------------------------------------------------------------------------------
Colon
Proximal small intestine CORRECT ANSWER
Pylorus
Terminal Ileum.
Page |
Glycerol, monoglycerides and long-chain fatty acids move into mucosalcells where they are re-esterified 924
into triglycerides, which are then packaged along with cholesterol into large lipoprotein particles
(chylomicrons). These are then transported out of the cell into the lacteals. Shortchain fatty acids pass
directly through the cells into the blood. Cholesterol is readily absorbed from the small intestine if bile,
fatty acids and pancreatic juice are present. Fat absorption is greatest in the upper parts of the small
intestine, but significant amounts are absorbed in the ileum.

194. A 43-year-old man presents with abdominal pain and is found on oesophagogastroduodenoscopy to
have a duodenal ulcer. He is treated with a proton pump inhibitor and Helicobacter pylori eradication
treatment, but the ulcer is persistent. Gastrin levels are checked and found to be raised. What syndrome
does this man have? Single best answer select one answer only.
Multiple endocrine neoplasia type I YOUR ANSWER
Multiple endocrine neoplasia type IIa
Multiple endocrine neoplasia type IIb
Zollinger-Ellison CORRECT ANSWER
None of the above.

ZollingerEllison syndrome results from a gastrin-secreting tumour of the islet cells of the pancreas or
rarely of the duodenum or the gastric antrum. This causes an increase in the levels of circulating gastrin.
It often presents as intractable duodenal ulceration with high incidence of bleeding and perforation. If the
ulcer perforates, the patient may present with the classical signs and symptoms of peritonitis.

Approximately 25% of patients with ZollingerEllison syndrome are associated with multiple endocrine
neoplasia type 1, with an associated adenoma of the parathyroid in approximately 25% of patients and
hyperplasia of the adrenal and thyroid in 10% of patients. Proton pump inhibitors that suppress acid
production and promote healing comprise first line treatment. H-2 antagonists may also be used, but are
less effective than proton pump inhibitors in reducing acid production. Surgery to remove the tumour is
only undertaken if the ulcers are resistant to the above medical treatment options.

195. You are reviewing a patient 1 day following major abdominal surgery, who has been having
respiratory difficulties and has been transferred to ICU overnight. Which respiratory physiological
variable is unaffected by the surgery? Select one answer only.
Anatomical dead space YOUR ANSWER
Functional residual capacity
Lung compliance CORRECT ANSWER
---------------------------------------------------------------------------------------------------------------------------------------------------------------------------------------------------------------
Dr Mohammed Shamsul Islam Khan, Medical Officer, Clinical Neuro-Surgery, National Institute of Neuro-Sciences and Hospital
Sher-E-Bangla Nagar, Dhaka-1207, Bangladesh. Mobile: +880 1713 455 662, +880 1685 811979. E-mail: drsikhan@gmail.com
MyPasTest MRCS A Online - Jan Exam 2015
10. Physiology; System Specific Physiology (399Qs)
----------------------------------------------------------------------------------------------------------------------------------
Minute ventilatory volume
Tidal volume.

Major abdominal surgery causes pain resulting in reduced tidal volumes, functional residual capacity and
anatomical dead space. Decreased minute ventilatory volume would also be a feature. In uncomplicated
major abdominal surgery, lung compliance is unaffected. However, if the patient were to develop adult
Page |
respiratory disease syndrome (ARDS) then lung compliance would fall. 925
196. As part of the pre-operative investigations for a patient requiring urgent surgery the anaesthetist
requests that you get the patient's functional residual capacity (FRC) checked. How can you measure
FRC? Select one answer only.
Exercise stress test YOUR ANSWER
Helium dilution technique CORRECT ANSWER
Nitrogen dilution technique
Spirometry
Wrights peak flow meter.

The FRC is the residual volume plus the expiratory reserve volume and represents < 50% of the vital
capacity. FRC decreases when supine and increases on standing. It also varies with height and body
build. Expiratory reserve can be measured directly by spirometry, but residual volume is measured by the
helium dilution technique or body plethysmography.

197. You are called to review a ward patient who is short of breath. You diagnose ARDS (acute respiratory
distress syndrome). What is an indirect cause of ARDS? Select one answer only.
Acute pancreatitis YOUR ANSWER (Correct Answer)
Acute renal failure
Aspiration
Lung contusion
Pneumonia.

The causes of ARDS can broadly be divided into direct and indirect insults to the lung. Direct insults
include lung contusion, aspiration and pneumonia. Indirect insults include massive blood transfusion,
pancreatitis and fat embolus. Renal failure may be associated with ARDS, especially in the presence of
multiorgan failure. It is, however, not a cause of ARDS.

198. One of your patients has respiratory failure. You phone the medical registrar who reviews the patient
and recommends BiPAP ventilation. The medical registrar asks what you know about this type of
ventilation. What would you tell them regarding BiPAP? Select one answer only.
It is a form of Continuous Positive Airway Pressure ventilation (CPAP) YOUR ANSWER
It is an invasive form of ventilation
---------------------------------------------------------------------------------------------------------------------------------------------------------------------------------------------------------------
Dr Mohammed Shamsul Islam Khan, Medical Officer, Clinical Neuro-Surgery, National Institute of Neuro-Sciences and Hospital
Sher-E-Bangla Nagar, Dhaka-1207, Bangladesh. Mobile: +880 1713 455 662, +880 1685 811979. E-mail: drsikhan@gmail.com
MyPasTest MRCS A Online - Jan Exam 2015
10. Physiology; System Specific Physiology (399Qs)
----------------------------------------------------------------------------------------------------------------------------------
It is suitable for cardiovascularly unstable patients
It is suitable for type-2 respiratory failure CORRECT ANSWER
The Frequency of pressure change is variable.

BiPAP is non-invasive bilevel mask ventilation. The level of inspiratory pressure is alternated between a Page |
high and low level at a fixed frequency to increase air flow into the lungs during inspiration. It is suitable
for type-2 respiratory failure and should be reassessed half an hour after initiating treatment: if the patient 926
is still retaining carbon dioxide, then mask ventilation is unlikely to succeed.

199. Your friends have persuaded you to participate in a 10K run for charity. As your legs begin to tire at
the 7K mark, you consider what influences increase blood-flow through muscle. Which of the following is
correct? Select one answer only.
Rise in tissue PO2 YOUR ANSWER
Fall in tissue PCO2
Parasympathetic stimulation
Bradykinin CORRECT ANSWER
Increase in temperature.

Blood flow in exercising muscle is mainly influenced by localmechanisms. Factors increasing blood flow
in muscle include: fall in tissue PO2, rise in tissue PCO2 , accumulation of potassium and vasodilator
metabolites (such as bradykinin). An increase in temperature would also dilate vessels, increasing the
cross-sectional area of the vascular bed and thereby reducing blood flow velocity. Blood flow in muscle
doubles after sympathectomy. The parasympathetic nervous system has no known effects on muscle
blood flow.

200. Anatomical dead space is decreased by......Single best answer question choose ONE true option
only.
Adrenaline YOUR ANSWER
Tracheostomy CORRECT ANSWER
A subject standing as opposed to lying supine
Increasing size of a subject
Increasing lung volume.

Anatomical dead space is the volume of conducting airways down to a level where rapid mixture of gas
already in the lungs with inspired gases takes place. The gas in this part of the lung does not take part in
gas exchange. Physiological dead space is the total lung volume not taking part in gas exchange, and is
usually the same as or more than the anatomical dead space. It includes the anatomical dead space, plus
alveolar dead space, so any alveoli that are ventilated but not perfused are included in the physiological
but not the anatomical dead space.
---------------------------------------------------------------------------------------------------------------------------------------------------------------------------------------------------------------
Dr Mohammed Shamsul Islam Khan, Medical Officer, Clinical Neuro-Surgery, National Institute of Neuro-Sciences and Hospital
Sher-E-Bangla Nagar, Dhaka-1207, Bangladesh. Mobile: +880 1713 455 662, +880 1685 811979. E-mail: drsikhan@gmail.com
MyPasTest MRCS A Online - Jan Exam 2015
10. Physiology; System Specific Physiology (399Qs)
----------------------------------------------------------------------------------------------------------------------------------

Factors increasing anatomical dead space are: a subject standing rather than sitting, increasing size of a
subject, increasing lung volume (e.g. during inspiration), and bronchodilatation due to drugs or
endogenous agents such as adrenaline. Because tracheostomy bypasses the upper respiratory tract, it
decreases anatomical dead space.
Page |
201. A normal healthy woman became pregnant. On her first antenatal visit she had haemoglobin 927
concentration of 12 g/dl. After 24 weeks of gestation she had haemoglobin of 10 g/dl with normal blood
picture. The most likely cause is: Single best answer question choose ONE true option only.
Folate-deficiency anaemia YOUR ANSWER
Iron-deficiency anaemia
Thalassaemia
Vitamin B12 deficiency
Physiological response CORRECT ANSWER.

In pregnancy, red cell volume also increases, but this frequently lags behind the plasma volume, resulting
in a reduced haematocrit and haemoglobin concentration. This has sometimes been referred to as the
physiological anaemia of pregnancy. However, the total amount of haemoglobin is greater during
pregnancy and the oxygen carrying capacity of the blood more than matches the increase in oxygen
consumption.

202. A 35-year-old woman is found to have depressed hearing at all frequencies of sound when tested by
air conduction but to have normal bone conduction for all frequencies. What is the most likely cause of
deafness in this woman? Single best answer question choose ONE true option only.
Destruction of the cochlea YOUR ANSWER
Damage to auditory association area
Fibrosis causing fixation of the ossicles CORRECT ANSWER
Lesion of the auditory nerve
Poor hair cell function in cochlea.

Because the cochlea is embedded in bone, vibrations in the bone can be transmitted directly to the
cochlear fluid. For this reason, damage to the ossicles or tympanic membrane would not be detected from
a bone conduction test.

203. You receive pre-op tests back on a patient regarding their respiratory status. What is the functional
residual capacity? Select one answer only.
Approximately 2.5 litres YOUR ANSWER (Correct Answer)
Approximately 60% of vital capacity
Increased when supine

---------------------------------------------------------------------------------------------------------------------------------------------------------------------------------------------------------------
Dr Mohammed Shamsul Islam Khan, Medical Officer, Clinical Neuro-Surgery, National Institute of Neuro-Sciences and Hospital
Sher-E-Bangla Nagar, Dhaka-1207, Bangladesh. Mobile: +880 1713 455 662, +880 1685 811979. E-mail: drsikhan@gmail.com
MyPasTest MRCS A Online - Jan Exam 2015
10. Physiology; System Specific Physiology (399Qs)
----------------------------------------------------------------------------------------------------------------------------------
The inspiratory capacity plus the inspiratory reserve volume
The tidal volume plus the expiratory reserve volume.

The functional residual capacity (FRC) is the residual volume plus the expiratory reserve volume and
represents less than 50% of vital capacity. It is decreased when supine and increased on standing. It
varies with height and body build. Expiratory reserve can be measured directly and residual volume
Page |
calculated by helium washing. 928
204. A 51-year-old male presents with an upper GI bleed. His systolic blood pressure is 118 and blood
results show an Hb of 11. Endoscopy reveals a duodenal ulcer with a visible bleeding vessel which is
successfully clipped and injected with adrenaline. Which is the most significant factor in predicting
rebleeding in this case? Single best answer - select one answer only.
Age YOUR ANSWER
Blood pressure
Ulcer size
Urea
Visible bleeding vessel at the time of endoscopy CORRECT ANSWER.

The most important factors in predicting rebleeding are: the presence of significant endoscopic stigmata
of recent haemorrhage with a visible bleeding vessel, haemodynamic instability, an ulcer on the posterior
wall of the duodenum or high on the lesser curve of the stomach, age over 60 years, shock or anaemia on
admission (Hb<10).

205. A 56-year-old lady presents with renal stones and is found to be hypercalcaemic. On further
investigation, imaging confirms a parathyroid adenoma. Which of the following actions does parathyroid
hormone have? Single best answer select one answer only.
Controls conversion of vitamin D to 25-hydroxycholecalciferol in the kidney YOUR ANSWER
Increases bone resorption CORRECT ANSWER
Increases renal excretion of calcium
Reduces bone resorption
Reduces gastrointestinal absorption of calcium.

Parathyroid hormone acts on the gut (increases dietary absorption of calcium), bones (increases
resorption) and kidneys (activates vitamin D) to increase calcium levels. Calcitonin acts to oppose
parathyroid hormone, and it therefore causes increased renal excretion and reduces bone resorption.
Vitamin D is produced in the skin by sunlight and is ingested in the diet.

It first undergoes 25-hydroxylation in the liver and then 1a-hydroxylation in the kidney to form the active
1,25-dihydroxycholecalciferol (1,25-DHCC). This last stage is under the control of PTH and phosphorus. A
rise in PTH or a fall in serum phosphate increases 1,25-DHCC synthesis. Oestrogen increases calcium
absorption and protects against the unopposed action of PTH.
---------------------------------------------------------------------------------------------------------------------------------------------------------------------------------------------------------------
Dr Mohammed Shamsul Islam Khan, Medical Officer, Clinical Neuro-Surgery, National Institute of Neuro-Sciences and Hospital
Sher-E-Bangla Nagar, Dhaka-1207, Bangladesh. Mobile: +880 1713 455 662, +880 1685 811979. E-mail: drsikhan@gmail.com
MyPasTest MRCS A Online - Jan Exam 2015
10. Physiology; System Specific Physiology (399Qs)
----------------------------------------------------------------------------------------------------------------------------------
206. You assist in intubating and ventilating a patient at a trauma call. What complications of artificial
ventilation should you observe the patient for? Select one answer only.
Basal atelectasis YOUR ANSWER
Cerebral oedema
Hypercarbia Page |
Hypertension 929
Surgical emphysema CORRECT ANSWER.

By adequately inflating the lungs, artificial ventilation will help to prevent the collapse of small airways.
However, overforceful ventilation pressures may cause a pneumothorax and subsequent surgical
emphysema. The use of positive end-expiratory pressure to reduce alveolar closing may lead to a positive
intrathoracic pressure, which in turn may reduce venous return to the heart, so reducing cardiac output.
An increased rate of artificial ventilation can be used to induce hypocarbia and so reduce intracranial
pressure. Acute gastric distension may occur with ventilation, and this usually responds well to the
insertion of a nasogastric tube.

207. A 54-year-old woman has undergone some blood tests as part of an employment health screen. She
reports she is in good health and, being very health conscious, takes regular vitamin and mineral
supplements. She is taking bendrofluazide 2.5 mg for hypertension and her blood pressure is 132/82
mmHg. The only abnormality is a serum calcium concentration of 2.94 mmol/l. Which of the following is
the most likely cause? Single best answer question choose ONE true option only.
Diuretic treatment YOUR ANSWER
High dietary calcium intake
High dietary vitamin D intake
Occult malignancy
Primary hyperparathyroidism CORRECT ANSWER.

Thiazides can cause hypercalcaemia but it is usually only mild. Vitamin D itself is physiologically inactive
and, whereas 1-hydroxylated derivatives can be a cause of hypercalcaemia, vitamin D which has to be
metabolised to activate it is less commonly so. Intestinal absorption of calcium is subject to tight
control, and a high intake does not cause hypercalcaemia. The two most common causes of
hypercalcaemia are primary hyperparathyroidism and malignancy. In an asymptomatic individual, primary
hyperparathyroidism is the more likely cause.

208. A 54-year-old man is admitted to HDU with a history of sudden onset left sided facial droop and limb
weakness. A diagnosis of stroke is made. Which of the following statements regarding cerebral blood
flow is correct? Single best answer select one answer only.
Is supplied entirely by the carotid arterial system YOUR ANSWER
Is decreased as intracranial pressure increases CORRECT ANSWER
Averages approximately 1 litre per minute per 100 g of brain
---------------------------------------------------------------------------------------------------------------------------------------------------------------------------------------------------------------
Dr Mohammed Shamsul Islam Khan, Medical Officer, Clinical Neuro-Surgery, National Institute of Neuro-Sciences and Hospital
Sher-E-Bangla Nagar, Dhaka-1207, Bangladesh. Mobile: +880 1713 455 662, +880 1685 811979. E-mail: drsikhan@gmail.com
MyPasTest MRCS A Online - Jan Exam 2015
10. Physiology; System Specific Physiology (399Qs)
----------------------------------------------------------------------------------------------------------------------------------
Is diminished with a loss of 10% of the circulating volume of blood in the normal young person
Increases at mean arterial pressures of less than 60 mmHg.

Blood supply to the brain is by the internal carotid and vertebral arteries, which form the circle of Willis.
The brain lies within the rigid skull. So an increase in the volume of the skull contents will increase the
intracranial pressure. Cerebral blood flow is proportional to the difference between mean arterial pressure
Page |
and intracranial pressure. CBF averages 50100 ml per min per 100 g of brain. It is sustained in 930
preference to other organs to which blood flow is diminished when there is loss of blood. Mean arterial
pressure must decrease to less than 60 mmHg before there is a decrease in CBF. This does not occur
until there is a loss of over 40% of the circulating volume in the normal individual (2 l itres in a 75 kg man).

209. Carotid sinus baroreceptors are located in the carotid sinus at the bifurcation of the external ad
internal carotids. They are stimulated by which of the following? Select one answer only.
Increased arterial PCO2 YOUR ANSWER
Fall in arterial PCO2
Anaemia
Decrease in pulse pressure
Rise in mean arterial pressure CORRECT ANSWER.

Baroreceptors are stretch receptors found in the adventitia of the bloodvessels and wall of the heart. The
arterial baroreceptors are located in the carotid sinus and aortic arch. When the arterial blood pressure
drops, the arterial wall is subjected to less stretch, and the sensory nerves coming from the carotid sinus
(sinus nerve) and from the aortic arch (depressor nerve) become less active and send fewer impulses.

Upon receiving fewer impulses from the baroreceptors, the cardiovascular centres respond by exciting
the sympathetic and inhibiting the parasympathetic nervous systems. This leads to an increase in heart
rate and stroke volume, and generalised constriction of arterioles (not brain or heart) and veins. On the
venous side, receptors are found in the wall of the left and right atria, at the entrances of the inferior vena
cava and superior vena cava and in the pulmonary circulation.

210. A 58-year-old man is admitted to A and E resus having sustained 60% burns following a house fire.
His carboxyhaemoglobin is 24%. Which of the following statements is correct? Single best answer -
select one answer only.
Carboxyhaemoglobin level of 50% is manifested by headache and confusion YOUR ANSWER
Carbon monoxide binds with the intracellular cytochrome system and produces abnormal cellular function
CORRECT ANSWER
Oxygen has a greater affinity for haemoglobin than carboxyhaemoglobin
Oxyhaemomoglobin dissociates less readily than carboxyhaemoglobin
The fall in the partial pressure of arterial oxygen (pa(O2)) is directly proportional to a rise in
carboxyhaemoglobin levels.

---------------------------------------------------------------------------------------------------------------------------------------------------------------------------------------------------------------
Dr Mohammed Shamsul Islam Khan, Medical Officer, Clinical Neuro-Surgery, National Institute of Neuro-Sciences and Hospital
Sher-E-Bangla Nagar, Dhaka-1207, Bangladesh. Mobile: +880 1713 455 662, +880 1685 811979. E-mail: drsikhan@gmail.com
MyPasTest MRCS A Online - Jan Exam 2015
10. Physiology; System Specific Physiology (399Qs)
----------------------------------------------------------------------------------------------------------------------------------
Carbon monoxide (CO) combines very readily with haemoglobin, having a greater affinity for haemoglobin
than oxygen (240 times greater) forming carboxyhaemoglobin (COHb). Carboxyhaemoglobin dissociates
less readily than oxyhaemoglobin, and once formed reduces the oxygen-carrying capacity of the blood,
leading to tissue anoxia. The dissolved oxygen in the plasma remains unaffected and the partial pressure
of arterial oxygen (pa(O2)) remains normal.

As well as binding preferentially with haemoglobin, CO binds with great affinity to other haem-containing
Page |
compounds, most importantly the intracellular cytochrome system. This combination may produce 931
abnormal cellular functioning, and in severe exposure produces a 'sick cell syndrome'.
Carboxyhaemoglobin level of 50% is manifested by hallucination, ataxia, syncope, convulsions and coma;
a level of 60% could lead to death.

211. Regarding the clinical physiology of the adrenal gland in Cushings disease, which of the following
pertains? Single best answer question choose ONE true option only.
The zona glomerulosa of the cortex is predominantly responsible for sex steroid production YOUR
ANSWER
The zona fasciculata is predominantly controlled by ACTH and is often hypertrophied CORRECT ANSWER
The zona reticularis is predominantly responsible for mineralocorticoid production
About 15% of glucocorticoid production takes place in the adrenal medulla
The zona fasciculata is primarily responsible for mineralocorticoid production.

The zona glomerulosa of the cortex is predominantly responsible for mineralocorticoid production, the
zona fasciculata for glucocorticoid production and the zona reticularis for sex corticoid production. The
adrenal medulla originates from the neural crest and hence there is almost complete demarcation of
function, with the medulla being responsible for the production of catecholamine-related compounds.

212. A 56-year-old lifelong smoker is admitted to CCU with a history of shortness of breath and chest
pain. His troponin is elevated and his ECG reflects ischaemic changes. A diagnosis of myocardial infarct
is made. Which of the following is correct regarding his coronary circulation? Select one answer only.
Is approximately 500ml/min at rest YOUR ANSWER
Is increased by pain
The right coronary artery typically supplies a third of the blood to the left ventricular muscle CORRECT
ANSWER
Is dependent on venous pressure
Occurs during systole.

Myocardial blood flow is approximately 250 ml/min at rest (which represents 5% of the cardiac output)
and is dependent on arterial pressure. The right coronary artery supplies a third of the blood to the left
ventricular muscle. Myocardial blood flow is seen only in diastole. Pain and vasopressin (ADH) may
reduce myocardial blood flow. Coronary vessels have both a- and -adrenergic receptors. Coronary

---------------------------------------------------------------------------------------------------------------------------------------------------------------------------------------------------------------
Dr Mohammed Shamsul Islam Khan, Medical Officer, Clinical Neuro-Surgery, National Institute of Neuro-Sciences and Hospital
Sher-E-Bangla Nagar, Dhaka-1207, Bangladesh. Mobile: +880 1713 455 662, +880 1685 811979. E-mail: drsikhan@gmail.com
MyPasTest MRCS A Online - Jan Exam 2015
10. Physiology; System Specific Physiology (399Qs)
----------------------------------------------------------------------------------------------------------------------------------
vessels generally have more a receptors and therefore vasoconstrict slightly with sympathetic
stimulation.

213. Calcium plays a vital role in cell regulation, signal transduction, release of neurotransmitters and
contraction of muscle cells. Which of the following statements regarding calcium physiology is correct?
Single best answer - select one answer only. Page |
Approximately 70% of the body's calcium is stored in bone YOUR ANSWER 932
Calcium is excreted in the stools and urine at a rate of 10-20 mg/d
Hypercalcaemia commonly occurs in Cushing's disease
Parathyroid hormone (PTH) mobilises calcium from bone and initiates calcium absorption from the gut
CORRECT ANSWER
The medullary cells of the thyroid gland produce calcitonin.

PTH increases bone resorption, renal calcium reabsorption and gastrointestinal calcium absorption.
Calcitonin is produced by the parafollicular cells of the thyroid gland and inhibits bone resorption. Bone
is the primary storage site for calcium, where approximately 1 kg - 99% of total body calcium - is stored.
Calcium is filtered (about 10 000 mg/d) as part of the glomerular filtrate. Most is reabsorbed.

Daily calcium excretion via urine and faeces is approximately 160-200 mg.
(http://jn.nutrition.org/content/138/1/166S.full) Cushing's disease is very rarely associated with
hypercalcaemia

214. In a small randomised double-blind trial of a new treatment in acute myocardial infarction, the
mortality in the treated group was half that in the control group, but the difference was not significant.
What can we conclude? Select one answer only.
The treatment is useless YOUR ANSWER
There is no point in continuing to develop the treatment
Reduction in mortality is so great that we should introduce the treatment
We should keep adding cases to the trial until the test for the comparison of two proportions shows a
significant difference
We should carry out a trial of much greater size CORRECT ANSWER.

A halving of mortality attributable to a new treatment would almost certainly be clinically significant.
Since the difference was not statistically significant the treatment may be useless, and the observed
reduction was due to random variation, or it MAY actually improve survival. The study has been
inconclusive, a larger sample would be needed to identify whether the drug actually does work (A is false,
B is false, C is false, E is true). If we keep adding cases to the trial and retesting, then the probability of
obtaining a falsely significant result will increase. Sequential trials need to be specially designed to take
account of multiple testing (D is false).
215. In a patient with anaemia and normal lungs: Single best answer question choose ONE true option
only.
---------------------------------------------------------------------------------------------------------------------------------------------------------------------------------------------------------------
Dr Mohammed Shamsul Islam Khan, Medical Officer, Clinical Neuro-Surgery, National Institute of Neuro-Sciences and Hospital
Sher-E-Bangla Nagar, Dhaka-1207, Bangladesh. Mobile: +880 1713 455 662, +880 1685 811979. E-mail: drsikhan@gmail.com
MyPasTest MRCS A Online - Jan Exam 2015
10. Physiology; System Specific Physiology (399Qs)
----------------------------------------------------------------------------------------------------------------------------------
Arterial p(O2) is reduced YOUR ANSWER
Arterial-venous O2 concentration difference is increased
Arterial O2 saturation is reduced
Cardiac output is reduced
Page |
p(O2) of mixed venous blood is reduced CORRECT ANSWER.
933
A patient with anaemia and normal lungs typically has a normal arterial p(O2). Because the position of the
oxygen dissociation curve is typically normal, the arterial oxygen saturation is also normal. If the oxygen
consumption and cardiac output are normal, the arterial-venous O2 concentration difference will also be
normal. As a matter of fact, cardiac output is sometimes reflexly increased in anaemia and if this occurs,
the arterial-venous O2 concentration difference will be decreased. Although the arterial p(O2) is typically
normal, the p(O2) of mixed venous blood must fall. This is because the venous oxygen concentration falls
to a very low level as the normal amount of oxygen is extracted and so the venous p(O2) is abnormally
low.

216. A 21-year-old male medical student who has been feeling non-specifically unwell for several days is
noticed to have slightly icteric sclerae by his girlfriend and has liver function tests performed. The results
of these are normal apart from a serum bilirubin concentration of 44 mol/l (317). His urine does not
contain bilirubin. Which of the following is the most likely diagnosis? Single best answer question
choose ONE true option only.
DubinJohnson syndrome YOUR ANSWER
Gilberts syndrome CORRECT ANSWER
Hereditary spherocytosis
Infectious mononucleosis
Rotor syndrome.
DubinJohnson, Rotor and Gilberts syndromes are all inherited disorders of bilirubin metabolism.
However, in the first two, there is a defect in the secretion of bilirubin from the liver and the bilirubin that
accumulates in the plasma is conjugated, water-soluble and thus is excreted in the urine.
Infectious mononucleosis can cause hepatitis and jaundice but an elevated transaminase activity would
be expected. Hereditary spherocytosis is a chronic haemolytic disorder due to a defect in the red cell
membrane (most frequently in spectrin, a structural protein). It can present with a wide range of severity,
from jaundice at birth to asymptomatic anaemia or jaundice in adults, but is much less common
(approximately 1:5000 in Northern Europeans) than Gilberts syndrome (approximately 1:20).
217. A 76-year-old female presents with fatigue, shortness of breath and weight loss. She is found to have
microcytic anaemia on routine blood tests. Which of the following statements is correct with regard to her
anaemia? Single best answer - select one answer only.
Alcoholism may be a cause YOUR ANSWER
It reduces the overall oxygen carrying capacity of blood CORRECT ANSWER
It causes cardiac output to reduce
---------------------------------------------------------------------------------------------------------------------------------------------------------------------------------------------------------------
Dr Mohammed Shamsul Islam Khan, Medical Officer, Clinical Neuro-Surgery, National Institute of Neuro-Sciences and Hospital
Sher-E-Bangla Nagar, Dhaka-1207, Bangladesh. Mobile: +880 1713 455 662, +880 1685 811979. E-mail: drsikhan@gmail.com
MyPasTest MRCS A Online - Jan Exam 2015
10. Physiology; System Specific Physiology (399Qs)
----------------------------------------------------------------------------------------------------------------------------------
It can cause bradycardia
It decreases total blood viscosity.

Anaemia is the reduction in the oxygen transport capacity of blood, usually due to a reduction of total
circulating red cell mass. In severe anaemia there may be evidence of a hyperdynamic circulation e.g.
tachycardias and murmurs. Cardiac output has to increase in order to keep up with physiological
Page |
demands and this may result in later cardiac failure. 934
218. High levels of circulating adrenaline (epinephrine) will produce which of the following physiological
responses? Select one answer only.
Glycosuria YOUR ANSWER (Correct Answer)
Excessive sweating
Bradycardia
Paroxysmal hypotension
Bronchospasm.

Sweating is a cholinergic impulse response. Adrenaline only has a minimal effect on sweating, causing
slight localised secretion. Tachycardia and paroxysmal hypertension are features of high circulating
levels of adrenaline. Hepatic glucose output is also seen, giving rise to hyperglycaemia and glycosuria.

219. A 58-year-old man suffered a stroke that mainly affected his thalamus. Which of the following
abnormalities is most likely to be seen in this patient after the stroke? Single best answer question
choose ONE true option only.
Increased sexual drive YOUR ANSWER
Increased temperature
Hyperaesthesia CORRECT ANSWER
Hypotonia
Thirst.

Cerebrovascular accidents can cause thalamic syndrome, which results in contralateral hemianaesthesia,
burning or aching sensation in one half of a body (hyperaesthesia), often accompanied by mood swings.
Ischaemia of the territory of the paramedian artery, if bilateral, causes serious troubles, including akinetic
mutism accompanied or not by oculomotor troubles.

220. A 58-year-old heavy smoker undergoes radiotherapy for laryngeal cancer. As a result of this, his
saliva production is reduced due to irradiation of the salivary glands. Which of the following would be
true regarding his saliva production? Single best answer select one answer only.
He would previously have secreted >3000ml/day YOUR ANSWER
It should have the same pH as plasma
It will contain a lower molar concentration of potassium than that found in plasma
---------------------------------------------------------------------------------------------------------------------------------------------------------------------------------------------------------------
Dr Mohammed Shamsul Islam Khan, Medical Officer, Clinical Neuro-Surgery, National Institute of Neuro-Sciences and Hospital
Sher-E-Bangla Nagar, Dhaka-1207, Bangladesh. Mobile: +880 1713 455 662, +880 1685 811979. E-mail: drsikhan@gmail.com
MyPasTest MRCS A Online - Jan Exam 2015
10. Physiology; System Specific Physiology (399Qs)
----------------------------------------------------------------------------------------------------------------------------------
It will contain amylase CORRECT ANSWER
It will produce hypertonic secretions.

The salivary glands normally produce between 0.75 and 1.5 litres of saliva per day. Saliva is always
hypotonic to plasma. The pH of saliva from resting glands is slightly acidic. During active secretion,
however, the saliva becomes more basic, and its pH increases to nearly 8. The concentration of
Page |
potassium in saliva is always much greater than its concentration in plasma. When salivary flow rates are 935
very low, salivary potassium levels are high. The serous acinar cells have zymogen granules that contain
salivary amylase; which has a major digestive function, breaking down starch.

221. Glucose is an essential substrate for the metabolism of most cells. The transport of glucose in the
renal tubular cells occurs via: Single best answer question choose ONE true option only.
Active transport YOUR ANSWER
Concentration gradient
Facilitated diffusion
Secondary active transport with sodium CORRECT ANSWER
Secondary active transport with potassium.

Glucose is an essential substrate for the metabolism of most cells. Glucose transport through biological
membranes requires specific transport proteins. Transport of glucose through the apical membrane of
renal tubular as well as intestinal epithelial cells depends on the presence of secondary active Na +-
glucose symporters, SGLT-1 and SGLT-2, which concentrate glucose inside the cells, using the energy
provided by co-transport of Na+ ions down their electrochemical gradient.

222. Aldosterone is secreted from the: Single best answer question choose ONE true option only.
Liver YOUR ANSWER
Zona glomerulosa of the adrenal cortex CORRECT ANSWER
Juxtaglomerular apparatus
Adrenal medulla
Zona fasciculata of the adrenal cortex.

The adrenal gland comprises an outer cortex and an inner medulla, which represent two developmentally
and functionally independent endocrine glands within the same anatomical structure. The adrenal
medulla secretes adrenaline (70%) and noradrenaline (30%). The adrenal cortex consists of 3 layers, or
zones.
The layers from the surface inwards may be remembered by the mnemonic GFR:
1. G = Zona glomerulosa (secretes aldosterone)
2. F = Zona fasciculata (secretes cortisol and sex steroids)
3. R = Zona reticularis (secretes cortisol and sex steroids).

---------------------------------------------------------------------------------------------------------------------------------------------------------------------------------------------------------------
Dr Mohammed Shamsul Islam Khan, Medical Officer, Clinical Neuro-Surgery, National Institute of Neuro-Sciences and Hospital
Sher-E-Bangla Nagar, Dhaka-1207, Bangladesh. Mobile: +880 1713 455 662, +880 1685 811979. E-mail: drsikhan@gmail.com
MyPasTest MRCS A Online - Jan Exam 2015
10. Physiology; System Specific Physiology (399Qs)
----------------------------------------------------------------------------------------------------------------------------------
Aldosterone is a steroid hormone that facilitates the reabsorption of sodium and water and the excretion
of potassium and hydrogen ions from the distal convoluted tubule and collecting ducts. Conns syndrome
is characterised by increased aldosterone secretion from the adrenal glands.

223. A 56-year old woman attends her general practitioner seeking advice on a heavy dependence on
alcohol. She is currently low of mood, unemployed and complains of upper abdominal pain, nausea, Page |
vomiting and retching. Which of the following symptoms are associated with chronic alcohol abuse?
Single best answer - Select one answer only. 936
Hightened libido YOUR ANSWER
Menstrual disturbance CORRECT ANSWER
Microcytosis
Primary thrombcythemia
Sydenhams chorea.

Chronic alcohol abuse may cause a Wernickes encephalopathy (a reversible condition) and Korsakoffs
syndrome (irreversible). Signs and symptoms of cerebellar ataxia are common in chronic alcoholism.
Other effects of chronic alcohol ingestion include generalised marrow depression, with lymphopenia and
immunosuppression. Macrocytosis is generally due to decreased folate levels, caused by chronic alcohol
abuse. Sydenhams chorea is associated with rheumatic fever. Chronic alcohol abuse may cause
menorrhagia as liver damage causes hypocoagulability. During end-stage liver disease, menstruation
may cease completely.

224. The Gaussian distribution is used in statistical analysis to determine two limits within which an
observation will fall. Which of the following is correct about Gaussian distribution? Single best answer -
select one answer only.
The median value may be less than the mean YOUR ANSWER
The standard deviation (SD) is a measure of how accurately the calculated mean approaches the true
population mean
The ANOVA test may be used to compare this set of values with one further set provided they are also
normally distributed
The modal value is always equal to the mean CORRECT ANSWER
25% of the values will have numerical values that are smaller than the mean value minus 5.96 standard
deviations.

The Gaussian distribution is more usually referred to as Normal distribution and is characterised by being
'bell-shaped' (unimodal) and symmetrical about its central value. Accordingly themean=median=mode.
The standard deviation (SD) is a measure of the spread of the data values. 95% of observations lie within
1.96 standard deviations. Hence, 5% lie outside the interval, 2.5% less than mean - 1.96 SD and 2.5%
greater than mean + 1.96 SD. Student's t-test is an appropriate way of comparing the means of two groups
of normally distributed values.

---------------------------------------------------------------------------------------------------------------------------------------------------------------------------------------------------------------
Dr Mohammed Shamsul Islam Khan, Medical Officer, Clinical Neuro-Surgery, National Institute of Neuro-Sciences and Hospital
Sher-E-Bangla Nagar, Dhaka-1207, Bangladesh. Mobile: +880 1713 455 662, +880 1685 811979. E-mail: drsikhan@gmail.com
MyPasTest MRCS A Online - Jan Exam 2015
10. Physiology; System Specific Physiology (399Qs)
----------------------------------------------------------------------------------------------------------------------------------
When the same group of individuals is assessed on three or more occasions, the assessment times can
be compared in pairs using either paired t-tests or Wilcoxon matched pairs rank-sum tests as appropriate.
However the number of comparisons needed is large if there are many groups to be compared and the
risk of a type I error (false positive result) increases rapidly. If the measure can be assumed to have a
Normal distribution, the assessment times can all be compared simultaneously using a repeated
measures analysis of variance (ANOVA).
Page |
225. Which of the following is true regarding carbon dioxide? Select one answer only. 937
It is carried on the haemoglobin molecule as carboxyhaemoglobin YOUR ANSWER
It makes haemoglobin a weaker acid and more effective buffer CORRECT ANSWER
In the blood it increases the oxygen binding power of haemoglobin
It is less soluble in blood than oxygen
It combines reversibly with myoglobin.

Carbon monoxide (CO) is carried on haemoglobin (Hb) as carboxyhaemoglobin. Carbon dioxoide


(CO2)makes Hb a weaker acid but decreases the oxygen binding power of Hb and it is 25 times more
soluble in blood than oxygen. Oxygen combines reversibly with myoglobin. CO 2 enters red blood cell
concentrates and via carbonic anhydrase forms hydrogen and bicarbonate ions. The bicarbonate diffuses
out of the cell in exchange for chloride ions, the so-called chloride shift.

226. A 21-year old man is admitted following a knife stab to the chest. He is placed on cardiopulmonary
bypass during exploratory surgery. Which of the following statements is correct regarding
cardiopulmonary bypass? Single best answer - select one answer only.
Associated neurological injury is eliminated by meticulous removal of air from the circuit YOUR ANSWER
Does not require anticoagulation
Is usually instituted via the ascending aorta and right atrium CORRECT ANSWER
Requires continued ventilation to maintain gas exchange
Should not be introduced via the femoral vessels.

Cardiopulmonary bypass allows surgeons to operate on the heart and great vessels by affording a
relatively bloodless operative field. Blood is drained from the circulation by gravity siphon into a
reservoir. From here it is oxygenated and filtered, and then pumped back into the circulation. The usual
configuration is right atrial drainage to the pump, and inflow back via the ascending aorta.

The femoral or even axillary vessels may also be used. Full heparinisation is required as the blood comes
into contact with foreign surfaces and would clot immediately. Meticulous removal of air from the circuit
aims to minimise neurological injury via air emboli, but cannot eliminate neurological sequelae. As the
bypass machine also oxygenates, continued ventilation is not required on bypass.

227. A 55-year-old diabetic man presents to accident and emergency in septic shock. He has a mottled
appearance to his lower limb and at surgical debridement a diagnosis of necrotising fasciitis is made. He
---------------------------------------------------------------------------------------------------------------------------------------------------------------------------------------------------------------
Dr Mohammed Shamsul Islam Khan, Medical Officer, Clinical Neuro-Surgery, National Institute of Neuro-Sciences and Hospital
Sher-E-Bangla Nagar, Dhaka-1207, Bangladesh. Mobile: +880 1713 455 662, +880 1685 811979. E-mail: drsikhan@gmail.com
MyPasTest MRCS A Online - Jan Exam 2015
10. Physiology; System Specific Physiology (399Qs)
----------------------------------------------------------------------------------------------------------------------------------
is moved to ITU post operatively and commenced on inotropes to maintain his blood pressure. Which of
the following is correct? Select one answer only.
Adrenaline stimulates a- and b-receptors YOUR ANSWER (Correct Answer)
Noradrenaline is predominantly a b-agonist
Dopexamine is a sympathomimetic suppressor Page |
Dobutamine increases systemic vascular resistance 938
Adrenaline may increase renal blood flow at higher doses.

At higher doses of adrenaline, a-mediated vasoconstriction reduces renal blood flow and can cause
oliguria and precipitate acute renal failure. Noradrenaline is predominantly an a-agonist. Dopexamine is a
potent splanchnic vasodilator, reducing afterload and improving blood flow to vital organs, including the
kidney. Dobutamine reduces systemic vascular resistance, decreasing afterload and ventricular filling
pressures, and is of use in cardiogenic shock and cardiac failure.

228. The secretion of testosterone by interstitial (Leydig) cells in the testes is stimulated by the secretion
of which of the following cell types? Single best answer question choose ONE true option only.
Pituitary acidophils YOUR ANSWER
Pituitary basophils CORRECT ANSWER
Primary spermatocytes
Sertoli cells
Spermatogonia.

In men, luteinising hormone (LH) acts upon the interstitial (Leydig) cells in the testes and is responsible
for testosterone production that exerts intratesticular activity in terms of the spermatogenesis and
endocrine activity as the 'male hormone'. LH is produced by gonadotrophs, which are basophilic cells in
the anterior pituitary gland.

229. A 20-year-old medical student presents with a history of epigastric pain and family history of MEN
syndrome. He undergoes endoscopy which shows erosions. He is non-smoker. Which of the following
statements is mostly likely to be correct?
Blood tests show a high gastrin level YOUR ANSWER (Correct Answer)
Family history is unlikely to be relevant
No further tests are required
Proton pump inhibitors are the best treatment
The aetiology is likely to be alcohol related.

This patient has Zollinger Ellison Syndrome. This is a rare syndrome and can be linked with Multiple
Endocrine Neoplasia Type 1. It is characterised by high gastrin levels as a result of a gastrin secreting

---------------------------------------------------------------------------------------------------------------------------------------------------------------------------------------------------------------
Dr Mohammed Shamsul Islam Khan, Medical Officer, Clinical Neuro-Surgery, National Institute of Neuro-Sciences and Hospital
Sher-E-Bangla Nagar, Dhaka-1207, Bangladesh. Mobile: +880 1713 455 662, +880 1685 811979. E-mail: drsikhan@gmail.com
MyPasTest MRCS A Online - Jan Exam 2015
10. Physiology; System Specific Physiology (399Qs)
----------------------------------------------------------------------------------------------------------------------------------
neuroendocrine tumour usually situated in the pancreas. Treatment involves proton pump inhibitors and
surgical excision of tumour or chemotherapy.

230. You attend a post mortem for one of your patients who you suspect died of adult respiratory distress
syndrome (ARDS). What histological features would help confirm this cause of death? Select one answer
only. Page |
Bulla formation YOUR ANSWER 939
Glandular hyperplasia
Granulomas
Interstitial fibrosis CORRECT ANSWER
Synovial membrane formation.

Physiological and histological features of ARDS include: increased capillary permeability; interstitial and
alveolar oedema; fibrin exudation; hyaline membrane formation; and, later, diffuse late interstitial and
alveolar fibrosis.

231. Phase 0 of the cardiac action potential relates to a: Single best answer question choose ONE true
option only.
Rapid efflux of calcium YOUR ANSWER
Rapid influx of calcium
Rapid influx of potassium
Rapid influx of sodium CORRECT ANSWER
None of the above.

Phase 0 rapid sodium influx


Phase 1 efflux of potassium
Phase 2 slow influx of calcium
Phase 3 efflux of potassium
Phase 4 sodium/calcium efflux, potassium influx

232. Which one of the following is MOST likely to increase during exercise? Single best answer question
choose ONE true option only.
Peripheral vascular resistance YOUR ANSWER
Pulmonary vascular resistance
Stroke volume CORRECT ANSWER
Diastolic pressure
Venous compliance.

---------------------------------------------------------------------------------------------------------------------------------------------------------------------------------------------------------------
Dr Mohammed Shamsul Islam Khan, Medical Officer, Clinical Neuro-Surgery, National Institute of Neuro-Sciences and Hospital
Sher-E-Bangla Nagar, Dhaka-1207, Bangladesh. Mobile: +880 1713 455 662, +880 1685 811979. E-mail: drsikhan@gmail.com
MyPasTest MRCS A Online - Jan Exam 2015
10. Physiology; System Specific Physiology (399Qs)
----------------------------------------------------------------------------------------------------------------------------------
During exercise, increased oxygen consumption and increased venous return to the heart result in an
increase in cardiac output and an increase in blood flow to both skeletal muscle and coronary circulation,
when oxygen utilization is greatest. The increase in cardiac output is due to an increase in both heart rate
and stroke volume. Systemic arterial pressure also increases in response to the increase in cardiac
output.
Page |
However, the fall in total peripheral resistance, which is caused by dilatation of the blood vessels within 940
the exercising muscles, results in a decrease in diastolic blood pressure. The pulmonary vessels undergo
passive dilatation as more blood flows into the pulmonary circulation. As a result, pulmonary vascular
resistance decreases. The decrease in venous compliance, caused by sympathetic stimulation, helps to
maintain ventricular filling during diastole.

233. You have a patient on the emergency list for an urgent laparotomy. The patient has an irregularly
irregular pulse with a rate of 180bpm, and the ECG confirms fast atrial fibrillation. The patient has only got
peripheral intravenous access. In order to control the rate and attempt to convert to sinus rhythm
urgently, which would be the most appropriate treatment for the fast atrial fibrillation? Single best answer
- select one answer only.
Amiodarone 10 mg/kg intravenous infusion over 20 minutes YOUR ANSWER
Digoxin 0.250.5 mg orally
Isoprenaline 2 mg/kg infusion over 2 minutes
Phentolamine 10 mg/ml in 20 minutes
Verapamil 510 mg intravenously CORRECT ANSWER.

Atrial fibrillation (AF) may be treated by intravenous or oral digoxin. The onset of action of oral digoxin is
slower. Digoxin does not convert the patient to sinus rhythm but merely controls the rate. Verapamil and
amiodarone can both convert AF into sinus. Verapamil can be given via a peripheral line (5-10mg).

Amiodarone should be administered via a central line if infusion is prescribed (5 mg/kg over 20120
minutes with ECG monitoring, max 1.2g over 24 hours). Isoprenaline increases contractility and heart
rate, and is used in severe bradycardia and heart block. Phentolamine is a short-acting drug used to
control hypertensive crises due to phaeochromocytoma.

234. A 67-year old life long smoker is seen pre-operatively for excision of squamous cell lung cancer.
Which of the following is true of functional residual capacity? Single best answer - select one answer
only.
It can be measured via spirometry YOUR ANSWER
It is the tidal volume plus the expiratory reserve volume
It is approximately 60% of vital capacity
It is increased when someone is supine

---------------------------------------------------------------------------------------------------------------------------------------------------------------------------------------------------------------
Dr Mohammed Shamsul Islam Khan, Medical Officer, Clinical Neuro-Surgery, National Institute of Neuro-Sciences and Hospital
Sher-E-Bangla Nagar, Dhaka-1207, Bangladesh. Mobile: +880 1713 455 662, +880 1685 811979. E-mail: drsikhan@gmail.com
MyPasTest MRCS A Online - Jan Exam 2015
10. Physiology; System Specific Physiology (399Qs)
----------------------------------------------------------------------------------------------------------------------------------
It is approximately 2.5 litres in a healthy adult man CORRECT ANSWER.

The FRC is the residual volume plus the expiratory reserve volume andrepresents < 50% of the vital
capacity. FRC decreases when supine andincreases on standing. It also varies with height and body
build. Expiratory reserve can be measured directly by spirometry, but residual volume is measured by the
helium dilution technique. Page |
235. Which of the following hormones produces contraction of the smooth muscle cells underlying the 941
milk-producing alveolar cells? Single best answer question choose ONE true option only.
FSH YOUR ANSWER
Oestrogen
Oxytocin CORRECT ANSWER
Progesterone
Prolactin.

The suckling stimulus provided by the feeding infant at the nipple sends afferent fibres to the
hypothalamic paraventricular and supraoptic nuclei, with subsequent release of oxytocin from the
posterior pituitary. Oxytocin stimulates contraction of the smooth muscle cells underlying the milk-
producing alveolar cells. The contractions express milk from the alveolar areas into the duct system
leading to the surface at the nipple. Without this 'milk letdown' reflex, milk remains unavailable to the
infant in spite of adequate production.

236. You are reviewing a patient who has had a post-operative myocardial infarction. He is complaining of
severe pain in the left femur, having undergone a left femoral nailing. What effect is the pain most likely to
have on myocardial blood flow? Single best answer - select one answer only.
Decreased YOUR ANSWER (Correct Answer)
Increased in diastole
Increased in systole
Increased throughout cardiac cycle
No effect.

Myocardial blood flow is 250 ml/min at rest, represents 5% of the cardiac output and is dependent on
arterial pressure. Flow is only seen in diastole. Pain and vasopressin (antidiuretic hormone) reduce
myocardial blood flow.

237. During digestion of a fatty meal, which hormone causes contraction of the gall bladder and relaxation
of the sphincter of Oddi? Single best answer question choose ONE true option only.
Cholecystokinin YOUR ANSWER (Correct Answer)
Gastrin
Insulin
---------------------------------------------------------------------------------------------------------------------------------------------------------------------------------------------------------------
Dr Mohammed Shamsul Islam Khan, Medical Officer, Clinical Neuro-Surgery, National Institute of Neuro-Sciences and Hospital
Sher-E-Bangla Nagar, Dhaka-1207, Bangladesh. Mobile: +880 1713 455 662, +880 1685 811979. E-mail: drsikhan@gmail.com
MyPasTest MRCS A Online - Jan Exam 2015
10. Physiology; System Specific Physiology (399Qs)
----------------------------------------------------------------------------------------------------------------------------------
Secretin
Somatostatin.

Cholecystokinin secretion from the duodenal and jejunal mucosa is stimulated by the presence of fatty
acids, amino acids and peptides in the lumen of the duodenum and jejunum. As well as causing
contraction of the gall bladder and relaxation of the sphincter of Oddi, it stimulates release of pancreatic
Page |
enzymes, and increases the secretin mediated secretion of HC0 3- by pancreatic duct cells. Its release is 942
inhibited by somatostatin.

238. A 67-year-old man with end stage emphysema is admitted in respiratory failure and commenced on
intermittent positive pressure ventilation. Which of the following statements is correct with regard to the
pathophysiology of IPPV? Select one answer only.
Increases splanchnic blood flow YOUR ANSWER
Decreased cardiac output CORRECT ANSWER
Hepatic failure
Hypertension
Lowered intracranial pressure.

IPPV creates a positive intrathoracic pressure (this is normally negative) and a compression tamponade,
thereby reducing venous return, cardiac output and therefore blood pressure. With a reduction in cardiac
output there is reduction in liver, kidney and intestinal blood flow. The alveoli may be subjected to high
inflation pressures and result in subsequent PCO2 barotrauma and pneumothorax. IPPV causes an
increase in intracranial pressure, as do positive end-expiratory pressure (PEEP) ventilation and
obstruction of central venous drainage.

239. A 35-year-old male patient is admitted to the surgical assessment unit with right upper quadrant pain.
His past medical history consists of crohns disease, previous laparotomy for small bowel resection and
asthma. What is the most likely risk factor responsible for his symptoms? Single best answer - select one
answer only.
Age YOUR ANSWER
Diet
Lack of bile salts CORRECT ANSWER
Increased haemosiderin
Sex.
This patient has gallstones because of a lack of bile salts which are usually involved in fat absorption. He
has crohns disease which resulted in a small bowel resection which is most commonly the terminal
ileum where bile salts are most predominantly absorbed. Bile salts aid with the absorption of fat by
forming micelles. Without this the fat will sediment out to form gallstones. Other risk factors are said to
include obesity, female sex, age over 40 and premenopausal status.
240. Cardiac output is decreased: Single best answer question choose ONE true option only.
---------------------------------------------------------------------------------------------------------------------------------------------------------------------------------------------------------------
Dr Mohammed Shamsul Islam Khan, Medical Officer, Clinical Neuro-Surgery, National Institute of Neuro-Sciences and Hospital
Sher-E-Bangla Nagar, Dhaka-1207, Bangladesh. Mobile: +880 1713 455 662, +880 1685 811979. E-mail: drsikhan@gmail.com
MyPasTest MRCS A Online - Jan Exam 2015
10. Physiology; System Specific Physiology (399Qs)
----------------------------------------------------------------------------------------------------------------------------------
During stimulation of sympathetic nerves to the heart YOUR ANSWER
On cutting the vagus nerves to the heart
By increasing the end diastolic volume of the heart
As a result of decreased pressure within the carotid sinus
Page |
Upon assuming the upright position CORRECT ANSWER.
943
Stimulation of the sympathetic nervous system results in a rise in heart rate and stroke volume and
therefore cardiac output increases. Cutting the vagus nerves to the heart results in an increase in heart
rate because of the abolition of vagal tone and therefore cardiac output increases.

If the end diastolic volume of the heart (pre-load) is increased, under normal physiological circumstances,
cardiac output is increased by the Frank-Starling mechanism. The exception is in the failing heart where
the Law of LaPlace becomes more important and cardiac output actually falls.

Arterial blood pressure is homeostatically regulated through the action of baroreceptors, principally
located in the carotid sinus and the wall of the aortic arch. If the carotid sinus pressure is reduced, the
baroreceptors become inactive and lose their inhibitory effect on the vasomotor centre in the brainstem.
The result is activation of the sympathetic nervous system. This produces a rise in heart rate, stroke
volume, mean systemic filling pressure and venous return, leading to an increase in cardiac output and
return of the mean arterial blood pressure to its original value.

Cardiac output falls when one stands up due to the pooling of blood on the venous side of the circulation,
which has a large capacitance. Stepping out of a hot bath exacerbates this pooling effect because
superficial cutaneous veins dilate in response to heat, increasing their capacitance even further. Under
normal circumstances, activation of the baroreceptor reflex compensates to some degree, preventing
syncope. However, in the elderly, or in patients on anti-hypertensives, inadequate compensation from the
baroreceptor reflex may result in a vasovogal syncope, or othostatic hypotension.

241. Secretin is released into the circulation in response to low duodenal pH. Which of the following is the
best description with regard to the action of secretin? Single best answer - select one answer only.
Decreases water and bicarbonate secretion from the duodenum YOUR ANSWER
Decreases the release of bile from the gallbladder
Inhibits the contraction of the gallbladder
Reduces insulin released from the pancreas
Stimulates the pancreas to secrete a bicarbonate-rich fluid CORRECT ANSWER.

Secretin is secreted by S cells found in the proximal small bowel. It increases the secretion of bicarbonate
by the duct cells of the pancreas (causes secretion of a watery, alkaline pancreatic juice) and biliary tract,
augments the action of cholecystokinin, decreases gastric acid secretionand causes pyloric sphincter
---------------------------------------------------------------------------------------------------------------------------------------------------------------------------------------------------------------
Dr Mohammed Shamsul Islam Khan, Medical Officer, Clinical Neuro-Surgery, National Institute of Neuro-Sciences and Hospital
Sher-E-Bangla Nagar, Dhaka-1207, Bangladesh. Mobile: +880 1713 455 662, +880 1685 811979. E-mail: drsikhan@gmail.com
MyPasTest MRCS A Online - Jan Exam 2015
10. Physiology; System Specific Physiology (399Qs)
----------------------------------------------------------------------------------------------------------------------------------
contraction. The secretion of secretin isincreased by acid in the duodenum and by products of protein
digestion.

242. Aldosterone causes the following effect. Single best answer question choose ONE true option only.
Na+, H2O loss and K+ conservation in the distal tubule YOUR ANSWER
Page |
Na+, H2O conservation and K+ loss in the distal tubule CORRECT ANSWER
944
H+ loss in the distal tubule
Na+, H2O loss and K+ conservation in the ascending limb of the loop of Henle
Na+, H2O conservation and K+ loss in the ascending limb of the loop of Henle.

Aldosterone is a steroid hormone produced by the zona glomerulosa of the adrenal cortex. Its release is
stimulated mainly by the rennin/ angiotensin system. ACTH also causes aldosterone production, as do
hyponatraemia, hyperkalaemia and hypovolaemia.

It regulates Na+/ K+ and water balance, as well as playing a role acid base balance. At the distal tubule, it
acts to increase the Na+/K+ permeability of the luminal surface of the cells, and causes resorption of Na+
and H2O in exchange for K+, which is excreted into the urine. It also acts in the collecting ducts to secrete
H+ ions, thereby regulating plasma HCO3- levels.

243. A 35-year-old man has a history of recurring attacks of pancreatitis, eruptive xanthomas and
increased plasma triglyceride levels (2000 mg/ dl) associated with chylomicrons. Deficiency of which of
the following is the likely cause of these symptoms? Single best answer question choose ONE true
option only.
ApoB-100 receptor YOUR ANSWER
ApoB-48
HMG-CoA reductase
Low-density lipoprotein (LDL) receptors
Lipoprotein lipase CORRECT ANSWER.

The features presented in this vignette are suggestive of hypertriglyceridaemia secondary to lipoprotein
lipase deficiency. Lipoprotein lipase is an enzyme that hydrolyses lipids in lipoproteins, like those found
in chylomicrons and very-low-density lipoproteins (VLDL), into three fatty acids and one glycerol
molecule. It requires apoC-II as a co-factor. LPL deficiency leads to hypertriglyceridaemia. Lipoprotein
lipase is specifically found in endothelial cells lining the capillaries. Insulin is known to enhance LPL
synthesis in adipocytes and its placement in the capillary endothelium.

244. One of your patients who underwent an AAA repair is on ITU and has a pulmonary artery occlusion
pressure (PAOP) of 10mmHg. The PAOP most closely represents the pressure in which of the following?
Select one answer only.
The aorta YOUR ANSWER

---------------------------------------------------------------------------------------------------------------------------------------------------------------------------------------------------------------
Dr Mohammed Shamsul Islam Khan, Medical Officer, Clinical Neuro-Surgery, National Institute of Neuro-Sciences and Hospital
Sher-E-Bangla Nagar, Dhaka-1207, Bangladesh. Mobile: +880 1713 455 662, +880 1685 811979. E-mail: drsikhan@gmail.com
MyPasTest MRCS A Online - Jan Exam 2015
10. Physiology; System Specific Physiology (399Qs)
----------------------------------------------------------------------------------------------------------------------------------
The left atrium CORRECT ANSWER
The left ventricle
The right atrium
The right ventricle.
Page |
PAOP most closely represents the pressure in the left atrium. PAOP can be used to exclude oedema of 945
cardiac origin. It may be useful in cardiogenic shock to allow more accurate fluid management. The
pulmonary artery wedge pressure is elevated in cardiogenic shock.

245. A 62-year-old woman is admitted unwell after a fall and a prolonged period on the floor. She is
dehydrated and hypothermic and is requiring admission to the intensive care unit. Which of the following
is the most likely correct finding with regard to her gastric pH?
Decreased but no intervention required YOUR ANSWER
Decreased and should have prophylactic treatment CORRECT ANSWER
Increased but no intervention required
Increased and should have treatment
No Change.

The gastrointestinal tract is selectively impaired in patients who are in shock. It is one of the first organs
to be affected and one of the last to be restored by resuscitation. Measurement of stomach perfusion can
be estimated on the basis of the intramural pH. It is measured by tonometry: a catheter is filled with saline
and allowed to equilibrate with the luminal PCO2, which is assumed to be the same as the PCO2 of the
superficial mucosa.

Determination of the arterial bicarbonate concentration will allow calculation of the intramural pH using
the HendersonHasselbalch equation. This measurement reflects splanchnic tissue oxygenation and is
an indirect and early means of estimating global tissue oxygenation. As the pH is decreased, patients
should be treated with a prophylactic proton pump inhibitor to prevent stress ulceration.

246. The action potential of skeletal muscle? Single best answer question choose ONE true option only.
Has a prolonged plateau phase YOUR ANSWER
Spread inwards to all parts of the muscle via the T tubes CORRECT ANSWER
Causes immediate uptake of Ca into the sarcoplasmic reticulum
Is longer than the action potential of cardiac muscle
Is not essential for contraction.
The action potential of the skeletal muscle spreads out from the motor end plate, through the T tube
system this causes mobilization of Ca2+ from the sarcoplasmic reticulum to the cytoplasm and this action
potential is essential for contraction.
---------------------------------------------------------------------------------------------------------------------------------------------------------------------------------------------------------------
Dr Mohammed Shamsul Islam Khan, Medical Officer, Clinical Neuro-Surgery, National Institute of Neuro-Sciences and Hospital
Sher-E-Bangla Nagar, Dhaka-1207, Bangladesh. Mobile: +880 1713 455 662, +880 1685 811979. E-mail: drsikhan@gmail.com
MyPasTest MRCS A Online - Jan Exam 2015
10. Physiology; System Specific Physiology (399Qs)
----------------------------------------------------------------------------------------------------------------------------------
The action potential of cardiac muscle is longer than that of the skeletal muscle and has plateau phase.

247. You have a 60-year-old patient who had a previous myocardial infarction just over a year ago. He is
going to undergo an elective surgical abdominal aortic aneurysm repair. On which post-operative day is
myocardial ischaemia most likely to occur?
Day 1 YOUR ANSWER Page |
Day 2 946
Day 3 CORRECT ANSWER
Day 4
Day 5.

Perioperative myocardial ischaemia occurs most commonly in those with a history of a previous
infarction, in the intraoperative period and on day 3 postoperatively. The more recent the MI, the greater
the risk of reinfarction. Close intraoperative monitoring has been shown to reduce the risk of infarction in
the higher-risk patient. The risk in patients without a cardiac history is < 1 in 70,000.

248. Splenectomy increases susceptibility to which of the following organisms? Single best answer
question choose ONE true option only.
Streptococcus pyogenes YOUR ANSWER
Schistosoma haematobium
Bacteroides fragilis
Neisseria meningitidis CORRECT ANSWER
Staphylococcus aureus.

The spleen plays an important role in the removal of dead and dying erythrocytes and in the defence
against microbes. Removal of the spleen (splenectomy) leaves the host susceptible to a wide array of
pathogens, but especially to encapsulated organisms.

Certain bacteria have evolved ways of evading the human immune system. One way is through the
production of a slimy capsule on the outside of the bacterial cell wall. Such a capsule resists
phagocytosis and ingestion by macrophages and neutrophils. This allows them not only to escape direct
destruction by phagocytes, but also to avoid stimulating T-cell responses through the presentation of
bacterial peptides by macrophages. The only way that such organisms can be defeated is by making them
more palatable by coating their capsular polysaccharide surfaces in opsonising antibody.

The production of antibody against capsular polysaccharide primarily occurs through T-cell independent
mechanisms. The spleen plays a central role in both the initiation of the antibody response and the
phagocytosis of opsonised encapsulated bacteria from the bloodstream. This helps to explain why the
asplenic individuals are most susceptible to infection from encapsulated organisms, notably

---------------------------------------------------------------------------------------------------------------------------------------------------------------------------------------------------------------
Dr Mohammed Shamsul Islam Khan, Medical Officer, Clinical Neuro-Surgery, National Institute of Neuro-Sciences and Hospital
Sher-E-Bangla Nagar, Dhaka-1207, Bangladesh. Mobile: +880 1713 455 662, +880 1685 811979. E-mail: drsikhan@gmail.com
MyPasTest MRCS A Online - Jan Exam 2015
10. Physiology; System Specific Physiology (399Qs)
----------------------------------------------------------------------------------------------------------------------------------
Streptococcus pneumoniae (pneumococcus), Neisseria meningitidis (meningococcus)and Haemophilus
influenzae.

The risk of acquiring such infections is reduced by immunising individuals against such organisms and
by placing patients on prophylactic penicillin, in most cases for the rest of their lives. In addition, asplenic
individuals should be advised to wear a MedicAlert bracelet to warn other health care professionals of Page |
their condition. 947
249. Regarding thyroid gland physiology which of the following is correct? Select one answer only.
greater amounts of tri-iodothyronine (T3) are secreted thanthyroxine (T4) YOUR ANSWER
iodine is transported into the gland along a concentrationgradient
tri-iodothyronine (T3) is mainly bound to albumin in plasma
thyroxine-binding globulin is produced in the liver CORRECT ANSWER
thyroxine has a half-life of approximately 24 hours.

Iodine is actively transported into the thyroid gland against a concentration gradient; normal iodine
concentration in the thyroid is 30 times greater than that in blood. Thyroid hormone secreted into the
blood consists of predominantly T4 and T3 in a ratio of 93%:7%. However, approximately 50% of T4
produced becomes deiodinated to T3 within a few days of secretion. Both T3 and T4 are mainly bound to
thyroxine-binding globulin, which is synthesised in the liver. T4 has a half-life of approximately 15 days,
with some activity persisting for up to 6 weeks.

250. A 67-year-old life-long smoker with known heart failure is admitted to the acute medical ward with
shortness of breath and peripheral oedema. Your consultant asks you to note his jugular venous
pressure and proceeds to quiz you on 'a', 'c' & 'v' waves. Which of the following is correct? Select one
answer only.
a wave refers to atrial contraction YOUR ANSWER (Correct Answer)
c wave refers to closure of the mitral and tricuspid valves
v wave refers to a full atrium
v wave occurs just before the carotid pulse
a wave is elevated in atrial fibrillation.
The jugular venous pulsation has a double waveform. The a wave corresponds to atrial contraction and
ends synchronously with the carotid artery pulse. The c wave occurs when the ventricles begin to
contract and is caused by bulging of the atrioventricular (AV) valves backwards towards the atria. The v
wave is seen when the tricuspid valve is closed, just before ventricular contraction with and just after
the carotid pulse. The v wave represents the gradual build-up of blood in the atria while the AV valves
are closed during ventricular contraction. The absence of a waves is a feature of atrial fibrillation.

251. With regards to the oxygen-haemoglobin dissociation curve, what results in a decreased affinity for
oxygen? Select one answer only.
---------------------------------------------------------------------------------------------------------------------------------------------------------------------------------------------------------------
Dr Mohammed Shamsul Islam Khan, Medical Officer, Clinical Neuro-Surgery, National Institute of Neuro-Sciences and Hospital
Sher-E-Bangla Nagar, Dhaka-1207, Bangladesh. Mobile: +880 1713 455 662, +880 1685 811979. E-mail: drsikhan@gmail.com
MyPasTest MRCS A Online - Jan Exam 2015
10. Physiology; System Specific Physiology (399Qs)
----------------------------------------------------------------------------------------------------------------------------------
Decrease in red cell 2,3-DPG YOUR ANSWER
Hypocapnia
Metabolic alkalosis
Pyrexia CORRECT ANSWER
Page |
Respiratory alkalosis.
948
The oxygenhaemoglobin dissociation curve is sigmoid-shaped, and a right shift is associated with a
decreased affinity for oxygen. The curve shifts to the right with an increase in PCO2, temperature, 2,3-DPG
and an increase in hydrogen-ion concentration (decreased pH). The curve shifts to the left with decreased
PCO2, temperature, 2,3-DPG and a reduced hydrogen-ion concentration (increased pH).

252. A 3-week-old baby exhibits projectile vomiting shortly after feeding and failure to thrive. On
examination an olive-shaped mass is palpable in the right upper quadrant of the abdomen. A clinical
diagnosis of pyloric stenosis is made. What biochemical laboratory features would support the
diagnosis? Single best answer question choose ONE true option only.
Hypokalaemia, metabolic alkalosis, low urinary pH YOUR ANSWER (Correct Answer)
Hyperkalaemia, metabolic acidosis, high urinary pH
Hypokalaemia, metabolic acidosis, high urinary pH
Hyperkalaemia, metabolic alkalosis, low urinary pH
Hypokalaemia, metabolic alkalosis, high urinary pH.

Following a diagnosis of pyloric stenosis, the first concern is to correct the metabolic abnormalities that
invariably coexist with the condition. The serum electrolytes and capillary gases should be measured and
corrected prior to surgery.

With prolonged vomiting, the infant becomes dehydrated, with a hypochloraemic metabolic alkalosis. The
alkalosis is a result of loss of unbuffered hydrogen ions in gastric juice with concomitant retention of
bicarbonate.

Fluid loss stimulates renal sodium reabsorption, but sodium can only be reabsorbed either with chloride,
or in exchange for hydrogen and potassium ions (to maintain electroneutrality). Gastric juice has a high
concentration of chloride and patients losing gastric secretions become hypochloraemic. This means that
less sodium than normal can be reabsorbed with chloride.

However, it appears that the defence of extracellular fluid volume takes precedence over acid-base
homeostasis and further sodium reabsorption occurs in exchange for hydrogen ions (perpetuating the
alkalosis) and potassium ions (leading to potassium depletion). This explains the apparently paradoxical
finding of acidic urine in patients with pyloric stenosis. Potassium is also lost in the gastric juice and thus
patients frequently become potassium-depleted and yet are losing potassium in their urine.
---------------------------------------------------------------------------------------------------------------------------------------------------------------------------------------------------------------
Dr Mohammed Shamsul Islam Khan, Medical Officer, Clinical Neuro-Surgery, National Institute of Neuro-Sciences and Hospital
Sher-E-Bangla Nagar, Dhaka-1207, Bangladesh. Mobile: +880 1713 455 662, +880 1685 811979. E-mail: drsikhan@gmail.com
MyPasTest MRCS A Online - Jan Exam 2015
10. Physiology; System Specific Physiology (399Qs)
----------------------------------------------------------------------------------------------------------------------------------
253. You are participating in your hospitals annual medics Vs surgeons 5-a-side football game. Which of
the following is associated with the increased cardiac output as a result of exercise? Select one answer
only.
Raised central venous pressure (CVP) YOUR ANSWER
Increase end-systolic volume
Page |
A small increase in end-diastolic volume
949
Decrease stroke volume
Increased myocardial contractility CORRECT ANSWER.

During vigorous exercise the CVP would remain unchanged or may fallwith dehydration. In exercise, the
stroke volume increases, with a decrease in end-systolic volume. With the increase in heart rate, the end-
diastolic volume may also decrease slightly as less time is spent by the heart in diastole.

254. A 46-year-old man is admitted in acute pulmonary oedema and undergoes pulmonary artery wedge
pressure monitoring. Which of the following statements is correct about pulmonary artery wedge
pressure? Single best answer - select one answer only.
It is usually 0-5mmHg YOUR ANSWER
It is low in cardiogenic shock
It correlates with left ventricular end systolic pressure
It provides an indirect measure of the right atrial pressure
It may be a useful parameter to measure following cardiac surgery CORRECT ANSWER.

Pulmonary artery wedge pressure (PAWP) usually lies between 6-12 mmHg. It is elevated in cardiogenic
shock. Measuring PAWP may help to differentiate between cardiogenic and non-cardiogenic pulmonary
oedema. It may be useful in cardiogenic shock to allow more accurate fluid management. Therefore it is
sometimes measured following cardiac surgery. Pulmonary artery wedge pressure correlates with left
ventricular end diastolic pressure.

255. A 21-year-old man comes off his motorbike at 65 mph and is admitted with a large subdural
haematoma. He undergoes intracranial pressure monitoring. The early phase of increased intracranial
pressure (ICP) causes which of the following? Single best answer - select one answer only.
Decreased pulse pressure YOUR ANSWER
Hyperventilation
Hypotension
Pupillary constriction CORRECT ANSWER
Tachycardia.
Raised intracranial pressure may give rise to Cushing's response with decrease in respiratory rate,
bradycardia, hypertension and increased pulse pressure. Rising intracranial pressure may result in
---------------------------------------------------------------------------------------------------------------------------------------------------------------------------------------------------------------
Dr Mohammed Shamsul Islam Khan, Medical Officer, Clinical Neuro-Surgery, National Institute of Neuro-Sciences and Hospital
Sher-E-Bangla Nagar, Dhaka-1207, Bangladesh. Mobile: +880 1713 455 662, +880 1685 811979. E-mail: drsikhan@gmail.com
MyPasTest MRCS A Online - Jan Exam 2015
10. Physiology; System Specific Physiology (399Qs)
----------------------------------------------------------------------------------------------------------------------------------
Cheyne-Stokes respiration and subsequent respiratory depression, but not hyperventilation. Pupils
constrict initially due to external compression of overlying sympathetic fibres, but later dilate when the
oculomotor nerve is compressed. Complete third nerve palsy may also occur. As the herniation
progresses, the contralateral oculomotor nerve may be compressed, producing bilateral pupil dilation.

256. A patient receives too many infusions after an operation resulting in a 20% increase in his blood Page |
volume. What is the physiological process that is most likely to correct this abnormality? Single best 950
answer question choose ONE true option only.
Reduced activity of arterial pressure sensors YOUR ANSWER
Increased activity of renal sympathetic nerves
Aldosterone release
Atrial natriuretic peptide (ANP) release CORRECT ANSWER
Venous dilatation.

The atria contain granulated cells that release peptides, atrial natriuretic peptide (ANP), in response to
stretch. This natriuretic agent also relaxes the peripheral vasculature and thereby opposes the actions of
the sympathetic and reninangiotensin systems.

257. A patient has chronic renal impairment, with a plasma creatinine of 225mol/l and the glomerular
filtration rate (GFR) of 25ml/min. Concerning renal function: Single best answer quest ion choose ONE
true opt ion only.
The GFR is the main factor determining the rate of urine production YOUR ANSWER
GFR can be measured by para-aminohippuric acid (PAH)
The normal GFR is 50mls/min
Inulin clearance can be used to estimate GFR CORRECT ANSWER
A normal plasma creatinine implies normal renal function.

In the normal adult human the GFR (or normal renal clearance) averages 125ml/min, or 180 litres/day. The
entire plasma volume (about 3 litres) can therefore be filtered and processed by the kidney approximately
60 times each day. The rate of urine production in humans is dominated by tubular function and not by
GFR. The GFR remains relatively constant through autoregulation.

After 35 years of age, GFR falls at about 1ml/min/year. By the age of 80, GFR has fallen to about 50% of its
youthful level. GFR can decrease by as much as 50% before plasma creatinine rises beyond the normal
range. Consequently, a normal creatinine does not necessarily imply normal renal function, although a
raised creatinine does usually indicate impaired renal function.

A substance used to measure the GFR must be freely filtered at the glomerulus, not be secreted by the
tubules, not be reabsorbed, not be metabolised or synthesized in the body, not alter the renal
function/GFR, be non-toxic and soluble in plasma. Such a substance is the polyfructose molecule, inulin.
However, it is too cumbersome to use in routine clinical practice. Instead, GFR is more commonly
---------------------------------------------------------------------------------------------------------------------------------------------------------------------------------------------------------------
Dr Mohammed Shamsul Islam Khan, Medical Officer, Clinical Neuro-Surgery, National Institute of Neuro-Sciences and Hospital
Sher-E-Bangla Nagar, Dhaka-1207, Bangladesh. Mobile: +880 1713 455 662, +880 1685 811979. E-mail: drsikhan@gmail.com
MyPasTest MRCS A Online - Jan Exam 2015
10. Physiology; System Specific Physiology (399Qs)
----------------------------------------------------------------------------------------------------------------------------------
quantified by measuring the 24-hour urinary creatinine excretion. Para-aminohippuric acid is used to
measure renal blood flow and not GFR.

258. Why is a sudden loud sound more likely to damage the cochlea than a loud sound that develops
slowly? Single best answer question choose ONE true option only.
A sudden sound carries more energy YOUR ANSWER Page |
A sudden sound directly damages the vestibular nerve 951
The fluid pressure in the scala tympani decreases as a sound becomes louder
There is a latent period before the attenuation reflex can occur CORRECT ANSWER
The tympanic membrane becomes flaccid as a sound becomes louder.

When a loud sound is transmitted into the central nervous system, an attenuation reflex occurs after a
latent period of 40-80 ms. The reflex involves the contraction of two muscles that pull the malleus and
stapes toward each other, thereby causing the entire ossicular system to develop a high degree of
rigidity. In turn, the ossicular conduction of low-frequency sounds to the cochlea can be reduced by as
much as 30-40 decibels. Since loud sounds are usually low frequency sounds, the attenuation reflex can
protect the cochlea from damage caused by loud sounds when they develop slowly.

259. A 52-year-old woman undergoes investigation for jaundice. She first noticed this symptom 2 months
ago, but for 4 months prior to that, she had been experiencing generalised pruritus. The results of liver
function tests are as follows: serum bilirubin 325 mol/l, aspartate aminotransaminase 55 U/l (1542),
alkaline phosphatase 436 U/l (80150), gamma-glutamyltransferase 82 U/l (1151), albumin 36 g/l, total
protein 82 g/l. Which of the following is the most likely diagnosis? Single best answer question choose
ONE true option only.
Alcoholic cirrhosis YOUR ANSWER
Carcinoma of the head of the pancreas
Cholangiocarcinoma
Primary biliary cirrhosis CORRECT ANSWER
Primary sclerosing cholangitis.
The high alkaline phosphatase concentration suggests cholestatic jaundice. Alcoholic cirrhosis is
common but is unlikely (though not excluded) by the only slightly elevated -glutamyltransferase.
Cholangiocarcinoma is a rare tumour. Carcinoma of the head of the pancreas frequently presents in this
manner (though weight loss is often present also), but the slight elevation in total protein with lownormal
albumin suggests a high globulin concentration, which suggests autoimmune liver disease. Primary
sclerosing cholangitis is a possibility but is commoner in men than women (3:1), and in 75% of cases is
associated with inflammatory bowel disease. Primary biliary cirrhosis is more common in women.

260. Liver is the main site of glycogenolysis and gluconeogenesis. Which of the following hormones
increases hepatic glycogenolysis and promote gluconeogenesis? Single best answer question choose
ONE true option only.
Cholecystokinin YOUR ANSWER
---------------------------------------------------------------------------------------------------------------------------------------------------------------------------------------------------------------
Dr Mohammed Shamsul Islam Khan, Medical Officer, Clinical Neuro-Surgery, National Institute of Neuro-Sciences and Hospital
Sher-E-Bangla Nagar, Dhaka-1207, Bangladesh. Mobile: +880 1713 455 662, +880 1685 811979. E-mail: drsikhan@gmail.com
MyPasTest MRCS A Online - Jan Exam 2015
10. Physiology; System Specific Physiology (399Qs)
----------------------------------------------------------------------------------------------------------------------------------
Glucagon CORRECT ANSWER
Growth hormone
Insulin
Secretin.
Page |
Glucagon is a 29-amino acid polypeptide acting as an important hormone in carbohydrate metabolism. 952
The polypeptide has a molecular mass of 3485 Da and was discovered in 1923 by Kimball and Murlin.
Glucagon helps maintain the level of glucose in the blood by binding to glucagon receptors on
hepatocytes, causing the liver to release glucose - stored in the form of glycogen - through a process
known as glycogenolysis. As these stores become depleted, glucagon then encourages the liver to
synthesise additional glucose by gluconeogenesis. This glucose is released into the bloodstream. Both of
these mechanisms lead to glucose release by the liver, preventing the development of hypoglycaemia.

261. You are assessing a 24-year-old male who has been stabbed in the right side of the chest. Which of
the following signs would lead you to suspect tamponade? Select one answer only.
Arrthythmias YOUR ANSWER (Correct Answer)
Decreased venous pressure on inspiration
Empty neck veins
Loud heard sounds
A midsystolic click.

Findings consistent with tamponade include Becks triad, Kussmauls sign, and pulsus paradoxus.
Becks triad is muffled heart sounds, distended neck veins (raised JVP), and hypotension. Kussmauls
sign is increased venous pressure with inspiration in spontaneous breathing. Pulsus paradoxus is a
pulse that fades on inspiration. An ejection click is associated with mitral valve prolapse.

262. You have a post-op patient on the intensive care unit who has had a Swan-Ganz catheter inserted
peri-operatively. What can directly be measured using the Swan-Ganz catheter? Select one answer only.
Cardiac index YOUR ANSWER
End-tidal CO2
FiO2
Left ventricular stroke work
Pulmonary artery occlusion pressure CORRECT ANSWER.

Pulmonary artery occlusion, (or wedge), pressure can be directly measured by SwanGanz
catheterisation. Cardiac index and left ventricular stroke work can be derived from these measurements
using the Fick principle. FiO2, (concentration of inspired oxygen), and end-tidal CO2, (concentration of
expired CO2), cannot be measured by SwanGanz catheterisation.

---------------------------------------------------------------------------------------------------------------------------------------------------------------------------------------------------------------
Dr Mohammed Shamsul Islam Khan, Medical Officer, Clinical Neuro-Surgery, National Institute of Neuro-Sciences and Hospital
Sher-E-Bangla Nagar, Dhaka-1207, Bangladesh. Mobile: +880 1713 455 662, +880 1685 811979. E-mail: drsikhan@gmail.com
MyPasTest MRCS A Online - Jan Exam 2015
10. Physiology; System Specific Physiology (399Qs)
----------------------------------------------------------------------------------------------------------------------------------
263. A 42-year-old man is put on a proton-pump inhibitor to suppress symptoms of oesophagitis. The cell
and membrane biology of the gastric acid pump has which of the following features? Single best answer
question choose ONE true option only.
Histamine-stimulated acid production is independent of the proton pump YOUR ANSWER
The proton is exchanged with magnesium ions Page |
Acetylcholine-stimulated acid production is independent of the proton pump 953
The proton pump spans the apical membrane of the gastric parietal cell CORRECT ANSWER
The proton pump spans the basolateral membrane of the gastric parietal cell.

The H+K+-ATPase is embedded in the apical membrane. The channel is susceptible to agents that bind to
cysteine residues (particularly 813 and 822). The proton pump is the final common pathway of histamine
and acetylcholine-stimulated production, and particularly explains the enhanced efficacy of proton-pump
inhibitors in comparison to H2 antagonists or acetylcholine antagonists (eg pirenzepine).

264. You are treating a patient who sustained a moderate blunt head injury, and who is having intracranial
pressure (ICP) monitoring. The ICP is normal. Between what mean systolic BP values is cerebral blood
flow auto-regulated? Select one answer only.
50-140mmHg YOUR ANSWER
50-160mmHg CORRECT ANSWER
50-200mmHg
80-160mmHg
80-200mmHg.

The cerebral blood flow (CBF) in normal adults is approximately 5055 ml/100 g of brain tissue/min. In
children, the flow is different: At 1 year of age the CBF is similar to adult levels; however, at 5 years of
age, the CBF is approximately 90 ml/100 g/min. This level gradually declines to adult levels by the mid- to
late teens. In normal physiological conditions, the CBF is usually maintained by both pressure and
chemical autoregulation.

The pressure autoregulation is facilitated by reflex constriction or dilatation of the precapillary cerebral
vasculature in response to mean systolic blood pressures between 50160 mm Hg. Chemical
autoregulation is maintained by constriction or dilatation of the precapillary vessels in response to
changes in the p(O2) or p(CO2 ) of the blood. Severe traumatic brain injury disrupts both these
autoregulatory mechanisms.

Cerebral perfusion pressure (CPP) is equal to mean arterial blood pressure (MABP) minus ICP. The MABP
in a normally perfused patient is about 9095 mmHg; the normal ICP in adults is 1015 mmHg. Hence the
normal CPP is approximately 7080 mmHg. It should ideally be maintained at a level of at least 6070 mm
Hg to ensure adequate CBF. It should also be noted that any rise in ICP would cause a corresponding fall
in CPP.

---------------------------------------------------------------------------------------------------------------------------------------------------------------------------------------------------------------
Dr Mohammed Shamsul Islam Khan, Medical Officer, Clinical Neuro-Surgery, National Institute of Neuro-Sciences and Hospital
Sher-E-Bangla Nagar, Dhaka-1207, Bangladesh. Mobile: +880 1713 455 662, +880 1685 811979. E-mail: drsikhan@gmail.com
MyPasTest MRCS A Online - Jan Exam 2015
10. Physiology; System Specific Physiology (399Qs)
----------------------------------------------------------------------------------------------------------------------------------
265. You are reviewing a patient in the neurosurgical clinic with muscle spasms. Which of the following
are true regarding skeletal muscle contraction? Single best answer - select one answer only.
Occurs spontaneously in response to high serum calcium levels YOUR ANSWER
Is prolonged and maintained in fast twitch fibres
Depends only on thick myofilaments moving Page |
Occurs in response to depolarisation of the sarcolemma CORRECT ANSWER 954
Ends when calcium ions are released from the sarcoplasmic reticulum.

Depolarisation of the plasma membrane triggers passive release of calcium ions from the sarcoplasmic
reticulum through specific channels along a gradient. The calcium ions bind to specific sites on the thin
filaments. Binding causes formation and breakage of crosslinks between thick and thin filaments. The
cycling of crosslinks results in movement of the myofilaments along each other. This is sustained until
the calcium ions are actively transported back into the sarcoplasmic reticulum and relaxation occurs.

There are two types of muscle fibres, slow and fast which produce prolonged, maintained contraction and
transient fatigueable contraction respectively. Whole muscle is composed of mixtures of fast and slow
twitch fibres in varying amounts. Sustained muscle spasm, tetany, occurs in response to low serum
calcium levels. This can occur following removal of the parathyroid glands at thyroidectomy.

266. One of your patients on ITU has a Swan-Ganz catheter in situ. The patients cardiac output has been
calculated as 6L/min. Which of the following factors increases cardiac output? Single best answer
select one answer only.
Acidosis YOUR ANSWER
Histamine CORRECT ANSWER
Long-term acclimatisation at altitude
Rapid arrhythmias
Standing from a lying position.

Cardiac output = heart rate x stroke volume. Cardiac output is decreased upon standing from a lying
position as venous return to the heart is suddenly reduced. Above a certain heart rate the stroke volume
falls, so reducing cardiac output. Cardiac output is increased only in the short term, when high altitude is
first reached. After this initial rise, cardiac output gradually returns to normal but the haematocrit level
increases to improve the bloods oxygen-carrying capacity. Acidosis and alkalosis decrease contractility,
thus decreasing stroke volume. Other causes of increased cardiac output include arteriovenous shunts,
hyperthyroidism, severe anaemia and Pagets disease.

267. A 65-year-old diabetic patient has poorly controlled diabetes mellitus. Which of the following
biochemical changes is associated with inadequately controlled diabetes? Single best answer question
choose ONE true option only.
Abnormally rapid fall of blood glucose following a meal YOUR ANSWER
Abnormally low concentration of fatty acids in blood
---------------------------------------------------------------------------------------------------------------------------------------------------------------------------------------------------------------
Dr Mohammed Shamsul Islam Khan, Medical Officer, Clinical Neuro-Surgery, National Institute of Neuro-Sciences and Hospital
Sher-E-Bangla Nagar, Dhaka-1207, Bangladesh. Mobile: +880 1713 455 662, +880 1685 811979. E-mail: drsikhan@gmail.com
MyPasTest MRCS A Online - Jan Exam 2015
10. Physiology; System Specific Physiology (399Qs)
----------------------------------------------------------------------------------------------------------------------------------
Increased protein breakdown CORRECT ANSWER
High rate of glycogen synthesis in liver
Decreased gluconeogenesis.

Metabolic effects of insulin include: stimulation of the formation of glycogen from glucose and inhibition Page |
of glycogenolysis; stimulation of fatty acid (FA) production from stored lipids and inhibition of FA release
into the blood; stimulation of FA uptake and storage; inhibition of protein catabolism and of 955
gluconeogenesis, in which glucose is synthesised (mostly from some amino acid, released by protein
catabolism). A lack of insulin or inadequately controlled diabetes will result in reversal of all these effects
of insulin, all of which contribute to increasing blood glucose levels, increased fat metabolism and
protein degradation.

268. You are treating a 20-year-old man, who weighs 70kg, who has been stabbed in the right side of the
chest. 1.5L of blood have collected in the chest drain. The normal adult blood volume is approximately
what percentage of the body weight? Select one answer only.
3% YOUR ANSWER
5%
7% CORRECT ANSWER
9%
11%.

Haemorrhage is defined as an acute loss of circulating blood. The normal adult blood volume is
approximately 7% of body weight (a 70-kg man has a 5-litre circulating blood volume), and in children it is
approximately 89% of their body weight. The blood volume of obese patients is estimated using their
ideal body weight, as their true weight gives an overestimation of blood volume.

269. You have a patient who is going to have hyperbaric oxygen therapy and are explaining this treatment
to him. Which of the following is correct regarding Hyperbaric oxygen therapy? Select one answer only.
Can be administered using an anaesthetic breathing circuit YOUR ANSWER
Can be used in the treatment of acute oxygen toxicity
Can be used in the treatment of carbon monoxide poisoning CORRECT ANSWER
Typically consists of one 20-minute session
Typically consists of one 30-minute session.

Hyperbaric oxygen refers to an oxygen tension significantly greater than 1 kPa. An anaesthetic breathing
circuit can only achieve approximately 1kPa, so a pressurised chamber is required for the administration
of hyperbaric oxygen. Typically, treatment involves several sessions of an hour or more. Among other
effects, hyperbaric oxygen therapy is thought to improve oxygen delivery to cells and to reduce smoke-
induced pulmonary oedema. Uses include the treatment of gas gangrene, necrotising fasciitis and carbon
monoxide poisoning (though there has been conflicting evidence as to its efficacy). Exposure to more
than 2 kPa tension of oxygen may cause acute oxygen toxicity, which can manifest as convulsions.
---------------------------------------------------------------------------------------------------------------------------------------------------------------------------------------------------------------
Dr Mohammed Shamsul Islam Khan, Medical Officer, Clinical Neuro-Surgery, National Institute of Neuro-Sciences and Hospital
Sher-E-Bangla Nagar, Dhaka-1207, Bangladesh. Mobile: +880 1713 455 662, +880 1685 811979. E-mail: drsikhan@gmail.com
MyPasTest MRCS A Online - Jan Exam 2015
10. Physiology; System Specific Physiology (399Qs)
----------------------------------------------------------------------------------------------------------------------------------
270. You are treating a patient who underwent a heart transplant 7 days ago. The patient has had a routine
recovery, with no complications. When assessing their pulse what would you normally expect to find?
Single best answer - select one answer only.
Atrial fibrillation rate 100bpm YOUR ANSWER
Sinus rhythm rate 70bpm
Page |
Sinus bradycardia rate 50 bpm
956
Sinus tachycardia rate 100bpm CORRECT ANSWER
Supraventricular tachycardia rate 120bpm.

In a denervated or transplanted heart the vagus nerves have been severed and therefore do not exert a
braking effect on the cardiac rate; this leads to an elevated resting heart rate, with a sinus tachycardia.

271. You review a patient who underwent emergency laparotomy for abdominal sepsis 36 hours ago. The
patient clearly has acute respiratory distress syndrome (ARDS). Which of the following would you expect
in ARDS? Select one answer only.
Focal infiltration on chest Xray YOUR ANSWER
Leakage of protein rich oedema fluid into the lung tissue CORRECT ANSWER
Only late occurrence of tachypnoea and tachycardia
Steroids promote recovery in the late phase
Thinning of the alveolar membrane.

Acute respiratory distress syndrome (ARDS) is a well-recognised complication of sepsis, trauma and
other pulmonary insults, both direct and indirect. The commonest clinical signs are tachypnoea,
tachycardia, cyanosis and high-pitched coarse crepitations on auscultation. The alveolar membrane
becomes thickened. Differential diagnoses include left ventricular failure, bronchitis, pneumonia and
asthma.

The underlying pathological process is leakage of cells and fluid through the capillary walls of the alveoli.
The alveolar membrane thickens and ultimately fibroses. In the early stages, tachycardia and tachypnoea
may be the only features. Later there is cyanosis and worsening dyspnoea. Typically the chest Xray
shows diffuse infiltration. The cornerstones of treatment are support of the circulation, respiratory
support by ventilation and the use of appropriate antibiotics. Steroids have been used in the treatment of
ARDS but there is no good evidence to support their use.

272. Which of the following respiratory physiology tests would be consistent with a diagnosis of
moderately established cryptogenic fibrosing alveolitis? Single best answer question choose ONE true
option only.
Diffusion capacity decreased, FEV1/FVC normal, total lung capacity reduced YOUR ANSWER (Correct
Answer)
Diffusion capacity increased, FEV1/FVC normal, total lung capacity increased
---------------------------------------------------------------------------------------------------------------------------------------------------------------------------------------------------------------
Dr Mohammed Shamsul Islam Khan, Medical Officer, Clinical Neuro-Surgery, National Institute of Neuro-Sciences and Hospital
Sher-E-Bangla Nagar, Dhaka-1207, Bangladesh. Mobile: +880 1713 455 662, +880 1685 811979. E-mail: drsikhan@gmail.com
MyPasTest MRCS A Online - Jan Exam 2015
10. Physiology; System Specific Physiology (399Qs)
----------------------------------------------------------------------------------------------------------------------------------
Diffusion capacity normal, FEV1/FVC reduced, total lung capacity reduced
Diffusion capacity decreased, FEV1/FVC normal, total lung capacity n ormal
Diffusion capacity decreased, FEV1/FVC increased, total lung capacity increased.

Diffusion capacity is characteristically decreased in restrictive lung disorders. FEV 1/FVC reduced would Page |
be seen in obstructive airways disease, which would be reversible in asthma and irreversible in COPD. In
restrictive conditions FEV1/FVC ratio is normal or increased. Total lung capacity is reduced in restrictive
957
lung disease, whilst it is normal or increased in obstructive airways disease.

273. A 45-year-old lady presents with generalised weakness and fatigue. She has a history of renal colic.
Her calcium is found to be raised at 3.1. A lower parathyroid adenoma is suspected. Which of the
following statements is most likely to be true? Single best answer select one answer only.
This parathyroid gland would have developed from the second branchial pouch YOUR ANSWER
This would be an unusual case, as the majority of primary hyperparathyroidism is due to diffuse hypertrophy
of the parathyroids
The adenoma could be localised by a combination of ultrasound and Sestamibi radinucleotide scan
CORRECT ANSWER
This ladys weakness and fatigue are incidental to the hypercalcaemia
Parathyroid tissue cannot be implanted elsewhere in the body after total parathyroid excision.

The upper and lower parathyroids develop from the fourth and third branchial pouches, respectively.
Primary hyperparathyroidism is usually due to a solitary secreting parathyroid adenoma.
Hyperparathyroidism presents as:
1. bones (arthralgia and osteoporosis)
2. stones (renal)
3. abdominal groans (pancreatitis, peptic ulcers and constipation)
4. moans (depression, fatigue and weakness)

Pre-operative parathyroid adenoma localisation can be done by:


1. Ultrasound
2. Sestamibi (MIBI) radionucleotide scan
3. technetium/thallium scan
Tertiary hyperparathyroidism occurs in renal failure, where the parathyroids function autonomously. In
such cases, subcutaneous reimplantation of half a parathyroid in the forearm allows easy access if
further resection is required.

274. The following factors stimulate renin release. Single best answer question choose ONE true option
only.
Is decreased by isoflurane in general anaesthesia YOUR ANSWER
Propranolol
Increase in plasma K+ concentration
---------------------------------------------------------------------------------------------------------------------------------------------------------------------------------------------------------------
Dr Mohammed Shamsul Islam Khan, Medical Officer, Clinical Neuro-Surgery, National Institute of Neuro-Sciences and Hospital
Sher-E-Bangla Nagar, Dhaka-1207, Bangladesh. Mobile: +880 1713 455 662, +880 1685 811979. E-mail: drsikhan@gmail.com
MyPasTest MRCS A Online - Jan Exam 2015
10. Physiology; System Specific Physiology (399Qs)
----------------------------------------------------------------------------------------------------------------------------------
Angiotensin II
Salt depletion CORRECT ANSWER.

A decrease in blood pressure and salt depletion stimulates renin release. -blockers and increased K+
levels inhibit renin release. Angiotensin II inhibits renin release through a negative feed back mechanism. Page |
275. A 21-year old male is admitted following an RTA in which he sustains an open tibial fracture with 958
heavy blood loss. He develops low blood pressure, and is catheterised and resuscitated for low urine
output. Which of the following is correct in acute renal failure? Single best answer - select one answer
only.
Arterial pCO2 is increased YOUR ANSWER
Blood pH is increased
Plasma bicarbonate is increased
Potassium levels are decreased
Respiratory rate is increased CORRECT ANSWER.

Acute renal failure results in a metabolic acidosis. So, the blood pH and plasma bicarbonate level drop.
To compensate, the respiratory rate increases and carbon dioxide levels are lowered. The biochemical
features of ARF include a hyperkalaemia, hyponatraemia and elevated urea and creatinine. Phosphate is
elevated and calcium lowered.

276. A 36-year-old female who has recently been diagnosed with polycystic kidneys is referred to a renal
clinic as it is noted her glomerular filtration rate is quite low. What is the normal glomerular filtration rate
approximately in a normal individual? Single best answer - select one answer only.
12ml/min YOUR ANSWER
24ml/min
64ml/min
120ml/min CORRECT ANSWER
240ml/min.
In the normal individual the glomerular filtration rate is kept fairly constant at 120 ml/min. The ultrafiltrate
is essentially plasma without the proteins (these are not able to cross the basement membrane of the
glomerulus). The GFR is governed by the rate of glomerular blood flow, which is governed by the tone of
BOTH the afferent and efferent arterioles.

This is kept constant over a range of blood pressures. Glucose is able to cross the glomerular basement
membrane into the ultrafiltrate. In normal individuals it is then actively re-absorbed. The resorption
mechanism has a maximal rate. If the glucose concentration of the serum, and consequently the
ultrafiltrate is too high, then the mechanism is overwhelmed and glycosuria occurs (as occurs in diabetes
mellitus).

---------------------------------------------------------------------------------------------------------------------------------------------------------------------------------------------------------------
Dr Mohammed Shamsul Islam Khan, Medical Officer, Clinical Neuro-Surgery, National Institute of Neuro-Sciences and Hospital
Sher-E-Bangla Nagar, Dhaka-1207, Bangladesh. Mobile: +880 1713 455 662, +880 1685 811979. E-mail: drsikhan@gmail.com
MyPasTest MRCS A Online - Jan Exam 2015
10. Physiology; System Specific Physiology (399Qs)
----------------------------------------------------------------------------------------------------------------------------------
277. Behind the carotid bifurcation lies the carodic body. What stimulates the carotid body? Select one
answer only.
A fall in arterial PCO2 YOUR ANSWER
A fall in arterial PO2 CORRECT ANSWER
A fall in blood pH Page |
A fall in metabolic rate 959
A fall in lactate level.

The carotid body (like the aortic body) is a peripheral chemoreceptor. It is stimulated by a drop in arterial
PO2 or a rise in arterial PCO2. This results in a compensatory increase in ventilation. The blood flow in the
carotid body is enormous: 2000 ml/100 g tissue per minute. The central chemoreceptors in the medulla on
the ventral surface of the brainstem monitor hydrogen ion concentration of cerebrospinal fluid. Carbon
dioxide readily passes through the bloodbrain barrier and parallels the hydrogen ion concentration in
the interstitial fluid of the brain.

278. Follicle-stimulating hormone (FSH) is a glycoprotein hormone synthesised and secreted by


gonadotrophs in the anterior pituitary gland. In men FSH: Single best answer question choose ONE true
option only.
Acts as an antagonist of testosterone YOUR ANSWER
Inhibits Sertoli cells
Inhibits sperm maturation
Secretion is completely suppressed by testosterone
Stimulates the synthesis of androgen-binding protein CORRECT ANSWER.

In men, FSH helps to maintain Sertoli cells (large cells in the seminiferous tubules that nourish the
developing sperm). FSH and testosterone synergise to stimulate the synthesis of androgen binding
protein (ABP). ABP keeps the levels of testosterone high in the tubular fluid. Sertoli cells cannot
synthesise androgens but rely on the diffusion of androgens from interstitial cells of Leydig. FSH is
necessary for spermatids to mature into sperm. Oestrogen and testosterone feedback do not completely
suppress FSH secretion; inhibin, produced by ovarian granulosa cells and testicular Sertoli cells, is
necessary for complete suppression.

279. Which of the following is correct regarding carcinoid syndrome? Select one answer only.
Is due to excessive secretion of somatostatin YOUR ANSWER
May cause constipation
May result in cardiac valve lesions CORRECT ANSWER
Can be confirmed by elevated vanillyl mandelic acid (VMA) levels
Is cured by excision of the appendix.

---------------------------------------------------------------------------------------------------------------------------------------------------------------------------------------------------------------
Dr Mohammed Shamsul Islam Khan, Medical Officer, Clinical Neuro-Surgery, National Institute of Neuro-Sciences and Hospital
Sher-E-Bangla Nagar, Dhaka-1207, Bangladesh. Mobile: +880 1713 455 662, +880 1685 811979. E-mail: drsikhan@gmail.com
MyPasTest MRCS A Online - Jan Exam 2015
10. Physiology; System Specific Physiology (399Qs)
----------------------------------------------------------------------------------------------------------------------------------
Carcinoid syndrome is due to the excess secretion of serotonin by argentaffin cells. The commonest sites
are the appendix and small bowel. Symptoms usually only occur with hepatic metastases. Diagnosis is by
raised urinary excretion of 5-hydroxy indole-acetic acid (5-HIAA), a metabolite of serotonin. VMA levels
are elevated in phaeochromocytoma. The syndrome relates to serotonin produced by liver metastases
and therefore appendicectomy cannot be curative upper gastrointestinal surgery.

280. A 58-year-old woman with long-standing atrial fibrillation suffered an embolic stroke that affected her
Page |
primary motor cortex. Where is the primary motor cortex located? Single best answer question choose 960
ONE true option only.
Cerebellar vermis YOUR ANSWER
Occipital lobe
Precentral gyrus CORRECT ANSWER
Postcentral gyrus
Sylvian fissure.

The primary motor cortex (or M1) works in association with premotor areas to plan and execute
movements. M1 contains large neurons known as Betz cells, which send long axons down the spinal cord
to synapse onto alpha-motor neurones, which connect to the muscles. Premotor areas are involved in
planning actions (in concert with the basal ganglia) and refining movements based upon sensory input
(this requires the cerebellum).

The human primary motor cortex is located in the dorsal part of the precentral gyrus and the anterior
bank of the central sulcus. The precentral gyrus is in front of the postcentral gyrus, from which it is
separated by the central sulcus. Its anterior border is the precentral sulcus, while inferiorly it borders to
the lateral fissure (Sylvian fissure). Medially, it is contiguous with the paracentral lobule.

281. Your consultant has asked you to prepare a presentation for his 2nd year medical students on the
anatomy and function of the distal convoluted tubule. The distal convoluted tubule is the site of which
one of the following: Single best answer - select one answer only.
Aldosterone-regulated reabsorption of sodium YOUR ANSWER (Correct Answer)
Cholride ion secretion
Principal water reabsorption in the kidney
Sodium secretion
The action of antidiuretic hormone (ADH).

The first part of the distal convoluted tubule is effectively an extension of the thick segment of the loop of
Henle, and is therefore relatively impermeable to water. The reabsorption of sodium in this segment is
regulated by aldosterone. Aldosterone also has the effect of increasing the secretion of potassium and
hydrogen ions into the tubular lumen. Approximately 5% of the filtered water is removed in this segment
of the kidney. ADH acts predominantly on the collecting ducts to increase their permeability to water, so
leading to water retention.
---------------------------------------------------------------------------------------------------------------------------------------------------------------------------------------------------------------
Dr Mohammed Shamsul Islam Khan, Medical Officer, Clinical Neuro-Surgery, National Institute of Neuro-Sciences and Hospital
Sher-E-Bangla Nagar, Dhaka-1207, Bangladesh. Mobile: +880 1713 455 662, +880 1685 811979. E-mail: drsikhan@gmail.com
MyPasTest MRCS A Online - Jan Exam 2015
10. Physiology; System Specific Physiology (399Qs)
----------------------------------------------------------------------------------------------------------------------------------

282. Which of the following organs has the greatest blood flow per 100 g of tissue? Single best answer
question choose ONE true option only.
Brain YOUR ANSWER
Heart Page |
Skin
961
Liver
Kidneys CORRECT ANSWER.

Organ Blood flow in ml/100g/min


Kidneys 420.0
Heart 84.0
Liver 57.7
Brain 54.0
Skin 12.8

283. Which of the following changes will decrease the rate of diffusion of a substance? Single best
answer question choose ONE true option only.
A decrease in the molecular weight of the substance YOUR ANSWER
An increase in the concentration gradient
An increase in temperature
An increase in the molecular weight of the substance CORRECT ANSWER
An increase in membrane permeability.
The rate of diffusion of a substance is proportional to the difference in the concentration of the diffusing
substance between the two sides of the membrane, the temperature of the solution, the permeability of
the membrane and, in the case of ions, the electrical potential difference between the two sides of the
membrane.

284. A 61-year-old female presents with persistent vomiting after eating of largely undigested food.
Prolonged vomiting in gastric outflow obstruction is associated with which of the following? Single best
answer - select one answer only.
Decreased ADH secretion YOUR ANSWER
Hypochloraemic alkalosis CORRECT ANSWER
Metabolic acidosis
Respiratory alkalosis
Reduced appetite.
Prolonged vomiting causes a hypochloraemichypokalaemic metabolic alkalosis. The loss of H+ and Cl-,
from gastric secretions and retention of alkaline pancreatic and duodenal secretions, result in a
---------------------------------------------------------------------------------------------------------------------------------------------------------------------------------------------------------------
Dr Mohammed Shamsul Islam Khan, Medical Officer, Clinical Neuro-Surgery, National Institute of Neuro-Sciences and Hospital
Sher-E-Bangla Nagar, Dhaka-1207, Bangladesh. Mobile: +880 1713 455 662, +880 1685 811979. E-mail: drsikhan@gmail.com
MyPasTest MRCS A Online - Jan Exam 2015
10. Physiology; System Specific Physiology (399Qs)
----------------------------------------------------------------------------------------------------------------------------------
hypochloraemic metabolic alkalosis. The loss of fluid volume causes a physiological response that
includes sympathetic stimulation, increase in ADH secretion and activation of the renin-angiotensin-
aldosterone system. In small bowel obstruction, there is also loss of alkaline intestinal secretions, which
prevents the development of metabolic alkalosis.

285. Renin, also known as angiotensinogenase, is a circulating enzyme released mainly by Page |
juxtaglomerular cells in the juxtaglomerular apparatus of the kidneys. Plasma renin levels are decreased
in patients with: Single best answer question choose ONE true option only.
962
Heart failure YOUR ANSWER
Primary aldosteronism CORRECT ANSWER
Renal artery stenosis
Salt restriction
Upright posture.

Most patients with primary aldosteronism (Conn's syndrome) have an adrenal adenoma. The increased
plasma aldosterone concentration leads to increased renal Na+ reabsorption, which results in plasma
volume expansion. The increase in plasma volume suppresses rennin release from the juxtaglomerular
apparatus and these patients usually have low plasma renin levels. Salt restriction and upright posture
decrease renal perfusion pressure and therefore increase renin release from the juxtaglomerular
apparatus. Secondary aldosteronism is due to elevated renin levels and may be caused by heart failure or
renal artery stenosis.

286. You have a patient with increased lung compliance. What increases lung compliance? Select one
answer only.
Emphysema YOUR ANSWER (Correct Answer)
A decrease in the elastic fibres in the lung parenchyma
An increase in the surface tension of the alveolar fluid
An open pneumothorax
Pulmonary contusion.
Lung compliance can be defined as the change in lung volume per unit change in airway pressure. It is
normally measured in the steepest part of the relaxation pressure curve (normal value 0.2 l/cmH2O). The
surface tension of the film of fluid that lines the alveoli is an important factor affecting lung compliance.
Lung compliance is reduced by decreased elastic fibres in the lung parenchyma, pulmonary contusion,
open pneumothorax and increased functional residual capacity. Any injury to the lung can reduce
compliance. A low lung compliance equates to having a 'stiffer' lung. Lung compliance is increased with
age and emphysema.

287. During routine antenatal check-up of a pregnant patient at 26 weeks' gestation it was discovered that
she had high blood pressure, proteinuria (400 mg/day), serum albumin of 2.0 g/dl, creatinine 0.8 mg/dl and
peripheral oedema. She had normal blood pressure on her first antenatal checkup. Which of the following

---------------------------------------------------------------------------------------------------------------------------------------------------------------------------------------------------------------
Dr Mohammed Shamsul Islam Khan, Medical Officer, Clinical Neuro-Surgery, National Institute of Neuro-Sciences and Hospital
Sher-E-Bangla Nagar, Dhaka-1207, Bangladesh. Mobile: +880 1713 455 662, +880 1685 811979. E-mail: drsikhan@gmail.com
MyPasTest MRCS A Online - Jan Exam 2015
10. Physiology; System Specific Physiology (399Qs)
----------------------------------------------------------------------------------------------------------------------------------
diagnoses is most appropriate for this patient? Single best answer question choose ONE true option
only.
Chronic renal failure YOUR ANSWER
Glomerulonephritis
Nephrotic syndrome Page |
Polycystic kidney disease 963
Pre-eclampsia CORRECT ANSWER.

Pre-eclampsia is said to be present when hypertension arises in pregnancy (pregnancy-induced


hypertension) in association with significant protein in the urine, usually after the 20th week of
pregnancy. Its cause remains unclear, although the principal cause appears to be a substance or
substances from the placenta causing endothelial dysfunction in the maternal blood vessels.

While blood pressure elevation is the most visible sign of the disease, it involves generalised damage to
the maternal endothelium and kidneys and liver, with the release of vasopressive factors only secondary
to the original damage. Pre-eclampsia may develop at varying times within pregnancy and its progress
differs among patients; most cases present pre-term. It has no known cure apart from ending the
pregnancy (induction of labour or abortion). It may also present up to 6 weeks post partum. It is the most
common, dangerous complication of pregnancy and it may affect both the mother and the fetus.

Pre-eclampsia is diagnosed when a pregnant woman develops high blood pressure (two separate
readings taken at least 6 h apart of 140/90 or more) and 300 mg of protein in a 24-h urine sample
(proteinuria). Swelling or oedema, (especially in the hands and face) was originally considered an
important sign for a diagnosis of pre-eclampsia, but in current medical practice only hypertension and
proteinuria are necessary for a diagnosis. However, unusual swelling, particularly of the hands, feet or
face, notable by leaving an indentation when pressed on, can be significant and should be investigated.

288. You are treating a 70Kg male patient with a vital capacity of 3000ml. What decreases vital capacity?
Select one answer only.
Increased surface area YOUR ANSWER
Male gender
Pain CORRECT ANSWER
Standing
Young age.

Vital capacity is proportionally related to surface area and is decreased by pain. It is 20-25% lower in
females. Vital capacity is decreases with increasing age. It is related to posture, lowest in a patient who is
lying down, highest with standing. Strength of respiratory muscles is proportionally related to vital
capacity. Decreased vital capacity is also seen in TB, asthma, and bronchitis.

---------------------------------------------------------------------------------------------------------------------------------------------------------------------------------------------------------------
Dr Mohammed Shamsul Islam Khan, Medical Officer, Clinical Neuro-Surgery, National Institute of Neuro-Sciences and Hospital
Sher-E-Bangla Nagar, Dhaka-1207, Bangladesh. Mobile: +880 1713 455 662, +880 1685 811979. E-mail: drsikhan@gmail.com
MyPasTest MRCS A Online - Jan Exam 2015
10. Physiology; System Specific Physiology (399Qs)
----------------------------------------------------------------------------------------------------------------------------------
289. A 38-year-old woman presents with a plasma thyroid-stimulating hormone (TSH) 12.5 mU/l (normal <
5 mU/l) and a T3 resin uptake of 19% (normal 25-35%). Which of the following clinical symptoms and signs
would you expect in this patient? Single best answer question choose ONE true option only.
Anxiety YOUR ANSWER
Increased body temperature
Page |
Palpitations
964
Periorbital swelling and lethargy CORRECT ANSWER
Tachycardia.

An increased TSH combined with a low T3 resin uptake (and low free T4) are characteristic of
hypothyroidism. Clinical signs and symptoms of hypothyroidism include dull facial expression, puffiness
and periorbital swelling caused by infiltration with mucopolysaccharides, decreased adrenergic drive,
lethargy, bradycardia and cold intolerance. The periorbital swelling must be distinguished from
exophthalmus due to increased retro-orbital tissue, which is a specific sign for hyperthyroidism caused
by Graves' disease. Tachycardia, fever, palpitations and anxiety are all seen in hyperthyroidism.
Additional clinical signs seen with hyperthyroidism may include heat intolerance, tremor, sweating and
sleeplessness.

290. A patient with a head injury opens his eyes following painful stimuli and withdraws from pain, and
makes incomprehensible sounds. What is his Glasgow Coma Scale score? Single best answer - select
one answer only.
5 YOUR ANSWER
6
7
8 CORRECT ANSWER
9.

The Glasgow Coma Score is arrived at by adding three components: eye opening (E), motor activity (M)
and verbal response (V). In this instance E = 2, M = 4 and V = 2. The scale is described below.
Eyes:
01. spontaneously open 4 points
02. open to speech 3 points
03. open to painful stimulus 2 points
04. none 1 point
Motor:
01. obeys commands 6 points
02. localises painful stimulus 5 points
03. withdraws from painful stimulus 4 points
04. abnormal flexion 3 points
05. extensor response 2 points
06. no movement 1 point
Verbal:
---------------------------------------------------------------------------------------------------------------------------------------------------------------------------------------------------------------
Dr Mohammed Shamsul Islam Khan, Medical Officer, Clinical Neuro-Surgery, National Institute of Neuro-Sciences and Hospital
Sher-E-Bangla Nagar, Dhaka-1207, Bangladesh. Mobile: +880 1713 455 662, +880 1685 811979. E-mail: drsikhan@gmail.com
MyPasTest MRCS A Online - Jan Exam 2015
10. Physiology; System Specific Physiology (399Qs)
----------------------------------------------------------------------------------------------------------------------------------
01. orientated 5 points
02. confused 4 points
03. inappropriate words 3 points
04. incomprehensible sounds 2 points
05. none 1 point

291. You are on the high dependency unit reviewing a patient with severe sepsis and the anaesthetic team
Page |
are initiating treatment with inotropes. Which of the following is correct with respect to inotropic agents? 965
Single best answer - select one answer only.
Adrenaline (epinephrine) stimulates alpha receptors only YOUR ANSWER
Noradrenaline (norepinephrine) is predominantly a beta agonist
Dopexamine is a splanchnic vasoconstrictor
Dobutamine increases systemic vascular resistance
Adrenaline may reduce renal blood flow at higher doses CORRECT ANSWER.

At higher doses of adrenaline (epinephrine), a mediated vasoconstriction reduces renal blood flow and
can cause oliguria and precipitate acute renal failure. Noradrenaline (norepinephrine) is predominantly an
alpha agonist. Dopexamine is a potent splanchnic vasodilator, reducing afterload and improving blood
flow to vital organs, including the kidneys. Dobutamine reduces systemic vascular resistance, decreasing
afterload and ventricular filling pressures, and is of use in cardiogenic shock and cardiac failure.

292. Hypothyroidism due to disease of the thyroid gland is associated with increased plasma level of?
Single best answer question choose ONE true option only.
Cholesterol YOUR ANSWER (Correct Answer)
Albumin
RT3
Iodide
Thyroid binding globulin (TBG).

Thyroid hormone lowers circulating cholesterol level. The plasma cholesterol level drops before the
metabolic rate rises.

293. One of your patients was admitted to the intensive care unit with Acute respiratory distress
syndrome (ARDS) over 24 hours ago. What changes would you expect to see on a chest x-ray? Select one
answer only.
Diffuse alveolar shadows YOUR ANSWER (Correct Answer)
Hilar lymphadenopathy
Lower lobe collapse
No changes
---------------------------------------------------------------------------------------------------------------------------------------------------------------------------------------------------------------
Dr Mohammed Shamsul Islam Khan, Medical Officer, Clinical Neuro-Surgery, National Institute of Neuro-Sciences and Hospital
Sher-E-Bangla Nagar, Dhaka-1207, Bangladesh. Mobile: +880 1713 455 662, +880 1685 811979. E-mail: drsikhan@gmail.com
MyPasTest MRCS A Online - Jan Exam 2015
10. Physiology; System Specific Physiology (399Qs)
----------------------------------------------------------------------------------------------------------------------------------
Pleural effusion.

ARDS is the most extreme manifestation of acute lung injury. It produces diffuse alveolar shadows on
CXR. Early radiographic signs are nonspecific. The final common pathway of ARDS involves neutrophil
activation, and the release of inflammatory mediators and free radicals causing increased alveolar
permeability. ARDS should be managed in the ITU/HDU environment as rapid deterioration can occur Page |
requiring advanced circulatory support.
966
294. The colon has a large absorptive capacity. Which of the following is the most likely component
actively absorbed by the colon? Single best answer - select one answer only.
Amino acids YOUR ANSWER
Bicarbonate
Bile salts
Sodium CORRECT ANSWER
Water.

Absorption in the colon can be active or passive. Sodium is absorbed actively. Water is absorbed but
passively and is dependent on the movement of other solutes.

295. You are treating a patient with hyperthyroidism. Which of the following statements is correct? Single
best answer - select one answer only.
With antithyroid drugs patients have a high recurrence rate when therapy is discontinued YOUR ANSWER
(Correct Answer)
Surgery is the most rapid method of permanent control and results in euthyroidism in about 50% of cases at
five years
Surgery has no permanent complications
Radio-iodine therapy gives permanent control and has a low rate of hypothyroidism
All other treatments, except surgery, are contraindicated in pregnancy and the very young.

01. Damage to recurrent laryngeal nerves in 1%


02. Damage to parathyroid glands in 1%
03. Permanent hypothyroidism (leaving 810 g remnants of thyroid) in less than 5% (five-year
follow-up).
04. Hypothyroidism rate is 90% or more in five years, probably 100% in 20 years
05. Hypothyroidism occurs in 70% of patients with Graves disease treated with radioiodine
and in 40% of other patients treated
06. Radioactive iodine is contraindicated in pregnancy. If the patient is taking carbimazole,
this should be changed to PTU and then back to carbimazole after the 1st trimester

296. A 47-year-old male with poorly controlled diabetes and hypertension is referred to a renal clinic due
to concerns regarding his renal function. It is decided to monitor his glomerular filtration rate (GFR) as an
---------------------------------------------------------------------------------------------------------------------------------------------------------------------------------------------------------------
Dr Mohammed Shamsul Islam Khan, Medical Officer, Clinical Neuro-Surgery, National Institute of Neuro-Sciences and Hospital
Sher-E-Bangla Nagar, Dhaka-1207, Bangladesh. Mobile: +880 1713 455 662, +880 1685 811979. E-mail: drsikhan@gmail.com
MyPasTest MRCS A Online - Jan Exam 2015
10. Physiology; System Specific Physiology (399Qs)
----------------------------------------------------------------------------------------------------------------------------------
outpatient. Which of the following is used in clinical practice to estimate GFR? Single best answer - select
one answer only.
Creatinine level YOUR ANSWER
Creatinine clearance CORRECT ANSWER
Inulin clearance Page |
Urea levels 967
Urine output.

The GFR varies during life. Adult levels are reached at 2 years of age and decline linearly from 40 years
due to glomerular sclerosis. In research laboratories, GFR can be measured by inulin clearance. Inulin is
a polymer of fructose that is not secreted, metabolised or reabsorbed by the kidney. This means that the
amount of inulin excreted in urine (per minute) equals the amount of inulin filtered at the glomerulus (per
minute). In clinical practice, GFR is estimated by measuring the creatinine clearance. A 50% increase in
GFR occurs during pregnancy. GFR increases after a protein rich meal.

297. If the ventromedial nucleus of the hypothalamus is destroyed the affected individual will have:
Single best answer question choose ONE true option only.
Loss of appetite YOUR ANSWER
Loss of circadian rhythm
Loss of regulation of antidiuretic hormone secretion
Loss of satiety CORRECT ANSWER
Loss of vision.

The ventromedial nucleus of the hypothalamus (sometimes referred to as the ventromedial


hypothalamus) has two parts: superior and anterior. The superior part is responsible for satiety. This
means if it is taken out of an animal, the animal will overeat because there is nothing telling the body that
the animal is satisfied. The anterior part is responsible for the female sexual drive.

298. In a study of 100 patients presenting to casualty with abdominal pain, 30 patients have acute
appendicitis, 21 of these have a temperature of greater than 37.5C. 28 of those without appendicitis have
a temperature greater than 37.5C. Which of the following statements is the most likely? Single best
answer - select one answer only.
The predicitive value of temperature greater than 37.5C as a marker of appendicitis is 21/30 YOUR
ANSWER
The predicitive value of the test is the same in different populations
The sensitivity of temperature greater than 37.5C as a marker of appendicitis is 21/49
The specificity of temperature greater than 37.5C as a marker for appendicitis is 42/70 CORRECT
ANSWER
The specificity of the test will depend upon the prevalence of appendicitis in the population to which it is
applied.
---------------------------------------------------------------------------------------------------------------------------------------------------------------------------------------------------------------
Dr Mohammed Shamsul Islam Khan, Medical Officer, Clinical Neuro-Surgery, National Institute of Neuro-Sciences and Hospital
Sher-E-Bangla Nagar, Dhaka-1207, Bangladesh. Mobile: +880 1713 455 662, +880 1685 811979. E-mail: drsikhan@gmail.com
MyPasTest MRCS A Online - Jan Exam 2015
10. Physiology; System Specific Physiology (399Qs)
----------------------------------------------------------------------------------------------------------------------------------
Temperature Acute Appendicitis Other reason
>37.5C 21 28
=37.5C 9 42
Total 30 70 Page |
01. Sensitivity = 21/30
02. Specificity = 42/70 968
03. The sensitivity is evaluated on those with appendicitis, and specificity on those without
appendicitis
04. Positive predictive value = 21/49

The predictive value depends on the prevalence of appendicitis in the sample and may be different in
another population eg patients attending general practitioners with abdominal pain.

299. The cerebral blood flow (CBF). Single best answer question choose ONE true option only.
Accounts for about 15% of the cardiac output YOUR ANSWER (Correct Answer)
Is decreased by hypercapnia
Is decreased by hypoxia
Is mainly controlled by sympathetic and parasympathetic activity
Is decreased by isoflurane in general anaesthesia.

Cerebral blood flow accounts for about 15% of the cardiac output. Hypoxia and hypercapnia increase the
CBF. An autonomic mechanism seems to be unimportant. Isoflurane increases CBF.

300. Under normal conditions the amount of O2 taken up is a function of pulmonary blood flow; that is,
normally, O2 transfer is perfusion-limited. Which of the following conditions would favour a diffusion
limitation of O2 transfer from alveolar to pulmonary capillary blood? Single best answer question
choose ONE true option only.
Breathing hyperbaric gas mixture YOUR ANSWER
Chronic obstructive lung disease
Increased ventilatory rate
Mild exercise
Pulmonary oedema CORRECT ANSWER.

Normally, O2 is transferred from air spaces to blood via a perfusion limited process. Therefore, O 2 moves
across the alveolar-capillary membrane by a process of simple diffusion and the amount of gas taken up
depends entirely on the amount of blood flow. Processes that impair diffusion of O 2 transform the normal
relationship to a diffusion limited process. Therefore, if O2 must move a greater distance because of a
thickened barrier, as would occur with increased extravascular lung water (pulmonary oedema) or cell
components (interstitial fibrosis or asbestosis), the diffusion process is limited.
---------------------------------------------------------------------------------------------------------------------------------------------------------------------------------------------------------------
Dr Mohammed Shamsul Islam Khan, Medical Officer, Clinical Neuro-Surgery, National Institute of Neuro-Sciences and Hospital
Sher-E-Bangla Nagar, Dhaka-1207, Bangladesh. Mobile: +880 1713 455 662, +880 1685 811979. E-mail: drsikhan@gmail.com
MyPasTest MRCS A Online - Jan Exam 2015
10. Physiology; System Specific Physiology (399Qs)
----------------------------------------------------------------------------------------------------------------------------------
Chronic obstructive lung diseases have little effect on pulmonary diffusion capacity. Breathing a
hyperbaric gas mixture would increase the driving force and may overcome diffusion limitation in patients
with mild fibrosis or interstitial oedema. Increasing the ventilatory rate will not have this effect and will
only serve to maintain a high gradient of O2 from air to blood. Strenuous, but not mild, exercise decreases
passage time and may also favour diffusion limitation.
Page |
301. A 77-year-old male has recently been diagnosed with prostate cancer and commenced on flutamide 969
(anti-androgen). Which of the following combinations most appropriately describe the physiological
effects of androgens? Single best answer - select one answer only.
Increased protein synthesis, increased spermatogenesis, increased erythropoiesis YOUR ANSWER
(Correct Answer)
Increased protein synthesis, increased spermatogenesis, reduced erythropoiesis
Increased protein synthesis, reduced spermatogenesis, reduced erythropoiesis
Reduced protein synthesis, increased spermatogenesis, increased erythropoiesis
Reduced protein synthesis, reduced spermatogenesis, reduced erythropoiesis.

Functions of the testis in the normal man can be described as: (1) developmental where formation of the
genitalia and reproductive organs is determined by the production of testosterone; (2) spermatogenetic-
stimulating spermatogenesis under the control of FSH and LH from the anterior pituitary; (3) secondary
developmental where testosterone acts to promote the development of secondary sexual
characteristics; and (4) anabolic effects where the deposition of muscle and bone is promoted along with
other organs such as the larynx and red blood cells (erythropoiesis). Most testosterone is produced in the
testis though a significant amount is produced in the adrenals.

302. You are reviewing a patient with a lumbar disc prolapse at L3/4 level in the neurosurgical outpatient
clinic. You cannot elicit the knee jerk reflex on the affected side despite reinforcement. Which of the
following is correct regarding this deep tendon reflex? Single best answer - select one answer only.
The knee jerk is supplied by the L5 nerve root YOUR ANSWER
Passive stretch of the intrafusal muscle spindles when tapping the tendon results in ipsilateral contraction of
the extensors of the knee CORRECT ANSWER
Efferent stimuli are transmitted via the dorsal roots of the spinal nerves
The synapse between afferent and efferent neurons occurs in the brainstem
The afferent neuron also synapses with inhibitory interneurons that result in relaxation of the contralateral
knee extensors.

The knee-jerk reflex is the best-known example of a monosynaptic stretch reflex. Passive stretch of the
intrafusal spindle fibres through tapping the tendon with a tendon hammer results in generation of
afferent action potentials. These are transmitted via the dorsal roots of the spinal nerves to the ipsilateral
ventral grey matter where the only synapse with the efferent neurone occurs. The action potential is

---------------------------------------------------------------------------------------------------------------------------------------------------------------------------------------------------------------
Dr Mohammed Shamsul Islam Khan, Medical Officer, Clinical Neuro-Surgery, National Institute of Neuro-Sciences and Hospital
Sher-E-Bangla Nagar, Dhaka-1207, Bangladesh. Mobile: +880 1713 455 662, +880 1685 811979. E-mail: drsikhan@gmail.com
MyPasTest MRCS A Online - Jan Exam 2015
10. Physiology; System Specific Physiology (399Qs)
----------------------------------------------------------------------------------------------------------------------------------
transmitted through the anterior nerve roots via an alpha motor neuron, which synapses with the motor
endplate of the muscle fibres of the knee extensors.

Acetylcholine crosses the motor endplate to generate a contraction of the muscle group. This results in a
decrease in the stretch of the sensory fibres and a consequent decrease in electrical afferent activity. In
the ventral grey matter synapses with interneurons occur. These result in inhibition of contraction of the Page |
flexors of the knee. The result is a rapid extension of the knee. 970
303. You are reviewing one of your post-op patients, an 85-year-old female, who is now 5 days following a
dynamic hip screw for fractured neck of femur. You notice that the patient has a rapid irregularly irregular
pulse. An ECG confirms fast atrial fibrillation. What is the most likely cause of the atrial fibrillation? Single
best answer - select one answer only.
Anxiety YOUR ANSWER
Fluid depletion
Hypothyroidism
Pneumonia CORRECT ANSWER
Pulmonary embolism.

Atrial fibrillation is seen with ischaemic heart disease, mitral valve disease and hyperthyroidism. It is a
common complication of cardiac surgery and fluid overload, especially in patients in intensive therapy
units (ITU). It can be caused by pneumonia. The commonest pulse abnormality seen with pulmonary
embolism is sinus tachycardia. At day 5 following a dynamic hip screw, this patient is likely to be
immobile and susceptible to chest infection, and pneumonia is the most likely cause of AF in this patient.

304. Which of the following hormones is synthesised in the hypothalamus and secreted from the
posterior pituitary? Single best answer question choose ONE true option only.
Anti diuretic hormone (ADH) YOUR ANSWER (Correct Answer)
Adrenocorticotrophic hormone (ACTH)
Corticotrophin releasing hormone (CRH)
Thyrotrophin releasing hormone (TRH)
Thyroid stimulating hormone (TSH).

Vasopressin (ADH) and oxytocin are synthesised in the hypothalamic nuclei and pass down axons to the
posterior pituitary where they are secreted into the blood stream. In contrast, the trophic hormones such
as CRH and TRH are secreted by the hypothalamus in response to neural stimuli, and drain into the
hypothalamohypophyseal portal vessels to the anterior pituitary. There is then resultant stimulation of
ACTH and TSH secretion. The other hormones produced by a similar mechanism by the anterior pituitary
are growth hormone (GH), prolactin (PRL), lutenising hormone (LH) and follicle stimulating hormone
(FSH).
---------------------------------------------------------------------------------------------------------------------------------------------------------------------------------------------------------------
Dr Mohammed Shamsul Islam Khan, Medical Officer, Clinical Neuro-Surgery, National Institute of Neuro-Sciences and Hospital
Sher-E-Bangla Nagar, Dhaka-1207, Bangladesh. Mobile: +880 1713 455 662, +880 1685 811979. E-mail: drsikhan@gmail.com
MyPasTest MRCS A Online - Jan Exam 2015
10. Physiology; System Specific Physiology (399Qs)
----------------------------------------------------------------------------------------------------------------------------------
305. You are treating a patient who underwent emergency laparotomy for faecal peritonitis 2 days ago.
The patient is tachycardic and hypotensive and has developed adult respiratory distress syndrome,
(ARDS). What is the most likely cause of this? Select one answer only.
Abdominal sepsis YOUR ANSWER (Correct Answer)
Fat embolism
Page |
Pancreatitits
971
Pulmonary fibrosis
Trauma.

ARDS is an acute diffuse inflammatory process resulting from direct or indirect pulmonary injury. It is
most commonly seen in sepsis but can also occur after trauma, burns, inhalation injuries, shock and
pancreatitis. In postoperative surgical patients, abdominal sepsis or central-line sepsis should be
considered.

Indirect or direct lung injury initiates abnormal behaviour and movement of neutrophils, platelets and
macrophages. Neutrophils and platelets attach to capillary endothelium causing capillary leakage. This
leads to oedema of lung tissue and movement of neutrophils and erythrocytes into the lung parenchyma.
Lung lymph flow is increased with thickening of the alveolarcapillary membrane. This results in impaired
oxygen diffusion and reduced lung compliance as the alveolus is surrounded by fluid. In addition, some
of the fluid in the pulmonary parenchyma may leak into the alveoli, giving the characteristic appearance
of a hyaline membrane.

ARDS is characterised by refractory hypoxaemia (paO2< 8 kPa at FiO2> 0.4), alveolar inflammation and
oedema, reduced total compliance (< 30 ml/cm water) and a paO2 (in mm Hg) : FiO2 ratio of < 200 (normal
is approximately 500). Pulmonary fibrosis in the later stages of the disease leads to a decrease in the
functional residual capacity, further decrease in lung compliance and an increase in the shunt effect.
Pulmonary signs are often minimal or non-specific in the early stages, the patient simply being
breathless, but, if untreated, they could soon become hypoxic and cyanotic. Chest X-ray may be normal in
the early stages but later shows bilateral diffuse pulmonary infiltration.

Treatment, in addition to eliminating the precipitating aetiology, involves ventilating the patient in
intensive care. Patients are usually nursed in the prone position. The tidal volume should be kept low
(approximately 6 ml/kg), as should the pulmonary capillary wedge pressure (high exacerbates pulmonary
oedema). The mortality of this condition is as high as 5070%.

306. A lesion of the suprachiasmatic nucleus of hypothalamus will affect: Single best answer question
choose ONE true option only.
Autonomic function YOUR ANSWER
Regulation of circadian rhythm CORRECT ANSWER
Regulation of water balance
Temperature regulation
Sexual maturity.
---------------------------------------------------------------------------------------------------------------------------------------------------------------------------------------------------------------
Dr Mohammed Shamsul Islam Khan, Medical Officer, Clinical Neuro-Surgery, National Institute of Neuro-Sciences and Hospital
Sher-E-Bangla Nagar, Dhaka-1207, Bangladesh. Mobile: +880 1713 455 662, +880 1685 811979. E-mail: drsikhan@gmail.com
MyPasTest MRCS A Online - Jan Exam 2015
10. Physiology; System Specific Physiology (399Qs)
----------------------------------------------------------------------------------------------------------------------------------
The suprachiasmatic nucleus (SCN) is a region of the brain, located in the hypothalamus, which is
responsible for controlling endogenous circadian rhythms. The SCN receives inputs from specialized
photoreceptive retinal ganglion cells, via the retinohypothalamic tract. Destruction of the SCN leads to a
complete loss of circadian rhythm. The SCN also controls 'slave oscillators' in the peripheral tissues,
which exhibit their own 24-hour rhythms, but are crucially synchronised by the SCN. The importance of
entraining our bodies to an exogenous cue, such as light, is reflected by several circadian rhythm sleep
disorders, in which this process does not function normally.
Page |
972
307. The gut transit times of two different laxatives A and B in two samples of UK patients are compared
using the t-test. Which of these statements about this study are the most likely? Single best answer -
select one answer only.
It is assumed that the gut transit times in the two groups are not normally distributed YOUR ANSWER
The null hypothesis assumes there is a difference between the two population groups
The null hypothesis assumes there is no difference between the two population groups CORRECT
ANSWER
The number of patients must be the same in each group
The trial would be valid even if patients had travelled to the tropics recently.

The t-test can be used to compare the means of two normally distributed samples. The samples do not
have to be of the same size. The null hypothesis assumes that there is no difference in the means of the
groups. A statistically significant result implies that one group has significantly longer transit times than
the other. A double-blind randomised control trial would be more convincing than an observational study
in which the patients had chosen their own laxative. Confounding factors are less likely to occur with a
randomised trial. If more of one group had recently returned from the tropics and having recently been in
the tropics affected gut transit time, then 'recently been in tropics' would be a confounding factor and
may make the results questionable.

308. One of your patients is short of breath, you take arterial blood gases. What gas results would lead
you to diagnose Respiratory insufficiency, (failure)? Select one answer only.
PaCO2 of 5KPa and a PaO2 of 10kPa on 40% O2 YOUR ANSWER
PaCO2 of 5KPa and a PaO2 of 15KPa on 40% O2
PaCO2 of 10KPa and a PaO2 of 10KPa on 40% O2
PaCO2 of 15 kPa and a PaO2 of 5KPa on 40% O2 CORRECT ANSWER
PaCO2 of 15KPa and a PaO2 of 10KPa on 40% O2.

Respiratory failure can be divided into type I and type II. Type I is where the PaO 2 is low and the PCO2 is
normal or low, and type II is where the PO2 is low and the PaCO2 is high. In practical terms this occurs
when the PO2 is < 8 kPa and the PACO2> 7 kPa. Certainly, an inability to clear secretions will lead to
respiratory insufficiency. This often occurs after abdominal surgery (especially upper) when pain
prevents deep inspiration and coughing, with subsequent basal atelectasis and retention of secretions.
The symptoms of respiratory insufficiency include confusion, agitation and drowsiness. In pancreatitis,
---------------------------------------------------------------------------------------------------------------------------------------------------------------------------------------------------------------
Dr Mohammed Shamsul Islam Khan, Medical Officer, Clinical Neuro-Surgery, National Institute of Neuro-Sciences and Hospital
Sher-E-Bangla Nagar, Dhaka-1207, Bangladesh. Mobile: +880 1713 455 662, +880 1685 811979. E-mail: drsikhan@gmail.com
MyPasTest MRCS A Online - Jan Exam 2015
10. Physiology; System Specific Physiology (399Qs)
----------------------------------------------------------------------------------------------------------------------------------
inflammatory mediators are released into the circulation and this can produce respiratory impairment
through acute lung injury, which may progress to fulminant adult respiratory distress syndrome.

309. You are reviewing a post-operative neurosurgical patient with a left posterior cerebral artery
occlusion. Which of the following conditions may result? Single best answer - select one answer only.
Left homonymous hemianopia YOUR ANSWER Page |
Right visual field loss CORRECT ANSWER 973
Macular degeneration
Total blindness in right eye
Paralysis of the left lateral rectus muscle.

The posterior cerebral artery supplies the cerebral peduncle and the optic tract. It supplies the
inferomedial surface of the temporal and occipital lobes. Occlusion causes a contralateral hemianopia
and hemianaesthesia. An incomplete lesion would result in macular sparing. Macular sparing probably is
caused by collateral vascular supply to the occipital pole from posterior branches of the middle cerebral
artery and preservation of the optic radiations, though bilateral representation of macular vision has also
been suspected.

310. Which of the following cells is cytotoxic? Single best answer question choose ONE true option
only.
CD4 T-cells YOUR ANSWER
CD8 T-cells CORRECT ANSWER
B cells
TH1 cells
TH2 cells.

Lymphocytes can be divided into two main subtypes T cells and B cells.
B cells (or plasma cells) secrete antibodies.

T cells can be divided into two further subtypes CD4 T-cells and CD8 T-cells. CD4 (helper) T-cells can
recognise antigen only in the context of MHC Class II, whereas CD8 (cytotoxic) T-cells recognise cell-
bound antigens only in association with Class I MHC. This is known as MHC restriction.

CD4 and CD8 T-cells perform distinct but somewhat overlapping functions. The CD4 helper T-cell can be
viewed as a master regulator. By secreting cytokines (soluble factors that mediate communication
between cells), CD4 helper T-cells influence the function of virtually all other cells of the immune system
including other T-cells, B-cells, macrophages and natural killer cells. The central role of CD4 cells is
tragically illustrated by the HIV virus which cripples the immune system by selective destruction of this T-
cell subset.
---------------------------------------------------------------------------------------------------------------------------------------------------------------------------------------------------------------
Dr Mohammed Shamsul Islam Khan, Medical Officer, Clinical Neuro-Surgery, National Institute of Neuro-Sciences and Hospital
Sher-E-Bangla Nagar, Dhaka-1207, Bangladesh. Mobile: +880 1713 455 662, +880 1685 811979. E-mail: drsikhan@gmail.com
MyPasTest MRCS A Online - Jan Exam 2015
10. Physiology; System Specific Physiology (399Qs)
----------------------------------------------------------------------------------------------------------------------------------
In recent years two functionally different populations of CD4 helper T-cells have been recognised TH1
cells and TH2 cells, each characterised by the cytokines that they produce. In general, TH1 cells facilitate
cell-mediated immunity, whereas TH2 cells promote humoral-mediated immunity.

CD8 cytotoxic T-cells mediate their functions primarily by acting as cytotoxic cells (i.e. they are T-cells
that kill other cells). They are important in the host defence against cytosolic pathogens. Two principal Page |
mechanisms of cytotoxicity have been discovered perforin-granzyme-dependent killing and Fas-Fas 974
ligand dependent killing.

311. The electrocardiogram (ECG) of a patient shows prolonged PR interval. The normal duration of PR
interval is: Single best answer question choose ONE true option only.
0.001-0.002 s YOUR ANSWER
0.12-0.20 s CORRECT ANSWER
0.25-0.35 s
0.35-0.45 s
0.45-0.50 s.

The PR interval is measured from the beginning of the P-wave to the beginning of the QRS complex. It is
usually 0.12-0.20 s. A prolonged PR indicates a first-degree heart block, while a shortening may indicate
an accessory bundle that depolarises the ventricle early, such as seen in Wolff-Parkinson-White
syndrome.

312. A 45-year-old woman was seen in the Accident and Emergency Department with extreme generalised
oedema with marked expansion of the extracellular space within the subcutaneous tissues, visceral
organs and body cavities. The woman has: Single best answer question choose ONE true option only.
Anasarca YOUR ANSWER (Correct Answer)
Angioedema
Apoptosis
Hyperthecosis
Haemochromatosis.

Anasarca (or 'generalised oedema') is a condition characterised by widespread swelling of the skin due to
effusion of fluid into the extracellular space. It is usually caused by either cardiac failure, liver failure or
renal failure. It can also be created from the administration of exogenous intravenous fluid. Certain plant-
derived anticancer chemotherapeutic agents, such as docetaxel, cause anasarca through a poorly
understood capillary leak syndrome. This condition is also called leucophlegmasia.

313. Insulin levels are not increased by which of the following? Single best answer question choose
ONE correct option only.
Amino acids YOUR ANSWER
---------------------------------------------------------------------------------------------------------------------------------------------------------------------------------------------------------------
Dr Mohammed Shamsul Islam Khan, Medical Officer, Clinical Neuro-Surgery, National Institute of Neuro-Sciences and Hospital
Sher-E-Bangla Nagar, Dhaka-1207, Bangladesh. Mobile: +880 1713 455 662, +880 1685 811979. E-mail: drsikhan@gmail.com
MyPasTest MRCS A Online - Jan Exam 2015
10. Physiology; System Specific Physiology (399Qs)
----------------------------------------------------------------------------------------------------------------------------------
-adrenergic stimulation
Hypokalaemia CORRECT ANSWER
Increased plasma glucose
Vagal nerve stimulation.
Page |
Glucose is the most important stimulus to insulin, causing its release from secretory granules in the - 975
cells of the pancreas, and release of newly synthesised insulin. As well as glucose, insulin release is
stimulated by amino acids, some fatty acids, and the gut hormones secretin and cholecystokinin. -
adrenergic and vagal stimulation also increase insulin levels.

One of the effects of insulin other than decreasing blood glucose is to cause an intracellular shift of K +.
This reduces serum levels of K+, but not total body K+. Hypokalaemia acts to inhibit insulin release.

314. Which of the following renal functions will be assessed if you are measuring the urine specific
gravity? Single best answer question choose ONE true option only.
Blood flow YOUR ANSWER
Concentration CORRECT ANSWER
Filtration
Reabsorption
Secretion.

Renal concentrating ability is reflected by the specific gravity. In humans, normal specific gravity values
range from 1.002 g/ml to 1.028 g/ml. Increased specific gravity (ie increased concentration of solutes in
the urine) is associated with dehydration, diarrhoea, vomiting, excessive sweating, glucosuria and the
syndrome of inappropriate antidiuretic hormone secretion (SIADH). Decreased specific gravity (ie
decreased concentration of solutes in urine) is associated with renal failure, pyelonephritis, diabetes
insipidus, acute tubular necrosis, interstitial nephritis and excessive fluid intake.

315. The ejection fraction is defined as: Single best answer quest ion choose ONE true opt ion only.
The ratio of the end diastolic volume to stroke volume YOUR ANSWER
The ratio of stroke volume to end diastolic volume CORRECT ANSWER
End diastolic volume minus end systolic volume
End systol ic volume d iv ided by stroke volume
End diastolic volume multiplied by stroke volume.

During diastole, filling of the ventricles normally increases the volume of each ventricle to about 120mls.
This volume is known as the end diastolic volume. Then, as the ventricles empty in systole, the volume
decreases about 70mls, which is known as the stroke volume. The remaining volume in each ventricle,

---------------------------------------------------------------------------------------------------------------------------------------------------------------------------------------------------------------
Dr Mohammed Shamsul Islam Khan, Medical Officer, Clinical Neuro-Surgery, National Institute of Neuro-Sciences and Hospital
Sher-E-Bangla Nagar, Dhaka-1207, Bangladesh. Mobile: +880 1713 455 662, +880 1685 811979. E-mail: drsikhan@gmail.com
MyPasTest MRCS A Online - Jan Exam 2015
10. Physiology; System Specific Physiology (399Qs)
----------------------------------------------------------------------------------------------------------------------------------
about 50mls, is known as the end systolic volume and acts as a reserve which can be utilised to increase
stroke volume in exercise.

The fraction of end diastolic volume that is ejected is called the ejection fraction usually equal to about
60%. The ejection fraction is often used clinically as an indirect index of contractility. It is a particularly
useful in assessing the state of the myocardium prior to aortic aneurysm repair where cross-clamping of
the aorta places particular stress on the myocardium.
Page |
976
316. Absorption of calcium from the digestive tract? Single best answer question choose ONE true
option only.
Takes place mostly in the proximal jejunum YOUR ANSWER (Correct Answer)
Is prevented by the presence of small amounts of phytic acid in the diet , even when an excess calcium is
ingested
Is facilitated by the presence of fat in food
Can be reversed (calcium is secreted into bowel lumen) when plasma calcium concentration is raised by a
calcium infusion
Is about as rapid as that of sodium.

Phytic acid produces insoluble calcium phytate, when all phytic acid has been precipitated the excess
calcium is absorbed. Fatty acids form insoluble calcium salts (soaps). The shift of calcium ions across
the intestinal mucosa is virtually one way. Sodium is absorbed at a speed fifty times that for calcium
absorption.

317. You are prescribing one of your patients routine medications and prescribe an Angiotensin II
inhibitor. What does angiotensin II do? Select one answer only.
It decreases thirst YOUR ANSWER
It is a weak arteriolar vasoconstrictor
It reduces blood pressure
It stimulates aldosterone release CORRECT ANSWER
It stimulates renin release.

Angiotensin II stimulates aldosterone synthesis and secretion through the activity of a specific receptor
found in the zona glomerulosa. Angiotensin II is one of the most potent endogenous vasoconstrictor
agents and inhibits renin release through a negative-feedback loop. Renin stimulates the formation of
angiotensin I from angiotensinogen. Angiotensin I is converted to angiotensin II in the lung by
angiotensin-converting enzyme (ACE).

318. ADH (Vasopressin) release in response to dehydration causes. Single best answer question choose
ONE true option only.
Decreased permeability of the collecting ducts to water YOUR ANSWER

---------------------------------------------------------------------------------------------------------------------------------------------------------------------------------------------------------------
Dr Mohammed Shamsul Islam Khan, Medical Officer, Clinical Neuro-Surgery, National Institute of Neuro-Sciences and Hospital
Sher-E-Bangla Nagar, Dhaka-1207, Bangladesh. Mobile: +880 1713 455 662, +880 1685 811979. E-mail: drsikhan@gmail.com
MyPasTest MRCS A Online - Jan Exam 2015
10. Physiology; System Specific Physiology (399Qs)
----------------------------------------------------------------------------------------------------------------------------------
Decreased urine osmolality
Increased Na+ resorption in the ascending limb of the loop of Henle
Increased Na+ resorption in the descending limb of the loop of Henle
Increased permeability of the collecting ducts to water CORRECT ANSWER.
Page |
ADH is released by the posterior pituitary in response to dehydration, from stimulation of osmoreceptors 977
adjacent to the supraoptic nucleus, as well as volume receptors in the aorta atria and great veins. Water
absorption in the collecting ducts is independent of sodium concentration, and is under the control of
ADH, which causes increased permeability of the ducts. Increased ADH levels will increase the osmolality
of the urine via this method.

In the descending limb of the loop of Henle, sodium and water are passively resorbed. The ascending
limb is impermeable to water, with active sodium resorption, producing a concentration gradient in the
renal medulla, which is essential for the maintenance of water balance.

319. Which one of the following hormones is secreted by the anterior pituitary? Single best answer quest
ion choose ONE true opt ion only.
Testosterone YOUR ANSWER
Oxytocin
TSH CORRECT ANSWER
CRH
ADH.

The pituitary gland (hypophysis) is the conductor of the endocrine orchestra. It is divided into both an
anterior part and posterior part. The anterior pituitary (adenohypophysis or pars distalis) secretes 6
hormones namely:
01. FSH/LH: Reproduction
02. ACTH: Stress response
03. TSH: Basal metabolic rate
04. GH: Growth
05. Prolactin: Lactation

The posterior pituitary (neurohypophysis or pars nervosa) secretes only 2 hormones:


01. ADH (vasopressin): Osmotic regulation
02. Oxytocin: Milk ejection and labour.

Testosterone is produced from Leydig cells in the testis and from the adrenal glands. CRH is produced by
the median eminence of the hypothalamus.

320. A 48-year-old man was admitted to the Accident and Emergency Department with exacerbation of his
chronic obstructive pulmonary disease. An arterial blood gas on room air showed respiratory acidosis.
---------------------------------------------------------------------------------------------------------------------------------------------------------------------------------------------------------------
Dr Mohammed Shamsul Islam Khan, Medical Officer, Clinical Neuro-Surgery, National Institute of Neuro-Sciences and Hospital
Sher-E-Bangla Nagar, Dhaka-1207, Bangladesh. Mobile: +880 1713 455 662, +880 1685 811979. E-mail: drsikhan@gmail.com
MyPasTest MRCS A Online - Jan Exam 2015
10. Physiology; System Specific Physiology (399Qs)
----------------------------------------------------------------------------------------------------------------------------------
The patient's renal excretion of potassium would be expected to: Single best answer question choose
ONE true option only.
Fall, since the filtered load of potassium to the tubules falls in acidosis YOUR ANSWER
Fall, since tubular secretion of potassium is inversely coupled to acid secretion CORRECT ANSWER
Rise, since acid and potassium excretion are coupled Page |
Rise, since acidosis is a stimulus to renin secretion by the juxtaglomerular apparatus 978
Rise, since acidosis increases the affinity of the aldosterone receptor for aldosterone.

Secretion of hydrogen ions and potassium ions by the renal tubule are inversely related. Therefore,
increased excretion of hydrogen ions during metabolic compensation for respiratory acidosis will result
in decreased secretion of potassium ions, (or increased retention). This would potentially cause a rise in
serum potassium. The filtered load of potassium depends only on plasma potassium concentration and
glomerular filtration rate, not on plasma pH.

321. During a marathon attempt a runner collapses and is admitted with severe acute dehydration. This
patient most likely has: Single best answer question choose ONE true option only.
Decreased baroreceptor firing rate CORRECT ANSWER
Decreased plasma osmolarity
High renal water excretion
Low plasma ADH levels
Low water permeability of collecting duct tubular.

Acute dehydration results in decreased plasma volume and increased plasma osmolarity, since more
water than salt is lost in sweat. The decrease in plasma volume leads to an inhibition of the baroreceptors
and a lower firing rate. The increase in plasma osmolarity leads to increased ADH secretion and high
plasma ADH levels, which increases water permeability of collecting duct cells. Therefore more water is
reabsorbed by the kidneys and renal water excretion is low.

322. The Haemoglobin Oxygen-Dissociation Curve is shifted to the left by: Single best answer question
choose ONE true option only.
An increase in pCO2 YOUR ANSWER
A fall in pH
A rise in temperature
An increase in 2,3-DPG
Fetal haemoglobin CORRECT ANSWER.

The haemoglobin oxygen dissociation curve is sigmoidal in shape which reflects the underlying
biochemical properties of haemoglobin. The significance of the sigmoidal curve is that haemoglobin
becomes highly saturated at high oxygen partial pressures (and is therefore highly efficient at collecting
---------------------------------------------------------------------------------------------------------------------------------------------------------------------------------------------------------------
Dr Mohammed Shamsul Islam Khan, Medical Officer, Clinical Neuro-Surgery, National Institute of Neuro-Sciences and Hospital
Sher-E-Bangla Nagar, Dhaka-1207, Bangladesh. Mobile: +880 1713 455 662, +880 1685 811979. E-mail: drsikhan@gmail.com
MyPasTest MRCS A Online - Jan Exam 2015
10. Physiology; System Specific Physiology (399Qs)
----------------------------------------------------------------------------------------------------------------------------------
oxygen), and releases a significant amount of oxygen at pressures which are fairly low, but not extremely
so (with the result that haemoglobin is highly effective at supplying oxygen where it is needed).

The effect of things that shift the curve to the right (raised CO2, lowered pH, increased temperature,
increase in 2,3-DPG) is to increase oxygen availability in the tissues. The effect of CO 2/H+ on O2 carriage
is known as the Bohr shift or effect. This is exactly what is needed in metabolising tissues; release of Page |
acids or CO2 thus liberates O2 to fulfil the metabolic needs of the tissue. Do not confuse this with the 979
effect of changes in O2 on CO2 carriage which is called the Haldane effect.

A shift of the oxygen dissociation curve to the left is characteristic of fetal haemoglobin. When compared
with adult haemoglobin, it is composed of 2 alpha and 2 gamma chains, instead of the usual 2 alpha and 2
beta chains of adult haemoglobin. This arrangement assists in the transfer of oxygen across the placenta
from the maternal to the fetal circulation. The corollary of this is that fetal tissue oxygen levels have to be
low to permit the release of oxygen from the haemoglobin.

323. The following metabolic changes occur in the ebb phase (first 24 hours) of response to injury?
Single best answer question choose ONE true option only.
Plasma pH increases YOUR ANSWER
The plasma level of free fatty acids decreases
Hypoglycemia
The plasma level of non protein nitrogen decreases
Plasma glycerol increases CORRECT ANSWER.

There is usually acidosis (pH decreases). Lipolysis increases leading to increase in fatty acids and
glycerol. There is hyperglycemia and an increased level of non protein nitrogen.

324. High titres of antithyroid microsomal and antithyroglobulin antibodies would suggest which of the
following diagnoses in a patient presenting with a complaint of tiredness? Single best answer - choose
ONE true option only.
Hashimotos thyroiditis YOUR ANSWER (Correct Answer)
Reidels thyroiditis
Graves disease
Hypoparathyroidism
Idiopathic hypothyroidism.

This finding in Hashimotos thyroiditis is characteristic, but lower titres can occur in Reidels thyroiditis
and Graves disease. High titres of these antibodies in euthyroid individuals indicate the possibility of
future thyroid failure, but this may be many years away; hence the need for thyroid function tests every 1
2 years in such individuals.
---------------------------------------------------------------------------------------------------------------------------------------------------------------------------------------------------------------
Dr Mohammed Shamsul Islam Khan, Medical Officer, Clinical Neuro-Surgery, National Institute of Neuro-Sciences and Hospital
Sher-E-Bangla Nagar, Dhaka-1207, Bangladesh. Mobile: +880 1713 455 662, +880 1685 811979. E-mail: drsikhan@gmail.com
MyPasTest MRCS A Online - Jan Exam 2015
10. Physiology; System Specific Physiology (399Qs)
----------------------------------------------------------------------------------------------------------------------------------
325. In calcium physiology which of the following statements is correct? Select one answer only.
Parathyroid hormone (PTH) mobilises calcium from bone and initiates calcium reabsorption from the gut
YOUR ANSWER (Correct Answer)
Calcitonin is produced by the medullary cells of the thyroid gland
approx 70% of the bodys calcium is stored in bone Page |
hypercalcaemia may occur in Cushings disease 980
calcium is excreted in the stools and urine at a rate of 1020 mg/day.

PTH increases bone resorption, calcium reabsorption and calcium absorption. Calcitonin is produced by
the parafollicular cells of the thyroid gland, and inhibits bone resorption. Bone is the primary storage site
for calcium, where approximately 1 kg (99% of total body calcium) is stored. Calcium is filtered.

326. The electrocardiogram (ECG) strip of a patient shows sagging of the ST segment, depression of the
T-wave and elevation of the U-wave. What is the most likely electrolyte abnormality responsible for these
ECG findings? Single best answer question choose ONE true option only.
Hyperkalaemia YOUR ANSWER
Hypermagnesaemia
Hypocalcaemia
Hypokalaemia CORRECT ANSWER
Hypomagnesaemia.

Cardiac effects of hypokalaemia are usually minimal until plasma potassium levels are less than 3 mmol/l.
Hypokalaemia leads to sagging of the ST segment, depression of the T-wave and elevation of the U-wave.
With marked hypokalaemia, the T-wave become progressively smaller and the U-wave becomes
increasingly larger. Sometimes, a flat or positive T-wave merges with a positive U-wave, which can be
confused with qT prolongation.

Hypokalemia can also cause premature ventricular and atrial contractions, ventricular and atrial
tachyarrhythmias and second- or third-degree atrioventricular block. Such arrhythmias become more
severe with increasingly severe hypokalaemia; eventually, ventricular fibrillation can occur. Patients with
significant pre-existing heart disease and/or those receiving digoxin are at risk of cardiac conduction
abnormalities, even with mild hypokalaemia.

327. Nociception (pain). Single best answer question choose ONE true option only.
Is transmitted faster through C fibers than through A delta fibers YOUR ANSWER
Pain impulse received in the dorsal horn can be modulated by other descending spinal inputs CORRECT
ANSWER
Opioids act on receptors in the peripheral nerves
Side effects of opioids can be reversed by neostigmine
Glycine is excitatory pain neurotransmitter.
---------------------------------------------------------------------------------------------------------------------------------------------------------------------------------------------------------------
Dr Mohammed Shamsul Islam Khan, Medical Officer, Clinical Neuro-Surgery, National Institute of Neuro-Sciences and Hospital
Sher-E-Bangla Nagar, Dhaka-1207, Bangladesh. Mobile: +880 1713 455 662, +880 1685 811979. E-mail: drsikhan@gmail.com
MyPasTest MRCS A Online - Jan Exam 2015
10. Physiology; System Specific Physiology (399Qs)
----------------------------------------------------------------------------------------------------------------------------------
Pain impulse received by dorsal horn can be modulated by other ascending and descending spinal inputs
(Gate Theory). Pain is transmitted faster in myelinated A delta fibers, opioids act on -receptors in the
central (not peripheral) nervous system and their effects can be reversed by naloxone. Glycine is an
inhibitory neurotransmitter.

328. A 42-year-old lady undergoes a TRAM breast reconstruction following mastectomy for breast cancer. Page |
The anaesthetist inserts a CVP line for monitoring. Which of the following statements regarding CVP is
correct? Single best answer - select one answer only. 981
It indicates left ventricular filling pressure in left ventricular failure YOUR ANSWER
It is most useful when taken as a one off measurement
In normovolemia there will be a fall in CVP sustained for 10 minutes
It can be estimated by examining the external jugular vein CORRECT ANSWER
It is high in hypovolaemia.

The central venous pressure (CVP) can be measured invasively by using a manometer after central
venous cannulation. The CVP can be estimated by examining the height of the pulsation in the external
jugular vein. The height of the pulsation should be compared with the angle of Louis as the patient sits at
45 to the horizontal. Zero is taken at the level of the right atrium. This is somewhat arbitrary and so one-
off measurements are not very useful unless there is a profoundly low CVP indicating profound
hypovolemia.

More useful is to measure the changes in CVP occurring during and after the administration of a bolus of
200 ml of colloid intravenously. If the patient is hypovolemic, there will be only a very small transient
change in CVP. In normovolemia there will be a rise in CVP, which will be sustained for 10 minutes or so.
In hypervolemia the rise in CVP will be sustained for longer. A high CVP indicates myocardial failure. CVP
indicates the right and left ventricular filling pressure only when these are equal. If there is ventricular
failure and the filling pressure of the two ventricles is not equivalent then CVP is a measure only of the
right atrial pressure.

329. On a weekend ward round, you review a 57-year-old woman who is 8 days post anterior resection
with a stapled anastomosis. She has been mobilising around the ward but last night felt some discomfort
in the left iliac fossa. Looking at her observation chart, at that point she was tachycardic. This morning
she spiked a temperature of 37.7oC. She still has some left iliac fossa discomfort. Your consultant asks
you to scan her to look for a leak. Which is the best dynamic test to organise? Single best answer - select
one answer only.
A Flexible sigmoidoscopy YOUR ANSWER
B Barium Enema
C CT scan with IV contrast
D Gastrograffin Enema CORRECT ANSWER
E Ultrasound Scan.
---------------------------------------------------------------------------------------------------------------------------------------------------------------------------------------------------------------
Dr Mohammed Shamsul Islam Khan, Medical Officer, Clinical Neuro-Surgery, National Institute of Neuro-Sciences and Hospital
Sher-E-Bangla Nagar, Dhaka-1207, Bangladesh. Mobile: +880 1713 455 662, +880 1685 811979. E-mail: drsikhan@gmail.com
MyPasTest MRCS A Online - Jan Exam 2015
10. Physiology; System Specific Physiology (399Qs)
----------------------------------------------------------------------------------------------------------------------------------
Anastomotic leak is a complication of bowel anastomosis and usually occurs within the first 14 days post
surgery. Patients can present severely unwell with peritonitis if a large leak but sometimes in small leaks
the patient can be less compromised. A high level of suspicion should be employed if any patient with a
primary anastomosis deteriorates. A small leak could seal off if plugged by omentum or can potentially
worsen resulting in peritonitis. A septic screen should be performed and investigations to rule out an
anastomotic leak should be arranged. Page |
982
A dynamic test is best to demonstrate an active leak from the anastomosis a water soluble enema such
as gastrograffin is the best test. A wisp of contrast maybe seen from the anastomosis on xray screening
which diagnoses an active leak. A Barium Enema offers the same diagnostic images but should not be
used where a leak is possible as leakage of barium into the peritoneal cavity can cause peritonitis. A CT
scan with just IV contrast gives a static image - it may show a collection of fluid or a gas bubble next to
the anastomosis with a high index of suspicion for leak but is not accurate as showing an active leak
such as with a contrast enema.

In a relatively well patient a CT may show fluid which may be difficult to differentiate between normal post
operative fluid and an active leak. Ultrasound would not be particularly useful here- it may show a fluid
collection only. A flexible sigmoidoscopy can be useful to look at the mucosa for evidence of ischaemia
but should be used with great caution as this could disrupt the anastomosis.

330. You are explaining to a patient that you need to place a central venous catheter and you have chosen
to place a femoral line. The patient asks what the complications of this procedure are. Which of the
following complications is most likely with a femoral central venous catheter? Single best answer - select
one answer only.
Cardiac arrhythmias YOUR ANSWER (Correct Answer)
Damage to the thoracic duct
Injury to the brachial plexus
Pericardial tamponade
Pneumothorax.

Recognised complications of central venous catheter insertion include pneumo- or haemothorax, and air
embolism due to the head-down position of insertion. Cardiac arrhythmias can be seen with the catheter
inserted too far. Pericardial tamponade, injury to the brachial plexus and thoracic duct damage are
rare.Other complications include pseudo-aneurysm formation, arterial puncture, haematoma formation
and infection. While all of the potential answers are know complications of central venous catheterisation,
the question specifies the femoral route making intrathoracic complications unlikely, except for cardiac
arrhythmias due to inserting the catheter too far. Other known complications of femoral central venous
access include bowel penetration, bladder penetration, psoas abscess and femoral nerve injury.

331. A 35-year old lady is found at her pre-operative assessment to be be tachycardic, with a HR of
120bpm, extremely anxious and to have a smoothly-enlarged goitre in her neck. A diagnosis of Graves
disease is confirmed. What are her bloods likely to show? Single best answer select one answer only.
---------------------------------------------------------------------------------------------------------------------------------------------------------------------------------------------------------------
Dr Mohammed Shamsul Islam Khan, Medical Officer, Clinical Neuro-Surgery, National Institute of Neuro-Sciences and Hospital
Sher-E-Bangla Nagar, Dhaka-1207, Bangladesh. Mobile: +880 1713 455 662, +880 1685 811979. E-mail: drsikhan@gmail.com
MyPasTest MRCS A Online - Jan Exam 2015
10. Physiology; System Specific Physiology (399Qs)
----------------------------------------------------------------------------------------------------------------------------------
Low thyroxine; which regulates carbohydrate metabolism YOUR ANSWER
Raised thyroxine; secreted by the colloid cells of the thyroid gland
Raised thyroxine level; mostly bound to albumin in the blood
Raised thyroxine-stimulating hormone (TSH) from the posterior pituitary
Page |
Thyroid-stimulating hormone receptor immunoglobulins CORRECT ANSWER.
983
Thyroxine regulates carbohydrate and lipid metabolism. It is secreted from the follicular cells of the
thyroid gland and is transported in the blood 99% bound to thyroid binding globulin. Its secretion is
controlled in a negative feedback system by thyroid stimulating hormone (TSH) from the anterior
pituitary. Graves disease is an autoimmune condition in which immunoglobulins form to TSH receptors.
These bind to the receptors and cause stimulation, resulting in excess thyroxine secretion and clinical
hyperthyroidism. TSH is suppressed through negative feedback.

332. Which of the following patients is most likely to have a hyperplastic arteriolosclerosis with fibrinoid
necrosis, petechial haemorrhages and microinfarcts in the kidneys, in conjunction with a markedly
elevated plasma renin? Single best answer question choose ONE true option only.
A 56-year-old man with an acute myocardial infarction YOUR ANSWER
A 6-year-old boy with albuminuria
A 62-year-old woman with end-stage renal disease
A 15-year-old man with recent streptococcal infection
A 45-year-old woman with scleroderma CORRECT ANSWER.

The diffuse form of scleroderma can be associated with hyperplastic arteriolosclerosis and malignant
hypertension. Post-streptococcal glomerulonephritis produces glomerular hypercellularity. End-stage
kidneys all look alike: thickened arteries, globally sclerotic glomeruli and interstitial scarring with chronic
inflammation. The child is most likely to have minimal-change disease and the kidney will be grossly
normal, will be normal on light microscopy and will show only fusion of foot processes on electron
microscopy. An acute myocardial infarction could cause decreased cardiac output with decreased renal
perfusion and ischaemia, leading to acute tubular necrosis.

333. You are asked to see an 82-year-old woman under the medical team with a lower respiratory tract
illness and non-tender abdominal distension. There are dilated loops on her abdominal film. Bloods
reveal sodium 138, potassium 3.1, urea 10, creatinine 118, white cell count 17, c-reactive protein 115. What
is the most likely diagnosis? Single best answer - select one answer only.
Bowel Cancer YOUR ANSWER
Constipation
Diverticulitis
Pseudo-obstruction CORRECT ANSWER
Small bowel obstruction.

---------------------------------------------------------------------------------------------------------------------------------------------------------------------------------------------------------------
Dr Mohammed Shamsul Islam Khan, Medical Officer, Clinical Neuro-Surgery, National Institute of Neuro-Sciences and Hospital
Sher-E-Bangla Nagar, Dhaka-1207, Bangladesh. Mobile: +880 1713 455 662, +880 1685 811979. E-mail: drsikhan@gmail.com
MyPasTest MRCS A Online - Jan Exam 2015
10. Physiology; System Specific Physiology (399Qs)
----------------------------------------------------------------------------------------------------------------------------------
Pseudo-obstuction is an example of non-mechanical bowel obstruction. It can be acute or chronic. Acute
is also known as Ogilvie syndrome. In 80% of cases, colonic pseudo-obstruction is associated with other
clinical conditions. The most common are cardio-respiratory illness, post-trauma or pelvic surgery.
Treatment should be based on correcting the associated cause such as the respiratory illness or
correcting electrolyte imbalances usually potassium, magnesium or phosphate. This may resolve the
obstruction. Otherwise consider endoscopic decompression. Neostigmine has also been used in some Page |
chronic cases. 984
334. Capacitation of sperms takes place in the uterus. Capacitation allows: Single best answer question
choose ONE true option only.
Decreased energy metabolism YOUR ANSWER
Enhanced motility CORRECT ANSWER
Inhibition of acrosome reaction
Release of FSH
Release of LH.

Ejaculated sperms are able to swim vigorously and appear structurally mature; however, they require a
period of incubation in the female reproductive tract before gaining the potential for fertilisation.
Glycoprotein molecules coating the surface of the sperm cell are solubilised by uterine fluid. These
molecules were originally acquired in the epididymis and seminal plasma. This process is called
capacitation. An oestrogen-dominated uterus is optimal for capacitation. Note that sperm survival is also
optimal and that ovulation occurs under these conditions. Capacitation allows: (1) increased energy
metabolism; (2) enhanced motility; and (3) the acrosome reaction or activation.

335. One of your patients has undergone an emergency laparotomy and is on the intensive care unit
being maintained on Controlled mechanical ventilation via an endotracheal route. What physiological
changes would you expect in someone with this type of ventilation? Select one answer only.
Decreased thoracic pressure, decreased venous return and decreased cardiac output YOUR ANSWER
Decreased thoracic pressure, increased venous return and decreased cardiac output
Increased thoracic pressure, decreased venous return and decreased cardiac output CORRECT ANSWER
Increased thoracic pressure, increased venous return and decreased cardiac output
Increased thoracic pressure, increased venous return and increased cardiac output.

Controlled mechanical ventilation requires intubation and hence necessitates Intensive Therapy Unit
admission. The patient must be sedated to tolerate the endotracheal tube. Oxygen concentration can
reach 100%, although this is avoided to prevent its toxic effects. Because of increased thoracic pressure,
venous return and hence cardiac output are decreased. Positive End Expiratory Pressure ventilation
increases the risk of barotraumas, which can result in the rapid formation of a tension pneumothorax.

336. A 36-year-old woman is diagnosed with a phaeochromocytoma. She is most likely to have increased
serum levels of: Single best answer question choose ONE true option only.
---------------------------------------------------------------------------------------------------------------------------------------------------------------------------------------------------------------
Dr Mohammed Shamsul Islam Khan, Medical Officer, Clinical Neuro-Surgery, National Institute of Neuro-Sciences and Hospital
Sher-E-Bangla Nagar, Dhaka-1207, Bangladesh. Mobile: +880 1713 455 662, +880 1685 811979. E-mail: drsikhan@gmail.com
MyPasTest MRCS A Online - Jan Exam 2015
10. Physiology; System Specific Physiology (399Qs)
----------------------------------------------------------------------------------------------------------------------------------
Aldosterone YOUR ANSWER
Cortisol
Noradrenaline CORRECT ANSWER
Oxytocin
Page |
Vasopressin.
985
A phaeochromocytoma is a neuroendocrine tumour of the medulla of the adrenal glands originating in the
chromaffin cells, which secretes excessive amounts of catecholamines, usually adrenaline and
noradrenaline. Extra-adrenal paragangliomas (often described as extra-adrenal phaeochromocytomas)
are closely related, though less common, tumours that originate in the ganglia of the sympathetic
nervous system and are named based upon the primary anatomical site of origin. Traditionally it is known
as the 10% tumour, since bilateral disease is present in approximately 10% of patients, approximately
10% of tumours are malignant and approximately 10% are located in chromaffin tissue outside the adrenal
gland.

337. You are reviewing a patient with autonomic dysfunction following a spinal cord injury. Which of the
following statements is correct regarding autonomic neurons? Single best answer - select one answer
only.
They arise from the spinal cord only YOUR ANSWER
They have ganglia near the target organ for the sympathetic system
They form in the mid-brain and the sacrum for the parasympathetic system CORRECT ANSWER
They have ganglia running in a chain alongside the spinal cord for the parasympathetic system
They have adrenaline as their major pre-ganglionic synaptic neurotransmitter.

The parasympathetic neurons are formed at the upper and lower extremities of the spinal cord, in the mid-
brain and hindbrain (fibres are carried by cranial nerves III, VII, IX and X) but also in the sacral spinal cord
(S2S4). The neurons of the sympathetic nervous system are formed throughout the thoracic and upper
lumbar spine. The ganglia of the sympathetic nervous system are found in a chain either side of the
spine. The ganglia of the parasympathetic nervous system tend to lie near the target organ (eg coeliac
ganglion which supplies the foregut). The neurotransmitter at the synapses in the ganglia of both the
sympathetic and parasympathetic nervous systems is acetylcholine. The synapses of the post-ganglionic
fibres with the end organs in the sympathetic nervous system involve catecholaminergic
neurotransmission.

338. One of your patients suffers a cardiac arrest on the ward. The monitor shows pulseless electrical
activity (PEA). The patient underwent an AAA repair 1 day ago and complained of abdominal pain prior to
arresting. Which is the most likely cause of the PEA arrest? Select one answer only.
Cardiac tamponade YOUR ANSWER
Electric shock

---------------------------------------------------------------------------------------------------------------------------------------------------------------------------------------------------------------
Dr Mohammed Shamsul Islam Khan, Medical Officer, Clinical Neuro-Surgery, National Institute of Neuro-Sciences and Hospital
Sher-E-Bangla Nagar, Dhaka-1207, Bangladesh. Mobile: +880 1713 455 662, +880 1685 811979. E-mail: drsikhan@gmail.com
MyPasTest MRCS A Online - Jan Exam 2015
10. Physiology; System Specific Physiology (399Qs)
----------------------------------------------------------------------------------------------------------------------------------
Hypovolaemia CORRECT ANSWER
Hypoxia
Tension pneumothorax.

The causes of a PEA arrest include hypovolaemia, hypoxia, hypothermia, hyper/hypokalaemia, tension Page |
pneumothorax, cardiac tamponade, drug toxicity, electrolyte abnormalities, thromboembolism and
mechanical obstruction. The treatment is basic life support, adrenaline (epinephrine; 1mg per 3 minutes) 986
and treatment of the underlying cause. Electric shock usually causes a ventricular fibrillatory arrest or
occasionally an asystolic arrest. The abdominal pain in this patient may represent leak/re-rupture, making
hypovolaemia the most likely cause in the above patient.

339. Which of the following is the most important direct stimulus to respiration? Single best answer quest
ion choose ONE true opt ion only.
Increased pCO2 of the CSF YOUR ANSWER
Increased H+ concentrat ion of the CSF CORRECT ANSWER
Decreased arterial pO2
Decreased arter ial pH
Decreased arterial pCO2.

Chemoreceptors involved with the control of respiration are present in the central nervous system and
peripherally. The central chemoreceptors are situated in the ventral medulla, and increase firing in
response to the H+ concentration of the brain extra cellular fluid, which is directly related to the H +
concentration in the CSF. H+ / HCO3 cannot easily cross the blood brain barrier, but CO2 does so readily.
This frees H+ions, causing a low CSF pH, increased firing of the central chemoreceptors and increased
ventilation.

Peripheral chemoreceptors are found in the carotid bodies and aortic arch, and increase their firing rate in
response to decreased PaO2, decreased arterial pH and increased paCO2. These are much less important,
however, in stimulating respiration than the central chemoreceptors.

340. Bile salt reuptake principally occurs in the: Single best answer question choose ONE true option
only.
Duodenum YOUR ANSWER
Jejenum
Ileum CORRECT ANSWER
Colon
Caecum.
90-95% of the bile salts are absorbed from the small intestine and then excreted again from the liver; most
are absorbed from the terminal ileum. This is known as the enterohepatic circulation. The entire pool
recycles twice per meal and approximately 6-8x per day.
---------------------------------------------------------------------------------------------------------------------------------------------------------------------------------------------------------------
Dr Mohammed Shamsul Islam Khan, Medical Officer, Clinical Neuro-Surgery, National Institute of Neuro-Sciences and Hospital
Sher-E-Bangla Nagar, Dhaka-1207, Bangladesh. Mobile: +880 1713 455 662, +880 1685 811979. E-mail: drsikhan@gmail.com
MyPasTest MRCS A Online - Jan Exam 2015
10. Physiology; System Specific Physiology (399Qs)
----------------------------------------------------------------------------------------------------------------------------------
Disruption of the enterohepatic circulation, either by terminal ileal resection or through a diseased
terminal ileum (e.g. Crohns disease), results in decreased fat absorption and cholesterol gallstone
formation. The latter is believed to result because bile salts normally make cholesterol more water-soluble
through the formation of cholesterol micelles. Loss of reuptake also results in the presence of bile salts in
colonic contents, which alters colonic bacterial growth and stool consistency.
Page |
341. You have a patient in the intensive care unit with a Swan-Ganz catheter in situ, which was inserted 987
approximately 4 hours ago. ECG changes are noted following insertion. Which ECG changes are most
often seen following Swan-Ganz catheterisation? Single best answer - select one answer only.
Transient left bundle branch block occurs in up to 5% of patients and usually resolves within 24 hours
YOUR ANSWER
Transient left bundle branch block occurs in up to 5% of patients and usually resolves within 1 week
Transient right bundle branch block occurs in up to 5% of patients and usually resolves within 24 hours
CORRECT ANSWER
Transient right bundle branch block occurs in up to 5% of patients and usually resolves within 1 week
Transient bifascicular block occurs in up to 5% of patients and usually resolves on removal of the catheter.

Transient right bundle branch block is seen in up to 5% of patients, but usually resolves within 24 hours.
It is the commonest cardiac anomaly with Swan-Ganz catheterisation.

342. You are auscultating a patient's chest as part of their pre-operative assessment and their first heart
sound is very prominent. What does the first heart sound correspond to? Select one answer only.
The closure of the atrioventricular valve YOUR ANSWER (Correct Answer)
The closure of the aortic valve
The closure of the pulmonary valve
The opening of the mitral and tricuspid valves
The opening of the pulmonary valve.

The first heart sound represents atrioventricular (AV) valve closure; the second heart sound represents
aortic and pulmonary valve closure.
343. You respond to a trauma call. The patient has bilateral chest drains inserted and has lost 1000ml of
fresh blood into one and 750ml of fresh blood into the other. The patient has a pulse rate of 80bpm. Which
of the following could ALL contribute towards suppressing the tachycardia response despite such
significant haemorrhage? Select one answer only.
Hyperthermia, a pacemaker, alpha-blockers and calcium antagonists YOUR ANSWER
Hyperthermia, a pacemaker, beta-blockers and calcium antagonists
Hypothermia, a pacemaker, alpha-blockers and calcium antagonists
Hypothermia, a pacemaker, beta-blockers and calcium antagonists CORRECT ANSWER
Hypothermia, a pacemaker, beta-blockers and dopamine antagonists.
---------------------------------------------------------------------------------------------------------------------------------------------------------------------------------------------------------------
Dr Mohammed Shamsul Islam Khan, Medical Officer, Clinical Neuro-Surgery, National Institute of Neuro-Sciences and Hospital
Sher-E-Bangla Nagar, Dhaka-1207, Bangladesh. Mobile: +880 1713 455 662, +880 1685 811979. E-mail: drsikhan@gmail.com
MyPasTest MRCS A Online - Jan Exam 2015
10. Physiology; System Specific Physiology (399Qs)
----------------------------------------------------------------------------------------------------------------------------------
Tachycardia in response to haemorrhage may be absent in: the elderly; patients on B-blockers and
calcium antagonists; patients with hypothermia; and patients who have a pacemaker. Infants develop a
tachycardia and the rate will be much higher than in adults. Athletes have a higher cardiac output and
stroke volume but a lower resting pulse than the average population. The usual responses to
hypovolaemia may not be manifest in athletes until a significant blood loss has occurred. Dopamine
antagonists e.g. metoclopramide and alpha-blockers, do not affect the response.
Page |
344. Menstrual bleeding serves as a sign that a woman has not become pregnant. Menstrual bleeding is a 988
consequence of: Single best answer question choose ONE true option only.
Progesterone withdrawal YOUR ANSWER (Correct Answer)
Proliferative phase of endometrium
Prostaglandin withdrawal
Secretory phase of endometrium
Thickening of endometrium.

In the absence of a pregnancy and without human chorionic gonadotrophin (hCG), the corpus luteum
demises and inhibin and progesterone levels fall. Progesterone withdrawal leads to menstrual shedding
(progesterone-withdrawal bleeding) and falling inhibin levels allow FSH levels to rise to raise a new crop
of follicles.

345. A 20-year-old man presents with mild jaundice following a flu-like illness. Following review by a
gastroenterologist, he has been told that a diagnosis of Gilberts syndrome is probable. Which laboratory
test is most likely to confirm this diagnosis? Single best answer question choose ONE true option only.
Absence of bilirubin in the urine YOUR ANSWER (Correct Answer)
Decreased serum haptoglobin concentration
Elevated serum aspartate aminotransferase (transaminase, AST) activity
Increased reticulocyte count
Increased urinary urobilinogen excretion.
In Gilberts syndrome, the excess bilirubin is unconjugated, and does not appear in the urine. The same is
true for jaundice secondary to haemolysis. However, in haemolytic jaundice, urinary urobilinogen is
increased (increased production of bilirubin, and hence of urobilinogen), the reticulocyte count may be
elevated and serum haptoglobin concentration decreased. Haemolysis may also cause a slight increase
in serum aminotransferase (transaminase) activity.

346. A 52-year-old female with longstanding rheumatoid arthritis is seen in a pre-assessment clinic prior
to a planned left total knee replacement. On examination she is noted to be overweight and struggles to
rise from a chair. She is noted to be on long term steroids for rheumatoid arthritis and is found to have a
high blood pressure, raised fasting plasma glucose and striae seen in her skin. Which of the following
conditions is she most likely to have developed secondary to her medication? Single best answer - select
one answer only.
Addisons disease YOUR ANSWER
---------------------------------------------------------------------------------------------------------------------------------------------------------------------------------------------------------------
Dr Mohammed Shamsul Islam Khan, Medical Officer, Clinical Neuro-Surgery, National Institute of Neuro-Sciences and Hospital
Sher-E-Bangla Nagar, Dhaka-1207, Bangladesh. Mobile: +880 1713 455 662, +880 1685 811979. E-mail: drsikhan@gmail.com
MyPasTest MRCS A Online - Jan Exam 2015
10. Physiology; System Specific Physiology (399Qs)
----------------------------------------------------------------------------------------------------------------------------------
Congenital adrenal hyperplasia
Conn syndrome
Cushing syndrome CORRECT ANSWER
Phaeochromocytoma.
Page |
Cushings syndrome is caused by persistently and inappropriately raised glucorticoid levels. In most 989
cases this results from administration of steroids for medical conditions e.g. rheumatoid arthritis, Crohns
disease etc.

A prolonged elevation of plasma cortisol would produce a clinical picture of Cushings syndrome. Protein
depletion is a feature as a result of excess protein catabolism. The amino acids liberated from the
catabolism of proteins are converted into glucose in the liver. A resultant hyperglycaemia is seen, and
this may precipitate diabetes mellitus.

Cortisol can exert a significant mineralocorticoid action, so causing sodium retention and potassium
loss. Excess cortisol also has the effect of decreasing bone formation and increasing bone resorption,
leading to osteoporosis and loss of bone mass.

347. A 78-year-old man undergoes a very prolonged transurethral resection of prostate (TURP). Which of
the following may occur? Single best answer select one answer only.
Bradycardia YOUR ANSWER (Correct Answer)
Hyperglycaemia
Hypernatraemia
Increased osmolality of the plasma
Increased total body sodium.

A considerable amount of glycine may enter the circulation following transurethral resection of the
prostate (TURP), especially if resection is prolonged. Metabolic breakdown products of glycine can cause
biochemical effects; hyponatraemia, which may be part of a transurethral resection syndrome. This leads
to reduced plasma osmolality.

Early symptoms and signs of this include nausea, bradycardia, confusion and headache. If left untreated,
seizures, hypoxia and cardiac events can ensue.

348. A patient awaiting surgery for a parathyroid adenoma is admitted with confusion and drowsiness.
The diagnosis of hypercalcaemic crisis is made. What should be the initial treatment? Single best answer
select one answer only.
Intravenous magnesium YOUR ANSWER
Intravenous fluids CORRECT ANSWER
Steroids

---------------------------------------------------------------------------------------------------------------------------------------------------------------------------------------------------------------
Dr Mohammed Shamsul Islam Khan, Medical Officer, Clinical Neuro-Surgery, National Institute of Neuro-Sciences and Hospital
Sher-E-Bangla Nagar, Dhaka-1207, Bangladesh. Mobile: +880 1713 455 662, +880 1685 811979. E-mail: drsikhan@gmail.com
MyPasTest MRCS A Online - Jan Exam 2015
10. Physiology; System Specific Physiology (399Qs)
----------------------------------------------------------------------------------------------------------------------------------
Calcitonin
Intravenous phosphate.

Hypercalcaemic crisis occurs when a patient with mild hypercalcaemia has a rapid rise in plasma calcium.
This is associated with rapid deterioration in the patients condition, in the form of confusion, drowsiness,
vomiting and dehydration. The first-line treatment of hypercalcaemia is intravenous fluid replacement.
Page |
990
Hypercalcaemic crises due to hyperparathyroidism do not respond to treatment with steroids. The
condition may be controlled pre-operatively by the careful use of calcitonin, mithramycin or IV phosphate.
Crises may be precipitated by intercurrent illness, operation or infection.

349. The rate at which a liquid meal leaves the stomach is? Single best answer question choose ONE
true option only.
Greater in the upright than in the supine position YOUR ANSWER
Proportional to the volume of stomach content CORRECT ANSWER
Greater if the meal contains fat
Slower if the meal is 5% glucose than if it is 50% glucose
Slower if vagotomy and drainage procedure (such as gastroenterostomy or pyloroplasty) has been
performed.

Gastric emptying accelerates on lying down. The rate of gastric emptying at any moment is proportional
to the volume present in the stomach at that moment.

When the fat reaches the duodenum it stimulates mixed hormonal and vagal mechanisms that slow the
rate of stomach emptying. An isotonic meal will empty at maximal rate but osmotically stronger or weaker
solutions will empty more slowly. Vagotomy may temporarily slow gastric emptying, but its long term
effect is to increase the rate of gastric emptying or leave it un changed so if a drainage procedure is
accompanied by vagotomy there will be a tendency towards accelerating gastric emptying.

350. You are assisting in ICU in performing a tracheostomy. The consultant asks you what physiological
affect this procedure will have? Select one answer only.
Reduce airway resistance YOUR ANSWER (Correct Answer)
Reduce lung compliance
Reduce physiological dead space
Reduce residual volume
Reduce vital capacity.

A tracheostomy will decrease the anatomical dead space but not the physiological dead space.
Anatomical dead space is the respiratory system volume exclusive of the alveoli. Physiological dead
space is the volume of gas not equilibrating with blood, or wasted ventilation. In normal healthy
---------------------------------------------------------------------------------------------------------------------------------------------------------------------------------------------------------------
Dr Mohammed Shamsul Islam Khan, Medical Officer, Clinical Neuro-Surgery, National Institute of Neuro-Sciences and Hospital
Sher-E-Bangla Nagar, Dhaka-1207, Bangladesh. Mobile: +880 1713 455 662, +880 1685 811979. E-mail: drsikhan@gmail.com
MyPasTest MRCS A Online - Jan Exam 2015
10. Physiology; System Specific Physiology (399Qs)
----------------------------------------------------------------------------------------------------------------------------------
individuals, the two dead spaces are identical. Airway resistance and lung compliance would also be
improved by tracheostomy and this is one of the advantages of such a procedure. A tracheostomy would
have no effect on the residual lung volume or vital capacity.

351. You have a patient who is hypoxic with a decreased respiratory rate. What stimulates an increase in
pulmonary ventilation? Select one answer only. Page |
Carotid body chemoreceptors YOUR ANSWER (Correct Answer)
991
Increased arterial PO2
Decreased arterial PCO2
Increased pH
Reflex activation in the respiratory centre in the cerebellum.

Pulmonary ventilation is stimulated by acidosis, hypercapnia and hypoxia, mediated via respiratory
chemoreceptors in the carotid and aortic bodies and medulla of the brain. The respiratory centre is not
located in the cerebellum, but in the medulla and pons.

352. The most important physiological function of the lymphatic system is to: Single best answer
question choose ONE true option only.
Transport fluid and proteins away from the interstitium to the blood YOUR ANSWER (Correct Answer)
Concentrate proteins in the lymph
Remove particulate materials from the interstitium
Transport antigenic materials to lymph nodes
Create negative pressure in the free interstitial fluid.

The lymphatic system performs all of the functions listed as options, but removal of fluid and proteins
(especially proteins) is the most important physiological function. In the absence of a lymphatic system,
the interstitial fluid protein concentration would increase greatly, causing widespread extracellular
oedema.
353. Pepsinogen is secreted from which cells of the stomach? Single best answer question choose ONE
true option only.
Parietal cells YOUR ANSWER
Chief cells CORRECT ANSWER
G cells
Mucous cells
IX.
There are many types of cells located within the gastric glands;
1. Parietal cells secrete HCl and intrinsic factor
2. Chief cells secrete pepsinogen, the precursor of pepsin
3. G cells secrete the hormone gastrin
4. Mucous cells secrete mucous
---------------------------------------------------------------------------------------------------------------------------------------------------------------------------------------------------------------
Dr Mohammed Shamsul Islam Khan, Medical Officer, Clinical Neuro-Surgery, National Institute of Neuro-Sciences and Hospital
Sher-E-Bangla Nagar, Dhaka-1207, Bangladesh. Mobile: +880 1713 455 662, +880 1685 811979. E-mail: drsikhan@gmail.com
MyPasTest MRCS A Online - Jan Exam 2015
10. Physiology; System Specific Physiology (399Qs)
----------------------------------------------------------------------------------------------------------------------------------
354. What is the half life of free triiodothyronine (T3) in the blood? Single best answer question choose
ONE true option only.
1 minute YOUR ANSWER
1 hour
1 day CORRECT ANSWER Page |
1 week 992
1 month.

Most of the T3 and thyroxine (T4) are carried in plasma bound to thyroxine binding globulin, and are
inactive in this state. Only 1% of T3 and 0.05% of T4 is free. T3is the active hormone, and is formed from
the intracellular deiodination of T4 by type 2 deiodinase. The half life of T4 is 1 week, and of T3 1 day,
suggesting that T4 acts as a source of T3, rather than an active hormone in its own right.

355. You have a post-operative patient on the intensive care unit following an emergency laparotomy 12
hours ago. The patients central venous pressure is decreased. Which of the following is most likely to
cause the decreased central venous pressure? Select one answer only.
Epidural anaesthesia YOUR ANSWER (Correct Answer)
Myocardial Infarction
Pericardial effusion with tamponade
Vasoconstriction in response to sepsis
500 ml gelofusine challenge.

Decreased central venous pressure can result from epidural anaesthesia and vasodilation. Sepsis can
result in a decreased central venous pressure due to vasodilation. High central venous pressure is an
indication of overfilling or cardiac or pulmonary pathology. A fluid challenge will result in a transient high
CVP in adequately filled patients and a maintained high CVP in over-filled patients.
356. You request a lung function test for a patient. What is lung spirometry routinely used to directly
measure? Select one answer only.
FEV1 YOUR ANSWER (Correct Answer)
FEV1:FVC ratio
Functional residual volume
The peak expiratory flow rate
Total lung capacity.

The spirometer is able to measure the forced vital capacity (not the total lung capacity), the FEV1 (forced
expiratory volume in 1 second) and so derive FEV1:FVC (forced vital capacity) ratio. Residual volume is
measured by helium dilution. The peak expiratory flow rate is routinely measured with a Wrights peak
flow meter, and FEV1 is the dynamic measure of flow in spirometry.
357. The brain is the least tolerant of all the body organs to ischaemia. Interruption of cerebral blood flow
for as little as 5 s may cause fainting (syncope), while even transient decreases in blood flow result in
---------------------------------------------------------------------------------------------------------------------------------------------------------------------------------------------------------------
Dr Mohammed Shamsul Islam Khan, Medical Officer, Clinical Neuro-Surgery, National Institute of Neuro-Sciences and Hospital
Sher-E-Bangla Nagar, Dhaka-1207, Bangladesh. Mobile: +880 1713 455 662, +880 1685 811979. E-mail: drsikhan@gmail.com
MyPasTest MRCS A Online - Jan Exam 2015
10. Physiology; System Specific Physiology (399Qs)
----------------------------------------------------------------------------------------------------------------------------------
dizziness. What percentage of cardiac output is received by brain? Single best answer question choose
ONE true option only.
1% YOUR ANSWER
15% CORRECT ANSWER
35% Page |
50% 993
75%.

The brain is the least tolerant of all the body organs to ischaemia. Interruption of cerebral blood flow for
as little as 5 s may cause fainting (syncope), while even transient decreases in blood flow result in
dizziness. The brain is encased in the rigid cranium and fluids (that comprise about 80% of all tissues) are
incompressible.

This means that the total volume within the cranial cavity must remain constant. As a result, an increase
in arterial vascular volume associated with arteriolar dilatation at one site must be accompanied by a
decrease in volume at some other site. The decrease in volume may occur in veins, cerebral ventricles
(cerebrospinal fluid) or at other points within the cranium.

The brain comprises about 2.5% of the total body weight and receives 15% of the cardiac output. Oxygen
extraction is relatively high, however, with venous oxygen levels approximating 13 vol per cent, an
arteriovenous oxygen difference of 7 vol per cent. These values are similar to those found in resting
skeletal muscle.

358. Which one of the following is true regarding the adrenal gland? Select one answer only.
Has a part known as the medulla, which secretes hormones, derived from cholesterol YOUR ANSWER
Produces hormones in response to corticotrophin-releasing factor acting on the receptors of the adrenal
gland itself
Is responsible for the stress response CORRECT ANSWER
Produces hormones from its cortex, which tend to act on extracellular receptor molecules
Produces hormones from its cortex, which tend to act via second messenger systems.

The adrenal gland consists of two parts the medulla and the cortex. The medulla secretes adrenaline and
noradrenaline in response to stress. This reaction is rapid and generates the early part of stress
response. The adrenal cortex produces a variety of hormones, which are all derived from cholesterol.
These include cortisol, progesterone, aldosterone and androgens. The release of these hormones is
controlled by the hypothalamo-pituitary axis, which culminates in stimulation of receptors in the gland
itself by adrenocorticotrophic hormone (ACTH).

ACTH is released in response to release of corticotrophic hormone (CRH) by the hypothalamus. These are
able to penetrate the plasma membrane of their target organs and therefore they tend to act on
intracellular receptors. These lipid hormones exert their effects through causing increased or decreased
expression of specific gene targets. The adrenal medullary hormones are not lipid soluble. They therefore
---------------------------------------------------------------------------------------------------------------------------------------------------------------------------------------------------------------
Dr Mohammed Shamsul Islam Khan, Medical Officer, Clinical Neuro-Surgery, National Institute of Neuro-Sciences and Hospital
Sher-E-Bangla Nagar, Dhaka-1207, Bangladesh. Mobile: +880 1713 455 662, +880 1685 811979. E-mail: drsikhan@gmail.com
MyPasTest MRCS A Online - Jan Exam 2015
10. Physiology; System Specific Physiology (399Qs)
----------------------------------------------------------------------------------------------------------------------------------
exert their effects through extracellular receptors, which exert their intracellular effects by second
messenger systems. The response to this type of hormone is much more rapid.

359. A patient is undergoing a gastroenterostomy. What is the best way to prevent Dumping Syndrome?
Excise antrum YOUR ANSWER
Page |
Excise pylorus
Preserve antrum
994
Preserve cardia
Preserve pylorus CORRECT ANSWER.

If the pylorus is removed, the risk of dumping syndrome is increased as this increases the rate of gastric
emptying.

360. You review a patient on the ward one day following laparotomy and diagnose type 2 respiratory
failure. Which of the following statements is correct with respect to type 2 respiratory failure? Select one
answer only.
In type-2 failure alveolar ventilation is inadequate YOUR ANSWER (Correct Answer)
In type-2 failure decreased PO2 is the most concerning feature
In type-2 failure the functional residual capacity is reduced
Type 2 failure is improved by the administration of opiates
In type-2 failure the patient may be breathless despite normal blood gases.
Type-1 failure is acute hypoxia due to reduced gas exchange secondary to damaged lung tissue, a right to
left shunt or ventilation/perfusion (VQ) mismatch. Functional capacity is reduced, so tidal exchange
occurs below the closing volume. There is decreased lung compliance. In type-2 failure, hypercarbia is a
feature and oxygen therapy above 28% may be contraindicated if the patient has a hypoxic respiratory
drive.

361. A 26-year-old novice athlete partakes in his first 10K race. Along the way he neglects to drink any
fluids. Which of the following occurs in dehydration? Single best answer - select one answer only.
Approximately 15% of the glomerular filtrate is reabsorbed in the proximal renal tubule YOUR ANSWER
Fluid in the distal convoluted tubule is hypertonic with respect to plasma
Interstitial fluid in the medulla of the kidney is hypotonic
Renal blood flow will increase
Urine output may decrease to around 600 ml/24 h CORRECT ANSWER.

Dehydration leads to splanchnic and visceral vasoconstriction, so renal blood flow falls. This causes a
drop in urine output and a minimum obligatory urine output of 500 ml/24 h may result. The secretion of
antidiuretic hormone (ADH) leading to increased water reabsorption is another mechanism by which this
occurs. The loops of Henle of the juxtamedullary nephrons dip deeply into the medullary pyramids before
---------------------------------------------------------------------------------------------------------------------------------------------------------------------------------------------------------------
Dr Mohammed Shamsul Islam Khan, Medical Officer, Clinical Neuro-Surgery, National Institute of Neuro-Sciences and Hospital
Sher-E-Bangla Nagar, Dhaka-1207, Bangladesh. Mobile: +880 1713 455 662, +880 1685 811979. E-mail: drsikhan@gmail.com
MyPasTest MRCS A Online - Jan Exam 2015
10. Physiology; System Specific Physiology (399Qs)
----------------------------------------------------------------------------------------------------------------------------------
draining into the distal convoluted tubules in the cortex. There is a graded increase in the osmolality at
the tips of the papillae (around 1200 mOsm/l approximately equal to four times that of plasma).

362. You are treating a patient with increased lung compliance. What would increase lung compliance?
Select one answer only.
A general anaesthetic YOUR ANSWER Page |
Hypervolaemia 995
Pulmonary fibrosis
The head-up position CORRECT ANSWER
The lithotomy position.

Lung compliance is inversely proportional to the elasticity of the lung. In emphysema, all elastic tissues
have been destroyed and compliance increases. Compliance increases in hypovolaemia (because there is
less blood in lung and it is easier to expand). Pulmonary fibrosis makes the lung more difficult to expand,
as does a general anaesthetic and any feet-up position.

363. A 62-year-old undergoes a gastric bypass. She attends the surgical assessment unit feeling unwell,
is seen by the consultant and told she has Dumping Syndrome. She is unsure what this means and you
have been asked to explain it to her. What are the likely symptoms she has been suffering? Single best
answer - select one answer only.
Collapse and chest pain YOUR ANSWER
Left upper quadrant pain, diarrhoea, chest pain
Pain before eating, dizziness and nausea
Postprandial pain, vomiting and dizziness CORRECT ANSWER
Shortness of breath and chest pain.

Dumping Syndrome following gastrectomy is due to absent gastric reservoir, abnormal postoperative
gastric motor function and hormonal secretion alteration. After eating, symptoms are related to the rapid
transit of hyperosmolar contents into the small bowel. Osmotic fluid shifts lead to the vasomotor
symptoms.

364. A 14-year-old boy is brought to his GP with a history of generalised lethargy and weight loss. Further
questioning reveals polydipsia and polyuria. A random blood sugar is very high and further testing
confirms a new diagnosis of type 1 diabetes mellitus. The endocrine component of the pancreas is
composed of the islets of Langerhans which are composed of , and cells. Which of the following
statements best describe the functions of these cells? Single best answer - select one answer only.
cells produce glucagon, cells produce insulin, cells produce somatostatin YOUR ANSWER (Correct
Answer)
cells produce insulin, cells produce glucagon, cells produce somatostatin
cells produce somatostatin, cells produce insulin, cells produce glucagon
---------------------------------------------------------------------------------------------------------------------------------------------------------------------------------------------------------------
Dr Mohammed Shamsul Islam Khan, Medical Officer, Clinical Neuro-Surgery, National Institute of Neuro-Sciences and Hospital
Sher-E-Bangla Nagar, Dhaka-1207, Bangladesh. Mobile: +880 1713 455 662, +880 1685 811979. E-mail: drsikhan@gmail.com
MyPasTest MRCS A Online - Jan Exam 2015
10. Physiology; System Specific Physiology (399Qs)
----------------------------------------------------------------------------------------------------------------------------------
and cells produce glucagon, cells produce insulin
and cells produce insulin, cells produce glucagon.

The islets of Langerhans in the pancreas comprise the endocrine pancreas. The Islets are themselves
composed of alpha, beta and delta cells. The alpha cells produce glucagons in response to low blood
glucose levels. This has the effect of breaking down stores of glycogen and fat to generate glucose and
Page |
fatty acids/ketone bodies respectively. The beta cells of the gland secrete insulin. Insulin promotes the 996
absorption of glucose into cells and the storage of fat deposits and glucagons.

Delta cells are least numerous and secrete somatostatin. Insulin promotes the storage of energy sources
when there is a rich supply (such as after a meal). Glucagon liberates energy stores when there is not a
ready supply. Type 1 diabetes mellitus occurs in young people due the development of anti-islet cell
antibodies. This results in diminished insulin levels and consequent dangerous derangement of glucose
and lipid metabolism.

365. A lesion of the lateral geniculate nucleus of the thalamus will affect: Single best answer question
choose ONE true option only.
Hearing YOUR ANSWER
Smell
Taste
Touch
Vision CORRECT ANSWER.

The lateral geniculate nucleus (LGN) of the thalamus is a part of the brain that is the primary processor of
visual information, received from the retina, in the central nervous system. The LGN receives information
directly from the retina and sends projections directly to the primary visual cortex. In addition, it receives
many strong feedback connections from the primary visual cortex. Although the function of the LGN is
unknown, it has been shown that it introduces coding efficiencies by cancelling out redundant
information from the retina, but there is almost certainly much more going on.

Like other areas of the thalamus, particularly other relay nuclei, the LGN probably helps the visual system
focus its attention on the most important information. That is, if you hear a sound slightly to your left, the
auditory system probably 'tells' the visual system, through the LGN, to direct visual attention to that part
of space. The LGN is also a station that refines certain receptive fields.

366. A patient with chest pain had ST depression on his electrocardiogram (ECG). The ST segment
ordinarily lasts about: Single best answer question choose ONE true option only.
0.02 s YOUR ANSWER
0.04 s
0.08 s CORRECT ANSWER
0.16 s

---------------------------------------------------------------------------------------------------------------------------------------------------------------------------------------------------------------
Dr Mohammed Shamsul Islam Khan, Medical Officer, Clinical Neuro-Surgery, National Institute of Neuro-Sciences and Hospital
Sher-E-Bangla Nagar, Dhaka-1207, Bangladesh. Mobile: +880 1713 455 662, +880 1685 811979. E-mail: drsikhan@gmail.com
MyPasTest MRCS A Online - Jan Exam 2015
10. Physiology; System Specific Physiology (399Qs)
----------------------------------------------------------------------------------------------------------------------------------
0.8 s.

The ST segment connects the QRS complex and the T-wave. This segment ordinarily lasts about 0.08 s
and is usually level with the PR segment. Upward or downward displacement (ST-segment elevation or
ST-segment depression) may indicate coronary ischaemia or myocardial infarction.
Page |
367. Gastric acid secretion is important for the PH of the stomach. Which one of the following statements
about gastric acid is correct? Single best answer - Select one answer only. 997
Controlled by pepsin YOUR ANSWER
H cells secrete gastrin
Passively secreted
Promoted by vagal nerve stimulation CORRECT ANSWER
Sympathetic nervous system activates the release.
The gastric mucosa is thrown into folds called rugae. Microscopically the mucosa is seen to consist of
pits and peaks. In the pits lie the gastric glands. These glands are composed of several types of cell.
Goblet cells secrete mucus. Parietal cells secrete HCl through the actions of the proton pump. Chief cells
secrete an inactive enzyme pepsinogen.

This is activated by low pH through cleavage to form pepsin. Pepsin digests proteins into smaller
polypeptides. G cells secrete gastrin. Gastrin acts in an autocrine fashion to promote acid secretion. Acid
secretion is promoted by the parasympathetic nervous system through the action of acetylcholine,
released in response to vagal nerve transmission. Parietal cells also secrete intrinsic factor which if
required for the adsorption of vitamin B12. This is not functional in pernicious anemia.

368. A 48-year old woman is diagnosed with primary hyperparathyroidism. Which of the following is likely
to be correct? Single best answer select one answer only.
She will also have diabetes mellitus YOUR ANSWER
She will be asymptomatic CORRECT ANSWER
She will have reduced parathyroid hormone (PTH) levels
She will have a raised phosphate
She will have MEN2b syndrome.

Asymptomatic primary hyperparathyroidism is found in greater than 50% of patients. In 85% of cases, it is
caused by a single parathyroid adenoma, but can be due to multiple gland adenomas/hyperplasia and
rarely even parathyroid carcinoma. Patients will have a raised serum calcium and parathyroid hormone
and a reduced phosphate (secondary to phosphaturia). There is no association with diabetes.
Hyperparathyroidism is found in MEN 1 and MEN 2a syndromes, but not in MEN 2b.

369. A 48-year-old man has been unable to eat for 7 days because of an obstructing lesion in the
oesophagus. Which of the following is the major source of fuel being oxidised by his skeletal muscles?
---------------------------------------------------------------------------------------------------------------------------------------------------------------------------------------------------------------
Dr Mohammed Shamsul Islam Khan, Medical Officer, Clinical Neuro-Surgery, National Institute of Neuro-Sciences and Hospital
Sher-E-Bangla Nagar, Dhaka-1207, Bangladesh. Mobile: +880 1713 455 662, +880 1685 811979. E-mail: drsikhan@gmail.com
MyPasTest MRCS A Online - Jan Exam 2015
10. Physiology; System Specific Physiology (399Qs)
----------------------------------------------------------------------------------------------------------------------------------
Single best answer question choose ONE true option only.
Muscle creatine phosphate YOUR ANSWER
Muscle glycogen
Muscle triglycerides
Serum fatty acids CORRECT ANSWER Page |
Serum glucose. 998
Starvation is a severe reduction in nutrient, vitamin and energy intake and is the most extreme form of
malnutrition. In humans, prolonged starvation (in excess of 1-2 months) causes permanent organ damage
and will eventually result in death. Note that the first priority of metabolism in starvation is to provide
sufficient glucose to the brain and other tissues (e.g. erythrocytes) that are absolutely dependent on this
fuel. However, precursors of glucose are not abundant, and most energy is stored in the fatty acyl
moieties of triacylglycerols.

Fatty acids cannot be converted into glucose, because acetyl CoA cannot be transformed into pyruvate.
In addition, the glycerol moiety of triacylglycerol can be converted into glucose but only a limited amount
is available. The only other potential source of glucose is amino acids derived from the breakdown of
proteins. However, proteins are not stored, and so any breakdown will necessitate a functional loss. Thus,
the second priority of metabolism in starvation is to preserve protein, which is accomplished by shifting
the fuel being used from glucose to fatty acids and ketone bodies.

370. Approximately 80% of phosphate is absorbed in the proximal convoluted tubule. Phosphate
reabsorption in the proximal convoluted tubule: Single best answer question choose ONE true option
only.
Is due to active co-transport with calcium ions YOUR ANSWER
Is due to active co-transport with chloride ions
Is due to passive diffusion down its electrochemical gradient
Is inhibited by calcitonin
Is inhibited by parathyroid hormone CORRECT ANSWER.

Regulation of phosphate excretion is accomplished primarily by parathyroid hormone (PTH), which


inhibits phosphate reabsorption in the proximal tubule. At high PTH concentration, as much as 40% of
filtered phosphate may be excreted. Reabsorption of phosphate by renal tubular cells occurs via a carrier
co-transport of phosphate and sodium. This mechanism is similarly to the reabsorption of glucose and
amino acids and is driven by the Na+ gradient built by the Na+/K+-ATPase at the basolateral membrane of
the tubular epithelium.

Co-transport with calcium or chloride does not occur. Reabsorption of phosphate occurs against its
electrochemical gradient and therefore cannot be passive. Calcitonin has only minor effects on renal
calcium and phosphate handling. It lowers serum calcium by suppressing bone osteoclasts, so shifting
the balance in favour of calcium deposition in the bone.

---------------------------------------------------------------------------------------------------------------------------------------------------------------------------------------------------------------
Dr Mohammed Shamsul Islam Khan, Medical Officer, Clinical Neuro-Surgery, National Institute of Neuro-Sciences and Hospital
Sher-E-Bangla Nagar, Dhaka-1207, Bangladesh. Mobile: +880 1713 455 662, +880 1685 811979. E-mail: drsikhan@gmail.com
MyPasTest MRCS A Online - Jan Exam 2015
10. Physiology; System Specific Physiology (399Qs)
----------------------------------------------------------------------------------------------------------------------------------
371. A 55-year-old woman with poorly-controlled polymyalgia rheumatica complains of weight gain,
muscle weakness and easy bruising. She is found to have increased abdominal girth, striae, bruising and
raised serum glucose levels. A diagnosis of Cushings syndrome is made. What is most likely to be the
cause of her Cushings syndrome? Single best answer select one answer only.
Iatrogenic steroids YOUR ANSWER (Correct Answer)
Page |
Medullary carcinoma of the thyroid
999
Pituitary tumour
Lung tumour secreting ACTH
Polymyalgia rheumatic.

This lady has Cushings syndrome and the commonest cause of this is iatrogenic through the
administration of steroids. In this case, this lady has poorly-controlled polymyalgia rheumatica and is
likely to have been on prolonged steroid treatment. Endogenous cases are caused by primary pituitary
disease (70%, referred to as Cushing Disease), primary adrenal tumours (15%) and ectopic ACTH-
secreting tumours (15%).

The diagnosis of Cushings syndrome depends on the loss of normal circadian rhythm as well as
persistent elevation of cortisol. It is necessary to take at least three serum samples to demonstrate the
loss of rhythm. The mean 24-hour cortisol levels can be estimated by collecting and measuring the 24-
hour urinary free cortisol, which gives an integrated measure of cortisol production. As a screening test it
is 95% accurate.

372. In the stomach, chief cells release pepsinogen. Pepsinogen is activated by: Single best answer
question choose ONE true option only.
Acid pH and pre-existing pepsin YOUR ANSWER (Correct Answer)
Cholecystokinin
Chymotrypsin
Gastrin and pepsin
Trypsin and acid pH.

Pepsin is expressed as a pro-form zymogen, pepsinogen, whose primary structure has an additional 44
amino acids. In other words, pepsinogen is an inactive precursor of the proteolytic enzyme, pepsin.
Following secretion from the peptic and mucous cells of the gastric glands, the pepsinogen comes into
contact with hydrochloric acid and previously formed pepsin, which split the pepsinogen to form active
pepsin.

373. A patient is admitted to accident and emergency with fulminant necrotising fasciitis and debrided.
Post-operatively, he develops low urine output and raised creatinine. Which of the following is correct?
Single best answer - select one answer only.
Acute renal failure is drug induced in 75% of cases YOUR ANSWER
Acute renal failure carries an overall mortality rate of 10%
---------------------------------------------------------------------------------------------------------------------------------------------------------------------------------------------------------------
Dr Mohammed Shamsul Islam Khan, Medical Officer, Clinical Neuro-Surgery, National Institute of Neuro-Sciences and Hospital
Sher-E-Bangla Nagar, Dhaka-1207, Bangladesh. Mobile: +880 1713 455 662, +880 1685 811979. E-mail: drsikhan@gmail.com
MyPasTest MRCS A Online - Jan Exam 2015
10. Physiology; System Specific Physiology (399Qs)
----------------------------------------------------------------------------------------------------------------------------------
Acute renal failure occurs in approximately 90% of critically ill patients
Acute renal failure is associated with distinct histological changes
Acute renal failure is usually the result of damage to the proximal tubule CORRECT ANSWER.

ARF occurs in about 30% of critically ill patients. Loss of renal function leads to a 60% increase in Page |
mortality and morbidity. The common causes of ARF include postoperative hypovolaemia, congestive
cardiac failure, radiocontrast and drugs (aminoglycosides, NSAIDs). Only 20% of ARF is drug-induced. In
1000
60% of cases, ARF is potentially avoidable, as it is the result of fluid or drug mismanagement.

The histological appearance of the kidney bears little relevance to the level of renal dysfunction a
severe functional renal deficit can occur with minimal histological changes. The proximal tubule is the
segment that is most susceptible to injury and reflects the relative hypoxia in the outer medulla and inner
cortex.

374. Starlings law of the heart describes the strength of the hearts systolic contraction. Which of the
following statements is correct? Single best answer - select one answer only.
Describes how the cardiac output is matched to mean arterial pressure YOUR ANSWER
Explains how congestion in the left or right side of the heart is prevented CORRECT ANSWER
Means that the more the heart is filled during systole the greater the force of contraction during diastole
Means that the more the heart is filled the lesser the volume of blood that will be ejected during systole
States that the energy of cardiac contraction is dependent upon the resting length of skeletal muscle fibre.

Starling described the intrinsic response of the heart to changes in right atrial and aortic pressure. With
an increase in ventricular filling during diastole there is an increase in fibre length, which results in an
increase in the force of contraction and stroke volume. This is an adaptation of stroke volume to venous
return.

375. The primary function of the skin circulation is to help maintain body temperature. Blood vessels
constrict to prevent heat loss and dilate to facilitate transfer of heat from the body core to the body
surface. What percentage of cardiac output is received by skin? Single best answer question choose
ONE true option only.
2% YOUR ANSWER (Correct Answer)
12%
20%
50%
70%.
The skin comprises 4-5% of the total body weight and receives about 2% of the cardiac output. The
arteria-venous oxygen difference is small (3 vol per cent), indicating that most of the blood flow is non-
nutrient flow.
---------------------------------------------------------------------------------------------------------------------------------------------------------------------------------------------------------------
Dr Mohammed Shamsul Islam Khan, Medical Officer, Clinical Neuro-Surgery, National Institute of Neuro-Sciences and Hospital
Sher-E-Bangla Nagar, Dhaka-1207, Bangladesh. Mobile: +880 1713 455 662, +880 1685 811979. E-mail: drsikhan@gmail.com
MyPasTest MRCS A Online - Jan Exam 2015
10. Physiology; System Specific Physiology (399Qs)
----------------------------------------------------------------------------------------------------------------------------------
376. You admit a patient known to have asthma who requires emergency surgery and you measure their
peak expiratory flow rate (PEFR). Which of the following statements is correct regarding the PEFR in
adults? Select one answer only.
It can be measured by a Wright's peak flow meter YOUR ANSWER (Correct Answer)
It increases with age
Page |
In normal males is equal to or greater than 400 l/min
1001
It is unchanged by training
It is unchanged in obstructive lung disease.

In normal adult males, the peak expiratory flow rate (PEFR) is greater than 500 l/min and in females
greater than 400 l/min. It is measured with a Wrights peak flow meter. It improves with training but
decreases with age. It is classically reduced in obstructive lung disorders such as asthma.

377. The acute blood loss of 1.5 litres leads to a decrease in? Single best answer question choose ONE
true option only.
The rate of oxygen extraction by the peripheral tissues YOUR ANSWER
Renin secretion
Platelet count
The cardiac output CORRECT ANSWER
Coronary and cerebral blood flow due to sympathetic overactivity.

The rate of oxygen extraction by the peripheral tissues is increased in response to acute blood loss, renin
secretion is also increased due to renal hypoperfusion. Platelet count is increased and cardiac output
decreased as the stroke volume decreases. The blood flow to the brain and the heart remains unchanged.
378. A 58-year-old man with a family history of MI is admitted with 3 vessel cardiac disease. He is offered
a CABG to alleviate his symptoms. During the process, he is placed on cardiopulmonary bypass. Which
of the following regarding bypass is correct? Select one answer only.
May produce severe postoperative neuropsychological problems YOUR ANSWER
Is complicated by acute respiratory distress syndrome in 60% of patients
Is complicated by a severe stroke in 10% of cases
Causes a thrombocythaemia
Serum cortisol levels fall upon induction of cardiopulmonary bypass CORRECT ANSWER.

Neurological complications are not uncommon following cardiopulmonary bypass and usually manifest
clinically as mild neuropsychological deficits, such as poor short-term memory, lack of concentration and
mood changes. These deficits are usually reversible and thought to result from cerebral microemboli or
hypoperfusion. Fewer than 1% of patients suffer a severe stroke. Cardiopulmonary bypass causes platelet
depletion and also reduces platelet activity. Acute pancreatitis occurs in 25% of open-heart procedures
and may be severe in 5% of cases. ARDS occurs in 1.5% of patients. Although serum cortisol levels rise
---------------------------------------------------------------------------------------------------------------------------------------------------------------------------------------------------------------
Dr Mohammed Shamsul Islam Khan, Medical Officer, Clinical Neuro-Surgery, National Institute of Neuro-Sciences and Hospital
Sher-E-Bangla Nagar, Dhaka-1207, Bangladesh. Mobile: +880 1713 455 662, +880 1685 811979. E-mail: drsikhan@gmail.com
MyPasTest MRCS A Online - Jan Exam 2015
10. Physiology; System Specific Physiology (399Qs)
----------------------------------------------------------------------------------------------------------------------------------
with major surgery in general, serum cortisol levels typically drop upon the induction of cardiopulmonary
bypass, likely as a consequence of haemodilution.

379. You are reviewing one of your patients lung function tests. The results are for a 25-year-old male,
who is normally fit and well and weighs 70Kg. What would you expect the normal vital capacity to be?
Select one answer only. Page |
Vital capacity (3.5L) = Inspiratory reserve volume (3L) + tidal volume (0.5L) YOUR ANSWER 1002
Vital capacity (4.5L) = Inspiratory reserve volume (3L) + expiratory reserve volume (1.5L)
Vital capacity (5L) = inspiratory reserve volume (3L) + tidal volume (0.5L) + expiratory reserve volume (1.5L)
CORRECT ANSWER
Vital capacity (6L) = Inspiratory reserve volume (3L) + tidal volume (0.5L) + expiratory reserve volume (1.5L)
+ residual volume (1L)
Vital capacity (2L) = Tidal volume (0.5L) + expiratory reserve volume (1.5L).

Vital capacity is equivalent to the sum of the inspiratory reserve volume, tidal volume and expiratory
reserve volume.

380. A 62-year-old woman undergoes a total gastrectomy for gastric cancer. She requires careful follow
up because of potential malabsorption and nutrient deficiencies. Which are the most likely nutrients that
she may be deficient in, if left untreated? Single best answer - select one answer only.
Iron, Vitamin B12 YOUR ANSWER (Correct Answer)
Iron, Vitamin A
Glucose, Vitamin B12
Glucose, Vitamin D
Vitamin C.

Iron absorption is dependent on iron being in the reduced ferrous state (Fe2+) and acid from the stomach
and duodenum is predominantly responsible for this. Intrinsic factor is a glycoprotein produced by the
stomach and is a cofactor required for the absorption of vitamin B12 (cyanocobalamin). Cyanocobalamin
becomes firmly bound to intrinsicfactor in the small intestine. This complex then becomes bound to
specific receptors in the terminal ileum, and cyanocobalamin is transferred across the intestinal
epithelium. Trypsin is required for this process to be efficient. Vitamins A, D, E and K are fat soluble
vitamins they require bile salts to be absorbed so therefore if the terminal ileum was excised bile salts
and therefore fat soluble vitamins cannot be absorbed.

381. With regard to CO2 transported in the blood, most of the CO2 is---Single best answer question
choose ONE true option only.
Dissolved in plasma YOUR ANSWER
In the form of carbamino compounds formed from plasma proteins

---------------------------------------------------------------------------------------------------------------------------------------------------------------------------------------------------------------
Dr Mohammed Shamsul Islam Khan, Medical Officer, Clinical Neuro-Surgery, National Institute of Neuro-Sciences and Hospital
Sher-E-Bangla Nagar, Dhaka-1207, Bangladesh. Mobile: +880 1713 455 662, +880 1685 811979. E-mail: drsikhan@gmail.com
MyPasTest MRCS A Online - Jan Exam 2015
10. Physiology; System Specific Physiology (399Qs)
----------------------------------------------------------------------------------------------------------------------------------
In the form of carbamino compounds formed from haemoglobin
Bound to Chloride
In the form of HCO3- CORRECT ANSWER.

Carbon dioxide is transported in three main ways: Page |


01. Carbamino compounds between CO2 and proteins. Most of these reactions are with the
globin portion of haemoglobin, accounting for 20-30% of the transported CO2.
1003
02. Dissolved CO2 accounts for about 10% of the transported CO2.
03. HCO3-accounts for about 60-70% of the transported CO2.

382. Theme: Abominal anatomy


A Proximal convoluted tubule
B Descending loop
C Ascending loop
D Distal convoluted tubule
E Collecting duct
F Bowman's capsule
G Ureter.

Choose the correct site for the following features of renal physiology from the list above. Each option
may be used once, more than once, or not at all.

Scenario 1
Facultative reabsorption of K+
A - Proximal convoluted tubule YOUR ANSWER
D - Distal convoluted tubule CORRECT ANSWER.

Scenario 2
Na+ reabsorption
B - Descending loop YOUR ANSWER
A - Proximal convoluted tubule CORRECT ANSWER.

Scenario 3
Glucose reabsorption
C - Ascending loop YOUR ANSWER
A - Proximal convoluted tubule CORRECT ANSWER.

Scenario 4
Low ion permeability, high water permaeability
D - Distal convoluted tubule YOUR ANSWER
B - Descending loop CORRECT ANSWER.

The proximal convoluted tubule is responsible for the absorption of water, sodium, potassium, chloride,
phosphate, bicarbonate, glucose and amino acids. The loop of Henle is divided into two parts: the thick
segment of the ascending limb is impermeable to water and urea and can pump NaCl into the lumen. The
---------------------------------------------------------------------------------------------------------------------------------------------------------------------------------------------------------------
Dr Mohammed Shamsul Islam Khan, Medical Officer, Clinical Neuro-Surgery, National Institute of Neuro-Sciences and Hospital
Sher-E-Bangla Nagar, Dhaka-1207, Bangladesh. Mobile: +880 1713 455 662, +880 1685 811979. E-mail: drsikhan@gmail.com
MyPasTest MRCS A Online - Jan Exam 2015
10. Physiology; System Specific Physiology (399Qs)
----------------------------------------------------------------------------------------------------------------------------------
distal convoluted tubule is impermeable to water and urea, but is also the site of action of aldosterone
which acts on a Na+/K+ pump to effectively excrete potassium and absorb sodium. The collecting ducts
are impermeable to water unless acted on by antidiuretic hormone and are slightly permeable to urea.

383. A 75-year-old woman is being followed by her GP for suspected developing primary hypothyroidism.
Which of the following biochemical changes would you most expect to occur first? Single best answer Page |
question choose ONE true option only.
1004
Fall in serum free thyroxine YOUR ANSWER
Fall in serum thyroxine-binding globulin
Fall in serum free triiodothyronine
Fall in serum total triiodothyronine
Increase in serum TSH CORRECT ANSWER.

Hypothyroidism develops gradually, often over many months or even years. In the early stages, free
thyroxine concentrations are maintained in the normal range by the increased secretion of TSH. Patients
with a slightly elevated TSH and lownormal thyroxine are said to have compensated or borderline
hypothyroidism. In some individuals, it appears that this state can be maintained without progression to
frank hypothyroidism. Triiodothyronine concentrations tend to fall later than thyroxine concentrations in
hypothyroidism; the concentration of thyroxine-binding globulin does not change significantly.

384. An 18-year old male starts to attend his local gym to increase his muscular physique. When
considering skeletal muscular contractions, which of the following statements is correct? Single best
answer - select one answer only.
Depends on thick and thin myofilaments moving along relative to each other YOUR ANSWER (Correct
Answer)
Ends when calcium ions are released from the sarcoplasmic reticulum
Is prolonged and maintained in fast twitch fibres
Occurs spontaneously in response to high serum calcium levels
Occurs in response to repolarisation of the sarcolemma.

Depolarisation of the plasma membrane triggers passive release of calcium ions from the sarcoplasmic
reticulum through specific channels along a gradient. The calcium ions bind to specific sites on the thin
filaments. Binding causes formation and breakage of crosslinks between thick and thin filaments. The
cycling of crosslinks results in movement of the myofilaments along each other. This is sustained until
the calcium ions are actively transported back into the sarcoplasmic reticulum and relaxation occurs.

There are two types of muscle fibres, slow and fast that produce which produce prolonged, maintained
contraction and transient fatigueable contraction respectively. Whole muscle is composed of mixtures of
fast and slow twitch fibres in varying amounts. Sustained muscle spasm, tetany, occurs in response to
low serum calcium levels. This occurs with removal of all the parathyroid glands at thyroidectomy.

---------------------------------------------------------------------------------------------------------------------------------------------------------------------------------------------------------------
Dr Mohammed Shamsul Islam Khan, Medical Officer, Clinical Neuro-Surgery, National Institute of Neuro-Sciences and Hospital
Sher-E-Bangla Nagar, Dhaka-1207, Bangladesh. Mobile: +880 1713 455 662, +880 1685 811979. E-mail: drsikhan@gmail.com
MyPasTest MRCS A Online - Jan Exam 2015
10. Physiology; System Specific Physiology (399Qs)
----------------------------------------------------------------------------------------------------------------------------------
385. Which of the following systemic effects are most likely to be caused by a space occupying lesion in
the brain? Single best answer question choose ONE true option only.
Bradycardia YOUR ANSWER (Correct Answer)
Hypotension
Tachycardia Page |
Tachypnoea 1005
Venous ulceration.

The cranium is a fixed volume containing blood, CSF and brain tissue in equilibrium. Increases in one
component can be compensated by a decrease in the other components without increasing intracranial
pressure (the Monroe-Kellie doctrine). Beyond a certain point, this compensation is insufficient, and
raised intracranial pressure results (greater than 10-15mmHg).

The effects of raised intracranial pressure are hydrocephalus, cerebral ischaemia (due to decreased
cerebral perfusion pressure) and systemic effects. The systemic effects include hypertension,
bradycardia, slowed respiration and gastric ulceration (Cushings ulcer). These are thought to be due to
autonomic dysregulation resulting from hypothalamic compression.

386. You are undertaking your first session at the gym with your new personal trainer who is monitoring
your heart rate whilst you pedal on the exercise bike. Regarding heart rate, which of the following is true?
Select one answer only.
Decreases on inspiration YOUR ANSWER
Increases with pressure on the eyeball
Decreases with sleep CORRECT ANSWER
Decreases after a meal
Decreases with pressure on the sinoatrial (SA) node.

Heart rate is determined by the electrical discharge of the SA node and is normally 6080/min. The SA
node spontaneously fires at 140/min and resting vagal tone reduces this to approximately 70/min.
Baroreceptors in the carotid and aortic sinuses produce a reflex response to hypertension by inhibiting
the SA node and decreasing the heart rate. Inspiration increases heart rate and decreases blood pressure
via stretch receptors in the lung and the respiratory centre in the medulla. The oculocardiac reflex
(Aschner-Dagnini reflex) is a decrease in pulse rate associated with compression of the eyeball.

387. You are assessing a patient in the Emergency Department and note that he has a raised
jugulovenous pressure. What can increase the JVP? Single best answer - select one answer only.
Decreased intrathoracic pressure YOUR ANSWER
Hypovolaemia
Inspiration
---------------------------------------------------------------------------------------------------------------------------------------------------------------------------------------------------------------
Dr Mohammed Shamsul Islam Khan, Medical Officer, Clinical Neuro-Surgery, National Institute of Neuro-Sciences and Hospital
Sher-E-Bangla Nagar, Dhaka-1207, Bangladesh. Mobile: +880 1713 455 662, +880 1685 811979. E-mail: drsikhan@gmail.com
MyPasTest MRCS A Online - Jan Exam 2015
10. Physiology; System Specific Physiology (399Qs)
----------------------------------------------------------------------------------------------------------------------------------
Standing
Superior vena cava (SVC) obstruction CORRECT ANSWER.

The jugulovenous pressure (JVP) is a way of measuring right atrial pressure by observing the atrial
pressure waveform in the neck veins (continuous column of blood from right atrium). It is affected by
atrial contraction and relaxation, the closure of the atrioventricular (AV) valves and venous relaxation and
Page |
filling. It is reduced on inspiration and standing (reduced venous return). It is increased in expiration, 1006
when supine, on exercise, if there is an increase in intrathoracic pressure, superior vena cava (SVC)
obstruction and increased blood volume (eg in pregnancy and overtransfusion).

388. A 40-year-old fit and well Polish woman presents to the ENT clinic with a solitary thyroid nodule. She
is asymptomatic. What is the most useful first line investigation? Single best answer select one answer
only.
CT scan YOUR ANSWER
MRI scan
Fine needle aspiration cytology CORRECT ANSWER
Technetium radioisotope scan
Core biopsy.

Thyroid nodules are common, and the majority are within multinodular goitres. The most useful initial
investigations are ultrasound and fine needle aspiration cytology of any dominant nodules. Thyroid
function tests in patients with multinodular goitre or a solitary thyroid nodule are usually normal. These
are therefore usually only necessary if you suspect a hot nodule in a symptomatic patient. Only around
1020% of cold nodules in adults and 50% in children are malignant. Further imaging is only necessary if
you suspect a very large goitre causing tracheal compression or retrosternal extension. In such cases, CT
of the neck and thorax is useful.

389. A 40-year-old obese woman comes to you in the surgical out-patient clinic complaining of right upper
quadrant abdominal pain, which occurs whenever she takes a fatty meal. Which of the following
substances may be involved in the pathophysiology of this woman's complaint? Single best answer
question choose ONE true option only.
Cholecystokinin YOUR ANSWER (Correct Answer)
Chymotrypsin
Gastrin
Secretin
Somatostatin.

The woman in this clinical vignette has symptoms suggestive of gallstones. During the ingestion of a
meal, vagal parasympathetic discharges stimulate gall-bladder contractions. Intraluminal fats or amino
acids in the intestine stimulate cholecystokinin (CCK) cells in the duodenum to release CCK, which
causes sustained gall-bladder contractions and relaxation of the sphincter of oddi. In the presence of
---------------------------------------------------------------------------------------------------------------------------------------------------------------------------------------------------------------
Dr Mohammed Shamsul Islam Khan, Medical Officer, Clinical Neuro-Surgery, National Institute of Neuro-Sciences and Hospital
Sher-E-Bangla Nagar, Dhaka-1207, Bangladesh. Mobile: +880 1713 455 662, +880 1685 811979. E-mail: drsikhan@gmail.com
MyPasTest MRCS A Online - Jan Exam 2015
10. Physiology; System Specific Physiology (399Qs)
----------------------------------------------------------------------------------------------------------------------------------
gallstones, inflammation of the gall-bladder occurs and CCK, by causing gall-bladder contractions in the
presence of gallstones, aggravates the inflammation.

390. Which of the following cells secretes intrinsic factor? Single best answer question choose ONE
true option only.
Goblet cells YOUR ANSWER Page |
Kupffer cells 1007
Peptic cells
Chief cells
Parietal cells CORRECT ANSWER.

Goblet cells are mucus-secreting cells, widely distributed throughout epithelial surfaces, but especially
dense in the gastrointestinal and respiratory tracts. Kupffer cells have phagocytic properties and are
found in the liver. They participate in the removal of ageing erythrocytes and other particulate debris.

The gastric mucosa contains many cell subtypes, including acid-secreting cells (also known as parietal or
oxyntic cells), pepsin secreting cells (also known as peptic, chief or zymogenic cells) and G-cells (gastrin-
secreting cells). Peptic cells synthesise and secrete the proteolytic enzyme, pepsin. Parietal cells actively
secrete hydrochloric acid into the gastric lumen, accounting for the acidic environment encountered in
the stomach.

However parietal cells are also involved in the secretion of the glycoprotein, intrinsic factor.
Intrinsic factor plays a pivotal role in the absorption of vitamin B12 from the terminal ileum. Autoimmune
damage to parietal cells leads to a lack of intrinsic factor and hydrochloric acid, leading to vitamin B12
deficiency and achlorhydria. This is known as pernicious anaemia. Pernicious anaemia is associated with
a 3-fold increase in gastric cancer risk.

391. Your consultant has asked you to prepare a presentation for his 2nd year medical students on the
anatomy and function of the loop of Henle. Which of the following is true? Single best answer - select one
answer only.
Chloride is transported from the tubular fluid into the ascending limb YOUR ANSWER
Tubular sodium concentration increases as the descending limb travels downwards CORRECT ANSWER
The descending limb is impermeable to urea
The low osmolality maintains the counter-current mechanism
Water moves out of the ascending limb into the interstitium.

The descending loop of Henle is permeable to water but the ascending loop is impermeable. Sodium,
potassium and chloride are co-transported out of the thick segment of the ascending limb. Therefore, a
gradient exists whereby the fluid in the descending loop of Henle becomes hypertonic as water moves
into the hypertonic interstitium (sodium level increases as the descending limb travels downwards). In the
---------------------------------------------------------------------------------------------------------------------------------------------------------------------------------------------------------------
Dr Mohammed Shamsul Islam Khan, Medical Officer, Clinical Neuro-Surgery, National Institute of Neuro-Sciences and Hospital
Sher-E-Bangla Nagar, Dhaka-1207, Bangladesh. Mobile: +880 1713 455 662, +880 1685 811979. E-mail: drsikhan@gmail.com
MyPasTest MRCS A Online - Jan Exam 2015
10. Physiology; System Specific Physiology (399Qs)
----------------------------------------------------------------------------------------------------------------------------------
ascending limb, the fluid in the tubules becomes more dilute and is hypotonic to plasma because of
movement of sodium and chloride out of the tubular lumen. This forms the mechanism of the counter-
current multiplier.

392. A 72-year-old patient presents with urinary retention. He describes pneumaturia and feculent debris
in his urine. He has had significant weight loss and a change in bowel habit over the last two months. A Page |
contrast enema shows an obstructing lesion in the distal sigmoid colon. He is otherwise fit and well. What
is the best definitive treatment in this case? Single best answer - select one answer only. 1008
Catheterise YOUR ANSWER
Neostigmine
Octreotide
Surgical resection of colonic lesion CORRECT ANSWER
Total parental nutrition.

This patient has developed a malignant colovesical fistula as a result of a sigmoid tumour. This patient
will initially need a catheter but it is not the definitive treatment requested. A fistula will not heal if there
is distal obstruction in this case an obstructing operative tumour. TPN maybe used in the treatment of a
high output fistulae but it is not definitive. Similarly Octreotide, a somatostatin statin analogue, may be
used in high output fistulae to reduce the volume of gastrointestinal secretion but is not a definitive
management here in a lower GI fistula. Risk factors for fistulae include malignancy, distal obstruction,
previous radiotherapy.

393. You are treating a patient who has sustained a significant blunt trauma to the left side of the chest.
The patient is peripherally cyanosed. Tissue hypoxia secondary to chest trauma may result from which of
the following? Select one answer only.
Alveolar hypersensitivity YOUR ANSWER
Hyaline membrane formation
Hypervolaemia
Hypocarbia
Pulmonary ventilationperfusion mismatch CORRECT ANSWER.

Pulmonary ventilationperfusion mismatch results from lung contusion, haematomas and alveolar
collapse. Reduced ventilation results in hypercarbia. Hypovolaemia from blood loss and changes in
intrathoracic pressure relationships (from tension or open pneumothoraces) clearly compromise oxygen
exchange.

394. You are called to review a ward patient who you suspect is in severe respiratory distress. What is a
reliable sign of severe respiratory distress? Select one answer only.
Bradycardia YOUR ANSWER
Cyanosis
---------------------------------------------------------------------------------------------------------------------------------------------------------------------------------------------------------------
Dr Mohammed Shamsul Islam Khan, Medical Officer, Clinical Neuro-Surgery, National Institute of Neuro-Sciences and Hospital
Sher-E-Bangla Nagar, Dhaka-1207, Bangladesh. Mobile: +880 1713 455 662, +880 1685 811979. E-mail: drsikhan@gmail.com
MyPasTest MRCS A Online - Jan Exam 2015
10. Physiology; System Specific Physiology (399Qs)
----------------------------------------------------------------------------------------------------------------------------------
Hypertension
Intercostal recession CORRECT ANSWER
Pulsus alternans.

Clinical signs of respiratory distress include use of accessory muscles, increased respiratory rate, Page |
sweating, pulsus paradoxus and inability to speak. CO2 retention can cause a bounding pulse,
vasodilation, tremor and reduced level of consciousness. Cyanosis is an unreliable sign. 1009
395. You request lung function tests for a pre-op patient. In order to measure which of the following lung
volumes, would you have to additionally request a helium washout technique? Select one answer only.
Forced expiratory volume in one second YOUR ANSWER
Functional residual capacity CORRECT ANSWER
Inspiratory capacity
Maximum ventilation volume
Vital capacity.

Inspiratory capacity is the maximum inspiration starting from the normal expiratory position. Total lung
capacity is the total volume of both lungs. V

ital capacity is the maximum volume of air that can be expelled from the lungs by forceful effort after a
maximum inspiration. Maximum ventilation volume is the greatest volume of air that can be breathed in a
given time (litres/minute).

Functional residual capacity (FRC) includes residual volume that cannot be expelled into a spirometer.
FRC (residual volume plus expiratory reserve volume) is measured by a helium washout technique.

396. A 70-year-old retired schoolteacher presents with nocturia and poor urinary flow. He is diagnosed
with benign prostatic hyperplasia (BPH). Which of the following is the most appropriate treatment for him.
Single best answer select one answer only.
Alfuzosin YOUR ANSWER (Correct Answer)
Amlodipine
Goserelin
Methyldopa
Oxybutynin.
Prazosin and alfuzosin are selective alpha blockers which relax prostatic smooth muscles. Finasteride is
a specific inhibitor of 5-alpha reductase which metabolises testosterone into the more potent androgen
dihydrotestosterone. It is an alternative to alpha blockers especially in larger prostates.

Oxybutynin is an anticholinergic drug which is used in the treatment of urinary frequency due to bladder
instability. Goserelin is an luteinising hormone-releasing hormone (LHRH) analogue which has a role in
the treatment of prostatic cancer.
---------------------------------------------------------------------------------------------------------------------------------------------------------------------------------------------------------------
Dr Mohammed Shamsul Islam Khan, Medical Officer, Clinical Neuro-Surgery, National Institute of Neuro-Sciences and Hospital
Sher-E-Bangla Nagar, Dhaka-1207, Bangladesh. Mobile: +880 1713 455 662, +880 1685 811979. E-mail: drsikhan@gmail.com
MyPasTest MRCS A Online - Jan Exam 2015
10. Physiology; System Specific Physiology (399Qs)
----------------------------------------------------------------------------------------------------------------------------------
Amlodipine and methyldopa are antihypertensives.

397. A 47-year-old patient is admitted with pancreatitis. His Modified Glasgow score is 3 based on his
blood results listed: wcc 11, pa02 7.8, Ur 2.3, LDH 555, ALT 140, Glucose 11.1. Which is the most likely
remaining results? Single best answer - select one answer only.
Albumin 28 YOUR ANSWER (Correct Answer) Page |
pC02 5 1010
Creatinine 115
Hb 8.9
Potassium 5.2.

The Modified Glasgow Score is one such Scoring System for pancreatitis done on admission. A point is
recorded for each criteria below. Note that this spells the mnemonic PANCREAS. A score of 3 or more
indicates a severe attack and ideally should have an ITU review/ monitored in ITU.

This patient scores 3 based on pa02, glucose and albumin levels.


01. PaO2 < 7.9kPa
02. Age > 55 years
03. Neutrophils : wcc> 15 x 10/l
04. Calcium < 2mmol/l
05. Renal Function : Urea> 16mmol/l
06. Enzymes : AST/ALT > 200Iu/L or LDH > 600iU/L
07. Albumin < 32 g/L
08. Sugar: Glucose > 10mmol/L
Other such scoring systems include Ransons Criteria and APACHE.

398. Gastric blood flow is influenced by both neural and humoral factors. Which of the following factors
reduces gastric blood flow? Single best answer question choose ONE true option only.
Acetylcholine YOUR ANSWER
Gastrin
Histamine
Vagal stimulation
Vasopressin CORRECT ANSWER.

Gastric blood flow is influenced by both neural and humoral factors. Vagal stimulation, gastrin, histamine
and acetylcholine stimulate gastric secretion and the production of vasodilator metabolites, which, in
turn, cause increased gastric blood flow. Acetylcholine and histamine also have direct vasodilator effects
on the gastric arterioles. Inhibitors of secretion, such as catecholamines and secretin, inhibit gastric acid
secretion and decrease gastric blood flow by reducing the concentration of local vasodilator metabolites.

Sympathetic stimulation mediated by noradrenaline causes vasoconstriction, as does circulating


vasopressin. The gastric circulation shows the autoregulatory escape phenomenon with sustained
---------------------------------------------------------------------------------------------------------------------------------------------------------------------------------------------------------------
Dr Mohammed Shamsul Islam Khan, Medical Officer, Clinical Neuro-Surgery, National Institute of Neuro-Sciences and Hospital
Sher-E-Bangla Nagar, Dhaka-1207, Bangladesh. Mobile: +880 1713 455 662, +880 1685 811979. E-mail: drsikhan@gmail.com
MyPasTest MRCS A Online - Jan Exam 2015
10. Physiology; System Specific Physiology (399Qs)
----------------------------------------------------------------------------------------------------------------------------------
sympathetic stimulation. Maintaining the integrity of gastric mucosal blood flow is important for
protecting against damage from the high acidity of stomach contents.

399. Which of the following hormones is secreted by the kidney in response to sympathetic nervous
stimulation? Single best answer question choose ONE true option only.
Aldosterone YOUR ANSWER (Correct Answer) Page |
Angiotensin I 1011
Angiotensin II
Erythropoetin
Renin CORRECT ANSWER.

Renin is produced by the juxtaglomerular apparatus of the kidney in response to hypovolaemia, via 3
mechanisms:
1. increased catecholamine levels secondary to sympathetic stimulation from arterial receptors
2. direct effect of hyponatraemia on the juxtaglomerular apparatus
3. reduction of renal perfusion pressure via afferent arteriolar baroreceptors.

Renin acts to cleave angiotensin I from angiotensinogen produced in the liver. Angiotensin converting
enzyme is present in many tissues, especially the lungs, and converts angiotensin I to angiotensin II.
Angiotensin II is a powerful vasoconstrictor, causing vasoconstriction of renal arteries, as well as a
positive inotropic effect on the heart. It also causes release of ADH and adrenaline. Along with
aldosterone, whose release is also stimulated, Angiotensin II conserves Na + and H2O in the gut.
Aldosterone acts to conserve Na+ and H2O in the distal renal tubule and collecting ducts. These
mechanisms combine to restore the plasma volume in hypovolaemia.
Erythropoetin is released by the kidney in response to hypoxia and high levels of the products of red cell
breakdown, and increases the rate of red cell production.

Courtesy:

Dr Mohammed Shamsul Islam Khan


MBBS (CMC); FCPS-II (Neuro-Surgery)
Medical Officer, Clinical Neuro-Surgery
National Institute of Neuro-Sciences and Hospital
Sher-E-Bangla Nagar, Dhaka-1207, Bangladesh.
Mobile: +880 1713 455 662, +880 1685 811979.
E-mail: drsikhan@gmail.com
---------------------------------------------------------------------------------------------------------------------------------------------------------------------------------------------------------------
Dr Mohammed Shamsul Islam Khan, Medical Officer, Clinical Neuro-Surgery, National Institute of Neuro-Sciences and Hospital
Sher-E-Bangla Nagar, Dhaka-1207, Bangladesh. Mobile: +880 1713 455 662, +880 1685 811979. E-mail: drsikhan@gmail.com

Vous aimerez peut-être aussi